Medida Integraç˜ao e Probabilidades

Transcrição

Medida Integraç˜ao e Probabilidades
Distribuiç ã o Estimada:
[email protected], 2.4, 2111.D
0.00002
0.000015
0.00001
5. ´ 10-6
50 000
100 000
150 000
200 000
Colecção Métodos Estocásticos para a
Matemática Financeira I
Medida Integração e Probabilidades
Notas de Lições
Manuel L. Esquı́vel
Professor Associado
de
Probabilidade e Processos Estocásticos
2 de Fevereiro de 2012
2
Prefácio
As the book is largely based on lectures, and, as I like my students to
follow my lectures, proofs are given in great detail; this may bore the
mature mathematician, but it will I believe be a great help to anyone
trying to learn the subject ab initio
In C. A. Rogers, Hausdorff Measures, Cambridge University Press
1970, page viii
Neste texto apresentam-se umas notas de lições da disciplina Medida Integração e
Probabilidades do terceiro ano da licenciatura em Matemática da FCT/UNL. É nesta
disciplina que pela primeira vez os alunos entram em contacto com o integral de Lebesgue
e as suas importantes ramificações na teoria das probabilidades segundo Kolmogorov.
Procurou-se criar pontes com os assuntos estudados nas disciplinas de análise matemática
e de probabilidades e estatı́stica de anos anteriores. Dado pretendermos uma primeira
introdução ao tema, o estudo de alguns resultados fundamentais da teoria requerendo
desenvolvimentos técnicos mais exigentes deverá ser feito nas obras clássicas referidas
nas bibliografias.
O primeiro capı́tulo tem como objectivo motivar a introdução da noção de medida
estabelecendo pontes com a teoria das probabilidades ensinada nos dois primeiros anos
e com o integral de Riemann estudado nas disciplinas de análise matemática. No sexto
capııtulo revê-se, num contexto geral, a lei dos grandes números que foi apresentada
no primeiro capı́tulo para as variáveis de Bernoulli. No capı́tulo sobre os espaços de
funções integráveis apresenta-se uma construção de espaços vectoriais de pares de funções
integráveis e respectivos domı́nios de definição mensuráveis. Esta construção pretende
enfatizar a razão pela qual se consideram habitualmente os espaços de classes de funções
mensuráveis.
Há um aspecto essencial da teoria da integração que não é abordado nestas notas; o
integral como funcional linear positivo sobre um espaço de funções contı́nuas definidas
num espaço localmente compacto. Esta lacuna deve-se sobretudo à orientação prosseguida nestas notas que é a de reforçar a ligação com a teoria das probabilidades. Uma
segunda justificação é a de que só no semestre seguinte ao semestre em que a disciplina
Medida Integração e Probabilidades é leccionada é que os alunos entram em contacto
com aspectos mais detalhados da topologia e da análise funcional.
i
ii
Conteúdo
Prefácio
ii
1 A Teoria da Probabilidade, segundo Borel, para sucessões de Bernoulli
1.1 A representação binária dos números de [0, 1] . . . . . . . . . . . . . . . .
1.2 O princı́pio de Borel . . . . . . . . . . . . . . . . . . . . . . . . . . . . . .
1.3 Leis dos Grandes Números . . . . . . . . . . . . . . . . . . . . . . . . . . .
1.4 Exercı́cios . . . . . . . . . . . . . . . . . . . . . . . . . . . . . . . . . . . .
1.5 Exercı́cios Complementares . . . . . . . . . . . . . . . . . . . . . . . . . .
1.6 Resoluções . . . . . . . . . . . . . . . . . . . . . . . . . . . . . . . . . . . .
Bibliografia . . . . . . . . . . . . . . . . . . . . . . . . . . . . . . . . . . . . . .
1
1
8
19
33
35
37
50
2 Medida, Probabilidade e Acontecimentos
2.1 Introdução . . . . . . . . . . . . . . . . . . . . . . . . .
2.2 Teoria da Medida . . . . . . . . . . . . . . . . . . . . .
2.3 Construção de medidas . . . . . . . . . . . . . . . . . .
2.4 Construção das medidas de Lebesgue-Stieltjes sobre R
2.5 Resoluções . . . . . . . . . . . . . . . . . . . . . . . . .
2.6 Comentários sobre as referências bibliográficas . . . .
2.7 Exercı́cios . . . . . . . . . . . . . . . . . . . . . . . . .
Bibliografia . . . . . . . . . . . . . . . . . . . . . . . . . . .
.
.
.
.
.
.
.
.
.
.
.
.
.
.
.
.
.
.
.
.
.
.
.
.
.
.
.
.
.
.
.
.
.
.
.
.
.
.
.
.
.
.
.
.
.
.
.
.
.
.
.
.
.
.
.
.
.
.
.
.
.
.
.
.
.
.
.
.
.
.
.
.
.
.
.
.
.
.
.
.
.
.
.
.
.
.
.
.
53
53
53
67
69
77
77
78
89
3 As Funções Mensuráveis
3.1 Introdução . . . . . . . .
3.2 As funções mensuráveis
3.3 Apêndice . . . . . . . .
3.4 Exercı́cios . . . . . . . .
Bibliografia . . . . . . . . . .
.
.
.
.
.
.
.
.
.
.
.
.
.
.
.
.
.
.
.
.
.
.
.
.
.
.
.
.
.
.
.
.
.
.
.
.
.
.
.
.
.
.
.
.
.
.
.
.
.
.
.
.
.
.
.
91
91
93
116
117
122
.
.
.
.
123
. 123
. 131
. 133
. 142
.
.
.
.
.
.
.
.
.
.
.
.
.
.
.
.
.
.
.
.
.
.
.
.
.
.
.
.
.
.
.
.
.
.
.
4 A Integração à Lebesgue
4.1 O Integral de Lebesgue . . . . . . . .
4.2 Propriedades importantes do integral
4.3 Os teoremas de convergência . . . .
4.4 Esperança matemática . . . . . . . .
.
.
.
.
.
.
.
.
.
.
.
.
.
.
.
.
.
.
.
.
.
.
.
.
.
.
.
.
.
.
.
.
.
.
.
.
.
.
.
.
. . . . . . . .
de Lebesgue
. . . . . . . .
. . . . . . . .
.
.
.
.
.
.
.
.
.
.
.
.
.
.
.
.
.
.
.
.
.
.
.
.
.
.
.
.
.
.
.
.
.
.
.
.
.
.
.
.
.
.
.
.
.
.
.
.
.
.
.
.
.
.
iii
CONTEÚDO
Bibliografia . . . . . . . . . . . . . . . . . . . . . . . . . . . . . . . . . . . . . . 162
5 Os Espaços de Funções Integráveis
5.1 Introdução . . . . . . . . . . . . . . . . . . . . . . . . . .
5.2 Os conjuntos de funções integráveis Lp . . . . . . . . . .
5.3 O espaço de Banach Lp ((X, A, µ)) . . . . . . . . . . . .
5.4 O espaço com produto interno L2 ((X, A, µ)) . . . . . . .
5.5 Conjuntos e funções convexas; a desigualdade de Jensen
5.6 Uma construção dos espaços Lp . . . . . . . . . . . . . .
5.7 Exercı́cios . . . . . . . . . . . . . . . . . . . . . . . . . .
Bibliografia . . . . . . . . . . . . . . . . . . . . . . . . . . . .
6 As leis dos grandes números
6.1 Introdução . . . . . . . . . . . . . .
6.2 A convergência em probabilidade .
6.3 A lei fraca dos grandes números . .
6.4 Uma lei forte dos grandes números
6.5 Sobre a simulação . . . . . . . . .
6.6 Exercı́cios . . . . . . . . . . . . . .
6.7 Resoluções . . . . . . . . . . . . . .
Bibliografia . . . . . . . . . . . . . . . .
7 Independência
7.1 Introdução . . . . . . . . . .
7.2 A noção de independência .
7.3 A lei do 0-1 de Kolmogorov
7.4 Exercı́cios complementares .
Bibliografia . . . . . . . . . . . .
.
.
.
.
.
.
.
.
.
.
.
.
.
.
.
.
.
.
.
.
.
.
.
.
.
.
.
.
.
.
.
.
.
.
.
.
.
.
.
.
.
.
.
.
.
.
8 A Esperança Condicional
8.1 A informação disponı́vel ou conhecida
8.2 O teorema de Radon-Nikodym . . . .
8.3 Definição e Propriedades . . . . . . . .
8.4 Exercı́cios . . . . . . . . . . . . . . . .
8.5 Resoluções . . . . . . . . . . . . . . . .
Bibliografia . . . . . . . . . . . . . . . . . .
.
.
.
.
.
.
.
.
.
.
.
.
.
.
.
.
.
.
.
.
.
.
.
.
.
.
.
.
.
.
.
.
.
.
.
.
.
.
.
.
.
.
.
.
.
.
.
.
.
.
.
.
.
.
.
.
.
.
.
.
.
.
.
.
.
.
.
.
.
.
.
.
.
.
.
.
.
.
.
.
.
.
.
.
.
.
.
.
.
.
.
.
.
.
.
.
.
.
.
.
.
.
.
.
.
.
.
.
.
.
.
.
.
.
.
.
.
.
.
.
.
.
.
.
.
.
.
.
.
.
.
.
.
.
.
.
.
.
.
.
.
.
.
.
.
.
.
.
.
.
.
.
.
.
.
.
.
.
.
.
.
.
.
.
.
.
.
.
.
.
.
.
.
.
.
.
.
.
.
.
.
.
.
.
.
.
.
.
.
.
.
.
.
.
.
.
.
.
.
.
.
.
.
.
.
.
.
.
.
.
.
.
.
.
.
.
.
.
.
.
.
.
.
.
.
.
.
.
.
.
.
.
.
.
.
.
.
.
.
.
.
.
.
.
.
.
.
.
.
.
.
.
.
.
.
.
.
.
.
.
.
.
.
.
.
.
.
.
.
.
.
.
.
.
.
.
.
.
.
.
.
.
.
.
.
.
.
.
.
.
.
.
.
.
.
.
.
.
.
.
.
.
.
.
.
.
.
.
.
.
.
.
.
.
.
.
.
.
.
.
.
.
.
.
.
.
.
.
.
.
.
.
.
.
.
.
.
.
.
.
.
.
.
.
.
.
.
.
.
.
.
.
.
.
.
.
.
.
.
.
.
.
.
.
.
.
.
.
.
.
.
.
.
.
.
.
.
.
.
.
.
.
.
.
.
.
.
.
.
.
.
.
.
.
.
.
.
.
.
.
.
.
.
.
.
.
163
163
163
173
176
176
186
191
192
.
.
.
.
.
.
.
.
.
.
.
.
.
.
.
.
.
.
.
.
.
.
.
.
193
. 193
. 193
. 198
. 199
. 201
. 202
. 207
. 207
.
.
.
.
.
209
. 209
. 210
. 213
. 215
. 217
.
.
.
.
.
.
219
. 219
. 220
. 223
. 236
. 239
. 239
9 O Teorema de Fubini, Leis Conjuntas e a Construção de Sucessões de
Variáveis Aleatórias Independentes
241
9.1 Introdução . . . . . . . . . . . . . . . . . . . . . . . . . . . . . . . . . . . . 241
9.2 A construção dos espaços produto e o teorema de Fubini . . . . . . . . . . 241
9.3 O Teorema de Fubini . . . . . . . . . . . . . . . . . . . . . . . . . . . . . . 245
9.4 Lei Conjunta de um Par de Variáveis Aleatórias . . . . . . . . . . . . . . . 249
iv
9.5
Construção de uma sucessão de variáveis
leis dadas . . . . . . . . . . . . . . . . . .
9.6 Exercı́cios Complementares . . . . . . . .
9.7 Resoluções . . . . . . . . . . . . . . . . . .
9.8 Exercı́cios . . . . . . . . . . . . . . . . . .
Bibliografia . . . . . . . . . . . . . . . . . . . .
aleatórias independentes com
. . . . . . . . . . . . . . . . .
. . . . . . . . . . . . . . . . .
. . . . . . . . . . . . . . . . .
. . . . . . . . . . . . . . . . .
. . . . . . . . . . . . . . . . .
.
.
.
.
.
250
251
254
262
263
v
vi
Capı́tulo 1
A Teoria da Probabilidade,
segundo Borel, para sucessões de
Bernoulli
Nous voici suffisament armés pour resoudre ”mathématiquement”le
problème da la capture du lion [. . . ] partageons le Sahara en deux: le
lion se trouvera certainement sur une des deux parties ainsi ob- tenues
[. . . ] . Considérons maintenant le demi-Sahara ainsi obtenu. Partageons-le en deux: notre lion se trouvera à coup sûr dans une des deux
moitiés. En continuant à procéder ainsi, nous obtenons des surfaces
emboı̂tées les unes dans les autres [. . . ] nous sommes sûrs de tenir le
lion.
In R. Péter, Jeux avec l’infini, p. 256
1.1
A representação binária dos números de [0, 1]
Consideremos o seguinte problema: como localizar um dado número x do intervalo ]0, 1]?
Uma ideia inspirada no algoritmo da caça ao leão no deserto do Saara é a seguinte. Na
primeira etapa, consideramos a partição do intervalo ]0, 1] em dois intervalos de igual
comprimento:
1
1
]0, 1] =]0, ] ∪ ] , 1] .
2
2
Seja α1 o código para o resultado da primeira etapa, definido da forma seguinte:
(
0 se x ∈]0, 12 ]
α1 = α1 (x) :=
1 se x ∈] 21 , 1] .
Na segunda etapa consideramos duas possibilidades e a ambas aplicamos o processo aplicado na primeira etapa. Na primeira possibilidade supomos que x ∈]0, 12 ] e consideramos
Capı́tulo 1
uma nova partição:
1 1
1
1
]0, ] =]0, 2 ]∪] 2 , ] ,
2
2
2 2
dando origem a um código α2 para o resultado da primeira possibilidade da segunda
etapa definido por:
(
0 se x ∈]0, 212 ]
α2 = α2 (x) :=
1 se x ∈] 212 , 12 ] .
Na segunda possibilidade, x ∈] 12 , 1] consideramos a partição:
1
2 3
3
] , 1] =] 2 , 2 ] ∪ ] 2 , 1]
2
2 2
2
e claro está, o código correspondente para esta segunda possibilidade:
(
0 se x ∈] 222 , 232 ]
α2 = α2 (x) :=
1 se x ∈] 232 , 1] .
Ou seja e em resumo para as duas posibilidades da segunda etapa α2 (x):
(
0 se x ∈]0, 212 ] ∪ ] 222 , 232 ]
α2 = α2 (x) :=
1 se x ∈] 212 , 12 ] ∪ ] 232 , 1] .
Exercı́cio 1. Considere um número x ∈]0, 1] arbitrário dado.
1. Represente graficamente as duas etapas do processo de codificação da posição de x que descrevemos
acima.
2. Detalhe todas as possibilidades da terceira etapa no processo que iniciámos acima, após ter efectuado uma representação gráfica dessa terceira etapa.
3. Procure dar a forma de αn para um dado n ∈ N∗ arbitrário.
Podemos aceitar agora que é válido o seguinte resultado.
Proposição 1. Dado x ∈]0, 1] existe uma sucessão (αn )n∈N∗ tal que:
(
n−1 − 1 ,
0 se x ∈] 22pn , 2p+1
2n ] p = 0, . . . , 2
αn = αn (x) :=
2p+2
n−1 − 1 .
1 se x ∈] 2p+1
2n , 2n ] p = 0, . . . 2
(1.1)
Demonstração. A demonstração é um exercı́cio que o leitor deve realizar, por exemplo,
pelo método de indução.
Observação 1. Note-se que, para cada n ∈ N∗ fica assim definida uma correspondência
que a cada x ∈]0, 1] associa αn (x) ∈ {0, 1}.
2
A Teoria da Probabilidade, segundo Borel, para sucessões de Bernoulli
Exercı́cio 2. Mostre que a correspondência x 7−→ αn (x) definida na fórmula 1.1 é uma aplicação de
]0, 1] em {0, 1}.
A questão que podemos colocar agora é a de saber até que ponto é que a sucessão
(αn )n∈N∗ , em que αn ∈ {0, 1}, permite localizar sem qualquer ambiguidade o ponto x.
Uma primeira ideia natural é a que decorre da observação que o ponto x pertence a uma
sucessão de intervalos encaixados, (In (x))n∈N∗ que foram sendo construı́dos pelas etapas
descritas acima e que tem as seguintes propriedades.
1. ∀n ∈ N∗ x ∈ In (x)
2. ∀n ∈ N∗ ∃m ∈ {0, . . . , 2n − 1} In (x) =] 2mn , m+1
2n ]
2m 2m+1
2m+1 2m+2
3. ∀n ∈ N∗ In (x) =] 2mn , m+1
2n ] ⇒ In+1 (x) =] 2n+1 , 2n+1 ] ∨ In+1 (x) =] 2n+1 , 2n+1 ] .
Exercı́cio 3. Seja, por definição tal como acima, In (x) o único intervalo da partição de ]0, 1] em 2n
intervalos iguais de comprimento 1/2n , definidos para cada n ∈ N∗ , ao qual x pertence.
1. Mostre que se verificam, de facto, as propriedades indicadas acima.
2. Mostre que se tem:
\
In (x) = {x} .
n∈N∗
O exercı́cio anterior dá, de facto, uma primeira resposta ao problema da localização
do número x. Uma segunda ideia consiste em representar o ponto x como limite de uma
sucessão de números dados a partir do algoritmo que estudámos, por exemplo como
limite da sucessão das extremidades inferiores (ou superiores) dos intervalos In (x). Pela
fórmula 1.1 sabemos que se para um dado m ∈ {0, . . . , 2n−1 } se tiver x ∈]m/2n , (m +
1)/2n ] então αn (x) é igual a 0 ou a 1 consoante m seja par ou ı́mpar. A nossa segunda
ideia consiste então em definir nestas condições, sn = sn (x) := m/2n . Naturalmente
sabemos que a sucessão (sn )n∈N∗ assim definida é convergente para x.
Exercı́cio 4. Justifique que, com a definição que acabámos de fazer, se tem que:
∀n ∈ N∗ sn < x ≤ sn +
1
2n
e conclua que então:
lim sn = x .
n→+∞
Podemos tornar mais preciso o que sabemos sobre sn se observarmos que, como:
]
m m+1
2m 2m + 1
2m + 1 2m + 2
,
] =] n+1 , n+1 ] ∪ ] n+1 , n+1 ] ,
n
n
2
2
2
2
2
2
se for sn = m/2n então, pela definição, ou se tem sn+1 = 2m/2n+1 ou se tem sn+1 =
(2m + 1)/2n+1 . Suponhamos que nos é dado s1 = α1 /2; sabemos, então, que α1 = 0 se
3
Capı́tulo 1
x ∈]0, 1/2] e α1 = 1 se x ∈]1/2, 1]. Pela observação que fizémos sabemos que s2 = 2α1 /22
ou s2 = (2α1 + 1)/22 . Temos pois que:
s2 =
α1
2α1 + 2
2
=
+ 2 , 2 ∈ {0, 1} ,
2
2
2
2
em que 2 = 1 se e só se, com as notações do processo de codificação que empregámos
acima, α2 = 1 pelo que:
α1 α2
s2 =
+ 2 .
2
2
À terceira etapa teremos que: s3 = 22 α1 /23 ou s3 = (22 α1 + 1)/23 pela primeira hipótese
ou s3 = (22 α1 + 2)/23 ou s3 = (22 α1 + 2 + 1)/23 , isto é, temos em geral que:
s3 =
22 α1 + 22 + 3
, 2 , 3 ∈ {0, 1} ,
23
em que 2 = 1 quando s2 = (2α1 + 1)/22 isto é quando e só quando, com as notações do
processo de codificação que empregámos acima, α2 = 1 sendo pois possı́vel escrever:
s3 =
22 α1 + 2α2 + 3
α1 α2
3
=
+ 2 + 3 , 3 ∈ {0, 1} .
3
2
2
2
2
Uns momentos de reflexão, recorrendo à fórmula 1.1 e eventualmente também à figura
que foi desenhada no exercı́cio 1 permitem-nos concluir que, de facto,:
s3 =
α1 α2 α3
+ 2 + 3 .
2
2
2
Podemos agora sumariar o que sabemos sobre os desenvolvimentos binários de um
dado número do intervalo ]0, 1] na seguinte proposição.
Proposição 2. Seja x ∈]0, 1] e a sucessão (αn )n∈N∗ definida pela proposição 1.
Então verifica-se que:
x=
+∞
X
αn (x)
n=1
2n
= lim
n→+∞
α
1
2
+
α2
αn +
·
·
·
+
.
22
2n
Demonstração. Suponha-se que para um dado qualquer∈ N∗ se tem:
sn =
α1 α2
αn
2n−1 α1 + 2n−2 α2 + · · · + αn
+ 2 + ··· + n =
.
2
2
2
2n
Por construção sabemos que:
x ∈ In (x) =]sn , sn +
4
1
].
2n
(1.2)
A Teoria da Probabilidade, segundo Borel, para sucessões de Bernoulli
Como se tem que
]sn , sn +
1
1
1
1
] =]sn , sn + n+1 ]∪]sn + n+1 , sn + n ] ,
n
2
2
2
2
vem pela definição adoptada que, no caso em que x ∈]sn , sn +
sn+1 = sn =
1
],
2n+1
αn+1 = 0 e,
α1 α2
αn
0
2n α1 + 2n−1 α2 + · · · + 2αn
2sn
=
=
+ 2 + · · · + n + n+1 ,
n+1
2
2
2
2
2
2
e no caso em que x ∈]sn +
1
,s
2n+1 n
sn+1 = sn +
+
1
2n+1
1
2n ]
=
tem-se αn+1 = 1 e,
α1 α2
αn
1
+ 2 + · · · + n + n+1 .
2
2
2
2
Se atendermos agora ao exercı́cio 4 fica concluı́da a demonstração. Com efeito, dado que
para todo o x ∈]0.1] e para todo o n ∈ N∗ se tem que αn (x) ∈ {0, 1}, aplicando o critério
de Weierstrass à série de funções a termos não negativos que figura no enunciado, temos
que:
+∞
+∞
X
1 − ( 12 )n+1
αn (x) X 1
1
≤
=
·
lim
=1,
2n
2n
2 n→+∞ 1 − 12
n=1
n=1
ficando assim demonstrado que a série converge sempre.
∗
Definição 1. A uma sucessão (αn )n∈N∗ ∈ {0, 1}N , verificando para um dado
número x ∈]0, 1] a fórmula 1.2, chamamos desenvolvimento binário ou
diádico de x.
Observação 2. Seja x ∈]0, 1] e seja (αn )n∈N∗ um desenvolvimento binário de x. Usaremos,
por vezes, a notação seguinte
x = .α1 α2 α3 . . . αn . . .
em analogia com a representação usual de um número real pelo seu desenvolvimento
decimal.
Podemos de alguma forma dizer que fica assim resolvida o problema da existência
para o desenvolvimento binário de um número do intervalo ]0, 1]. Uma outra questão
natural é a da unicidade deste desenvolvimento. A proposição que apresentaremos seguidamente, após alguns exemplos, responde a esta questão.
Exercı́cio 5.
1. Considere x = 1/2 e a convenção sobre notação que fizémos na observação 2.
Mostre que:
1
.1000 . . . 0 · · · =
2
e que
1
.0111 . . . 1 · · · = .
2
5
Capı́tulo 1
2. Dê outros exemplos de números de ]0, 1] admitindo dois desenvolvimentos binários distintos.
3. Formule uma conjectura sobre a forma geral dos números que admitem dois desenvolvimentos
binários distintos. Por exemplo, pode dizer-se que todos os números racionais têm dois desenvolvimentos binários distintos? Porquê ?
Proposição 3. Seja x ∈]0, 1] e duas sucessões (βn )n∈N∗ e (γn )n∈N∗ , distintas,
verificando:
+∞
+∞
X
X
βn
γn
x=
e
x
=
.
n
2
2n
n=1
n=1
Necessariamente tem-se que:
∃n0 ∈ N∗ , ∃m0 ∈ {0, 1, . . . , 2n0 } x =
m0
.
2n0
Demonstração. A hipótese feita sobre as sucessões no enunciado, implica que para um
dado n ∈ N∗ se tem que βn 6= γn . Caso contrário as sucessões não seriam distintas.
Consideremos, pois, n0 o primeiro ı́ndice da sucessão verificando a condição βn 6= γn . É
possı́vel considerar o primeiro ı́ndice uma vez que o conjunto dos inteiros é bem ordenado
e por isso todo o subconjunto dos inteiros não vazio tem primeiro elemento. Observemos
que se tem:
+∞
+∞
+∞
X
βn X γn βn0 − γn0 X βn − γn 0 =| x − x |= −
≥
.
− 2n
2n 2n0
2n n=1
n=1
n=n0 +1
Dado que se tem (βn0 − γn0 )/2n0 = 1/2n0 virá necessariamente:
+∞
X
1
βn − γn = n0 ; ,
n
2
2
n=n0 +1
o que só pode acontecer se:
∀n ∈ N∗ , n ≥ n0 + 1 (βn = 1 ∧ γn = 0) ∨ (βn = 0 ∧ γn = 1) .
Em qualquer caso x tem a forma que consta no enunciado, dado que, por exemplo, na
primeira possibilidade acima:
γ1 γ2
γn
2n0 −1 γ1 + 2n0 −1 γ2 + · · · + γn0 −1 + γn0
+ 2 + · · · + n00 =
.
2
2
2
2n0
P
n
n0
e também, atendendo a que +∞
n=n0 +1 (1/2 ) = 1/2 ,
x=
x=
6
β1 β2
βn
1
2n0 −1 β1 + 2n0 −1 β2 + · · · + βn0 −1 + (βn0 + 1)
+ 2 + · · · + n00 + n0 +1 + · · · =
.
2
2
2
2
2n0
A Teoria da Probabilidade, segundo Borel, para sucessões de Bernoulli
Pode observar-se ainda, uma vez que β1 = γ1 , . . . , βn0 −1 = γn0 −1 e que:
2n0 −1 β1 + 2n0 −1 β2 + · · · + βn0 −1 + (βn0 + 1)
2n0 −1 γ1 + 2n0 −1 γ2 + · · · + γn0 −1 + γn0
=
,
2n0
2n0
se tem γn0 = βn0 + 1 e, finalmente, como γn0 , βn0 ∈ {0, 1} vem γn0 = 1 e βn0 = 0.
A proposição anterior e sobretudo a sua demonstração permitem-nos concluir quais
são, exactamente, os pontos do intervalo [0, 1] que têm dois desenvolvimentos binários
distintos.
Definição 2. Os pontos, do intervalo [0, 1], da forma x = m/2n para n ∈ N∗ e
m ∈ {0, 1, . . . , 2n } designam-se por números diádicos.
Os desenvolvimentos binários distintos de um número diádico distinguem-se porque
um deles tem zeros, e só zeros, a partir de uma certa ordem enquanto que o outro tem
uns, e só uns, a partir de uma certa ordem.
Definição 3. A um desenvolvimento binário (αn )n∈N∗ tal que:
∃n0 ∈ N∗ ∀n ∈ N∗ n > n0 ⇒ αn = 0 .
chamamos desenvolvimento binário degenerado. Todos os outros são desenvolvimentos binários não degenerados.
∗
Notação 1. O conjunto B := {0, 1}N é o conjunto das sucessões de Bernoulli.
O conjunto Bndeg ⊂ B é o conjunto das sucessões de Bernoulli não degeneradas
definido como o subconjunto das sucessões de Bernoulli que constituem desenvolvimentos
binários não degenerados.
O resultado importante seguinte sobre o conjunto B irá permitir-nos concluir sobre a
cardinalidade dos intervalos não triviais de R. Para facilitar as notações vamos introduzir
uma noção que poderá revelar-se útil noutros contextos.
Definição 4. O delta de Kronecker δnm é dado para cada n, m ∈ N∗ por:
(
1 se n = m
m
δn :=
(1.3)
0 se n =
6 m.
Proposição 4. O conjunto B é infinitoa e não é numerávelb .
a
Um conjunto é infinito se e só se pode ser posto em bijecção com um seu subconjunto
próprio; pode verificar-se que se um conjunto contem um subconjunto infinito então o conjunto
é também infinito.
b
Um conjunto é numerável se for finito ou se puder ser posto em bijecção com o conjunto
dos inteiros N.
7
Capı́tulo 1
Demonstração. Seja por definição para cada m ∈ N∗
am = (δ1m , δ2m , . . . , δnm , . . . ) .
É claro que am ∈ B e verifica-se imediatamente que para m, p ∈ N∗ se m 6= p então
am 6= ap . Como o conjunto {am : m ∈ N∗ } é infinito e está contido em B temos que B é
infinito. Para demonstar que B é não numerável vamos supor o contrário e deduzir uma
contradição usando o processo diagonal de Cantor. Suponhamos que B é numerável e
consideremos (b(m) )m∈N∗ uma enumeração de B Note-se que para cada m ∈ N∗ se tem
(m)
que b(m) = (bn )n∈N∗ . Seja agora por definição c = (cn )n∈N∗ definido para n ∈ N∗ por:
cn := 1 − b(n)
n .
(1.4)
Verifica-se imediatamente que, por construção c ∈ B. Pela definição da fórmula 1.4,
para todo o m ∈ N∗ tem-se que c 6= b(m) pelo que c ∈
/ B, o que é contraditório.
(m)
Exercı́cio 6. Disponha as sucessões (bn )n∈N∗ e c = (cn )n∈N∗ numa tabela e justifique a denominação,
processo diagonal de Cantor para o processo seguido na construção de c.
Exercı́cio 7. Mostre que a correspondência de ]0, 1] em Bndeg que a x associa o correspondente desenvolvimento binário não degenerado é uma aplicação bem definida que é bijectiva. Conclua que ]0, 1]
é infinito e não numerável.
Exercı́cio 8. Mostre que qualquer intervalo de R não vazio e não reduzido a um ponto é infinito e não
é numerável. Mostre que R é infinito e não é numerável.
1.2
O princı́pio de Borel
[. . . ] Nous [. . . ] porterons notre attention sur une fraction decimale de
10 chiffres après la virgule: a = 0, 3141592653. On peut se demander
quelle est la probabilité d’obtenir precisément ce nombre a en tirant au
sort successivement les divers chif- fres au moyen, par exemple, de
roues analogues à celles que l’on emploie pour le tirage au sort des
numéros gagnants d’une loterie.
In E. Borel, Probabilité et certitude, p. 65.
8
A Teoria da Probabilidade, segundo Borel, para sucessões de Bernoulli
1.2.1
Breve apresentação do modelo de Kolmogorov
Supomos que o leitor está familiarizado com as ideias e as técnicas elementares da teoria
das probabilidades. Estas ideias foram apresentadas a partir de modelos para fenómenos
simples que convém recordar agora. Por exemplo, foram estudados o modelo de Bernoulli
para o lançamento de uma moeda ao ar, o modelo binomial para o lançamento de um
número finito de moedas idênticas ao ar, o modelo de Pascal, para o lançamento de de
um dado, ou de um número finito de dados idênticos ao ar, etc. Estes modelos, para
os fenómenos que admitem a descrição simples em linguagem natural que acabámos
de fazer, foram estudados identificando-se, em cada caso ou exemplo, um ou vários
acontecimentos a que se procurava atribuı́r uma probabilidade a partir de hipótese de
regularidade e/ou simetria postuladas para o modelo.
Exemplos simples e o teorema de Stone
Tomemos por exemplo o modelo, a que chamaremos modelo binomial, para o lançamento
de um certo número n ∈ N∗ de moedas idênticas e equilibradas ao ar. Pode considerar-se
o mesmo modelo para n lançamentos independentes de uma moeda equilibrada ao ar.
O lançamento de uma moeda tem dois resultados possı́veis: cara e coroa. Se convencionarmos que o resultado cara é representado pelo algarismo um e o resultado coroa é
representado pelo algarismo zero então para representar um lançamento de um número
n de moedas idênticas e equilibradas ao ar podemos usar um n-uplo de {0, 1}n . Um
lançamento de um número n de moedas idênticas e equilibradas ao ar é encarado como
uma realização de um fenómeno e o conjunto
{0, 1}n = {0, 1} × · · · × {0, 1} = {(α1 , . . . , αn ) : ∀i ∈ {0, . . . , n} αi ∈ {0, 1}} ,
como a totalidade das realizações do fenómeno em estudo. Foram estudados problemas
tais como atribuir uma probabilidade a um acontecimento definido como subconjunto
de realizações do fenómeno. Por exemplo, considere-se o seguinte problema. Determine
a probabilidade do seguinte acontecimento a que chamará A: no lançamento só há uma
moeda com caras, todas as outras são coroas. Partindo do pressuposto que as moedas
são equilibradas tem-se que:
∀(α1 , . . . , αn ) ∈ {0, 1}n P [{(α1 , . . . , αn )}] =
1
.
2n
Como se tem:
A = {(1, 0, . . . , 0), (0, 1, . . . , 0), . . . (0, 0, . . . , 1)} ,
virá que P[A] = n/2n . O modelo para o estudo probabilı́stico de um dado fenómeno tem
assim como componentes:
• O conjunto das realizações do fenómeno, denotado usualmente por Ω.
• Os acontecimentos definidos como subconjuntos de realizações. Sobre os acontecimentos é possı́vel efectuar operações lógicas como a conjunção de dois ou mais
9
Capı́tulo 1
acontecimentos, a disjunção de dois ou mais acontecimentos, é possı́vel considerar
o acontecimento contrário de um dado acontecimento, o acontecimento certo (Ω)
e, o seu contrário, o acontecimento impossı́vel (∅).
• A probabilidade de um dado acontecimento que, geralmente, é determinada a
partir de hipóteses de simetria impostas ao modelo (como por exemplo supor que
as moedas são equilibradas) e que corresponde a uma quantificação da confiança
que temos na realização desse acontecimento.
Note-se que nem sempre todas estas três componentes aparecem perfeitamente identificadas num modelo probabilı́stico de um dado fenómeno. No modelo do lançamento
de um número n de moedas idênticas e equilibradas ao ar convencionámos que uma
realização seria o resultado do lançamento apenas porque essa convenção permite obter
respostas a algumas das questões que é possı́vel colocar sobre o fenómeno. Um outro
modelo poderia exigir a especificação de todas as condições fı́sicas que estão subjacentes
a um dado lançamento e, se tal fosse feito daria origem a um modelo, certamente, muito
complicado e difı́cil de tratar. Ainda que a escolha do conjunto das realizações fosse
outra é sempre possı́vel considerar acontecimentos do tipo: só há uma moeda com caras,
todas as outras são coroas e por isso o conjunto das realizações não é essencial no modelo.
O essencial, num dado modelo, é que seja dada a famı́lia A dos acontecimentos dotada
das operações boleanas:
• Disjunção de acontecimentos: dados dois acontecimentos A e B, o acontecimento
A + B realiza-se quando se realiza A ou quando se realiza B.
• Conjunção de acontecimentos: dados dois acontecimentos A e B, o acontecimento
A × B realiza-se quando se realizam A e B.
• Negação de um acontecimento: dado um acontecimentos A o acontecimento ¬A
realiza-se quando se não se realiza A.
• Existe um acontecimento certo 1 e o seu contrário, o acontecimento impossı́vel 0.
Exercı́cio 9. Mostre que (A, +, ×) é uma álgebra de Boole, isto é, mostre que:
1. ∀A, B, C ∈ A A + (B × C) = (A + B) × (A + C).
2. ∀A, B, C ∈ A A × (B + C) = (A × B) + (A × C).
3. ∀A ∈ A A + 0 = A , A × 0 = 0 , A + 1 = 1 , A × 1 = A.
4. ∀A ∈ A A + ¬A = 1 , A × ¬A = 0 , ¬(¬A) = A.
Com efeito, um resultado notável - o teorema de Stone - diz-nos que sempre que
nos seja dada (A, +, ×) uma álgebra de Boole, representando para nós a famı́lia de
acontecimentos de um dado modelo probabilı́stico, existe um conjunto Ω e (B, ∪, ∩),
uma álgebra de Boole de subconjuntos de Ω, tais que (A, +, ×) é isomorfa a (B, ∪, ∩).
Fica assim sempre resolvido o problema da existência ou da determinação do conjunto
10
A Teoria da Probabilidade, segundo Borel, para sucessões de Bernoulli
das realizações para um dado modelo desde que estejam determinados os acontecimentos
do modelo. A demonstração deste teorema pode ser consultada nas obras [6][p. 9] e [5][p.
174] e sendo técnicamente acessı́vel faz apelo a noções topológicas não triviais.
Um exemplo fundamental: as sucessões de Bernoulli
Consideremos a generalização do exemplo estudado anteriormente que corresponde ao
modelo para o lançamento de uma infinidade numerável de moedas independentes e
equilibrada ao ar. Podemos considerar como espaço das realizações:
∗
Ω := {0, 1}N = {(α1 , α2 , . . . , αn , . . . ) : ∀n ∈ N∗ αn ∈ {0, 1}} .
Podemos considerar o acontecimento A definido, tal como antes: saiu apenas uma cara
e todas as outras moedas saı́ram coroas. Podemos representar o acontecimento como
reunião das realizações que o compõem, isto é, usando a notação relativa ao delta de
Kronecker introduzida com a fórmula 1.3:
A=
+∞
[
{(δ1n , δ2n , . . . , δnn , . . . )} .
n=1
Observemos que nesta representação temos que um dado acontecimento é representado
por uma união de uma infinidade numerável de acontecimentos elementares, cada um
dos quais corresponde a uma única realização. Este exemplo mostra que o conceito de
álgebra de acontecimentos é insuficiente para o estudo de modelos envolvendo conjuntos
infinitos de realizações. O conceito útil, nestes casos é o de álgebra-σ que passamos a
detalhar seguidamente.
O modelo de Kolmogorov
O que acabámos de expor torna natural o modelo de Kolmogorov para um fenómeno
probabilı́stico. Assim neste modelo considera-se um trio (Ω, B, P) em que:
• Ω é o conjunto das realizações;
• B é uma álgebra σ de subconjuntos de Ω isto é B ⊂ P(Ω) e verifica:
1. Ω ∈ B.
2. ∀A ∈ B Ω − A ∈ B.
3. ∀A1 , A2 , · · · ∈ B ∪n∈N∗ An ∈ B.
• P é uma função aditiva σ e normalizada de conjuntos, denominada probabilidade,
isto é uma aplicação de B em [0, 1] que verifica
1. P[Ω] = 1.
2. ∀A1 , A2 , · · · ∈ B dois a dois disjuntos,Pisto é tais que para m, n distintos,
Am ∩ An = ∅ se verifica P[∪n∈N∗ An ] = n∈N∗ P[An ].
11
Capı́tulo 1
Os estudos que vamos prosseguir serão feitos usando como quadro conceptual subjacente o modelo de Kolmogorov.
Exercı́cio 10. Mostre que no modelo de Kolmogorov se verifica que:
1. ∀A, B ∈ B , A ∪ B ∈ B.
2. ∀A, B ∈ B , A ∩ B = ∅ ⇒ P[A ∪ B] = P[A] + P[B].
3. P[∅] = 0.
1.2.2
O princı́pio de Borel
Na secção 1.1 partimos de um dado ponto do intervalo unitário da recta real e elaborámos um processo para o localizar, definindo o desenvolvimento binário associado a
esse número. Este desenvolvimento binário, que varia com o ponto do intervalo unitário
considerado, é dado por uma sucessão de variáveis - funções do ponto considerado - que
tomam valores num conjunto de dois elementos, zero e um.
Nesta secção vamos inverter o ponto de partida considerando como variável inicial
não o ponto do intervalo unitário mas as sucessões de variáveis que tomam valores em
{0, 1}. Poderemos assim introduzir o princı́pio de Borel que estipula a equivalência
entre os dois processos seguintes: tirar ao acaso um número do intervalo unitário e
efectuar uma infinidade de lançamentos independentes, ao ar, de uma moeda equilibrada.
A formulação matemática rigorosa do princı́pio de Borel poderá ser estudada com a
profundidade adequada no final destas lições.
Considere-se α uma variável aleatória com a lei de Bernoulli de parâmetro 1/2. Quer
isto dizer que, por exemplo:
P[α = 1] =
1
= 1 − P[α = 0] .
2
Esta variável aleatória pode representar, por exemplo, o modelo de um lançamento ao ar
de uma moeda equilibrada. Com efeito, o resultado de um tal lançamento tem duas eventualidades que, usualmente, designamos por cara ou coroa e que, no modelo da variável
de Bernoulli, representaremos por 1 e 0 respectivamente. Por outro lado a moeda sendo
equilibrada aponta para que se devam considerar iguais as probabilidades de cada uma
das eventualidades e, daı́, o parâmetro 1/2 no modelo. Considere-se agora uma sucessão
(αn )n∈N∗ de variáveis aleatórias independentes e identicamente distribuı́das com α, isto
é cada uma delas sendo uma variável aleatória com a lei de Bernoulli de parâmetro 1/2.
Esta sucessão é, naturalmente, um modelo matemático aceitável, para uma infinidade
numerável de lançamentos independentes, ao ar, de uma moeda equilibrada. Defina-se
agora a variável aleatória X por:
X=
+∞
X
αn
n=1
12
2n
.
A Teoria da Probabilidade, segundo Borel, para sucessões de Bernoulli
Dado que αn ∈ {0, 1} a série no membro à direita, na definição de X acima, é sempre
convergente pelo que a definição de X faz sentido. A questão natural neste contexto é
a de determinar a distribuição ou a lei de X. Sabendo que, por construção, X toma
valores em [0,1], considere-se a ∈]0, 1] e procuremos determinar:
FX (a) := P[X ≤ a] = P
" +∞
X αn
n=1
2n
#
≤a
.
Dado que todas as variáveis αn tomam os valores zero e um e fazem-no P
independenten
mente uma das outras é natural pensar que o resultado da soma da série +∞
n=1 αn /2 é
um qualquer número do intervalo [0, 1]. Daı́ que seja natural, pelo princı́pio usual das
probabilidades elementares, afirmar que:
P[X ≤ a] =
quantidade de pontos em [0, a]
.
quantidade de pontos em [0, 1]
Admitindo1 apenas que uma forma de levantar a indeterminação que decorre do facto
de se ter para qualquer intervalo [0, a] que a ”quantidade de pontos em [0, a]”é infinita,
consiste em substituir ”quantidade de pontos em [0, a]”por ”comprimento de [0, a]”,
teremos:
comprimento de [0, a]
=a.
(1.5)
P[X ≤ a] =
comprimento de [0, 1]
Fazendo fé nesta apresentação, muito intuitiva, podemos concluir que X tem a lei uniforme sobre [0, 1]. O que acabámos de ver justifica admitirmos como válido o seguinte
princı́pio.
Princı́pio 1 (de Borel; primeira formulação). O modelo probabilı́stico correspondente
a tirar um número ao acaso no intervalo [0, 1] é equivalente ao modelo correspondente
a efectuar, uma infinidade numerável de vezes, lançamentos, independentes, de uma
moeda equilibrada ao ar 2 .
Observação 3. O Princı́pio de Borel é de facto um teorema. Pode mostrar-se que X tem
distribuição uniforme em [0, 1] se e só se (αn (x))n∈N∗ , a sucessão dada pelo desemvolvimento binário não degenerado de X, é uma sucessão de Bernoulli, isto é, é uma sucessão
de variáveis aleatórias independentes e identicamente distribuı́das com a lei de Bernoulli
de parâmetro 1/2 (veja-se [4, p. 56]).
1
Esta admissão é, nesta altura, uma hipótese arbitrária, mas muito rica em consequências, que poderá
vir a ser justificada rigorosamente com o auxı́lio do processo de construção dos espaços de medida
produto.
2
Como veremos mais adiante neste curso, este princı́pio é de facto um teorema da teoria da medida
e do integral. Com efeito, Steinhauss demonstrou que o espaço de medida ([0, 1], L([0, 1]), λ) é isomorfo,
no sentido da teoria da medida (existência de uma bijecção bi-mensurável) ao espaço produto {0, 1}N .
13
Capı́tulo 1
A medida de Lebesgue
No estudo heurı́stico que fizémos acima para a determinação da lei da variável X fomos
levados a utilizar o comprimento de um subintervalo de [0, 1], como o valor da probabilidade de uma variável uniformemente distribuı́da em [0, 1] tomar os seus valores
nesse intervalo (veja-se a fórmula 1.5). Para utilizarmos o modelo de Kolmogorov é necessário seguir esta associação um pouco mais longe. Por analogia com as propriedades
enunciadas na subsecção 1.2.1 para a função aditiva σ de probabilidade no modelo de
Kolmogorov supomos que, denotando por λ a função definida sobre os subintervalos de
[0, 1] tal que,
∀a, b ∈ [0, 1] , a ≤ b , λ([a, b]) = λ(]a, b]) = λ(]a, b[) = λ([a, b[) = b − a ,
se tem que λ, como aplicação, se pode estender aos subconjuntos de [0, 1] que se podem
representar como união de subintervalos de [0, 1] de tal forma que, por exemplo:
∀a, b, c, d ∈ [0, 1] , a ≤ b < c ≤ d , λ([a, b] ∪ [c, d]) = λ([a, b]) + λ([c, d]) ,
(1.6)
verificando-se uma propriedade semelhante para qualquer outro tipo de intervalos. Estudaremos adiante a construção rigorosa de uma aplicação definida numa famı́lia de
subconjuntos de [0, 1] que verifica a propriedade 1.6 e a que chamaremos a medida de
Lebesgue. De momento, admitiremos que uma tal aplicação existe.
Princı́pio 2 (de Borel; segunda formulação). Se E for um contecimento no modelo correspondente a efectuar, uma infinidade numerável de vezes, lançamentos, independentes,
de uma moeda equilibrada ao ar e Ẽ for o acontecimento correspondente no modelo probabilı́stico correspondente a tirar um número ao acaso no intervalo [0, 1] então:
P[E] = λ(Ẽ) .
1.2.3
Aplicações do princı́pio de Borel
Caras no primeiro lançamento
Consideremos, no modelo do lançamento de uma infinidade de moedas ao ar, o acontecimento, E dado por sai caras no primeiro lançamento. Tem-se então que:
n
o
∗
E := (ωn )n∈N∗ ∈ {0, 1}N : ω1 = 1 .
O acontecimento correspondente, no modelo da tiragem de um número ao acaso no
intervalo [0, 1], é dado por:
(
)
+∞
X
1
α
1
n
e := x ∈ [0, 1] : x = +
E
, αn ∈ {0, 1} =] , 1] .
2
2n
2
n=2
Pelo princı́pio de Borel tem-se que:
e = λ(] 1 , 1]) = 1 .
P[E] = λ(E)
2
2
14
A Teoria da Probabilidade, segundo Borel, para sucessões de Bernoulli
Exercı́cio 11. Considere, para N ∈ N∗ , o espaço de probabilidade Ω = {0, 1}N munido da famıılia de
acontecimentos P(Ω), que é a famı́lia de partes de Ω, e da probabilidade P definida sobre os acontecimentos por:
#A
#A
∀A ∈ P(Ω) P[A] =
= N .
#Ω
2
1. Mostre que se A = {(α1 , . . . , αn ) ∈ Ω : α1 = 1} ∈ P(Ω), então, P[A] = 1/2.
2. Compare o resultado obtido na alı́nea anterior com o estudo que fez neste primeiro exemplo de
aplicação do princı́pio de Borel.
Os primeiros N lançamentos formam uma sucessão prescrita
Suponha que fixa um inteiro N ∈ N∗ e que fixa uma sequência, com comprimento N
de zeros e uns, denominada (a1 , . . . , an ). No modelo do lançamento de uma infinidade
de moedas ao ar, considere o acontecimento, E, os primeiros N lançamentos coincidem
com a sequência (a1 , . . . , an ). Tem-se então que:
n
o
N∗
∗
E := (ωn )n∈N ∈ {0, 1} : ω1 = a1 , · · · , ωn = an .
Da mesma forma que utilizámos, no exemplo anterior, o acontecimento correspondente
no modelo da tiragem de um número ao acaso no intervalo [0, 1] é dado por:
(
)
+∞
X
a
a
a
α
1
2
n
N
e := x ∈ [0, 1] : x =
E
+ 2 + ··· + N +
, αn ∈ {0, 1} .
2
2
2
2n
n=N +1
Considere-se o ponto s do intervalo ]0, 1] definido por:
s :=
N
X
ak
k=1
2k
,
e Observando agora
isto é a parte prescrita da representação binária
ponto x ∈ E.
P+∞ de um
0
αn
que a parte não prescrita, dada por s := n=N +1 2n , pode tomar valores no intervalo
0
]0, 1/2N ], pois αn = 0 ou αn = 1, e que x = s + s tem-se:
e =]s, s + 1 ] .
E
2N
O princı́pio de Borel diz-nos agora que:
e = λ(]s, s + 1 ]) = 1 .
P[E] = λ(E)
2N
2N
Exercı́cio 12. Considere o modelo introduzido no exercı́cio 11 com as correspondentes notações.
1. Mostre que se a1 , . . . , aN ∈ {0, 1} e A = {(a1 , . . . , an )} ∈ P(Ω), então, P[A] = 1/2N .
2. Compare o resultado obtido na alı́nea anterior com o estudo que fez neste segundo exemplo de
aplicação do princı́pio de Borel.
15
Capı́tulo 1
Caras na N -ésima jogada
Na descrição deste exemplo adoptamos um estilo mais sumário. Fixa-se um inteiro N
não nulo e, no modelo da tiragem de uma infinidade de moedas equilibradas ao ar,
consideramos o acontecimento definido por sai caras à N-ésima jogada. Tem-se que:
n
o
∗
E := (ωn )n∈N∗ ∈ {0, 1}N : ωN = 1 .
Do mesmo modo que nos exemplos anteriores tem-se que:
(
)
N
−1
+∞
X
X
α
1
α
n
n
e := x ∈ [0, 1] : x =
E
+ N +
, αn ∈ {0, 1} .
2n
2
2n
n=1
n=N +1
Se considerarmos, por definição:
s = s(α1 , . . . , αN −1 ) =
N
−1
X
n=1
αn
1
+ N ,
n
2
2
e Como (α1 , . . . , αN −1 ) é um elemento arbitrário de {0, 1}N −1
então ]s, s + 1/2N ] ⊂ E.
e como #{0, 1}N −1 = 2N −1 há exactamente 2N −1 escolhas distintas para s. Observe-se
0
0
0
agora que para duas dessas escolhas s e s os intervalos ]s, s + 1/2N ] e ]s , s + 1/2N ] são
disjuntos (porquê?), logo:
[
1
e=
E
]s(α1 , . . . , αN −1 ), s(α1 , . . . , αN −1 ) + N ]
2
N −1
(α1 ,...,αN −1 )∈{0,1}
sendo a reunião disjunta, pelo que, por aplicação do princı́pio de Borel:
e = 2N −1 × λ(]s, s + 1 ]) = 1 .
P[E] = λ(E)
2N
2
Exercı́cio 13. Considere o modelo introduzido no exercı́cio 11 com as correspondentes notações.
1. Mostre que se A = {(α1 , . . . , αN ) ∈ Ω : αN = 1} ∈ P(Ω), então, P[A] = 1/2.
2. Compare o resultado obtido na alı́nea anterior com o estudo que fez neste terceiro exemplo de
aplicação do princı́pio de Borel.
Nos primeiros N lançamentos há exactamente k caras
Neste último exemplo de aplicação do princı́pio de Borel consideramos fixos um inteiro
não nulo N e um inteiro k ∈ {1, . . . , N } e, no modelo de uma infinidade de lançamentos
de uma moeda ao ar, o acontecimento dado por: nos primeiros N lançamentos há,
exactamente, k em que saiem caras. Então:
(
)
N
X
∗
N
E := (ωn )n∈N∗ ∈ {0, 1} :
ωn = k
n=1
16
A Teoria da Probabilidade, segundo Borel, para sucessões de Bernoulli
e, do mesmo modo que nos exemplos anteriores tem-se que:
(
)
+∞
N
X
X
α
n
e := x ∈ [0, 1] : x =
E
∧
αn = k , αn ∈ {0, 1} .
2n
n=1
n=1
Consideremos, tal como no exemplo anterior,
s = s(α1 , . . . , αN ) =
N
X
αn
n=1
2n
,
e Havendo C N formas de escolher (α1 , . . . , αN ) ∈
então, também, ]s, s + 1/2N ] ⊂ E.
k
{0, 1}N de tal modo que haja exactamente k elementos da sequência iguais a 1, tem-se
que:
e=
E
[
(α1 ,...,αN −1 )∈{0,1}N −1 ,
]s(α1 , . . . , αN −1 ), s(α1 , . . . , αN −1 ) +
PN
n=1
1
]
2N
αn =k
sendo a reunião disjunta, pelo que, por aplicação do princı́pio de Borel:
e = C N × λ(]s, s +
P[E] = λ(E)
k
1
1
]) = CkN × N .
2N
2
Exercı́cio 14. Considere o modelo introduzido no exercı́cio 11 com as correspondentes notações.
P
N
N
1. Mostre que se A = {(α1 , . . . , αN ) ∈ Ω : N
n=1 αn = k} ∈ P(Ω), então, P[A] = Ck × 1/2 .
2. Compare o resultado obtido na alı́nea anterior com o estudo que fez neste quarto exemplo de
aplicação do princı́pio de Borel.
Observação 4. Os exercı́cios, sugeridos no final de cada um destes exemplos de aplicação
do princı́pio de Borel, mostram que sempre que o acontecimento descrito envolve apenas
um número de jogadas limitado por um certo inteiro, a determinação da correspondente
probabilidade pode fazer-se com recurso a um modelo em que o espaço de probabilidade
é finito. Esse tipo de modelos foi estudado na disciplina introdutória de Probabilidades
e Estatı́stica.
O exemplo seguinte mostra que tal não é o caso se a definição do acontecimento fizer
intervir um número de jogadas não limitado.
Ruı́na no jogo de Bernoulli
Suponha que decide apostar no jogo que descrevemos a seguir. O jogador inicia o jogo
com uma fortuna de X unidades monetárias (u.m.) e considera uma infinidade numerável
de lançamentos ao ar de uma moeda equilibrada, ordenados pelos inteiros não negativos
e não nulos. Para cada tal inteiro observa o resultado do correspondente lançamento.
17
Capı́tulo 1
Se saiu cara o jogador ganha uma u.m. e a sua fortuna acresce de uma u.m. e, se saiu
coroa, o jogador perde uma u.m. e a sua fortuna decresce de uma u.m.
O jogador pode contabilizar a sua fortuna, após N lançamentos, definindo-a como a
soma da fortuna inicial com a totalidade dos ganhos e perdas obtidos nos primeiros N
lançamentos. O jogo termina num dado lançamento, com número de ordem N0 , assim
que a fortuna, após N0 lançamentos, for igual a zero; é, por definição, a ruı́na do jogador.
Observe-se que o conjunto de todas as realizações do jogo que conduzem à ruı́na é um
acontecimento bem definido.
Exercı́cio 15. Considere como modelo para o lançamento de uma infinidade numerável de moedas ao ar
∗
o espaço {0, 1}N com a convenção de que um resultado de caras, num dado lançamento, é representado
por 1 e um resultado de coroa é representado por 0.
1. Mostre que o acontecimento dado pela ruı́na do jogador se pode representar como a disjunção de
uma infinidade numerável de acontecimentos definidos por: ocorre a ruı́na do jogador à k-ésima
jogada, para k = 1, . . . ,.
2. Conclua que o acontecimento, ocorre a ruı́na do jogador, não pode ser definido por condições
relativas a um número de lançamentos limitado por um dado inteiro positivo não nulo.
Com o objectivo de formalizar convenientmente o acontecimento dado pela ruı́na do
jogador, vamos definir as funções de Rademacher3
1.2.4
As funções de Rademacher
As funções de Rademacher têm como domı́nio natural o intervalo ]0, 1]. Seja, pois,
x ∈]0, 1] e (αn (x))n∈N∗ o correspondente desenvolvimento binário não degenerado.
Definição 5. Para k ∈ N∗ , a k-ésima função de Rademacher, Rk , é dada por:
Rk (x) := 2αk (x) − 1 .
Observação 5. É imediato verificar que:
(
+1 se αk (x) = 1
Rk (x) :=
−1 se αk (x) = 0 .
(1.7)
(1.8)
Em consequência, se convencionarmos que, no jogo definido no exemplo 1.2.3, à saı́da de
caras - no lançamento - corresponde 1 e à saı́da de coroa corresponde 0, fica claro que
Rk (x) descreve o ganho, ou perda, obtido na jogada de ordem k da realização do jogo
(αn (x))n∈N∗ .
3
Hans Adolph Rademacher (1892–1969), matemático de origem alemã radicado nos E.U.A. após fuga
de Breslau, por motivo das suas posições pacifistas, no tempo do nazismo; contribuı́u, notalvelmente,
nas áreas da análise e da teoria analı́tica dos números (veja-se [3]).
18
A Teoria da Probabilidade, segundo Borel, para sucessões de Bernoulli
Exercı́cio 16. Mostre que no contexto do exemplo 1.2.3 e da observação anterior 5 se verificam as
proposições seguintes.
1. Para k ∈ N∗ , função Sk (x) definida, no intervalo ]0, 1] como a soma dos ganhos e perdas até à
jogada k inclusivé, na realização do jogo (αn (x))n∈N∗ é dada por:
Sk (x) :=
k
X
Rl (x) .
l=1
2. No modelo da tiragem ao acaso de um número do intervalo ]0, 1] o acontecimento E dado pela
ruı́na do jogador é dado por:
+∞
[
e=
ek ,
E
E
k=1
onde se tem
ek := {x ∈]0, 1] : ∀l ∈ {1, 2, . . . , k − 1} Sl (x) + X > 0 ∧ Sk (x) + X = 0} .
E
ek )k∈N∗ são dois a dois disjuntos.
3. Os acontecimentos (E
A determinação da probabilidade da ruı́na do jogador, P[E], pelo princı́pio de Borel levaria à determinação de
+∞
[
ek ) ,
λ(
E
k=1
Efectuaremos esta determinação quando estudarmos em detalhe a medida de Lebesgue.
As propriedades elementares da medida de Lebesgue, evocadas na secção 1.2.2 não se
e nem E
ek se podem representar, em geral, como uniões
podem utilizar uma vez que nem E
finitas de intervalos.
1.3
Leis dos Grandes Números
Considere-se o modelo para o fenómeno de uma infinidade numerável de lançamentos
independentes, ao ar, de uma moeda equilibrada em que se representa o resultado caras
por 1 e o resultado coroa por 0. O espaço das realizações do fenómeno é, naturalmente
∗
Ω = {0, 1}N e uma realização ω ∈ Ω do fenómeno será representada por uma dada
sucessão de zeros e uns, ω = (αn )n∈N∗ . Considere-se para cada n ∈ N∗ :
sn (ω) := α1 + · · · + αn
(1.9)
Uns momentos de reflexão permitirão ao leitor convencer-se que a quantidade sn (ω) representa o número de vezes em que o lançamento resultou em caras nos primeiros n
lançamentos da realização ω. Na hipótese da moeda ser equilibrada, o resultado caras
(1) e o resultado coroa (0) são igualmente prováveis em cada lançamento. Em consequência, para ω - realização tı́pica do fenómeno - sn (ω)/n, que representa a frequência
da ocorrência de uns nos primeiros n lançamentos, deverá estar próxima de 1/2, desde
que n seja suficientemente grande.
19
Capı́tulo 1
Com auxı́lio de software adequado4 determinámos uma sequência a := (an )n∈N∗ que
simula uma sequência de 10000 zeros e uns independentes e tal que cada termo da
sequência valha zero ou um, com igual probabilidade.
P Sabemos nque a uma tal sequência
se associa, de modo único, um número real x(a) = 10000
n=1 an /2 . No caso da simulação
efectuada, obtivémos, como primeira aproximação, x(a) = 0.738724. Seguidamente,
apresentamos uma representação gráfica da sequência
n
sn (a)
1X
=
ai , n = 1, . . . , 10000 .
n
n
i=1
0.52
0.51
2000
4000
6000
8000
10000
0.49
0.48
0.47
0.46
Figura 1.1: Uma ilustração da lei forte dos grandes números
Observação 6. A sequência (sn (a)/n)n=1,...,10000 manifesta, claramente, uma tendência
assimptótica para o valor 1/2 não deixando de apresentar, também, oscilações notáveis
em torno deste valor.
0
Insistimos no facto desta figura depender da sequência a. Uma sequência a , distinta
de a daria lugar, em geral, a uma figura distinta da apresentada. No entanto, a propósito
dessa outra figura poder-se-ia repetir, ipsis verbis, o que afirmámos no parágrafo anterior,
0
substituindo(sn (a)/n)n=1,...,10000 por (sn (a )/n)n=1,...,10000 .
As leis dos grandes números, no contexto do modelo em estudo - as sucessões de
Bernoulli - são formulações matemáticas que descrevem o comportamento assimptótico
da sucessão (sn (ω)/n)n∈N∗ , para realizações ω ∈ Ω tı́picas do fenómeno. No seguimento
dos nossos estudos esclareceremos o sentido a dar ao termo tı́pico neste contexto.
1.3.1
Lei Fraca dos Grandes Números
Na primeira formulação do comportamento assimptótico da sucessão (sn )n∈N∗ , que vamos expor a seguir, será posto em evidência que, para qualquer > 0, a probabilidade
4
Usámos o programa MathematicaTM. Em apêndice pode consultar o Notebook que produziu os
resultados apresentados a seguir.
20
A Teoria da Probabilidade, segundo Borel, para sucessões de Bernoulli
do acontecimento composto pelas realizações do fenómeno em que a distancia de sn /n a
1/2 excede , tende para zero quando n tende para mais infinito.
Para formular a lei fraca dos grandes números consideramos o modelo da tiragem de
um número ao acaso no intervalo ]0, 1]. Ao abrigo do princı́pio de Borel sabemos que
este é um modelo equivalente ao modelo inicial. Seja, então, x ∈]0, 1] e (αn (x))n∈N∗ o
correspondente desenvolvimento binário não degenerado. Seja por definição:
∀n ∈ N∗ sn (x) = α1 (x) + α2 (x) + · · · + αn (x) .
Teorema 1 (Lei fraca dos grandes números).
sn (x) 1
∀ > 0
lim λ
x ∈]0, 1] :|
− |> =0.
n→+∞
n
2
(1.10)
Antes de demonstrarmos este resultado importante devemos consagrar algum tempo
a estudar algumas das possı́veis interpretações do enunciado. De forma intuitiva, a ideia
que este resultado encerra consiste em fixar um minorante para a distancia entre sn (x)/n
e 1/2, que denominámos , considerando-se em seguida o conjunto das realizações x em
que sn (x)/n dista de 1/2 mais do que . À medida que n cresce para infinito, a medida
de Lebesgue desse conjunto tende para zero. Dito de outra forma, considere-se > 0 e
n0 fixos. Dado que:
|
sn0 (x) 1
− |> n0
2
⇔
(
sn0 (x)
1
sn (x)
1
< − ) ∧ ( 0
> + ) ,
n0
2
n0
2
pode interpretar-se a lei fraca dos grandes números, pelo princı́pio de Borel, dizendo que
a probabilidade do conjunto das realizações x ∈]0, 1] em que sn0 (x)/n0 dista de 1/2 mais
do que é arbitrariamente pequena desde que n0 seja suficientemente grande.
O exercı́cio seguinte de simulação computacional tem como objectivo possibilitar ao
leitor uma familiarização com a variedade de aspectos que pode assumir a sucessão cujo
comportamento assimptótico se estuda.
Exercı́cio 17 (Facultativo). Escolha software (uma folha de cálculo ou linguagem de programação adequada) que permita gerar com facilidade números pseudo-aleatórios5 . Fixe = 0.015.
1. Programe a obtenção de um número pseudo-aleatório que simule uma variável aleatória X tal que:
P[X = 1] =
1
= P[X = 0] .
2
2. Programe uma lista L que simule
P(X1 , . . . , XN ), ou seja, N observações independentes da variável
aleatória X e determine sN = N
i=1 Xi . O valor de N deverá ser fixado inicialmente em 250 e
depois aumentar até aos 10000 em passos de 250.
5
Os números pseudo-aleatórios são gerados em computador a partir de programas baseados em congruências ou outros processos simples e não são, por isso, verdadeiramente aleatórios. No entanto,
têm propriedades estatı́sticas que os tornam adequados para representar números, realmente, aleatórios.
Veja-se para um primeiro contacto com a noção de número pseudo aleatório [10][p. 738] ou [8][p. ] e
para um estudo mais aprofundado [7][p. 161].
21
Capı́tulo 1
3. Crie uma lista C(N ) à qual junta 1 (um) se | sN /N − 1/2 |> e 0 (zero) no caso contrário.
4. Repita a alı́nea 2 e a seguinte M vezes e determine #{i ∈ C(N ) : i = 1}/M . O valor de M
deverá ser fixado em função do equipamento e da linguagem que utilizar e poderá ser igual a 300.
5. Represente graficamente #{i ∈ C(N ) : i = 1}/M em função de N , segundo as indicações dadas
na alı́nea 2 e comente o resultado obtido.
A figura seguinte representa o resultado obtido numa resolução deste exercı́cio6 . Se
assumirmos que #{i ∈ C(N ) : i = 1}/M representa uma estimativa para a probabilidade
do acontecimento relevante no enunciado da lei fraca dos grandes números, constata-se
na figura que esta probabilidade tende para zero quando N cresce.
0.35
0.3
0.25
0.2
0.15
0.1
0.05
2000
4000
6000
8000
10000
Figura 1.2: Uma ilustração da lei fraca dos grandes números
Demonstração. (Lei fraca dos Grandes Números) A demonstração repousa sobre uma
desigualdade importante, denominada desiguladade de Tchebychev, que o leitor provavelmente já estudou na disciplina introdutória de Probabilidades e Estatı́stica.
No primeiro passo da demonstração consideramos uma transformação elementar do
conjunto cuja probabilidade se vai estimar. Para tal, utilizam-se as funções de Rademacher. Assim, tem-se que:
(
)
n
sn (x) 1
1
1X
x ∈]0, 1] :|
− |> =(a) x ∈]0, 1] :|
αi (x) − |> =(b)
n
2
n
2
i=1
(
)
n
X
(1.11)
= x ∈]0, 1] :|
(2αi (x) − 1) |> 2n =(c)
i=1
= x ∈]0, 1] :| Sn (x) |2 > (2n)2
,
em que as igualdades indicadas se justificam pelas razões seguintes.
6
Veja-se em apêndice o conjunto de instruções no programa Mathematica TM que permitiu obter
esta figura.
22
A Teoria da Probabilidade, segundo Borel, para sucessões de Bernoulli
(a) Por definição: sn (x) =
Pn
i=1 αi (x);
(b) Cálculo algébrico simples;
P
(c) Pela definição Sn (x) = ni=1 Ri (x) e porque se pode elevar, ao quadrado, ambos
os membros, não negativos, de uma desigualdade sem introduzir novas soluções.
No segundo passo vamos estudar uma versão da desigualdade de Tchebytchev, para
as funções em escada, e uma aplicação desta desigualdade à estimativa em estudo. As
funções em escada são as funções constantes por intervalos.
Definição 6. Uma função f :]0, 1] −→ R é em escada no intervalo ]0, 1] sse:
∃ n ∈ N∗ ; 0 = c0 < c1 < · · · < cn−1 < cn = 1 ; α1 , . . . , α1 ∈ R :
∀i = 0, . . . , n − 1 ∀x ∈]ci , ci+1 ] f (x) = αi+1 .
(1.12)
De acordo com esta definição, uma função definida no intervalo ]0, 1] é em escada7
se este intervalo se pode decompor num partição de um número finito de subintervalos,
fechados à direita e abertos à esquerda, de tal forma que f seja constante sobre cada um
dos subintervalos.
Exercı́cio 18. Sejam f e g funções em escada no intervalo ]0, 1] e γ um número real.
1. Mostre que γ × f , f + g e f × g são ainda funções em escada no intervalo ]0, 1].
P
2. Mostre que Sn (x) = n
i=1 Ri (x) é uma função em escada no intervalo ]0, 1].
3. Mostre que | Sn (x) |2 é uma função em escada no intervalo ]0, 1].
Proposição 5 (Desigualdade de Tchebytchev). Seja f uma função em escada
no intervalo ]0, 1] tomando valores não negativos. Então, para qualquer > 0:
1
λ ({x ∈]0, 1] : f (x) > }) <
Z
1
f (x)dx .
0
Demonstração. (Desigualdade de Tchebytchev) Observe-se que uma função constante
por intervalos com domı́nio limitado, é necessáriamente integrável à Riemann. Seja f
uma função em escada no intervalo ]0, 1] tomando valores não negativos. Com as notações
da definição 1.12 o integral de Riemann da função f pode facilmente ser calculado como
7
Esta definição de função em escada particulariza a noção de função constante por intervalos pelo
que poderemos vir a encontrar, noutros contextos, outras definições, mais gerais, de função em escada.
23
Capı́tulo 1
o resultado da soma, finita, das áreas dos rectangulos ]ci , ci+1 ] × [0, αi+1 ]. Assim tem-se
que:
Z 1
n−1
X
f (x)dx =
(ci+1 − ci ) × αi+1 .
0
i=0
Seja > 0 arbitrário. Dado que a função f toma valores não negativos, os números αi
para i = 1, . . . , n são não negativos pelo que o valor da soma à direita só pode decrescer
se lhe retirarmos os termos em que αi ≤ . Logo:
n−1
X
X
i=0
i∈{0,...,n−1},αi+1 >
(ci+1 −ci )×αi+1 ≥
X
(ci+1 −ci )×αi+1 > ×
(ci+1 −ci ) .
i∈{0,...,n−1},αi+1 >
Observe-se que a soma mais à direita representa a medida de Lebesgue do conjunto
dos pontos x ∈]0, 1] tais que f (x) > . Com efeito, este subconjunto é formado pelos
intervalos ]ci , ci+1 ] para os quais αi+1 > . Em consequência:
Z
1
X
f (x)dx > ×
0
(ci+1 − ci ) = × λ ({x ∈]0, 1] : f (x) > }) ,
i∈{0,...,n−1},αi+1 >
tal como querı́amos demonstrar.
Exercı́cio 19. Represente graficamente, em pelo menos duas figuras distintas, uma ilustração da demonstração da desigualdade de Tchebytchev.
Observação 7. Em consequência da desigualdade de Tchebytchev temos que:
sn (x) 1
λ
x ∈]0, 1] :|
− |> = λ {x ∈]0, 1] :| Sn (x) |2 > (2n)2 } ≤
n
2
Z 1
1
≤ 2 2
| Sn (x) |2 dx
4n 0
(1.13)
O terceiro passo e último passo da demonstração da lei fraca dos grandes números
consiste em estimar o integral que figura no termo mais à direita na cadeia de desigualdades 1.13 e que envolve as funções de Rademacher. Observe-se que:

! n
n
X
X
X
| Sn (x) |2 =
Ri (x) 
Rj (x) =
Ri (x)Rj (x) =
i=1
=
n
X
i=1
24
Ri2 (x) +
j=1
X
i,j∈{1,...,n},i6=j
i,j∈{1,...,n}
Ri (x)Rj (x) .
A Teoria da Probabilidade, segundo Borel, para sucessões de Bernoulli
Em consequência tem-se que:
Z
1
2
| Sn (x) | dx =
0
n Z
X
i=1
1
Ri2 (x)dx
0
X
+
i,j∈{1,...,n},i6=j
Z
1
Ri (x)Rj (x)dx =
0
=n,
uma
j, qualquer integral da forma
R 1 vez que pelo exercı́cio 26 se tem que, para iR 6=
1 2
R
(x)R
(x)dx
é
nulo
e
para
qualquer
i
o
integral
R
i
j
i (x)dx vale 1.
0
0
Para concluir a demonstração basta observar que pela estimativa obtida em 1.13 e
pelo cálculo que acabámos de efectuar:
λ
Z 1
sn (x) 1
1
1
x ∈]0.1] :|
− |> < 2 2
| Sn (x) |2 dx =
n
2
4n 0
4n2
(1.14)
pelo que se segue a conclusão enunciada.
1.3.2
Lei Forte dos Grandes Números
O enunciado da lei forte dos grandes números aproxima-se mais da formulação aproximada e intuitiva que consta da observação 6 e da figura a que esta observação se
refere. Em contrapartida, a demonstração é mais elaborada. Em traços largos e com as
notações da introdução a esta secção - a secção das leis dos grandes números - (veja-se
∗
a página 19), o enunciado seguinte diz-nos que para ω ∈ {0, 1}N , uma realização tı́pica
do fenómeno, se tem que limn→+∞ sn (ω)/n = 1/2. Ou ainda, de outra forma, é nula a
probabilidade do conjunto das realizações do fenómeno em que limn→+∞ sn (ω)/n ou não
existe ou, existindo, difere de 1/2. Tal como aconteceu para a lei fraca, formulamos a lei
forte sob a interpretação que o princı́pio de Borel nos permite. Seja então x ∈]0,
Pn1], o correspondente desenvolvimento binário não degenerado (αn (x))n∈N∗ e sn (x) = i=1 αi (x).
Teorema 2 (Lei Forte dos Grandes Números).
sn (x)
1
λ
x ∈]0, 1] : lim
=
=1.
n→+∞
n
2
(1.15)
Comentário 1. Fica assim esclarecido o significado de realização tı́pica do fenómeno
no contexto das matérias que estamos a estudar. Com efeito, a lei forte dos grandes
números diz-nos que o conjunto das realizações do fenómeno x ∈]0, 1] em que se verifica
limn→+∞ sn (x)/n = 1/2 tem medida de Lebesgue 1 ou, no modelo equivalente - segundo
o princı́pio de Borel - tem probabilidade 1. Estas realizações podem, pois, considerarse tı́picas do fenómeno observado uma vez que é nula a probablidade de observar uma
realização do fenómeno em que o comportamento da sucessão (sn (x)/n)n∈N∗ seja distinto
do anunciado pela lei forte dos grandes números.
25
Capı́tulo 1
Observação 8. Uma vez que se tem a seguinte partição:
[
sn (x)
1
]0, 1] =
x ∈]0, 1] : lim
=
n→+∞
n
2
[
sn (x)
1
x ∈]0, 1] : lim
não existe ou difere de
n→+∞
n
2
e que λ(]0, 1]) = 1, é óbvio que o enunciado do teorema na fórmula 1.15, é equivalente
a:
sn (x)
1
=0.
(1.16)
λ
x ∈]0, 1] : lim
não existe ou difere de
n→+∞
n
2
Dado que este último conjunto não se pode em geral decompor na união finita de intervalos disjuntos, torna-se necessário esclarecer o que entendemos por conjunto de medida
de Lebesgue nula.
Os conjuntos de medida de Lebesgue nula
Seja I(]0, 1]) a famı́lia dos intervalos de R contidos em ]0, 1].
Definição 7. Seja A ⊂]0, 1]. A tem medida de Lebesgue nula, escrevendo-se
então λ(A) = 0, se e só se:
∀ > 0 ∃(In )n∈N ∈ I(]0, 1])
N
A⊂
+∞
[
n=0
In ∧
+∞
X
λ(In ) < .
(1.17)
n=0
Comentário 2. Segundo esta definição, um conjunto tem medida de Lebesgue nula se
e só se, para cada > 0 se pode encontrar uma cobertura do conjunto por intervalos
tal que a soma da série dos comprimentos desses intervalos seja inferior a . Afirma-se
assim que um conjunto tem medida de Lebesgue nula se for subconjunto de uma união
- eventualmente infinita numerável - de intervalos cuja soma da série dos comprimentos
é tão pequena quanto se queira.
Na proposição seguinte apresentam-se algumas propriedades e alguns exemplos notáveis
de conjuntos de medida de Lebesgue nula.
26
A Teoria da Probabilidade, segundo Borel, para sucessões de Bernoulli
Proposição 6. Sejam A e B subconjuntos de ]0, 1] e seja Q o conjuntos dos
números racionais.
1. A ⊂ B ∧ λ(B) = 0 ⇒ λ(A) = 0
2. ∀x ∈]0, 1] λ({x}) = 0
3. Seja (An )n∈N uma sucessão de subconjuntos de ]0, 1] com medida de Lebesgue nula. Então:
+∞
[
λ(
An ) = 0 .
n=0
4. λ(Q∩]0, 1]) = 0 e (R − λ(Q∩]0, 1])) = 1
Demonstração. Para demonstrar a propriedade 1, observe-se que como A ⊂ B, se se
tiver que
+∞
+∞
[
X
B⊂
In ∧
λ(In ) < ,
n=0
n=0
+∞
[
+∞
X
então também se verifica que:
A⊂
In ∧
n=0
λ(In ) < ,
n=0
pelo que se tem λ(A) = 0.
A propriedade 2 decorre do facto de se ter:
∀ > 0 ∀x ∈]0, 1] ∀n ∈ N{x} ⊂]x −
2n+3
,x +
2n+3
[.
Assim sendo tem-se que:
{x} ⊂
+∞
[
]x −
n=0
, x + n+3 [ e
n+3
2
2
+∞ X
λ ]x −
n=0
2
,x +
n+3
+∞
X
1
[
=
= <
n+3
n+2
2
2
2
n=0
A demonstração da propriedade 3 é um pouco mais elaborada mas alguns minutos de
reflexão tornam-na acessı́vel. Seja (Am )m∈N uma sucessão de subconjuntos de ]0, 1] com
(m)
medida de Lebesgue nula e seja > 0. Para cada m ∈ N seja (In )n∈N uma sucessão
de intervalos de ]0, 1] tais que, segundo a definição de conjuntos de medida de Lebesgue
nula, se tenha:
+∞
+∞
[
X
Am ⊂
In(m) e
λ(In(m) ) < m+2 .
2
n=0
n=0
(m)
Observe-se que (In )(n,m)∈N×N é uma famı́lia indexada por um conjunto infinito numerável N×N. Com efeito - veja o exercı́cio 20 - existe uma aplicação bijectiva b = (b1 , b2 )
27
Capı́tulo 1
de N em N × N. Seja agora por definição:
(b (n))
Jn := Ib2 1(n)
∈ I(]0, 1]) .
É imediato verificar que:
+∞
[
[
Am ⊂
m=0
In(m)
=
+∞
[
Jn .
n=0
(n,m)∈N×N
Por outro lado tem-se também que:
+∞
X
λ(Jn ) =
n=0
X
(n,m)∈N×N
λ(In(m) )
=(a)
+∞
X
+∞
X
m=0
n=0
!
λ(In(m) )
≤
+∞
X
m=0
1
= <.
2m+2
2
Note-se que a igualdade referida com o ı́ndice (a) decorre do facto de para as somas de
séries duplas a termos positivos ser indiferente a ordem pela qual se efectuam as somas
das duas séries simples (veja-se [9][p. 175 e 196]).
A demonstração da propriedade 4 recorre àos resultados obtidos nas propriedades 2
e 3 uma vez que Q sendo numerável Q∩]0, 1] também é, pelo que se pode escrever:
Q∩]0, 1] =
+∞
[
{rn } .
n=0
Como para qualquer n ∈ N se tem que λ({rn }) = 0 - pela propriedade 2 - conclui-se que
λ(Q∩]0, 1]) = 0, pela propriedade 3. A igualdade mais à direita no enunciado decorre
de se ter a partição:
]0, 1] = (Q∩]0, 1]) ∪ (R − Q∩]0, 1])
e λ(]0, 1]) = 1.
Observação 9. Com um raciocı́nio semelhante ao utilizado a demonstração da propriedade 4, pode mostrar-se que todo o subconjunto de ]0, 1] infinito e numerável tem medida
de Lebesgue nula.
Exercı́cio 20 (Facultativo). O objectivo deste exercı́cio é mostrar que existe uma bijecção entre N × N
e N, construindo-a.
1. Represente graficamente R := {0, 1, 2, 3, 4, 5} × {0, 1, 2, 3, 4, 5} no referencial ortonormado usual
do plano e o caminho, no plano, definido pela seguinte sequência de pontos desse reticulado: [1] =
(0, 0), [1] = (0, 0), [2] = (1, 0), [3] = (0, 1), [4] = (0, 2), [5] = (1, 1), [6] = (2, 0), [7] = (3, 0), [8] =
(2, 1), [9] = (1, 2), [10] = (0, 3), [11] = (0, 4), [12] = (1, 3), [13] = (2, 2), [14] = (3, 1),[14] = (4, 0),
etc. Observe que o caminho assim definido passa uma e uma só vez pelos pontos do reticulado
R. Assim sendo, o algoritmo para a construção do caminho é um bom ponto de partida para a
definir a bijecção procurada. Formule, em linguagem natural as regras de contrução da extensão
deste caminho a N × N.
28
A Teoria da Probabilidade, segundo Borel, para sucessões de Bernoulli
2. Para um par (n, m) ∈ N2 defina, P (n, m), o peso desse par, por P (n, m) := n + m. Represente
graficamente uma pirâmide com o vértice em [1] = (0, 0) e colocando na mesma linha os pares com
o mesmo peso, ordenando cada linha da esquerda para a direita sendo a ordem para os pontos,
em cada linha, a ordem indicada na construção do caminho referida na alı́nea 1.
3. Mostre que o número de ordem do elemento mais à esquerda em cada linha é:
(n + m)(n + m + 1)
+1
2
e verifique que os pares da linha de peso (n + m) se podem representar por:
(n + m) − (−1)n+m (n + m)
(n + m) + (−1)n+m (n + m)
+ (−1)n+m k,
− (−1)n+m k ,
2
2
para k = 0, 1, . . . , n + m.
4. Conclua que a aplicação N que ao par (0, 0) associa o inteiro N(0, 0) = 1 e ao par (n, m) associa
o inteiro:
N(n, m) =
n + (−1)n+m n
m − (−1)n+m m
(n + m)(n + m + 1)
+1+
+
,
2
2
2
é uma bijecção de N × N em N∗ . Conclui-se, imediatamente, que a aplicação M := N − 1 é a
bijecção pretendida.
Os números normais
Em abstracto, a lei forte dos grandes números afirma que o conjunto dos números não
normais tem medida de Lebesgue nula. Tomando em consideração as propriedades dos
conjuntos de medida de Lebesgue nula e a observação 9 ocorre, naturalmente, a seguinte
questão: será o conjunto dos números não normais
sn (x)
1
c
∗
N = x ∈]0, 1] : (
)n∈N não converge para
n
2
um conjunto númeravel?
De facto, tal não acontece. Este conjunto é infinito e não numerável. Esta constatação, que vamos justificar seguidamente, confirma-nos, de outro modo, que a lei forte
dos grandes números é um resultado não trivial.
Considere-se Φ a correspondência que a um número x ∈]0, 1], tendo um dsenvolvimento binário não degenerado (αn (x))n∈N∗ , ou seja, com as notações introduzidas na
observação 2,
x = .α1 (x)α2 (x)α3 (x) . . . αn (x) . . .
associa o número Φ(x) ∈]0, 1] tendo o desenvolvimento binário não degenerado, (αn (Φ(x)))n∈N∗ ,
dado por:
Φ(x) = .α1 (Φ(x))α2 (Φ(x))α3 (x) . . . αn (Φ(x)) · · · =
= .α1 (x)11α2 (x)11α3 (x)11 . . . 11αn (x)11 . . .
Uns segundos de reflexão convencerão o leitor que o desenvolvimento de Φ(x) é construı́do
a partir do desenvolvimento de x por um processo simples. Com efeito, na sucessão
29
Capı́tulo 1
dada por (αn (Φ(x)))n∈N∗ , os termos de ordem 1, 4, 7, . . . , 3n + 1, . . . para n = 0, 1, 2, . . .
correspondem os termos α1 (x), α2 (x), α3 (x), . . . , αn+1 (x), . . . ; para k 6= 3n + 1, em que
n = 0, 1, 2, . . . , tem-se que αk (Φ(x)) = 1. Em sı́ntese pode dizer-se que:
(
αn (x) se k = 3n + 1, n ∈ N
∀k ∈ N∗ αk (Φ(x)) =
(1.18)
1 no caso contrário.
Dado que, por construção, a sucessão (αn (Φ(x)))n∈N∗ é não degenerada, o número de
]0, 1]
+∞
X
αk (Φ(x))
Φ(x) :=
2k
k=1
fica determinado univocamente pelo que Φ é uma aplicação injectiva de ]0, 1] em ]0, 1].
Exercı́cio 21. Redija cuidadosamente a demonstração de que Φ é uma aplicação bijectiva de ]0, 1] em
Φ(]0, 1]).
Observe-se agora que, por construção, para n ∈ N∗ e qualquer x ∈]0, 1]:
s3n (Φ(x)) ≥ 2n ,
pelo que
2
sn (Φ(x))
s3n (Φ(x))
2n
≥ lim inf
≥ lim inf
=
n∈N∗
n∈N∗ 3n
n
3n
3
e, logo, Φ(x) ∈
/ N. Em consequência, tem-se que:
lim inf
∗
n∈N
Φ(]0, 1]) ⊂ Nc ⊂]0, 1] .
Uma vez que Φ é bijectiva, e ]0, 1] em Φ(]0, 1]), tem-se que:
#]0, 1] = #Φ(]0, 1]) ≤ #Nc ≤ #]0, 1] .
O teorema de Cantor–Schroeder–Bernstein permite-nos concluir que:
#Nc = #]0, 1] ,
isto é, que Nc tem a mesma cardinalidade que ]0, 1] sendo, por isso, infinito e não
numerv́el.
Observação 10. Os números normais que, de acordo com a lei forte dos grandes números,
são os números tı́picos do intervalo ]0, 1] têm propriedades muito interessantes8 . No
entanto, é muito difı́cil exibir um número normal. Sabe-se que o número n cujo desenvolvimento decimal é dado por
n := .1234567891011121314151617 . . .
é normal (veja-se [2][p. ]) mas por exemplo, ainda se ignora se π − 3 ou e − 2 são normais
ou não.
8
Uma dessas propriedades é que no desenvolvimento binário (ou decimal), dum número normal, se
podem encontrar todos os livros escritos até hoje ou que se venham a escrever um dia (veja-se [1][p.
68–75]).
30
A Teoria da Probabilidade, segundo Borel, para sucessões de Bernoulli
A demonstração da lei forte dos grandes números
A demonstração que vamos apresentar inicia-se de modo semelhante à da lei fraca mas
melhora-se a velocidade de convergência, para zero, da estimativa da medida de Lebesgue
do conjunto relevante.
Demonstração. Considerem-se as notações usadas na demonstração da lei fraca dos grandes números. Em resultado de mais uma aplicação da desigualdade de Tchebytchev,
tem-se que, para n ∈ N∗ :
sn (x) 1
− |> = λ {x ∈]0, 1] :| Sn (x) |4 > (2n)4 } <
λ
x ∈]0, 1] :|
n
2
(1.19)
Z 1
1
4
<
| Sn (x) | dx .
16n4 4 0
Mais uma vez, da mesma forma que na demosntração da lei fraca, note-se que:

! n
! n
!
n
n
X
X
X
X
| Sn (x) |4 =
Ri (x) 
Rj (x)
Rl (x)
Rk (x)
i=1
=
=
+
j=1
X
l=1
k=1
Ri (x)Rj (x)Rl (x)Rk (x) =
i,j,l,k∈{1,...,n}
n
X
X
Ri4 (x) +
Ri (x)Rj3 (x)
i=1
i,j∈{1,...,n},i6=j
X
X
+
Ri2 (x)Rj2 (x)+
i,j∈{1,...,n},i6=j
Ri (x)Rj (x)Rl2 (x)+
i,j,l∈{1,...,n},i6=j6=l,i6=l
+
X
Ri (x)Rj (x)Rl (x)Rk (x) .
i,j,l,k∈{1,...,n},i6=j6=l6=k6=i6=l,j6=k
R1
Invocando novamente o exercı́cio 26, para estimar o 0 | Sn (x) |4 dx tal como fizémos
na demonstração da lei fraca, tem-se:
Z 1
Z 1
n Z 1
X
X
4
4
| Sn (x) | dx =
Ri (x)dx +
Ri2 (x)Rj2 (x)dx =
0
i=1
0
i,j∈{1,...,n},i6=j
0
= n + C24 × C2n = n + 3n(n − 1) = 3n2 − 2n ≤ 3n2 .
considerando apenas os termos não nulos. Reportando esta estimativa na desigualdade 1.19 temos que:
Z 1
sn (x) 1
1
3
− |> <
| Sn (x) |4 dx ≤
.
(1.20)
λ
x ∈]0, 1] :|
4
4
n
2
16n 0
16n2 4
Iremos em seguida aproveitar esta estimativa construir uma cobertura adequada do
conjuntodos números não normais cuja medida de Lebesgue seja tão pequena quanto
queiramos. Para tal necessitamos de um resultado simples de análise que é enunciado
no lema seguinte.
31
Capı́tulo 1
Lema 1. Para qualquer η > 0 existe uma sucessão (n )n∈N∗ de números reais positivos,
tal que:
1. A sucessão (n )n∈N∗ decresce para zero:
lim ↓ n = 0
n→+∞
2. A velocidade de convergência para zero da sucessão (n )n∈N∗ é contrabalançada
pela velocidade de convergência para +∞ da sucessão (n2 )n∈N∗ de tal forma que:
+∞
X
n=1
3
<η
16n2 4n
Demonstração. Procura-se uma sucessão (n )n∈N∗ decrescente para zero e tal que a série
de termo geral (3/16n2 4 )n∈N∗ seja convergente, tendo uma soma majorada estritamente
por η. Tal é possı́vel se se tiver para β > 1:
n2 4n ' nβ .
Seja então, por definição, para qualquer n = 1, 2, . . . :
4n :=
c3
√ ,
16 n
onde c é uma constante, em relação a n ∈ N∗ , cujo valor determinaremos em função
√
de η. É imediato verificar que a sucessão de termo geral n := (3c/ n)1/4 satisfaz as
duas condições do enunciado desde que se escolha c, em função de η, suficientemente
grande.
Para concluirmos a demonstração vamos mostrar que (N )c tem medida de Lebesgue
nula, segundo a definição. Como Sn é uma função em escada, pode deduzir-se que o
conjunto:
sn (x) 1
x ∈]0, 1] :|
− |> = {x ∈]0, 1] :| Sn (x) |> 2n}
n
2
é uma união finita de subintervalos de ]0, 1]. Seja η > 0 qualquer. Seja (n )n∈N∗ uma
sucessão que satisfaça as condições do lema 1 acima. Observe-se agora que se
+∞
\
sn (x) 1
x∈
− |≤ n ,
x ∈]0, 1] :|
n
2
n=1
então
lim |
n→+∞
sn (x) 1
− |= 0 ,
n
2
pelo que x ∈ N, isto é:
N ⊂
c
+∞
[
n=1
32
sn (x) 1
− |> n
x ∈]0, 1] :|
n
2
.
A Teoria da Probabilidade, segundo Borel, para sucessões de Bernoulli
Dado que o conjunto mais à direita nesta inclusão é uma união numerável de intervalos
e que, pela construção do lema e pela estimativa 1.20:
X
+∞ +∞
X
3
sn (x) 1
− |> n } ≤
<η,
λ {x ∈]0, 1] :|
n
2
16n2 4n
n=1
n=1
tem-se, pela definição de conjunto de medida de Lebesgue nula, que λ(Nc ) = 0, isto é, a
lei forte dos grandes números.
1.4
Exercı́cios
Alguns dos enunciados seguintes figuram também nas folhas de exercı́cios.
Exercı́cio 22. Prove que o conjunto B das sequências de Bernoulli não é numerável usando o argumento
da diagonal de Cantor.
Exercı́cio 23. Seja ω ∈ I =]0, 1].
P
i
1. Mostre que ω pode ser escrito na forma ∞
i=1 ai /2 , ai = 0, 1. Mostre que esta expansão é única
quando nos restringimos a desenvolvimentos binários não degenerados.
P
i
2. Mostre que para qualquer inteiro k, ω ∈ I pode ser escrito na forma ∞
i=1 ai /k , ai = 0, 1, ..., k −1.
Mostre ainda que esta expansão é única quando nos restringimos a desenvolvimentos k-ários não
degenerados.
[1]
Exercı́cio 24. Um número é escolhido ao acaso em [0, 1]. Qual a probabilidade de:
1. o primeiro algarismo não nulo do desenvolvimento decimal ser 1 ?
2. o segundo algarismo significativo do desenvolvimento decimal ser 5 ?
Exercı́cio 25. Um jogador tem uma fortuna inicial de uma unidade monetária, calcule a probabilidade [1]
de ruina na 1, na 3 e na 5 jogada. Mostre que a probabilidade de eventual ruina é pelo menos de 70 %.
[2]
Exercı́cio 26. Mostre que
Z
Rγ1 Rγ2 ...Rγn dx = 0 ou 1
I
para qualquer sequência γ1 ≤ γ2 ≤ ... ≤ γn . Quando é que a resposta é 1?
ek , [1]
Exercı́cio 27. Sejam Rk :]0, 1] −→ R, k ∈ N∗ as funções de Rademacher. Defina, para cada k ∈ N∗ , R
a função de Rademacher sobre toda a recta real, requerendo que a função seja periódica de periodo um,
ek (x+1) = R
ek (x) para todo o x ∈ R. Com esta definição mostre que R
ek+1 (x) = R
ek (2x)
ou seja, fazendo R
ek (x) = R
e1 (2k−1 x).
e, por indução, que R
[2]
Exercı́cio 28. Mostre que
Rn (x) = −sgn[sin(2π2n−1 x)]
excepto num número finito de pontos.(Para qualquer número a, sgn(a) é 1 se a > 0 e -1 se a ≤ 0).
Mostre, também, que:
en (x) = −sgn[sin(2π2n−1 x)]
R
excepto numa infinidade numerável de pontos.
33
Capı́tulo 1
[1]
Exercı́cio 29. Prove que
2t − 1 =
∞
X
−k
Rk (t)2
.
k=1
[1]
Exercı́cio 30. Qualquer número ω ∈]0, 1] tem um desenvolvimento ternário
∞
X
ak 3−k
k=1
com ak = 0, 1, 2 (ver exercı́cio 23). Podemos tornar este desenvolvimento único selecionando, sempre
que exista ambiguidade, o desenvolvimento não degenerado. Com esta convenção, define-se
Tk (ω) = ak − 1.
Desenhe o gráfico de Tk para k = 1, 2, 3. Consegue encontrar uma fórmula geral?
[1]
Exercı́cio 31. Obtenha uma fórmula recursiva para os Tk semelhante à fórmula encontrada para os Rk
no exercı́cio 28.
[3]
Exercı́cio 32. Mostre que o conjunto dos números não normais é denso em [0, 1].
[2]
Exercı́cio 33.
1. Mostre que existe uma constante positiva c tal que, para todo n ∈ N,
Z 1
[Sn (x)]6 dx ≤ cn3 .
0
2. Seja An = {w ∈]0, 1] : |Sn (w)| > εn}. Mostre que λ (An ) ≤ cε−6 n−3 .
[2]
Exercı́cio 34. Mais geralmente, mostre que existe um número positivo ck tal que, para todo o n ∈ N,
Z 1
[Sn (x)]2k dx ≤ ck nk .
0
[3]
Exercı́cio 35. Prove o seguinte refinamento da lei forte dos grandes números:
∀δ >
[2]
1
2
lim
n→+∞
Sn
=0.
nδ
Exercı́cio 36. Prove que
1
Z
etSn (x) dx =
0
Indicação : Prove por indução. Escreva
Z 1
Z
etSn (x) dx =
0
et + e−t
2
(1.21)
n
.
1
etSn−1 (x) eRn (x) dx.
0
Divida o intervalo [0, 1] em 2n−1 intervalos iguais onde em cada um deles Rn−1 é constante. Mostre que
t
Z
Z
e + e−t
etSn (x) dx =
etSn−1 (x) dx
2
J
J
se J é um desses sub-intervalos.
34
A Teoria da Probabilidade, segundo Borel, para sucessões de Bernoulli
[2]
Exercı́cio 37. Do exercı́cio anterior, obtenha a fórmula
2k t
n Z 1
d
e + e−t
Sn (x)2k dx =
.
dt
2
0
t=0
[2]
Exercı́cio 38. Seja f uma função monótona não negativa definida no intervalo unitário. Prove a
desigualdade de Chebyshev
Z
1 1
λ({ω ∈ I; f (ω) > α}) <
f (x)dx
α 0
com o integral da direita sendo o integral de Riemann.
1.5
Exercı́cios Complementares
∗
Exercı́cio 39. Seja, para cada α ∈ [0, 1] com desenvolvimento binário (αn )nεN∗ ∈ [0, 1]N e, para cada
n ∈ N∗ seja Pn (α), a posição de um ponto na recta real, definida, por indução, por:
• Po (α) = 0
• ∀n ≥ 1 Pn (α) ∈ Z ⇒ Pn+1 (α) = Pn (α) + Rn+1 (α)
Mostre que para todo o α ∈ [0, 1], salvo talvez num conjunto de medida de Lebesgue nula, se verifica:
lim
n→∞
Pn+1 (α)
=0.
n+1
∗
Exercı́cio 40. Considere o jogo dado por (αn )n∈N∗ ∈ {0, 1}N , uma sucessão infinita de lançamentos
de uma moeda equilibrada ao ar. Sempre que numa tiragem saia coroas (0), o jogador perde 1 unidade
monetária, sempre que numa tiragem saia caras (1), o jogador ganha 1 unidade monetária.
1. Mostre que a soma dos ganhos e perdas, à N –ésima tiragem, é dada por:
N
X
Rh (α) .
h=1
2. Para 0 < δ < 1 seja AN (δ) o acontecimento definido por: ”A soma dos ganhos e perdas, à Nésima
jogada, é maior ou igual que δN e menor ou igual que −δN ”. Mostre que:
P[AN (δ)] ≤
1
.
δ2 N
Exercı́cio 41. Seja α ∈]0, 1] e consideremos o seu desenvolvimento ternário não degenerado (αn )n∈N∗ ∈
∗
{0, 1, 2}N . Seja β ∈ {0, 1, 2}, fixo, e defina-se:
(
2
αk = β
Rk (α) =
−1 αk 6= β .
1. Mostre que
R 1 PN
o
h=1
Rh (α) dα = 0.
R1
Ri (α)Rj (α)dα = 0 desde que i 6= j.
o
2
R 1 PN
3. Mostre que o
R
(α)
dα = 2N .
h
h=1
2. Mostre que
35
Capı́tulo 1
Exercı́cio 42. Diremos que um conjunto A ⊆ R verifica a propriedade T se e só se, sendo J a famı́lia
de todos os intervalos de de R,
∀ > 0 ∃p ∈ N ∃I1 , · · · , Ip ∈ J A ⊆
p
[
Ik ∧
h=1
p
X
λ(Ik ) < .
h=1
1. Mostre que se A verificar a propriedade T, então A é de medida de Lebesgue nula.
2. Mostre que o fecho de um conjunto, na topologia ususal de R, que verifica a propriedade T, também
verifica a propriedade T.
3. Dê exemplo de um conjunto de medida de Lebesgue nula, que não verifica a propriedade T.
Exercı́cio 43. Seja o jogo dado por uma sucessão infinta de lançamentos independentes de uma moeda
∗
equilibrada ao ar. Seja α ≡ (αn )n∈N∗ ∈ {0, 1}N uma realização do jogo. Sempre que numa tiragem saia
coroa (0) o jogador perde +1 e sempre que saia caras (1) o jogador ganha 1. Suponha que o jogador tem
uma fortuna inicial de dois. Determine a probabilidade de ruı́na à segunda jogada. Poderá tirar proveito
da representação gráfica de um esquema em árvore.
Exercı́cio 44. Seja α ∈]0, 1] e consideremos o correspondente desenvolvimento binário (αn )n∈N∗ ∈
∗
{0, 1}N . Seja por definição
n
1X
Xn (α) =
Rh (α) .
n
h=1
R1
1. Obtenha uma expressão para o etXn (α) dα .
2K
R1
2. Baseando-se no resultado que obteve na alı́nea anterior determine uma expressão para: o Xn (α)
dα
.
Exercı́cio 45. Considere o conjunto
o
n
∗
M := α ≡ (αn )n∈N∗ ∈ {0, 1}N : ∃no ∈ N∗ ∀n ≥ no αn = 1 .
1. Mostre que λ(M ) = 0 .
2. Conclua que, com probabilidade um, no desenvolvimento binário de α ∈ M a sequência 10 aparece
uma infinidade de vezes.
∗
Exercı́cio 46. Considere o jogo dado por (αn )n∈N∗ ∈ {0, 1}N , uma sucessão infinita de lançamentos
de uma moeda equilibrada ao ar. Sempre que, ao N–ésimo lançamento saia coroas (0), o jogador perde
1/2N unidades monetárias e sempre que ao N–ésimo lançamento, saia caras (1), o jogador ganha 1/2N
unidades monetárias.
1. Determine a expressão da soma dos ganhos e perdas ,ao N–ésimo lançamento, usando as funções
de Rademacher.
P
n
2. Determine a probabilidade da soma dos ganhos e perdas, no jogo α = ∞
n=1 αn /2 , ser positiva.
Exercı́cio 47. Seja α ∈]0, 1] e consideremos o seu desenvolvimento não degenerado em base 3, (αn )n∈N∗ ∈
{0, 1, 2}N . Seja β ∈ {0, 1, 2}. Defina-se:
(
2
αk = β
Rk (α) =
−1 αk 6= β
36
A Teoria da Probabilidade, segundo Borel, para sucessões de Bernoulli
1. Mostre que:
1
Z
t(
e
PN
h=1
Rh (α))
dα =
o
2e−t + e2t
3
N
.
2. Em função da alı́nea anterior, obtenha uma expressão para:
1
Z
0
N
X
!2L
Rh (α)
dα .
h=1
Exercı́cio 48. Seja ω ∈]0, 1]; Mostre que, para cada inteiro k fixo, ω se pode representar na forma
ω=
∞
X
ai
,
i
k
i=1
com ai ∈ {0, 1, 2, · · · , k − 1}. Mostre que esta representação é única se se verificar que:
∀i ∈ N∗ ∃j ≥ i aj 6= 0 .
Exercı́cio 49. Mostre que a correspondência τ :]0, 1] −→]0, 1] que a um ponto α ∈]0, 1] com desenvolvimento binário não degenerado (αn )n∈N∗ associa o ponto β := τ (α) com desenvolvimento binário não
degenerado (βn )n∈N∗ dado por:
(
αk+1 n = 3k + 1, k ≥ 0
βn :=
1
n 6= 3k + 1, k ≥ 0
pode ser usada para definir uma aplicação injectiva τ , que verifica: τ (]0, 1]) é um conjunto não numerável
de números não normais.
1.6
Resoluções
Resolução:[Exercı́cio 4] Para a resolução do exercı́cio vamos proceder por indução. Seja
x ∈]0, 1] qualquer fixo. Como vimos imediatemente a seguir ao enunciado do exercı́cio,
a propriedade é válida para n = 1, 2, 3. Suponhamos que se tem então:
sn (x) =
α1 (x) α2 (x)
αn (x)
m(x)
+
+ ··· +
= n
2
n
2
2
2
2
em que m(x)/2n é a extremidade inferior do intervalo de comprimento 1/2n a que x
pertence. Tem-se pois:
m(x) m(x) + 1
2m(x) 2m(x) + 1
2m(x) + 1 2m(x) + 2
x∈
,
=
,
∪
,
2n
2n
2n+1
2n+1
2n+1
2n+1
Se se tiver x ∈]2m(x)/2n+1 , (2m(x) + 1)/2n+1 ] então tem-se pelas definições αn+1 (x) = 0
e por isto:
sn+1 (x) =
α1 (x) α2 (x)
αn (x) αn+1 (x)
m(x)
2m(x)
+
+ ··· +
+
= sn (x) = n = n+1
2
n
n
2
2
2
2
2
2
37
Capı́tulo 1
e sn+1 (x) continua a ser a extremidade inferior do intervalo de comprimento 1/2n+1 a
que x pertence. Do mesmo modo se se tiver que x ∈](2m(x)+1)/2n+1 , (2m(x)+2)/2n+1 ]
então αn+1 (x) = 1,
sn+1 (x) = sn (x) +
1
2n+1
=
m(x)
1
2m(x) + 1
+ n+1 =
n
2
2
2n+1
e sn+1 (x) é ainda a extremidade inferior do intervalo de comprimento 1/2n+1 a que x
pertence ficando assim demonstrada, por indução, a propriedade enunciada no exercı́cio,
uma vez que se tem:
1
x ∈ sn (x), sn (x) + n ,
2
dado que o comprimento dos intervalos à ordem n é 1/2n . Pelo teorema das sucessões
enquadradas tem-se que limn→+∞ sn (x) = x.
♦
Resolução:[Exercı́cio 22] Suponhamos C numerável. Podemos então representá-lo
na forma C = {Cm : m ∈ N∗ } ficando entendido que se m 6= n então Cm 6= Cn . Pela
definição, isto é,
(
)
∞
X
αh
C = α ∈ [0, 1] : α =
, αh ∈ [0, 2]
zh
h=1
podemos então associar a cada Cm o seu desenvolvimento ternário Cm = (αhm )h∈N∗ com
αhm ∈ {0, 2} .
Representemo-los sob a forma de um quadro

1 1 1
1

 C1 = α1 α2 α3 · · · αp



2 2 2
2


 C2 = α1 α2 α3 · · · αp
C3 = α13 α23 α33 · · · αp3


..


.



 C = αp αp αp · · · αp .
p
1
2
3
p
9
e seja β = (βn )n∈N∗ ζ com a seguinte representação em base 3
∀n ∈ N∗ βn = 2 − αnn · · · (∗)
(1.22)
Como para todo o n ∈ N αnn ∈ {0, 2} será também βn ∈ {0, 1} pelo que por construção
β ∈ C. Mas por outro lado: ∀m ∈ N∗ β 6= Cm . Com efeito, caso contrário, se para
mo ∈ N∗ fosse β = Cmo então terı́amos ∀n ∈ N∗ βn = αnmo .
mo . A hipótese feita de
Ora isto contradiz a definição 1.22 que implica que βmo 6= αm
o
que C é numerável engendra uma contradição pelo que C não é numerável.
Observação 11. C é, obviamente, infinito.
9
38
Observe-se, no quadro, a diagonal usada para construir β.
A Teoria da Probabilidade, segundo Borel, para sucessões de Bernoulli
♦
Resolução:[Exercı́cio 26] Observe-se que podemos supôr sempre que γ1 < γ2 <
· · · < γn . Com efeito, seja p1 ∈ {1, . . . , n} o primeiro inteiro tal que γp1 +1 > γp1 . Tem-se
então que:
Rγ1 · Rγ2 · · · Rγn = Rγp1p · Rγp1 +1 · · · Rγn
1
Seja agora p2 ∈ {p1 + 1, . . . , n} o primeiro inteiro tal que γp2 +1 > γp2 . Tem-se, em
consequência, que:
Rγ1 · Rγ2 · · · Rγn = Rγp1p Rγp2p−p1 · Rγp2 +2 · · · Rγn
1
2
Por indução podemos pois encontrar um número finito de inteiros p1 , · · · , pm ∈
{1, · · · , n} tais que:
−pm−1
Rγ1 · Rγ2 · · · Rγn = Rγp1p Rγp2p−p1 · Rγp3p−p2 · · · Rγpm
pm
1
2
3
onde p1 < p2 < · · · < pm .
Observando que para todo o p inteiro e γ inteiro
Rγ2p ≡ 1 e Rγ2p+1 ≡ Rγ2p · Rγ ≡ Rγ
bastará observar as paridades dos expoentes p1 , p2 −p1 , · · · , pm −pm−1 guardando apenas
aqueles que são ı́mpares sejam q1 , q2 , · · · , q` tendo-se então que:
Rγ1 · · · · · Rγn = Rγi1 · · · Rγiq
em que i1 , · · · , iq ∈ {p1 , · · · , pm } e correspondem aos q1 , · · · , q` . É óbvio que i1 < · · · <
iq .
R1
Sejam então γ1 < Γ2 inteiros e determine-se 0 Rγ1 (α)Rγ2 (α)dα.
Observe-se que sobre cada um dos intervalos diádicos em que Rγ1 é constante, e vale
+1 ou −1, Rγ2 oscila um número par de vezes entre os valores +1 e −1. Com efeito, num
intervalo de comprimento 2γ11 , onde Rγ1 é constante, há 2γ11 : 2γ12 = 2γ2 −γ1 intervalos de
comprimentos 2γ12 que é obviamente um número par. Assim sendo, podemos escrever:
1
Z
Rγ1 (α)Rγ2 (α)dα =
γ1 −1
k=2
X
Z
(±1)
0
k+1
2γ1
k
2γ 1
k=0
Rγ2 (α)d(α)
Como o integral de Rγ2 (α), sobre um intervalo 2kγ1 , k+1
2γ1 , vale zero temos que o
integral de Rγ1 Rγ2 em [0, 1] é zero. Do mesmo modo se procede para 3 funções com
γ1 < γ2 < γ3 .
γ1 −1
Z 1
Z k+1
k=2
X
2γ1
Rγ2 (α) · Rγ3 (α)dα
Rγ1 (α)Rγ2 (α)Rγ3 (α)dα =
(±1)
0
k+1
2γ1
Z
k
2γ1
Z
Rγ2 (α) · Rγ3 (α)dα =
`=(k+1)2γ2 −γ1 −1
=
k
2γ1
k=0
X
`=k2γ2 −γ1
(±1)
Z
`+1
2γ 2
`
2γ 2
(k+1) γ −γ
2 2 1
2γ2
k
2γ 2
2γ2 −γ1
Rγ2 (α) · Rγ3 (α)d(α) =
Rγ3 (α)dα = 0
39
Capı́tulo 1
Por indução teremos que para qualquer n, inteiro não nulo e γ1 < γ2 < · · · < γn
inteiros não nulos será
Z 1
Rγ1 · · · · · Rγn dα = 0
0
É óbvio que se γ1 ≤ γ2 ≤ · · · ≤ γn o integral valerá 1 quando e só quando fôr:
−pm−1
Rγ1 · · · Rγn = Rγp1p Rγp2p−p1 · Rγp3p−p2 · · · Rγpm
pm
1
2
3
com p1 , p2 − p1 números pares. Nesta circunstância teremos Rγ1 · · · · · Rγn ≡ 1 pelo que
o integral valerá 1, se houver pelo menos um expoente ı́mpar o integral será zero.
♦
Resolução:[Exercı́cio 27] As fórmulas que se pretende demonstrar dizem respeito à
extensão periódica de Rk , a késima função de Rademacher, a R. Esta extensão que denotaremos por R̃k é, por definição, uma função definida em R com perı́odo 1, verificando
portanto:
• ∀x ∈ R
R̃k (n + 1) = R̃k (n)
• ∀x ∈ [0, 1] R̃k (n) = Rk (n)
S
É certo que R = n∈Z ]n, n + 1]. Esta igualdade resulta de ser N bem ordenado e R
arquimediano. Em consequência, para cada x ∈ R existe Nn ∈ Z e xn ∈]0, 1] tais que
n = Nn + xn . (Ponha-se xn = n − Nx onde Nn é tal que x ∈]Nn , Nn + 1].) Podemos
pois escrever:
R̃k+1 (x) = R̃k+1 (Nx + xn ) =(1) R̃k+1 (αn ) =(2) R̃k+1 (αn ) =(3) Zαk+1 − 1
onde justificamos:
(1) por ser R̃k+1 periódica de perı́odo 1 e Nx ∈ Z i.e. Nn múltiplo inteiro do perı́odo;
(2) por ser R̃k+1 extensão de R̃k+1 a R e xn ∈]0, 1];
(3) se fôr αn ≡ (αn )n∈N∗ o desenvolvimento binário de αn .
Da mesma forma, podemos escrever:
α2 α3
αn+1
R̃k (2x) =R̃k (2Nx + 2xn ) = R̃k (2αn ) = R̃k α1 +
+ 2 + ··· + n + ··· =
2
2
2
!
!
∞
∞
X αn+1
X αn+1
=(5) R̃k
=(6) Rk
=(7) Zαk+1 − 1
n
2
2n
(4)
n=1
n=1
onde justificamos:
(4) e (6) por argumentos semelhantes aos argumentos para justificar (1) e (2) res-pectivamente;
(5) por serα1 = 0 ou α1 = 1 e R̃k ser periódica de perı́odo 1;
40
A Teoria da Probabilidade, segundo Borel, para sucessões de Bernoulli
(7) dado que R̃k (β) é por definição 2βk − 1 em que βk é o k ésimo termo do desenvolvimento binário de β, isto é, é o inteiro de {0, 1} que se encontra como numerador
da fracção que tem como denominador 2k .
É trivial que: R̃k (x) = R̃k−1 (2n) = R̃k−2 (22 n) = R̃1 (2k−1 n) usando-se, tal como já
foi demonstrado, que:
R̃k (x) = R̃k−1 (2x).
♦
Resolução:[Exercı́cio 28] Vejam-se as notações de correcção do exercı́cio 27 anterior.
Demonstraremos o seguinteresultado:
R̃n (x) = − sinal sen(2π2n−1 x) ,
excepto talvez numa famı́lia numerável de pontos.
Observe-se que, em função do que já foi visto em [5],
R̃n (n) = R̃1 (2n−1 n)
pelo que a igualdade que pretendemos demonstrar é equivalente a:
R̃1 (2n−1 n) = − sinal sen(2π2n−1 n) .
Com a mudança de variável y = 2n−1 x temos ainda
R̃1 (Y ) = − sinal [sen(2πy)]
Dado que y = Ny + αy com Ny ∈ Z e αy ∈]0, 1] será:
R̃1 (Ny + αy ) = − sinal [sen(2πNy + 2παy )] .
Ainda atendendo à periodicidade, de R̃1 e do seno, e ao facto de R̃1 ser extensão de
R1 e αy ∈]0, 1]
R1 (αy ) = − sinal [sen(2παy )] .
1
Ora, para αy ∈ 0, 2 , R1 (αy ) = −1 e como nestas condições 2παy ∈]0, π], temos
que − sinal[sen(2παy ] = −1 se αy 6= 21 e +1 se αy = 12 . Para αy ∈] 12 , 1] temos que
2παy ∈]π, 2π] pelo que R1 (αy = +1 e − sinal [sen(2παy )] = 1 . Em resumo, o resultado
1
com N ∈ Z e
que pretendemos demonstrar é válido em R salvo para x = 21 + N x 2n−1
n ≥ 1.
♦
Resolução:[Exercı́cio 29]
∞
X
k=1
Rk (t)2−k =(1)
∞
X
k=1
(2αk − 1)2−k =(2) 2
∞
X
αk
k=1
2k
−
∞
X
1 (3)
= 2t − 1
2k
k=1
Onde as justificações são como a seguir se indica:
41
Capı́tulo 1
(1) Por definição Rk (t) = 2αk − 1 onde t ≡ (αk )k∈N∗ é ????? desenvolvimento binário
de t ∈]0, 1];
(2) A série cujo termo geral é a somaPdos termos
gerais de duas séries convergentes
∞ αk é
convergente
visto que α2kk ≤ 21k e
é uma série convergente. Ora
k=1 2k
P∞ 1
k=1 2k = 1.
P
αk
(3) Por definição t = ∞
k=1 2k .
♦
Resolução:[Exercı́cio 30] A função Tk é uma função em escada que só toma três
valores (−1, 0, 1) sendo constante em cada um dos 3k intervalos de comprimento 31k em
que fica dividido o intervalo [0, 1]. A função Tk é contı́nua em ]0, 1] salvo nos pontos
da forma 3mk com m ∈ {0, · · · , 3k }, admitindo nestes pontos os limites à esquerda e à
direita, a saber, respectivamente
Lc (
m
m
m
Tk (x) = Tk ( k ) 6= Lc :=
Tk (x) =: Ld ( k ) .
) :=
limm
limm
k
m
m
z
z
z
n→ k , x< k , m6=0
n→ k , x> k , m6=0
z
z
z
z
A forma geral do gráfico de Tk pode ser descrito do modo seguinte. Tk toma, sucessivamente, os valores −1, 0, 1 à medida de x ∈]0, 1] vai crescendo de zero para 1, nos
sucessivos intervalos de comprimento z1k começando por tomar o valor −1 no primeiro
intervalo, isto é ]0, z1k ]. Para o intervalo de ordem m ∈ {1, · · · , z k }, isto é ] m−1
, zmk ], se
zk
fôr m = p3 + r com p ∈ N e r ∈ {1, 2, 0} a divisão euclidiana de m por 3, será


−1 se r = 1
Tk (n) =
0 se r = 2


1 se r = 0.
♦
Resolução:[Exercı́cio 31] A fórmula de recorrência a obter é para k ∈
∀x ∈ R
T̃k+1 (n) = T̃k (3x). A prova faz-se considerando T̃k a extensão periódica de perı́odo 1 de
Tk a R e usando o mesmo tipo de argumentos que os que foram usados nas resoluções
dos exercı́cios 27 e 28.
♦
Resolução:[Exercı́cio 32] Observe-se que se α ∈]0, 1] for um número diádico então α
∗
terá necessariamente um desenvolvimento binário da forma (αn )n∈N∗ ∈ {0, 1}N tal que
para no ∈ N∗ dado será:
∀n > no αn = 1
N∗
Assim sendo teremos que para n > no será
sn (α)
n − no
≥
n
n
Em consequência limn→+∞ sn (α)/n = 1, pelo que α é não-normal. Se denotarmos
D o conjunto dos números diádicos então temos que D ⊆ No , onde N representa os
números normais.
42
A Teoria da Probabilidade, segundo Borel, para sucessões de Bernoulli
É trivial agora que D = [0, 1]. Seja x ∈ [0, 1]. Suponhamos que x 6= 0, 1 e que para
Log
> 0 se tem: X =]n − , n − [⊆]0, 1[. Seja no tal que 2n1o < 2 no > 1 − Log2
terá que
m
existir então pelo menos um ponto diádico da forma 2no no intervalo X . Com efeito,
no },
este intervalo tem comprimento 2 e os pontos diádicos, da forma 2m
no , m ∈ {0, · · · , 2
estão separados uns dos outros por uma distância que vale 2n1o ; pela escolha de no , temos
1
2no <2 .
o
o
Em consequência D ⊆ N ⊆ [0, 1] como D = [0, 1] terá que ser N = [0, 1].
Observação 12. Se x = 0 ou 1 far-se-á a mesma prova mas com um intervalo da forma
]0, [ ou ]1 − , 1[
♦
Resolução:[Exercı́cio 33]
(SN (x))2 =
N
X
!2
Ri (x)
=
i=1
N
X
N
X
Ri2 +
i=1
i,j=1
i6=j
2
= (SN
(x))3 = N 3 + 3N 2

Ri Rj + 3N 
i,j=1
i6=j
Z
1
[SN (x)]6 dx = N 3 + 3N 2
o
N
X

+
o
1


N
X
2
i,j=1
i6=j
1
Z
Ri Rjdx + 3N
o
o



Ri Rj  + 
N
X
3

Ri Rj 
i,j=1
i6=j

Z
i,j=!
i6=j
Z
Ri Rj [SN (x)]6 =
i,j=1
i6=j

N
X
N
X
Ri Rj = N +
1


N
X
2

Ri Rj  dx+
i,j=!
i6=j
3
N
X

Ri Rj  dx
i,j=1
i6=j
Sabemos que
Z
1
Rγ1 Rγ2 · · · Rγn dx = 1
0
se com γ1 ≤ γ2 ≤ · · · ≤ γn podemos extrair γk1 < γk2 < · · · < γkt com kt ≤ n tal que
Rγ1 · Rγ2 · · · · · Rγn = Rγαk1 · Rγαk2 · · · · · Rγαkt
1
1
t
em que k1 , k2 , · · · , kt são pares.
Este integral vale zero no caso contrário.
R1
Daqui concluı́mos que 0 Ri Rjdx = 0, i ≤ j, donde
N Z
X
i,j=1
i6=j
1
Ri Rj dx = 0
o
43
Capı́tulo 1
Resta-nos estudar os restantes casos

2
Z 1 X
N

Ri , Rj  dx
o
i,j=1
2
N
N
X
X


Ri Rj  =
Ri Rj RK Rl


i,j=1
i6=j

Z
o
1


N
X
i,j,k,l=1
i6=j
K6=l
2
Z
N
X

Ri , Rj  dx =
i,j=1
i6=j
i,j,k,l=1
i6=j
K6=l
1
Ri Rj RK Rl dx = M N (N − 1)
o
Vejamos quais são os casos em que o integral vale um, ou seja, em que as igualdades
entre os ı́ndices são tais que as funções têm expoentes pares. Os casos existentes são:
1. Rα4 , este caso não é possı́vel, pois implicarı́a i = j = K = l, o que não pode
acontecer
2. Rα2 Rβ2 , α 6= β. Este caso já é possı́vel. Seja M ∈ N o número de caos deste tipo
existente. Para cada caso, temos N (N − 1) ocorrências
Rα2 Rβ2
”α- existem N números disponı́veis para este ı́ndice
”β- como α 6= β existem N − 1 números disponı́veis para este ı́ndice

3
Z 1 X
N


Ri Rj  dx

o



i,j=1
i6=j
N
X
i,j=1
i6=j
3

Ri Rj  =
X
Rα1 Rα2 Rα3 Rα4 Rα5 Rα6
αi ε{1,··· ,N }
iε{1,··· ,N }
α1 6=α2
α3 6=α4
α5 6=α6
Este caso é análogo ao anterior, tendo em conta que os casos são:
1. Rα6
2. Rα4 Rβ2 , α 6= β
3. Rα2 Rβ2 Rγ2 , α 6= β 6= γ
44
A Teoria da Probabilidade, segundo Borel, para sucessões de Bernoulli
Não sendo possı́vel 1 e 2, pois exigem igualdade entre os ı́ndices que não podem
acontecer devido às condições α1 6= α2 , α3 6= α4 , α5 6= α6 . O único caso existente é 3.
Seja s o número de casos deste tipo existentes. Para cada caso temos N (N − 1)(N − 2)
ocorrências.
3






Z 1 X


 dx = sN (N − 1)(N − 2)

R
R
R
R
R
R
α
α
α
α
α
α
6
5
4
3
2
1


o  α ε{1,··· ,N }

i

 iε{1,···
,6}

 α1 6=α2
α3 6=α4
α5 6=α6
1
Z
[SN (x)]6 dx = N 3 + 3M N 2 (N − 1) + sN (N − 1)(N − 2) ≤
o
≤ n3 + δ(3N 3 − 3N 2 + N 3 − 3N 2 + 2N ) =
= N 3 + δ(4N 3 − 3N 2 + 2N ) = (4δ + 1)N 3 −
− 3δN 2 + 2N ≤ (4δ + 1)N 3 + 2N 3 = (4δ + 3)N 3
(NOTA: ≤→
N εN
; Seja δ = max(M, 1))
δεN
An = {wε]0, 1] : |Sn (w)| > εn} = wε]0, 1] : |Sn (w)|6 > ε6 n6
Aplicando a desigualdade de Chebyshev, temos:
Z 1
1
1
µ(An ) ≤ 6 6
[SN (x)]6 dx ≤ 6 6 C3 n3 = ε−6 n−3 C3
ε n o
ε n
♦
Resolução:[Exercı́cio 34] O método é semelhante ao que foi aplicado para os casos
em que K = 2 ou 3. Usaremos a fórmula multinomial a saber:
(R1 + R2 + · · · + RN )P =
X
P1 + P2 + · · · + PN = P
P1 , · · · , PN ∈ N
P!
PN
RP1 RP2 · · · RN
.
P1 !P2 ! · · · PN ! 1 2
Tomando em conta o que sabemos sobre integrais de produtos de funções de Rademacher:
Z 1
Z 1
X
2k!
2PN
2P1 2P2
2K
R1 R2 · · · RN dα
(R1 +· · ·+RN ) dα =
2P1 ! · · · 2PN !
o
o
P1 + p2 + · · · + PN = K
P1 , · · · , PN ∈ N
45
Capı́tulo 1
dado que os outros termos, os que correspondem aos expoentes ı́mpares nas funções de
Rademacher, desaparecem quando integrados sobre o intervalo [0, 1]. R1
N
Observe-se que teremos N termos da forma o Rk2K (α)dα e teremos
termos da
2
forma
Z 1
2k!
R2K−2 (α) · Ri2 (α)dα para k 6= i
(2k − 2)!2! o k
que corresponde a um dos p1 , · · · , pN igual a 2K − 2 e um outro igual a 2. Termos deste
tipo darão origem na soma a uma quantidade do tipo:
2k!
N (N − 2)
×
(2K − 2)!2!
2!
isto é, a um termo em N 2 . Podemos supôr que N > K dado que se pretende uma
majoração para todo o N .
A questão consiste então em saber qual a maior potência de N que é possı́vel obter
dado que, para K fixo , esta potência é que condicionará o crescimento da expressão.
Quando forem, K entre P1 , · · · , PN , iguais a 1 será, então, para os termos correspondentes:
Z 1
2k!
Rk21 (α) · · · Rk2k (α)dα
2! · · · 2! o
Kvezes
N
−K+1)
= N (N −1)···(N
havendo
termos desta espécie que somados darão um termo
K!
K
da forma:
2K!
2
K −1
1
K
1−
1−
N
1−
N
N
N
2k · K!
A maior potência existente é assim N k o que mostra que para uma certa constante CK
será:
Z 1
(SN (α))2k dα ≤ CK N K .
o
♦
10 ]
Resolução:[Exercı́cio 34; segunda resolução


[SN (x)]2K = N +
N
X
K

Ri Rj 
=
i,j=1
i6=j
Z
o
10
46
1
[SN (x)]2K dx =
K
X
K−M
N
X


CM N M 
Ri Rj 

i,j=1
i6=j
M =0
K
X

CM N M
M =0
Esta resolução foi proposta pelo Dr. João Cabral.
Z
o
1


N
X
i,j=1
i6=j
K−M

Ri Rj 
dx
A Teoria da Probabilidade, segundo Borel, para sucessões de Bernoulli

Z
o
1


N
X
K−M

Ri Rj 
dx =
i,j=1
i6=j
1
Z
X
Rα1 Rα2 · · · Rα2K−2M −1 Rα2K−2M dx(1)
αi ε{1,cdots,N }
iε{1,2,cdots,2(KM )−1,2(KM )}
α1 6=α2 ;α2 6=α3 ;··· ;α2K−2M −1 6=α2K−2M
o
Analogamente ao exercı́cio 15, é necessário analisar quando é que
Z 1
Rα1 Rα2 · · · Rα2K−2M −1 Rα2K−2M dx = 1
o
O caso que envolve o maior número de ocorrências é do tipo
Rβ2 1 Rβ2 2 · · · Rβ2 K−M −1 Rβ2 K−M ; β1 6= β2 6= · · · 6= βK−M −1 6= βK−M
onde temos N (N − 1) · · · (N − (K − M − 1))(N − (K − M )) ocorrências. Qualquer
outro caso envolvendo expoentes pares, envolveria igualdades entre os βi , diminuindo o
número de factores no produto N (N −1)(N −2) · · · (N −(K −M )), donde seria reforçado
por este valor. Seja tr o número de casos possı́veis em (1), o número de ocorrências é
sempre majorado por N (N − 1) · · · (N (K − M )), donde
Z
X
1
Rα1 Rα2 · · · Rα2K−2M −1 Rα2K−2M dx ≤
αi ε{1,··· ,N }
(ε{1,2,··· ,2(K−M )−1,2(K−M )}
α1 6=α2 ,α3 6=α4 ,··· ,α2(K−M )−1 6=α2(K−M )
o
≤ tM
Z
1
2K
[SN (x)]
o
dx ≤
K
X
N (N − 1)
N (K − M )
···
≤ tM N K−M
≤N
≤N
M
CM N tM N
K−M
=N
K
PK
M =0 cM tM
M =0
CK
♦
Resolução:[Exercı́cio 35] Seguimos a demonstração da lei forte dos grnades números
e consideramos:
AN () = {α ∈]0, 1] : |SN (α)| > N } =
n
o
= α ∈]0, 1] : (SN (α))2k > 2k N 2k
para k inteiro qualquer. Pela desigualdade de Chebyshev
Z 1
1
Ck
λ(AN ()) < 2k 2k ×
(SN (α))2k dα < 2k k
N
N
0
usando também a majoração obtida no exercı́cio anterior.
47
Capı́tulo 1
Tal como na prova de lei P
forte consideramos agora que é uma função de N N tal
Ck
< δ onde δ é um parâmetro positivo fixado
que limN →∞ N = 0 e ainda ∞
N <1 P2k
N
Nk
a
Nb
previamente. Bastará para tal que 2k
N =
com a.b constantes tais que:
∞
X
∞
Ck X 1
Ck
=
<δ
P2k
a
N K−b
N Nk
N <1
N <1
Para tal bastará que K − b > 1 e que a seja suficientemente grande dados K, b e δ.
Pondo BN = {α]0, 1] : |SN (α)| > N N } será obviamente:
∞
∞
X
CK
Ck X 1
λ(BN ) ≤ 2k K e
< δ.
λ(BN ) =
a
N K−b
N N
N <1
N <1
Para α ∈
√
2k
T∞
N <1
1
será |SN (α)| ≤ N · N pelo que para > SN1(α)
≤ N N 2 − =
+
N2
1
b
−− 2k
2
aN
.
b
Esta última quantidade terá como limite zero se for 12 − − 2k
< 0 ou de forma
K−(K−2)
K−b
= K1 . Podemos
equivalente: > 2K . Podemos pois concluir assim. Dado >
2K
nessa altura mostrar que (BN )N ∈N forma uma cobertura de
α ∈]0, 1] : lim
N →∞
SN (α)
1
N 2 +
C
=0
P
o que ficou escrito acima.
♦
tal que ∞
N =1 λ(BN ) < δ, de acordo com
PN
p p+1
Resolução:[Exercı́cio 36] SN (x) = i=1 Ri (x) constante em ] 2N , 2N ], pε{0, 1, · · · , 2N −
1}
1. N = 1, S1 (x) = R1 (x)
(
R1 (x) =
−1, xε]0, 12 ]
1, xε] 12 , 1]
2. Suponhamos que SN (x) é constante em ] 2pN , p+1
], pε{0, 1, · · · , 2N − 1}
2N
SN +1 (x) =
N
+1
X
i=1
Ri (x) =
N
X
Ri (x) + RN +1 (x) = SN (x) + RN +1 (x)
i=1
Consideremos o intervalo ] 2pN , p+1
] , com pε{0, 1, · · · , 2N }. Sabemos, por hipótese
2N
que SN (x) é constante neste intervalo seja Kp esse valor
48
p p+1
2p 2p + 1
2p + 1 2p + 2
,
=
,
∪
,
2N 2N
2N +1 2N +1
2N +1
2N
A Teoria da Probabilidade, segundo Borel, para sucessões de Bernoulli
RN +1 (x) =
SN +1 (x) =



−1, xε] 2N2p+1 , 22p+1
N +1 ]


2p+2
1, xε] 22p+1
N +1 , 2N +1 ]

2p
2p+1

 Kp − 1, xε] 2N +1 , 2N +1 ]


Kp +
(2)
2p+2
1, xε] 22p+1
N +1 , 2N +1 ]
A participação de oredem N +1 obtem-se a partir da partição de ordem N , dividindo
cada um dos intervalos desta em 2 de amplitude 2N1+1 . Podemos então concluir, a partir
de (2), o resultado desejado. Consideremos a partição de ordem N − 1
N −1
2−1
[ p
p+1
,
]0, 1] =
2N −1 2N −1
p=0
Seja Jp =
Z
i
p
, p+1
2N −1 2N −1
tSN (x)
e
i
Z
dx =
e
Jp
t(SN −1 (x)+RN (x))
Jp
tK tRN (x)
e
2p+1
2N
= etK
Z
etK
= N −1
2
R1
Demonstremos que
o
1. N = 1
Z 1
Z
tR1 (x)
e
dx =
2p
2N
tK
e−t dx +
e−t + et
2N
etSN (x) dx =
1/2
tR1 (x)
e
Z
dx = e
Z
2p+1
2N
2p+2
2N
et dx
e−t + et
2
e−t +et
2
N
tR1 (x)
dx+
e
= etK
Z
o
etSN (x) dx =
]0, 1] =
2N
−1
[
p=0
e−t +et
2
Z
dx +
2p+2
2N
2p+1
2N
!
dx
=
e−t + et
=
2N
etSN −1 (x) dx.
Jp
por indução
Z
dx =
1/2
R1
tRN (x)
!
1
Z
e
2p
2N
2p+1
2N
=
o
2. Suponhamos que
etSN −1 (x) · etRN (x) dx =
=
Jp
= intJp e
o
Z
1/2
Z
−t
1
e dx+
o
1/2
et dx =
e−t + et
2
N
p p+1
Jp , com Jp = N , N
2
2
49
Capı́tulo 1
Z
1
e
tSN +1 (x)
Z
dx =
o
tSN +1 (x)
N
2 −1
Jp
∪p=o
e
dx =
N −1 Z
2X
p=o
etSN +1 (x) dx =
Jp
2N −1
−t
Z
X e−t + et Z
e + et
tSN (x)
=
etSN (x) dx =
e
dx =
2N −1
2
2
J
J
∪
p
p
p=o
p=o
−t
Z 1
N
+1
e + et
etSN (x) dx =
=
.
2
o
R1
etSN (x) dx,
♦
ou seja, g(t) é um integral pa-
Resolução:[Exercı́cio 37] Seja g(t) = o
ramétrico, donde
Z 1
SN (x)etSN (x) dx
g 0 (t) =
o
É fácil obter que
g
(2K)
1
Z
(t) =
[SN (x)]2K etSN (x) dx
o
R1
donde g (2K) (0) = o [SN (x)]2K dx
Mas, por outro lado, sabemos que
Z
1
tSN (x)
e
dx =
o
donde g (2K) (t) =
g
(2K)
d 2K
dt
Z
e−t +et
2
1
(0) =
"
dx(=)
o
Z
=
1
N
N
2K
[SN (x)]
e−t + et
2
d
dt
2K e−t + et
2
N #
=
t=0
[SN (x)]2K dx
o
Resolução:[Exercı́cio 36] .
♦
♦
Bibliografia
[1] Émile Borel. Probabilité et certitude. Presses Universitaires de France, Paris, 1950.
[2] G. H. Hardy and E. M. Wright. An introduction to the theory of numbers. The
Clarendon Press Oxford University Press, New York, fifth edition, 1979.
50
A Teoria da Probabilidade, segundo Borel, para sucessões de Bernoulli
[3] Glenn James and Robert C. James. Mathematics dictionary. 5th ed., multilingual.
Van Nostrand Reinhold, 1992.
[4] Olav Kallenberg. Foundations of modern probability. Probability and its Applications (New York). Springer-Verlag, New York, second edition, 2002.
[5] Paul Malliavin. Integration and probability, volume 157 of Graduate Texts in Mathematics. Springer-Verlag, New York, 1995. With the collaboration of Hélène Airault,
Leslie Kay and Gérard Letac, Edited and translated from the French by Kay, With
a foreword by Mark Pinsky.
[6] Jacques Neveu. Bases mathématiques du calcul des probabilités. Préface de R.
Fortet. Deuxième édition, revue et corrigée. Masson et Cie, Éditeurs, Paris, 1970.
[7] Harald Niederreiter. Random number generation and quasi-Monte Carlo methods,
volume 63 of CBMS-NSF Regional Conference Series in Applied Mathematics. Society for Industrial and Applied Mathematics (SIAM), Philadelphia, PA, 1992.
[8] Sheldon M. Ross. Simulation. Statistical Modeling and Decision Science. Academic
Press Inc., San Diego, CA, second edition, 1997.
[9] Walter Rudin. Principles of mathematical analysis. McGraw-Hill Book Co., New
York, third edition, 1976. International Series in Pure and Applied Mathematics.
[10] Stephen Wolfram. The Mathematicar book. Wolfram Media, Inc., Champaign, IL,
fourth edition, 1999.
51
52
Capı́tulo 2
Medida, Probabilidade e
Acontecimentos
...Ali estava ele, o presidente dos Estados Unidos, rodeado por pessoas
importantes. ...Foi então que alguém disse: Senhor Lincoln, este
homem fez uma longa jornada para lhe falar. Tem uma questão para
lhe pôr. ... o presidente disse: Sim qual é a questão? Eu disse: Senhor
Lincoln, de que comprimento pensa que devem ser as pernas de um
homem?... E o senhor Lincoln, disse alguma coisa? Respondeu?...
Respondeu. Disse que deviam ter o comprimento suficiente para
chegarem ao chão.
Woody Allen, Side Effects, 1975, in Prosa Completa, Woody Allen,
Gradiva & Produções Fictı́cias, 2004, páginas 373 e 374.
2.1
Introdução
Apresentamos um condensado das principais noções que importa conhecer sobre a teoria
da medida e sobre a aplicação desta à teoria das probabilidades. Dado que o texto
pretende ser apenas um auxiliar para o estudo independente e individual, os exercı́cios
constituem parte fundamental do texto devendo ser trabalhados completamente. Nos
textos apresentados nas referências as matérias aqui expostas poderão ser aprofundadas
pelos alunos mais exigentes. No entanto, a leitura aconselhada como livro de texto é [11]
uma vez que estas notas tomam esta obra como estrutura básica inicial.
2.2
2.2.1
Teoria da Medida
Álgebras e Álgebras–σ
Começamos por definir famı́lias de subconjuntos de um conjunto dado que representarão
as famı́lias de conjuntos medı́veis. Seja X um conjunto e P(X) o conjunto de todos os
subconjuntos de X.
53
Capı́tulo 2
Definição 8. Seja A ⊂ P(X). A é uma álgebra, sobre X, se e só se:
1. X ∈ A;
2. ∀A ∈ A Ac = X \ A ∈ A;
3. ∀A, B ∈ A A ∪ B ∈ A.
Exemplo 1. Verifique cuidadosamente que P(X) é uma álgebra sobre X.
O exercı́cio seguinte constrói uma álgebra importante na qual se baseia a nossa
intuição de conjunto linear admitindo como medida a noção de comprimento.
Exercı́cio 50. Sobre R o conjunto dos números reais, considere I = I(R) o conjunto dos intervalos de
R e A(I) o conjunto das partes de R que se podem representar como uniões finitas de intervalos, isto é:
(
)
N
[
∗
A(I) := E ⊂ R; ∃N ∈ N ∃I1 , . . . IN ∈ I E =
In .
n=1
Então, A(I) é uma álgebra sobre R.
1. Sejam A = |a1 , a2 | e B = |b1 , b2 | dois intervalos de R com extremidades verificando a1 ≤ a2 e
b1 ≤ b2 em que a1 , a2 , b1 , b2 ∈ R ∪ {+∞} ∪ {−∞} e em que | ∈ {[, ]}. Mostre que a intersecção de
dois intervalos é ainda um intervalo, isto é, com as escolhas adequadas dos parêntesis rectos [ e ]:
A ∩ B = |a1 , a2 | ∩ |b1 , b2 | = | sup(a1 , b1 ), inf(a2 , b2 )| .
2. Mostre que a diferença de dois intervalos pode sempre representar-se como união finita de intervalos dois a dois disjuntos.
3. Conclua que A(I) é uma álgebra sobre R.
Uma questão natural é a de saber se existem exemplos de álgebras não triviais. O
teorema seguinte mostra que dada uma qualquer famı́lia de partes de um conjunto, que
não seja necessariamente uma álgebra, existe sempre uma álgebra sobre esse conjunto
contendo a famı́lia dada, minimal no sentido da inclusão.
Teorema 3. Seja E ⊂ P(X) uma famı́lia de partes de X. Existe então A(E)
uma álgebra de partes de X que contem E e que é minimal no sentido da inclusão, denominada a álgebra gerada por E.
Demonstração. Seja A(E) a famı́lia de todas as álgebras sobre X contendo E. Então
A(E) 6= ∅ uma vez que P(X) ∈ A(E). Seja por definição:
A(E) :=
\
C∈A(E)
54
C = {A ⊆ X : ∀C ∈ A(E) A ∈ C} .
Medida, Probabilidade e Acontecimentos
Então é imediato verificar1 que A(E) é uma álgebra e que E ⊂ A(E). Se G for uma
qualquer álgebra sobre X contendo E então G ∈ A(E) e logo A(E) ⊂ G pelo que A(E) é
minimal.
A noção de álgebra é insuficiente para cobrir todos os casos com interesse. No
caso do plano euclideano não é possı́vel representar um cı́rculo como união finita de
rectângulos, mas sabemos desde Arquimedes 2 , que se pode obter a área do cı́rculo
como soma das áreas de uma infinidade de triângulos disjuntos encaixados no cı́rculo de
forma a constituı́rem um polı́gono inscrito neste. A noção seguinte permite superar esta
dificuldade.
Definição 9. Seja F ⊂ P(X). F é uma álgebra–σ, sobre X, se e só se:
1. X ∈ F;
2. ∀A ∈ F Ac ∈ F;
3. ∀(An )n∈N ∈ FN
S
n∈N An
∈ F.
Da mesma forma que para a noção de álgebra, é possı́vel demonstrar facilmente a
existência de álgebras-σ não triviais.
Teorema 4. Seja E ⊂ P(X) uma famı́lia de partes de X. Existe então σ(E)
uma álgebra–σ de partes de X que contem E e que é minimal no sentido da
inclusão, denominada a álgebra–σ gerada por E.
Demonstração. Seja F(E) a famı́lia de todas as álgebras–σ sobre X contendo E. Então
F(E) 6= ∅ uma vez que P(X) ∈ F(E). Seja por definição:
\
σ(E) :=
C = {A ⊆ X : ∀C ∈ F(E) A ∈ C} .
C∈F(E)
Então é imediato verificar que σ(E) é uma álgebra–σ e que E ⊂ σ(E). Se G for uma
qualquer álgebra–σ sobre X contendo E então G ∈ A(E) e logo σ(E) ⊂ G pelo que σ(E)
é minimal.
Definição 10. A um par (X, F) em que X é um conjunto e F é uma álgebra–σ
sobre X chamamos espaço mensurável.
O exemplo seguinte é fundamental uma vez que permite construir álgebras-σ naturais
nos espaços euclideanos usuais.
1
Sempre que no texto aparecer uma afirmação deste tipo é imperativo proceder à verificação. De
acordo com L. C. Young – veja-se [12][p. viii] – ...the reader should take no piece of mathematics on
trust if he is ever to grasp it.
2
Veja-se o artigo sobre Arquimedes na Wikipedia http://en.wikipedia.org/wiki/Archimedes. Arquimedes usou o método da exaustão.
55
Capı́tulo 2
Exemplo 2. Seja (E, τ ) um espaço topológico, isto é E é um conjunto e τ ⊂ P(E) é uma
topologia - ou o conjunto dos abertos - sobre E. Relembre que uma famı́lia de partes
de E, seja τ , é uma topologia sobre E, se e só se verifica os axiomas seguintes:
1. E ∈ τ ;
2. A famı́lia τ é fechada para as intersecções finitas, isto é:
\
Ai ∈ τ .
∀A1 , . . . An ∈ τ
1≤i≤n
3. A famı́lia τ é fechada para as uniões quaisquer, isto é, para qualquer conjunto I:
[
∀(Ai )i∈I ∈ τ I
Ai ∈ τ .
i∈I
Então, por definição, B(E) := σ(τ ) é a álgebra–σ de Borel de (E, τ ).
Exercı́cio 51. Sobre R, considere τu a famı́lia de subconjuntos de ω ⊂ R que verifica:
∀x ∈ ω ∃ > 0 ]x − , x + [⊂ ω .
1. Mostre que τu é uma topologia sobre R, denominada a topologia usual. A álgebra–σ gerada por
τu B = B(R) = σ(τu ), é a álgebra–σ de Borel de R.
2. Mostre que todo o aberto de R é união numerável de intervalos abertos.
3. Mostre que B(R) = σ(I).
4. Mostre que B(R) = σ(A(I)).
Observação 13. É imediato verificar que toda a álgebra-σ é uma álgebra. A proposição
recı́proca não é verdadeira. Com efeito, a álgebra A(I) - definida no exercı́cio 50 - não é
uma álgebra-σ uma vez que Q, o conjunto dos números racionais, que pertence a σ(A(I))
- como comprovaremos em seguida - não se pode escrever como união finita de intervalos.
Para verificarmos esta última afirmação observamos, primeiramente, que todo o intervalo
aberto (ou aberto à direita e fechado à esquerda ou, ainda, aberto à esquerda e fechado
à direita), não vazio, contem números irracionais e, em seguida, que os únicos intervalos
fechados que não contêm irracionais são da forma [r, r] para r racional. Como Q é infinito
numerável tem-se que:
[
Q=
[rn , rn ] ,
n∈N
sendo (rn )n∈N uma enumeração dos racionais. Para verificarmos que que Q ∈ σ(A(I))
observamos que, sendo Q sendo uma união numerável de fechados - conjuntos cujos
complementares são abertos - é um elemento da álgebra-σ de Borel de R que, pelo
exercı́cio 2, é exactamente σ(A(I)).
No exercı́cio seguinte constrói-se a álgebra-σ de Borel sobre Rn .
56
Medida, Probabilidade e Acontecimentos
Exercı́cio 52. Sobre Rn , considere τu = τu (Rn ) a famı́lia de subconjuntos de ω ⊂ Rn que verifica:
∀x ∈ ω ∃ > 0 B(x, ) := {y ∈ Rn || x − y ||< } ⊂ ω ,
onde
∀x = (x1 , . . . xn ) || x ||:=
n
X
!1
2
x2i
,
i=1
é a norma euclidiana usual que, de facto, pode ser substituı́da por qualquer outra norma.
1. Mostre que τu é uma topologia sobre Rn , denominada a topologia usual sobre este conjunto. A
álgebra–σ gerada por τu , que representamos por B(Rn ) = σ(τu (Rn )), é a álgebra–σ de Borel de
Rn .
2. Mostre que:
B(Rn ) = σ(A({I1 × I2 × · · · × In : I1 , . . . , In ∈ I})) .
2.2.2
Funções de Conjuntos Aditivas e Aditivas–σ
Nesta secção pretendemos formalizar o conceito de medida. A propriedade caracterı́stica
de uma uma medida é a seguinte: se se decompuser o conjunto na reunião de dois
subconjuntos medı́veis disjuntos então a medida do conjunto deverá ser a mesma que a
soma das medidas dos subconjuntos. Esta propriedade denomina-se aditividade. Mais
geralmente, no intuito de poder aproximar convenientemente conjuntos medı́veis mais
gerais, também deverá verificar-se a propriedade seguinte que generaliza a anterior. Se
um dado conjunto a medir se puder decompor numa partição (com uma quantidade
infinita numerável de elementos) de subconjuntos medı́veis então a medida do conjunto
devrá coincidir com a soma (da série) das medidas dos elementos da partição. Esta
generalização da aditividade às decomposições numeráveis denomina-se aditividade-σ.
Seja X um conjunto e A uma álgebra sobre X.
−
Definição 11. Uma aplicação µ definida em A, tomando valores em R+ :=
[0, +∞] e tal que
∃A ∈ A µ(A) < +∞ ,
é uma função de conjuntos.
1. A função de conjuntos µ é aditiva se e só se:
∀A, B ∈ A A ∩ B = ∅ ⇒ µ(A ∪ B) = µ(A) + µ(B) .
2. A função de conjuntos µ é aditiva-σ se e só se:
!
∀(An )n∈N ∈ A
N
(
[
An ∈ A) ∧ (∀m 6= n ⇒ An ∩ Am = ∅)
⇒
n∈N
!
⇒µ
[
n∈N
An
=
+∞
X
µ(An ) .
n=0
57
Capı́tulo 2
Os dois exemplos seguintes são fundamentais e devem ser estudados com demora e
aplicação que forem necessárias para uma completa familiarização com as ideias envolvidas.
Exercı́cio 53 (A medida de contagem). Seja um conjunto X e a álgebra P(X). Verifique que a aplicação
µc definida sobre P(X) por:
(
+∞ se #A ≥ ℵ0
µc (A) =
#A
se #A < +∞ .
é uma função aditiva-σ de conjuntos sobre P(X).
Exercı́cio 54 (A medida de Lebesgue). Seja |a, b| ∈ I um qualquer intervalo. Seja a aplicação λ definida
em I por
(
+∞
se a = −∞ ∨ b = +∞
λ(|a, b|) =
b − a se a, b ∈ R .
A aplicação λ formaliza a noção de comprimento de um intervalo da recta real.
1. Mostre que toda a união finita de intervalos se pode representar como união finita de intervalos
disjuntos dois a dois. Conclua que é possı́vel extender λ a A(I) como função aditiva de conjuntos.
2. Mostre que λ, definida na alı́nea anterior, é aditiva-σ.
O teorema seguinte explicı́ta algumas das propriedades importantes que decorrem
dos axiomas de definição da aditividade e da aditividade-σ.
Teorema 5. Seja µ uma função de conjuntos sobre uma álgebra A. Então:
1. Se µ for aditiva-σ então:
µ(∅) = 0 .
2. Se µ for aditiva-σ então µ é aditiva.
3. Se µ for aditiva então:
∀A, B ∈ A A ⊂ B ⇒ µ(A) ≤ µ(B) .
4. Se µ for aditiva então:
∀A1 , . . . , An ∈ A
(∀m 6= n ⇒ Am ∩ An = ∅) ⇒ µ(
n
[
Ai ) =
i=1
n
X
µ(Ai ) .
i=1
5. Se µ for aditiva então verifica-se a propriedade de subaditividade, isto é,
∀A1 , . . . , An ∈ A µ(
n
[
i=1
58
Ai ) ≤
n
X
i=1
µ(Ai ) .
Medida, Probabilidade e Acontecimentos
Demonstração. Para a demonstração de 1 basta observar que sendo µ uma função aditiva
de conjuntos, existe A ∈ A tal que µ(A) < +∞. Seja a sucessão (Bn )n≥2 tal que Bn = ∅.
Uma vez que A = A ∪ ∪+∞
n=2 Bn , sendo A e Bn disjuntos, tem-se então pela aditividade-σ
que:
!
+∞
+∞
X
[
µ(∅) ,
µ(A) = µ A ∪
Bn = µ(A) +
n=2
n=2
o que só acontece, sendo µ(A) < +∞, se for µ(∅) = 0.
Para a demonstração de 2 considerem-se A, B ∈ A disjuntos e a sucessão definida,
indutivamente, por:
C1 = A , C2 = B , ∀n ∈ N (n ≥ 3 ⇒ Cn = ∅) .
Então, tem-se que para n ∈ N, Cn ∈ A,
+∞
[
Cn = A ∪ B ∈ A ,
n=1
e que para n 6= m, Cn ∩ Cm = ∅. Logo, dado que µ é aditiva-σ:
! +∞
+∞
+∞
[
X
X
µ(A ∪ B) = µ
Cn =
µ(Cn ) = µ(A) + µ(B) +
µ(∅) .
n=1
n=1
n=3
Como, por 1, µ(∅) = 0 vem µ(A ∪ B) = µ(A) + µ(B).
Demonstre-se a tese 3 do teorema, considerando A, B ∈ A tendo-se A ⊂ B. Como
B se pode particionar em dois conjuntos disjuntos pertencentes à álgebra A na forma
B = A ∪ (B − A), tem-se, pela aditividade,
µ(B) = µ(A) + µ(B − A) .
Como µ(B − A) ≥ 0 tem-se necessariamente µ(A) ≤ µ(B).
Para a tese 4 é suficiente uma demonstração por indução. Para a união de dois
conjuntos a propriedade verifica-se dado que µ é aditiva. Suponha-se verificada a hipótese
de indução:
!
n
n
[
X
∀A1 , . . . , An ∈ A (∀m 6= n ⇒ Am ∩ An = ∅) ⇒ µ
Ai =
µ(Ai ) .
i=1
i=1
Sejam agora B1 , . . . , Bn , Bn+1 ∈ A, conjuntos dois a dois disjuntos. Então os conjuntos
(B1 ∪, . . . , ∪Bn ) e Bn+1 são disjuntos pelo que pela aditividade:
µ ((B1 ∪ · · · ∪ Bn ) ∪ Bn+1 ) = µ(B1 ∪ · · · ∪ Bn ) + µ(Bn+1 ) .
P
como, pela hipótese de indução, µ((B1 ∪ · · · ∪ Bn ) = ni=1 Bi tem-se finalmente que:
µ ((B1 ∪ · · · ∪ Bn ) ∪ Bn+1 ) = µ(B1 ∪ · · · ∪ Bn ∪ Bn+1 ) =
n+1
X
µ(Bi ) .
i=1
59
Capı́tulo 2
Para a demonstração de 5 basta para observar que se A, B são quaisquer conjuntos da
álgebra, pode particionar-se a união A ∪ B na união A ∪ B = (A − B) ∪ (A ∩ B) ∪ (B − A)
de três conjuntos dois a dois disjuntos. Pela aditividade e pela monotonia:
µ(A ∪ B) = µ(A − B) + µ(A ∩ B) + µ(B − A) ≤ µ(A − B) + 2µ(A ∩ B) + µ(B − A) =
= µ((A − B) ∪ (A ∩ B)) + µ((B − A) ∪ (A ∩ B)) = µ(A) + µ(B) .
A generalização a um número finito de conjuntos é imediata pela associatividade da
reunião de conjuntos e por indução.
A definição seguinte é motivada pelo modelo de Kolmogorov para as probabilidades.
Definição 12. Seja (X, F) um espaço mensurável. Se µ for uma função aditiva
de conjuntos sobre F então ao trio (X, F, µ) chamaremos espaço de medida.
Se µ(X) = 1 ao trio (X, F, µ) chamaremos espaço de probabilidade e a µ
chamamos medida de probabilidade sobre (X, F).
Exercı́cio 55. Seja X = {a, b, c} e sobre este espaço F = σ({{a, b}}), isto é, a álgebra-σ gerada pelo
conjunto {a, b}.
1. Mostre que F = {∅, {a, b}, {c}, X}.
2. Seja a função de conjuntos definida por
∀A ∈ F µ(A) =
(
1
0
se A = {c}
se A = {a, b} .
Mostre que (X, F, µ) é um espaço de probabilidade.
3. Qual é a medida dos conjuntos {a} e {b}? Porquê?
A seguinte definição contempla uma situação que ocorre com frequência. Diz respeito
aos subconjuntos dos conjuntos de medida nula que podem não ser mensuráveis. Esses
conjuntos são considerados desprezáveis relativamente à medida. No entanto, sendo
subconjuntos de conjuntos de medida nula seria natural que tivessem também medida
nula.
60
Medida, Probabilidade e Acontecimentos
Definição 13. Seja (X, F, µ) um espaço de medida.
1. Um conjunto A ∈ P(X) diz-se µ-desprezável se e só se:
∃B ∈ F
(A ⊂ B ∧ µ(B) = 0) ,
isto é se A é subconjunto de um conjunto mesurável com medida nula.
Note-se que os conjuntos µ-desprezáveis não são, em geral, mensuráveis.
2. Seja P uma dada propriedade que os elementos do conjunto X podem,
ou não, ter escrevendo-se no primeiro caso P(x). A propriedade diz-se
válida µ-quase por toda a parte (µ-qptp) se e só se o conjunto:
{x ∈ X : P(x)}c ,
for µ-desprezável. Se (X, F, µ) for um espaço de probabilidade e a propriedade P for válida µ-qptp também se diz que a propriedade P é válida
µ-quase certamente (µ-qc).
3. O espaço de medida (X, F, µ) diz-se completo se e só se:
∀A ∈ P(X), B ∈ F
(A ⊂ B ∧ µ(B) = 0) ⇒ (A ∈ F ∧ µ(A) = 0) ,
isto é se todos os conjuntos µ-desprezáveis forem mensuráveis. Neste caso
os conjuntos µ-desprezáveis têm medida nula.
Observação 14. Verifica-se, sem dificuldade, que os conjuntos de medida µ nula são
conjuntos µ-desprezáveis. O exercı́cio 55 mostra que a recı́proca não é verdadeira uma
vez que os conjuntos µ-desprezáveis {a} e {b} não sendo mensuráveis não têm medida.
Exercı́cio 56. Seja (X, F, µ) um espaço de medida não completo.
1. Mostre que todo o subconjunto de um conjunto µ-desprezável é ainda µ-desprezável.
2. Mostre que se (Nn )n∈N for uma famı́lia numerável de conjuntos µ-desprezáveis então
um conjunto µ-desprezável.
S
n∈N
Nn é
O teorema seguinte mostra que é sempre possı́vel completar um espaço de medida de
forma a que todos os conjuntos inicialemente desprezáveis sejam também mensuráveis.
A ocorrência de um espaço de medida não completo pode assim ser corrigida com este
teorema, se tal for esse o nosso interesse.
61
Capı́tulo 2
Teorema 6 (Completação de um espaço de medida). Seja (X, F, µ) um espaço
de medida não completo. Seja N = N(µ) a classe dos subconjuntos de X que
são µ desprezáveis. Seja, por definição, a famı́lia de partes de X:
e := {A ∪ N : A ∈ F, N ∈ N}
F
e por:
e a correspondência de conjuntos definida para A ∪ N ∈ F
µ
e[A ∪ N ] := µ[A] .
e µ
e e, tal que, a
Então, (X, F,
e) é um espaço de medida completo tal que F ⊂ F
restrição de µ
e a F coincide com µ.
e é uma álgebra-σ. Como ∅ é µ-desprezável tem-se
Demonstração. Verifiquemos que F
e
e
que ∅ = ∅ ∪ ∅ ∈ F e X = X ∪ ∅ ∈ F; note-se que, com a mesma justificação, se tem que
e Seja E = A ∪ N , com A ∈ F e N ∈ N um elemento de F.
e Por definição de
F ⊂ F.
conjunto µ desprezável, existe B ∈ F tal que µ(B) = 0 e N ⊂ B. Uma representação
gráfica no plano com diagramas de Venn convencerá o leitor de que:
E c = (A ∪ N )c = (A ∪ B)c ∪ (B \ (N ∪ A)) .
e Seja (En )n∈N uma famı́lia
Como (A ∪ B)C ∈ F e (B − N ) ∈ N tem-se que E c ∈ F.
e Pela definição:
numerável de elementos de F.
∀n ∈ N ∃An ∈ F, Nn ∈ N En = An ∪ Nn .
S
S
Como n∈N An ∈ F, visto F ser álgebra-σ e pelo exercı́cio 56 se tem que n∈N Nn ∈ N,
pode concluir-se que:
!
!
[
[
[
e.
An ∪
Nn ∈ F
En =
n∈N
n∈N
n∈N
Verifiquemos que a correspondência µ
e define, de facto, uma função de conjuntos. Para
e não depende
tal basta-nos verificar que o valor que µ
e toma num dado conjunto E ∈ F
da decomposição deste na reunião de um elemento de F com um elemento de N. Sejam
então A1 , A2 ∈ F e N1 , N2 ∈ N tais que E = A1 ∪ N1 e E = A2 ∪ N2 . Tem-se então que:
µ
e(E) = µ
e(A1 ∪ N1 ) := µ(A1 ) ≤ µ(A1 ∪ A2 ) =: µ
e ((A1 ∪ A2 ∪ (N1 ∪ N2 )) =
=µ
e ((A1 ∪ N1 ∪ (A2 ∪ N2 )) = µ
e(E ∩ E) = µ
e(E) =
=µ
e(A2 ∪ N2 ) := µ(A2 ) .
Podemos pois concluir que µ(A1 ) ≤ µ(A2 ). Do mesmo modo se procederia para concluir
que µ(A2 ) ≤ µ(A1 ) podendo, em seguida, afirmar-se que µ
e está bem definida uma vez
que:
µ
e(E) = µ(A2 ) = µ(A1 ) .
62
Medida, Probabilidade e Acontecimentos
e eµ
Note-se que se E ∈ F então E = E ∪ ∅ ∈ F
e(E) = µ
e(E ∪ ∅) = µ(E), o que mostra
que a restrição de µ
e a F coincide com µ. Verifiquemos que µ
e é uma função de conjuntos
aditiva-σ. Tem-se que µ
e(∅) = µ
e(∅ ∪ ∅) = µ(∅) = 0 pelo que µ
e é uma função de conjuntos.
e
Seja (En )n∈N uma famı́lia numerável de elementos de F dois a dois disjuntos. Seja
(An )n∈N ∈ FN e (Nn )n∈N ∈ NN tais que
∀n ∈ N En = An ∪ Nn .
Uma vez que An ⊂ En para cada n ∈ N tem-se, necessariamente, que os elementos da
famı́lia (An )n∈N são dois a dois disjuntos. Em conclusão e dado que µ sendo medida é
aditiva-σ, podemos afirmar que:
!
µ
e(
[
n∈N
En ) = µ
e
[
n∈N
An
!
∪
[
n∈N
Nn
:= µ(
[
n∈N
An ) =
+∞
X
µ(An ) =:
n=0
+∞
X
µ
e(En ) ,
n=0
o que mostra que µ
e é aditiva-σ. Falta-nos verificar que o espaço de medida que conse um conjunto µ
truı́mos é completo. Consideremos N
e-desprezável e verifiquemos que este
e
e tal que µ
e ⊂ E. Por
conjunto é F-mensurável. Por definição, existe E ∈ F
e(E) = 0 e N
e pode escrever-se que E = A ∪ N , com A ∈ F e N ∈ N. Note-se que
definição de F
0=µ
e(E) = µ(A) e que:
e =N
e ∩E =N
e ∩ (A ∪ N ) = (N
e ∩ A) ∪ (N
e ∩ N) .
N
e ∩ A ⊂ A e µ(A) = 0 tem-se, por definição que N
e ∩ A é µ-desprezável. Pelo
Como N
e
exercı́cio 56 podemos concluir, primeiramente, que N ∩ N e µ-desprezável e finalmente,
e é µ-desprezável. Para finalizar basta observar que
pela decomposição acima, que N
e
todos os conjuntos µ-desprezáveis são F-mensuráveis.
A seguinte proposição é da maior importância por mostrar que se um dado conjunto
puder ser aproximado por uma sucessão crescente de conjuntos medı́veis então a medida
do conjunto é dada pelo limite da medida dos conjuntos aproximantes.
63
Capı́tulo 2
Proposição 7 (Continuidade inferior e superior da medida). Seja (X, F, µ) um
espaço de medida e (An )n∈N ∈ FN .
1. Se An ↑ A, isto é se,
∀n ∈ N An ⊆ An+1 ∧ A =
[
An ,
n∈N
tem-se então que:
µ(A) = lim ↑ µ(An ) .
n→+∞
2. Se se verifica que para n0 ∈ N se tem µ(An0 ) < +∞ e ainda, An ↓ A, ou
seja,
\
∀n ∈ N An+1 ⊆ An ∧ A =
An ,
n∈N
tem-se então que:
µ(A) = lim ↓ µ(An ) .
n→+∞
Demonstração. Suponhamos que existe n0 ∈ N tal que µ(An0 ) = +∞. Então para
n ≥ n0 virá, pela monotonia µ(An ) = +∞ pelo que o resultado anunciado segue imediatamente.
Suponhamos que para qualquer n ∈ N se tem que µ(An ) < +∞. A representação
gráfica, com diagramas de Venn, das hipóteses admitidas no primeiro enunciado sugerem
que se se considere a sucessão:
B0 := A0 , B1 := A1 − A1 , . . . , Bn := An − An−1 , . . .
É imediato verificar que (Bn )n∈N ∈ FN , que os elementos desta sucessão são dois a dois
disjuntos e que:
[
[
Bn =
An .
n∈N
n∈N
Assim, dado que µ é aditiva-σ,
! +∞
+∞
[
X
X
µ(A) = µ
Bn =
µ(Bn ) = µ(A0 ) +
µ(An − An−1 ) .
n=0
n∈N
n=1
Como se tem,
+∞
X
n=1
µ(An − An−1 ) =
lim
N →+∞
N
X
(µ(An ) − µ(An−1 )) =
n=1
vem, tal como se pretendia, que:
µ(A) = µ(A0 ) +
lim µ(AN ) − µ(A0 ) =
N →+∞
64
lim (µ(AN ) − µ(A0 )) ,
N →+∞
lim µ(AN )
N →+∞
Medida, Probabilidade e Acontecimentos
Para demonstrar a segunda tese da proposição podemos utilizar a primeira tese
considerando para n > n0 , por definição:
Cn := An0 − An .
É imediato verificar que (Cn )n∈N ∈ FN é uma sucessão crescente pelo que, em virtude
da primeira tese,
!
+∞
[
µ
lim
µ(Cn ) .
Cn =
n→+∞ , n>n0
n=n0 +1
Observe-se agora que, por um lado,
+∞
[
µ
+∞
[
!
Cn
=µ
+∞
[
!
(An0 ∩ Acn )
= µ An0 ∩
Acn
=
n=n0 +1
n=n0 +1
n=n0 +1
!c !
+∞
\
= µ(An0 ) − µ
!
An
n=n0 +1
e por outro,
lim
n→+∞ , n>n0
µ(Cn ) =
lim
n→+∞ , n>n0
µ(An0 − An ) = µ(An0 ) −
lim
n→+∞ , n>n0
µ(An ) .
Considerando que se tem, por hipótese, µ(An0 ) < +∞ pode concluir-se que:
+∞
\
µ
!
An
n=n0 +1
=
lim
n→+∞ , n>n0
µ(An ) ,
o que implica o resultado que se quer demonstrar uma vez que se verifica,
+∞
\
n=0
An =
+∞
\
An e
n=n0 +1
lim µ(An ) =
n→+∞
lim
n→+∞ , n>n0
µ(An ) ,
por ser a sucessão decrescente e pelas propriedades usuais dos limites.
Corolário 1. Seja µ uma função de conjuntos sobre uma álgebra A. Se µ for
aditiva-σ então verifica-se a propriedade de subaditividade-σ, isto é,
∀(An )n∈N ∈ A
N
+∞
[
n=0
An ∈ A ⇒ µ(
+∞
[
n=0
An ) ≤
∞
X
µ(An ) .
n=0
65
Capı́tulo 2
Demonstração. A demonstração do corolário 1 é consequência da tese 5 no teorema 5 e
da proposição 7 uma vez que que se pode dizer que:
∀N ∈ N µ(
N
[
An ) ≤
n=0
N
X
µ(An ) ≤
n=0
∞
X
µ(An ) .
n=0
Sendo (∪N
n=0 An )N ∈N uma sucessão crescente de conjuntos mensuráveis, com
+∞
[
(
N
[
An ) =
N =0 n=0
+∞
[
An ,
n=0
tem-se que:
µ
+∞
[
n=0
N
[
!
An
=
lim µ
N ⇒+∞
n=0
!
An
≤
lim
N ⇒+∞
N
X
µ(An ) ≤
n=0
∞
X
µ(An ) ,
n=0
tal como querı́amos demonstrar.
Uma primeira aplicação não trivial da continuidade das medidas é o seguinte resultado.
Lema 2 (Lema de Fatou). Seja (An )n≥1 uma sucessão de conjuntos mensuráveis num espaço de medida (XA, µ). Então:


+∞
[ \
µ
Am  ≤ lim inf µ(Am ) ,
n=1 m≥n
n→+∞ m≥n
ou seja, de uma forma mais sintética e fácil de reter:
µ(lim inf An ) ≤ lim inf µ(An )
n∈N
n∈N
Demonstração. Observe-se que, para cada n ≥ 1 a sucessão
\
Am = An ∩ An+1 ∩ . . .
m≥n
é crescente e que para m ≥ n se verifica sempre que ∩m≥n Am ⊆ Am tendo-se pela monotonia que para cada m ≥ n se verifica µ(∩m≥n Am ) ≤ µ(Am ) pelo que µ(∩m≥n Am ) ≤
inf m≥n µ(Am ). Atendendo á primeira observação efectuada e pela continuidade da medida vem agora que:






+∞
\
\
[ \
lim µ 
Am  = µ  lim
Am  = µ 
Am  ≤ lim inf µ(Am ) ,
n→+∞
m≥n
n→+∞
tal como querı́amos demonstrar.
66
m≥n
n=1 m≥n
n→+∞ m≥n
Medida, Probabilidade e Acontecimentos
2.3
Construção de medidas
Uma questão natural é a de saber em que condições gerais se podem obter medidas
sobre espaços mensuráveis. O teorema fundamental mostra que se partirmos de uma
função aditiva de conjuntos sobre uma álgebra de conjuntos então é possı́vel estender
essa função a uma álgebra-σ contendo a álgebra inicial de forma a que fique uma função
aditiva-σ de conjuntos.
Definição 14. Seja X um conjunto e I ⊂ P(X). A famı́lia I diz-se um sistemaπ se e só se:
∀A, B ∈ I A ∩ B ∈ I ,
isto é, I é fechado relativamente à operação de intersecção finita.
Observação 15. Note-se que uma álgebra ou uma álgebra-σ são necessariamente sistemasπ.
Os exercı́cios seguintes introduzem duas noções muito importantes para a construção
de espaços de medida.
Definição 15. Seja X um conjunto e D uma famı́lia de subconjuntos de S. Dizse que D é um sistema-d (em algumas obras também se usa a denominação
sistema-λ) sobre X se:
1. X ∈ D;
2. Se A, B ∈ D e A ⊆ B então B \ A ∈ D;
3. Se An ∈ D e An ↑ A, então A ∈ D.
Exercı́cio 57. Mostre que uma famı́lia Σ de subconjuntos de X é uma álgebra-σ se e só se Σ for um
sistema-π e um sistema-d.
Da mesma forma que foi feito para as álgebras e para as álgebras-σ é possı́vel definir
o sistema-d gerado por uma famı́lia arbitrária de conjuntos.
Definição 16. Supondo que C é uma famı́lia de subconjuntos de S, defina-se
d(C) como sendo a intersecção de todos os d-sistemas que contêm C. Isto é,
d(C) é o mais pequeno (no sentido da inclusão) d-sistema que contém C.
O exercı́cio seguinte é um resultado técnico importante. Tem como primeira aplicação
notável o teorema que se lhe segue sobre a unicidade de medidas.
Exercı́cio 58 (Lema de Dynkin). Suponha que I é um π-sistema.
67
Capı́tulo 2
1. Seja D1 := {B ∈ d(I) : B ∩ C ∈ d(I), ∀C ∈ I}. Verifique que D1 = d(I).
2. Defina-se o conjunto D2 := {A ∈ d(I) : B ∩ A ∈ d(I), ∀B ∈ d(I)}. Prove que D2 = d(I).
3. Conclua que d(I) = σ(I). Em consequência desta igualdade, vê-se que qualquer sistema-d que
contenha um π-sistema, contém a álgebra-σ gerada por esse sistema-π.
O teorema seguinte é de grande utilidade nos processos gerais de construção de
medidas.
Teorema 7 (Unicidade e sistemas-π). Seja X um conjunto e I ⊂ P(I) um
sistema-π. Sejam µ1 e µ2 duas medidas sobre (X, σ(I)) tais que µ1 (X) =
µ2 (X) < +∞ e ainda,
∀A ∈ I µ1 (A) = µ2 (A) .
Então, µ1 ≡ µ2 , isto é,
∀A ∈ σ(I) µ1 (A) = µ2 (A) .
Demonstração. Mostra-se que o conjunto D := {F ∈ Σ : µ1 (F ) = µ2 (F )} é um sistemad e aplica-se o exercı́cio 58. Com efeito, por hipótese tem-se que X ∈ D. Sejam agora
A, B ∈ D. Tem-se então que”:
µ1 (A \ B) = µ1 (A) − µ1 (B) = µ2 (A) − µ2 (B) = µ2 (A \ B) ,
pelo que A \ B ∈ D. Se tivermos uma sucessão de conjuntos não decrescente (An )n∈N∗ ∈
∗
DN , então pela continuidade da medida:




[
[
µ1 
An  = lim ↑ µ1 (An ) = lim ↑ µ2 (An ) = µ2 
An  ,
n≥1
n→+∞
n→+∞
n≥1
pelo que ∪n≥1 An ∈ D. Para tterminar observe-se que sendo D um sistema-d e tendose, por hipótese, que I ⊂ D o lema de Dynkin mostra que σ(I) ⊂ D permitindo-nos
concluir.
O teorema seguinte é um dos resultados fundamentais da teoria da medida. Com
efeito, este resultado mostra que conhecendo uma álgebra sobre um conjunto e uma
função de conjuntos aditiva-σ sobre essa álgebra, é sempre possı́vel estender essa função
de conjuntos á álgebra-σ gerada pela álgebra, de forma a que seja uma medida sobre
essa álgebra-σ. Fica ssim estabelecida uma metodologia geral para construir um espaço
de medida. Esta metodologiua será aplicada adiante para a construção das medidas de
Borel-Stieltjes sobre a álgebra-σ de Borel sobre R.
Teorema 8 (Extensão de Carathéodory). Seja X um conjunto, A uma álgebra
sobre X e µ uma função aditiva-σ sobre A. Existe então µ
e uma extensão de µ
a σ(A) tal que (X, σ(A), µ
e) é um espaço de medida.
68
Medida, Probabilidade e Acontecimentos
Demonstração. Veja-se [11][p. 195] para uma demonstração tecnicamente trabalhosa
deste teorema.
Exemplo 3. Por aplicação do teorema de extensão de Carathéodory ao conjunto, à
álgebra e à função aditiva-σ de conjuntos do exercı́cio 54 (veja-se também o exercı́cio 50)
obtem-se o espaço de Lebesgue sobre a recta real (R, L(R), λ) em que se a álgebra
σ de Borel de R for B = B(R) = σ(I(R)) então L(R) é a álgebra σ completada de
B(R) por aplicação do teorema 6 e λ é a extensão a L da função aditiva-σ definida no
exercı́cio 54.
Observação 16. Pode mostrar-se que o espaço de Borel, (R, B(R), λ), não é completo.
2.4
Construção das medidas de Lebesgue-Stieltjes sobre R
A famı́lia dos intervalos de R tem um conjunto de propriedades simples que pode ser
formalizado com as noções desenvolvidas a seguir. Note-se que em geral estudaremos
classes de subconjuntos de um conjunto arbitrário. Como exemplos importantes teremos
subconjuntos da recta real. No entanto, as noções expostas podem aplicar-se a situações
mais gerais e constituem um modo eficaz de estudar a construção de medidas em espaços
abstractos. Uma referência relevante para esta secção é [3].
2.4.1
Semi-anéis
Já estudámos um exemplo de famı́lia de subconjuntos de um conjunto dado com as
propriedades dadas na definição seguinte; trata-se da famı́lia dos intervalos de R. Seja
Ω um conjunto e S ⊆ P(Ω).
Definição 17. S é um semi-anel sobre Ω se e só se:
1. ∀A, B ∈ S
A∩B ∈S
2. ∀A,
SN B ∈ S
j=1 Cj
∃N ∈ N∗
∃C1 , . . . , CN ∈ S
2 a 2 disjuntos A \ B =
Esta definição tem as seguintes consequências de verificação imediata.
• S é estável por intersecção finita.
• ∅ ∈ S.
Exemplo 4. As seguintes famı́lias de conjuntos de R são semi-anéis.
• Sd = {[a, b[⊆ R; a ≤ b}
• Se = {]a, b[⊆ R; a ≤ b}
• J o conjunto dos intervalos de R .
69
Capı́tulo 2
O exercı́cio seguinte mostra como a partir de dois semi-anéis nos espaços factores se
pode construir um semi-anel no produto cartesiano desses factores. Esta construção é
importante para a definição das medidas sobre espaços produto cartesiano de espaços de
medida.
0
Exercı́cio 59. Verifique que Se , e J são semi-anéis sobre R. Considere S semi-anel sobre Ω e S semi0
anel sobre Ω . Sobre o produto cartesiano
n
o
0
0
0
0
Ω × Ω = (ω, ω ) : ω ∈ Ω , ω ∈ Ω ,
considere a famı́lia se subconjuntos deste produto cartesiano dada por:
0
0
0
0
S ⊗ S := {A × A : A ∈ S , A ∈ S }.
0
0
Mostre que S ⊗ S é um semi-anel sobre Ω × Ω .
Indicação: Mostre que:
0
0
0
0
(A × A ) ∩ (B × B ) = (A ∩ B) × (A ∩ B )
0
0
0
0
0
(A × A ) \ (B × B ) = (A − B) × A ∪ (A ∩ B) × (A − B )
2.4.2
Álgebras e Anéis
Na recta real, se considerarmos a famı́lia Il dos intervalos limitados tem-se que R ∈
/ Il .
Esta situação motiva a definição de anel de partes de um conjunto.
Definição 18. S é um anel
a
sobre Ω sse:
1. ∀A, B ∈ S A ∪ B ∈ S
2. ∀A, B ∈ S A \ B ∈ S
a
Anel boleano ou clan em Francês.
Observação 17. Pode definir-se um anel-σ como sendo uma famı́lia de conjuntos que
verifica:
1. ∀A1 , A2 . . . An , · · · ∈ S ∪+∞
n=1 An ∈ S
2. ∀A, B ∈ S A \ B ∈ S
Proposição 8. Seja S um anel sobre Ω
1. ∅ ∈ S
2. ∀A, B ∈ S A ∩ B ∈ S
3. ∀A, B ∈ S A∆B ∈ S
70
Medida, Probabilidade e Acontecimentos
Demonstração. A demonstração decorre das seguintes observações,
1. S 6= ∅ , A ∈ S ⇒ A \ A = ∅ ∈ S
2. A ∩ B = A ∪ B − ((A \ B) ∪ (B \ A))
3. A∆B = (A \ B) ∪ (B \ A)
e aplicando as condições da definição.
Proposição 9. Seja S um semi-anel de partes de Ω. Seja C o conjunto das
uniões finitas de elementos de S. Então C é um anel.
Demonstração. É simples verificar que C é fechado para as uniões finitas uma vez que a
união de duas uniões finitas é uma união finita. Sejam A, B ∈ S ; por hipótese tem-se
A=
n
[
Aj B =
j=1
m
[
Bk com Aj , Bk ∈ S .
h=1
Então, as operações elementares sobre conjuntos dizem-nos que:
"
#
n
m
n
m
n \
m
[
[
[
[
[
A\B =
Aj −
Bk =
Aj −
Bk =
(Aj − Bk )
j=1
k=1
j=1
k=1
j=1 k=1
Como S é semi-anel e Aj , Bk ∈ S tem-se que Aj − Bk é união finita de elementos de S e
logo está em C.
Verifique-se que C é estável por intersecções finitas:


!
n
m
[
[
[

Aj  ∩
Bk =
Aj ∩ B k .
j=1
k=1
(j,k)∈{1,...,n}×{1,...,m}
Como Aj ∩ Bk ∈ S, a conclusão é imediata.
Teorema 9. Seja S semi-anel de partes de Ω.
1. Então, C(S), o anel gerado por S, pode obter-se como o conjunto das
uniões finitas de elementos de S.
2. Tem-se também que C(S) é o conjunto de uniões finitas de elementos de
S dois a dois disjuntos.
71
Capı́tulo 2
Demonstração. Pode verificar-se sem dificuldade que C(S) é a intersecção de todos os
anéis contendo S. É um anel e é mais pequeno, no sentido da inclusão, contendo S.
Com as notações da proposição anterior queremos provar que C = C(S). Por definição
C(S) ⊆ C ; Se A ∈ C então por definição de C e por definição de anel, A ∈ C(S) logo
C ⊆ C(S) e C = C(S)
S
Seja A ∈ C(S), então A = nj=1 Aj com Aj ∈ S . Considere-se:
B1 = A1 , B2 = A2 − A1 , . . . , k ≥ 2, Bk = Ak −
k−1
[
Aj ,
j=1
T
isto é, Bk = k−1
j=1 (Ak − Aj ). Como Ak − Aj é união finita de elementos dois a dois
disjuntos de S logo temos o resultado pretendido.
Definição 19. Um anel C sobre X é um anel unitário se e só se X ∈ C.
Proposição 10. A é uma álgebra sobre X se e só se for um anel unitário.
Demonstração. É uma verificação imediata que decorre de se ter que Ac = X \ A e
A \ B = A ∩ Bc.
Exemplo 5. S(J) é uma álgebra uma vez que R =] − ∞, +∞[∈ J .
2.4.3
Funções Aditivas e Aditivas-σ de Conjuntos sobr um Semi-anel
Estendemos noções já estudadas ao caso em que as famı́lias de conjuntos medı́veis têm
apenas as propriedades de semi-anel.
Definição 20. Seja S semi-anel sobre Ω e µ : S 7−→ R+ uma função aditiva de
conjuntos.
1. µ é aditiva sobre S sse:
∀A, B ∈ S (A ∪ B ∈ S ∧ A ∩ B = ∅) ⇒ (µ(A ∪ B) = µ(A) + µ(B))
2. µ é aditiva-σ sobre S sse:
∀(An )n∈N
∞
[
!
An ∈ S ∧ (∀m,n∈N n 6= m ⇒ An ∩ Am = ∅)
n=1
 
⇒ µ 
∞
[
n=j
72

An  =
∞
X
n=0

µ(An ) .
⇒
Medida, Probabilidade e Acontecimentos
Proposição 11 (Monotonia). Seja S semi-anel sobre Ω e µ : S 7−→ [0, +∞]
função aditiva de conjuntos, então:
∀A, B ∈ S A ⊆ B ⇒ µ(A) ≤ µ(B) .
S
S
Demonstração. Temos B = (B \P
A) ∪ A = ni=1 (Ci ∪ A) = ni=1 Di conjuntos disjuntos.
Logo, pela aditividade µ(B) = ( ni=1 µ(Ci )) + µ(A), logo µ é monótona
Exemplo 6 (As medidas de Lebesgue-Stieltjes). Seja F uma função monótona crescente,
continua à direita. Seja Se = {]a, b] : a, b ∈ R , a ≤ b} e, por definição:
µF (]a, b]) := F (b) − F (a+ ) = F (b) − F (a).
Então µF é aditiva. Com efeito se ]a, b] e ]c, d] são elementos de Se tais que ]a, b]∩]c, d] = ∅
e se, por exemplo, b ≤ c então se ]a, b]∪]c, d] ∈ Se terá que ser b = c e ]a, b]∪]c, d] =]a, d]
e logo
µF (]a,b]∪]c, d]) = µF (]a, d]) = F (d) − F (a) = F (d) − F (c) + F (c) − F (a) =
= F (d) − F (c) + F (b) − F (a) = µF (]a, b]) + µF ([c, d])
Proposição 12 ([5]). Com as notações anteriores µF é aditiva-σ sobre Se .
S
Demonstração. Seja (]ak , bk ]) tal que ]a, b] = ∞
k=1 ]ah , bk ] sendo a famı́lia de intervalos
P
dois a dois disjuntos. É óbvio que µF (]a, b]) ≥ ∞
k=1 µF (]ak , bk ]) uma vez que:
!
N
N
X
[
∀N ∈ N∗ µF (]a, b]) ≥(a) µF
µF (]ak , bk ]) ,
]ak , bk ] =
k=1
h=1
em que a desigualdade (a) se justifica porque a aditividade implica a monotonia. Seja
agora > 0. Como F é contı́nua à direita tem-se que existem δ, δ1 , . . . , δk , · · · ∈ R∗+ tais
que F (aS+ δ) − F (a) < e F (bk + δk ) − F (bk ) < /2k . Observe-se que: [a + δ, b] ⊂
]a , b + δk [ . Pelo facto de [a + δ, b] ser compacto existe N ∈ N∗ tal que
]a, b] ⊆ ∞
k=1
SN k k
[a + δ, b] ⊆ k=1 ]ak , bk + δk [. Tem-se então que
µF (]a + δ, b]) ≤ µF ([a + δ, b]) ≤(b)
N
X
µF (]ak , bk + δk ]) ≤(c)
k=1
≤
N
X
N X
k=1
µF (]ak ; bk ]) +
≤
2k
!
µF (]ak , bk ])
+,
k=1
em que (b) se justifica pela subaditividade e (c) porque
]ak , bk + δk ] =]ak , bk ]∪]bk , bk + δk ] .
73
Capı́tulo 2
Por outro lado, ]a, b] =]a, a + δ]∪]a + δ, b], logo
µF (]a, b]) = µF (]a + δ, b]) + µF (]a, a + δ]) = µF (]a + δ, b]) + F (a + δ) − F (a) ≤
!
!
N
∞
X
X
≤
µF (]ak , bk ]) + 2 ≤
µF (]ak , bk ]) + 2
k=1
k=1
o que permite concluir a demonstração.
Observação 18. No caso em que para se tem F (x) = x, para x ∈ R, iremos obter a
medida de Lebesgue sobre Se .
2.4.4
Extensões de funções aditivas e aditivas-σ
As funções aditivas e aditivas-σ sobre semi-anéis podem-se prolongar com essas propriedades aos anéis (às álgebras) geradas por esses semi-anéis.
Seja agora µ uma função aditiva sobre um semi-anel S. Seja µ̃ Sa extensão natural
de µ a C(S), o anel gerado por S, definida para A ∈ C(S) com A = nj=1 Aj dois a dois
disjuntos e Aj ∈ S, por:
n
n
X
X
µ̃(A) :=
µ̃(Aj ) =
µ(Aj )
j=1
j=1
Proposição 13. µ̃ está bem definida e é uma função aditiva sobre C(S).
S
Demonstração. Verifiquemos que µ̃ está bem definida. Seja A = m
k=1 Bk uma outra
decomposição de A como união disjunta de elementos de S . Como


!
n
m
n [
m
[
[
[
A=A∩A=
Aj  ∩
Bk =
(Aj ∩ Bk ) ⊂ Aj ∩ Bk ∈ S ,
j=1
j=1 k=1
k=1
tem-se que
µ̃(A) =
n
X
µ̃(Aj ) =
j=1
=(d)
n X
m
X
j=1 k=1
n
X
µ̃(Aj ∩ A) =
j=1
µ(Aj ∩ Bk ) =(e) =
n
X
j=1
m
X
µ̃ Aj ∩
m
[
h=1
!
Bk
=
n
X
j=1
µ̃
m
[
!
(Aj ∩ Bk )
=
k=1
µ̃(Bk )
k=1
em que (d) e (e) se justificam por µ̃ ser aditiva em S. Logo µ̃ está bem definida e a
aditividade é apenas uma verificação trivial. Com efeito, seja A = ∪ni=1 Ai e B = ∪m
j=1 Bj
dois elementos de C(S), isto é duas uniões disjuntas de elementos de S. Note-se que
podemos supor que A e B são disjuntos pois, caso contrário, considerarı́amos A ∪ B =
74
Medida, Probabilidade e Acontecimentos
(A \ B) ∪ (A ∩ B) ∪ (B \ A) que é uma reunião finita de elementos do anel disjuntos dois
a dois. Então se definirmos para k ∈ {1, . . . , m + n}:
(
Ak
k = 1, . . . , n
Ck =
Bk−n k = n + 1, . . . , n + m ,
teremos que (Ck )1≤k≤m+n é uma sequência de conjuntos dois a dois disjuntos tais que:
! m+n
m+n
n
m+n
[
X
X
X
µ̃(A ∪ B) = µ̃
Ck =
µ(Ck ) =
µ(Ak ) +
µ(Bk−n ) = µ̃(A) + µ̃(B) ,
k=1
k=1
k=1
k=n+1
tal como se pretendia demonstrar.
Proposição 14. Com as notações anteriores, se µ for aditiva-σ sobre S então
µ̃ é aditiva-σ sobre C(S).
Demonstração. Seja A = ∪+∞
n=1 An uma união disjunta de elementos de C(S). Sabemos
que é sempe possı́vel representar cada elemento A e An de C(S) como uma união finita
pn
n
n
n
de elementos de S, ou seja, A = ∪m
j=1 Bj e An = ∪i=1 Ai . Defina-se agora Ci,j := Ai ∩Bj .
n e A = ∪ C n tem-se então que:
Uma vez que Bj = ∪n,i Ci,j
n
i,j i,j
µ̃(A) =(a)
m
X
µ(Bj ) =(b)
j=1
m X
X
j=1 n,i
n
µ(Ci,j
) =(c)
+∞ X
X
n
µ(Ci,j
) =(d)
n=1 j,i
+∞
X
µ(An ) ,
n=1
em que (a) resulta da definição, (b) de µ ser aditiva-σ, (c) resulta de da série ser a
termos positivos e, por isso, comutativamente convergente e (d) das somas em i e j
serem finitas.
O corolário seguinte mostra como construir a medida associada a uma função monótona crescente contı́nua à direita. Inicialmente e por aplicação do teorema de Carathéodory, temos a possibilidade de construir uma sucessão de medidas sobre a sucessão
de intervalos encaixados (]n, n + 1])n∈Z munidos das respectivas álgebras-σ de Lebesgue.
Note-se que, para cada n ∈ Z, o anel gerado por
Sne = {]a, b]; a, b ∈ R n < a ≤ b ≤ n + 1}
é, de facto, uma álgebra.
Corolário 2. Com as notações usadas anteriormente e para cada n ∈ Z, µF é
aditiva-σ sobre a álgebra gerada por Sne = {]a, b]; a, b ∈ R n < a ≤ b ≤ n + 1}.
Em consequência, existe µ̃nF , a medida de Lebesgue-Stieltjes associada a F ,
extensão de µF a σ(Sne ) = L(]n, n + 1]) que é aditiva-σ, isto é, tal que µ̃nF é uma
medida sobre L(]n, n + 1]).
75
Capı́tulo 2
Demonstração. É uma aplicação do teorema de extensão de Carathéodory (veja-se o
teorema 8 ou [11, p. 20]).
Podemos agora definir uma medida sobre o conjunto dos números reais ”colando”as
medidas µ̃nF sobre cada um dos intervalos ]n, n + 1].
Proposição 15. Seja agora sobre L(R) a função de conjuntos definda por:
+∞
X
∀A ∈ L(R) µ̃F (A) :=
µ̃nF (A∩]n, n + 1]) .
n=−∞
Tem-se que (R, L(R), µ̃F ) é um espaço de medida denominado o espaço de medida de Lebesgue-Stieltjes associado a F .
Demonstração. Considerando a extensão natural da noção de soma de série aos caso em
que os ı́ndices variam em Z, tem-se que:
µ̃F (∅) =
lim
N →+∞
n=+N
X
µ̃nF (∅ ∩]n, n + 1]) = 0 .
n=−N
Seja agora uma sucessão (Ap )p≥1 de elementos de L(R) dois a dois disjuntos. Note-se
que tendo-se Ap , ]n, n + 1] ∈ L(R) então 3 :
Ap ∩]n, n + 1] ∈ L(]n, n + 1]) =(a) {B ⊆]n, n + 1] : B ∈ L(R)}
Então, pela definição de µ̃F tem-se que:

µ̃F 
+∞
[

Ap  =
+∞
X

µ̃nF ]n, n + 1] ∩
n=−∞
p=1
=
+∞ X
+∞
X
=

µ̃nF (]n, n + 1] ∩ Ap ) =

+∞
X
Ap  =
p=1
n=−∞ p=1
+∞
X
+∞
[
µ̃nF 
n=−∞
+∞
X
+∞
X
p=1
n=−∞
+∞
[

]n, n + 1] ∩ Ap  =
p=1
!
µ̃nF (]n, n + 1] ∩ Ap )
=
µ̃F (Ap ) .
p=1
Note-se que a terceira igualdade, a contar da esquerda resulta de µ̃nF ser uma medida e,
por isso, aditiva-σ e que a quarta é consequência de estarmos em presença de séries a
termos não negativos pelo que se pode trocar a ordem pela que efectuamos as somas.
3
76
Veja-se, para a segunda igualdade o exercı́cio correspondente das aulas práticas.
Medida, Probabilidade e Acontecimentos
2.5
Resoluções
Resolução:[Exercı́cio 57] É óbvio que uma álgebra-σ verifica as propriedades de definição tanto de um sistema-π como de um sistema-λ. Seja agora Σ verificando os
axiomas relativos às noções de sistema-π como de um sistema-λ. Pelo primeiro e pelo
segundo axioma tem-se que X \ X = ∅ ∈ Σ. Os mesmos axiomas implicam que se A ∈ Σ
então X \ A ∈ Σ. Em consequência desta última observação tem-se que a união finita
de elementos de Σ pertence a Σ. Com efeito, para dois conjuntos A, B ∈ Σ tem-se que
A ∪ B = (Ac ∩ B c )c ∈ Σ, dado que a intersecção de dois quaisquer conjuntos de Σ está
em Σ; uma aplicação do princı́pio de indução justifica esta afirmação. Para concluirmos,
∗
seja (An )n∈N∗ ∈ ΣN e considere-se para cada n ≥ 1, Bn := ∪np=1 An . Verifica-se sem
dificuldade que (Bn )n∈N∗ é uma sucessão crescente de elementos de Σ que converge para
∪n≥1 An ∈ Σ pela terceira propriedade dos sistemas-λ.
♦
Resolução:[Exercı́cio 58] Verifique-se que D1 = d(I). Por definição, D1 ⊆ d(I). Verificase sem dificuldades que D1 é um sistema-d. O segundo axioma, o menos imediato a
verificar, resulta de se ter para A, B ∈ D1 e C ∈ d(I) :
(A ∩ C) \ (B ∩ C) = (A \ B) ∩ C .
(2.1)
Para concluir, basta observar que I ⊂ D1 uma vez que I ⊂ d(I) e que I é um sistema-π.
Para verificar que D2 = d(I) pode seguir-se a mesma metodologia. Por definição,
também, D2 ⊆ d(I). A verificação do segundo axioma faz-se, também, recorrendo à
fórmula 2.1. Seja agora I ∈ I ⊂ d(I). Seja B ∈ d(I) = D1 qualquer, em que a igualdade
resulta da primeira alı́nea. Pela propriedade que define D1 tem-se que B ∩ I ∈ d(I), o
que mostra que I ∈ D2 . Conclui-se tal como na primeira alı́nea.
A terceira alı́nea do exercı́cio resulta de que, em consequência da segunda alı́nea,
d(I) é um sistema-pi. Tal resulta da igualdade D2 = d(I) e da propriedade de definição
de D2 . Pelo exercı́cio 57, temos que d(I) sendo um sistema-d e um sistema-π é uma
álgebra-σ, logo σ(I) ⊂ d(I). Como por outro lado se tem sempre que
I ⊂ σ(I) ⇒ d(I) ⊂ σ(I) ,
dado que σ(I) é um sistema-d, tem-se a igualdade.
2.6
♦
Comentários sobre as referências bibliográficas
O texto que desenvolvemos acima é apenas uma introdução ao tema da teoria da medida
que nos permitirá prosseguir, com uma segurança mı́nima, no estudo da integração
segundo Lebesgue e no desenvolvimento da teoria moderna das probabilidades. O leitor
desejoso de aprofundar este tema pode socorrer-se da obras referidas abaixo. Referimos
em particular sobre o tema dominante deste capı́tulo as seguintes obras.
Em [4] poderá encontrar uma exposição da construção das medidas sobre os anéis-σ
gerados por semi-anéis.
77
Capı́tulo 2
Em [1] pode ler-se uma construção muito expedita das medidas sobre álgebras-σ
geradas por álgebras de conjuntos.
Em [6] encontrar-se-á uma construção das medidas sobre sobre álgebras-σ geradas
por álgebras de conjuntos bem como a extenão ao cao em que a álgebra-σ é gerada por
um anel.
2.7
[1]
Exercı́cios
Exercı́cio 60. Consideremos o conjunto A = {a, b, c, d}. Indique, justificando, quais dos seguintes
subconjuntos de P(A) são σ-álgebras.
1. {{a}, {b}, {c}, {d}, {a, b}, {a, c}, {a, d}, {b, c}, {b, d}, {c, d},
{a, b, c}, {a, b, d}, {a, c, d}, {b, c, d}, {a, b, c, d}, ∅}.
2. {{a}, {c}, {b, c, d}, {a, b, d}, {a, c}, {a, b, c, d}, ∅}.
3. {{a}, {b}, {c}, {d}, {a, b}, {a, c}, {a, d}, {b, c}, {b, d}, {c, d},
{a, b, c}, {a, b, d}, {a, c, d}, {b, c, d}, {a, b, c, d}}.
4. {{b}, {b, c}, {a, c, d}, {a, d}, {c}, {a, b, c, d}, ∅}.
5. {{a, b}, {c, d}, {a, b, c, d}, ∅}.
[1]
Exercı́cio 61. Determine as σ-álgebras geradas por
1. os conjuntos do exercı́cio anterior que não são σ-álgebras.
2. {{i ∈ N : i é impar}}, em N.
3. {{a, b}, {c, d}, {a, b, c, d}, ∅}, em A = {a, b, c, d}.
4. {{a}, {b}, {c}}, em A = {a, b, c, d}.
[1]
Exercı́cio 62. Consideremos em A = {a, b, c, d} as seguintes σ-álgebras
F1 = {{a}, {c}, {b, c, d}, {a, b, d}, {a, c}, {b, d}{a, b, c, d}, ∅}
F2 = {{b}, {b, c}, {a, c, d}, {a, d}, {a, b, d}, {c}, {a, b, c, d}, ∅}.
Justifique se F = F1 ∩ F2 é uma σ-álgebra. Em caso afirmativo determine F.
[1]
Exercı́cio 63. Seja X um conjunto e F uma σ-álgebra em X. Prove que na definição de σ-álgebra a
condição
[
∀ {An }n∈N ∈ FN ,
An ∈ F
n∈N
é equivalente a
∀ {An }n∈N ∈ FN ,
\
An ∈ F.
n∈N
[1]
Exercı́cio 64. Verifique que
[a, b] =
∞ \
n=1
[1]
a−
1
1
,b +
.
n
n
Exercı́cio 65. Mostre que qualquer σ-álgebra de subconjuntos de R que contenha todos os intervalos
abertos também contém todos os intervalos fechados (veja, a tı́tulo de indicação, o exercı́cio 64 ou observe
que [a, b] = (] − ∞, a[∪]b, +∞[)c e que ] − ∞, a[= ∪∞
n=1 ]a − n, a[).
78
Medida, Probabilidade e Acontecimentos
[1]
Exercı́cio 66. Verifique que
]a, b[=
∞ h
[
n=1
[1]
a+
εi
ε
,b −
n
n
sendo
ε=
b−a
.
2
Exercı́cio 67. Mostre que qualquer σ-álgebra de subconjuntos de R que contenha todos os intervalos
fechados também contém todos os intervalos abertos (veja, a tı́tulo de indicação, o exercı́cio 66).
[2]
Exercı́cio 68. Seja (Hi )i∈I uma qualquer famı́lia de σ-álgebras em Ω. Mostre que
\
H=
Hi ,
i∈I
é também uma σ-álgebra em Ω.
Exercı́cio 69. [7]. Seja X um conjunto e E = {A} a famı́lia de partes de X reduzida ao subconjunto [1]
A de X. Mostre que a álgebra-σgerada por E é exactamente:
σ(E) = {∅, A, Ac , X} .
Indicação : A σ-álgebra gerada é minimal no sentido da inclusão.
Exercı́cio 70. [7]. Seja X um conjunto não numerável. Mostre que a σ-álgebra gerada pela famı́lia [2]
dos conjuntos singulares em X, é a σ-álgebra dos conjuntos que, ou são numeráveis ou têm como
complementar um conjunto numerável, i.e.:
σ(({x})x∈X ) = {A ⊆ X : card(A) ≤ ℵ0 ∨ card(Ac ) ≤ ℵ0 } .
Indicação : Relembre que o conjunto união de uma famı́lia numerável de conjuntos numeráveis é
ainda um conjunto numerável.
Exercı́cio 71. [2]. Seja X um conjunto e E = (An )n∈N uma partição numerável do conjunto X. [2]
Determine a forma geral de um elemento de σ(E), a σ-álgebra gerada por E.
Indicação : Relembre que a intersecção de dois quaisquer elementos distintos de uma partição é o
vazio, que a união de todos os elementos é o conjunto inteiro e que estas propriedades caracterizam as
partições.
Exercı́cio 72. Seja X um conjunto, (Xn )n∈N uma sucessão de partes de X duas a duas disjuntas. [3]
Seja para cada n ∈ N uma σ-álgebra sobre Xn , a que chamaremos Cn . Mostre que se se tiver:
[
S
Y =
Xn
D = D ⊆ Y : ∀n ∈ N ∃An ∈ Cn D = n∈N An ,
n∈N
então (Y, D) é um espaço mensurável.
Indicação : Para a verificação do segundo axioma deverá determinar a forma geral do complementar
em Y de um subconjunto de Y .
[3]
Exercı́cio 73. Seja Z o conjunto dos inteiros relativos. Mostre que a famı́lia
C = {A ⊆ Z : ∀n ≥ 1, (2n) ∈ A ⇔ (2n + 1) ∈ A} ,
é uma σ-álgebra sobre Z.
Indicação : Procure determinar a forma geral de um elemento de C.
79
Capı́tulo 2
0
Exercı́cio 74. Seja X um conjunto e seja C uma σ-álgebra de partes de X. Seja X um subconjunto [1]
de X. Mostre que:
0
0
C = CX 0 = {A ∩ X : A ∈ C} ,
0
é uma σ-álgebra sobre X .
1. Seja (R, B(R) o espaço de Borel sobre a recta real. Determine B(R)N .
2. Seja (R, B(R)) o espaço de Borel sobre a recta acabada. Determine B(R)R .
0
Definição 21. Á σ-álgebra CX 0 chamamos σ-álgebra traço de C sobre X .
[2]
Exercı́cio 75. Com as mesmas notações do exercı́cio anterior mostre que, se para E se tem, C = σ(E)
i.e. C é a σ-álgebra gerada por E, então se chamarmos a :
0
EX 0 = {A ∩ X : A ∈ E} ,
0
o traço da famı́lia E sobre X , ( que não é necessáriamente uma σ-álgebra, visto que E não o é) temos
que: CX 0 = σ(EX 0 ).
Indicação : Relembre a propriedade de minimalidade da σ-álgebra gerada por uma famı́lia de partes
de um conjunto.
[1]
Exercı́cio 76. [7]. Sejam X e Y dois conjuntos, f ∈ Y X e C uma σ-álgebra sobre Y . Mostre que a
imagem recı́proca de C por f (i.e. {f −1 (C) : C ∈ C}) é uma σ-álgebra sobre X. Deduzir o primeiro
resultado do exercı́cio anterior.
Indicação : Verifique os axiomas usando os resultados dos exercı́cios da secção (1).
[1]
Exercı́cio 77.
X, mostre que
σ-álgebra sobre
Indicação
[3]
Exercı́cio 78. Seja X um conjunto e A uma σ-álgebra sobre X. Seja também X ⊆ X um subconjunto
0
de X. Considere AX 0 o traço de A em X . Mostre que são equivalentes as seguintes propriedades:
Com as mesmas notações do exercı́cio anterior, e supondo que D é uma σ-álgebra sobre
{B ⊆ Y : f −1 (B) ∈ D} é uma σ-álgebra sobre Y . Deduzir que a ”elevação”de uma
0
0
X a X (onde X , é um subconjunto de X) é uma σ-álgebra sobre X.
: Para a segunda parte, aplique o resultado da primeira, à injecção canónica.
0
0
X ∈A
0
e AX 0 = {A ⊆ X : A ∈ A} .
Indicação : Verifique que o conjunto explicitado na segunda proposição é de facto uma σ-álgebra.
[2]
Exercı́cio 79. Sejam X e Y dois conjuntos e f ∈ Y X uma aplicação. Seja E uma famı́lia de partes
de Y . Mostre que se tem então:
f −1 (σ(E)) = σ(f −1 (E)) .
Indicação : A demonstração um pouco elaborada deste facto extremamente útil pode encontrar-se
em [3].
80
Medida, Probabilidade e Acontecimentos
[2]
Exercı́cio 80. Este exercı́cio e os seguintes são fundamentais, na medida em que caracterizam a σálgebra de Borel em R, através de quatro das suas classes geradoras fundamentais. Os resultados a
seguir formulados, serão de uma utilização constante.
Mostre que são iguais entre si as σ-álgebra sobre R geradas pelas quatro classes fundamentais de
subconjuntos de R.
E1 = {]a, b[ : a, b ∈ R a < b}
E2 = {]a, +∞[ : a ∈ R}
E3 = {]a, b[ : a, b ∈ Q a < b}
E4 = {]a, +∞[ : a ∈ Q}
Indicação : Utilize a minimalidade da σ-álgebra gerada por uma famı́lia de partes. Poderá mostrar,
que um elemento arbitrário de uma classe, se pode exprimir como união ou intersecção numerável, de
elementos de outra classe.
Exercı́cio 81. Mostre que a σ-álgebra de Borel sobre R é exactamente a σ-álgebra gerada pela classe [3]
E3 . (é também a σ-álgebra gerada por uma qualquer das outras três classes).
Indicação : Relembre que qualquer aberto de R é união numerável de intervalos abertos com extremidades racionais.
Exercı́cio 82 (Problema de revisão). Este exercı́cio seguinte descreve a estrutura da mais pequena σálgebra sobre o espaço de partida gerada por uma famı́lia de aplicações tomando valores num espaço
mensurável. Caso o espaço de partida seja um espaço métrico, o espaço de chegada seja um espaço de
Borel e as aplicações sejam contı́nuas, então a σ-álgebra associada é simplesmente a σ-álgebra de Borel
no espaço de partida.
Seja X um conjunto, (Y, θ) um espaço mensurável e F = (fi )i∈I uma famı́lia de aplicações de X [3]
em Y . Considere a famı́lia de partes de Xdefinida por:
[ −1
U=
fi (θ) = {A ⊆ X : ∃i ∈ I ∃B ∈ θ A = fi−1 (B)} .
i∈I
1. Mostre que U não é em geral uma σ-álgebra.
2. Considere:
A = A(F, θ) = σ(U) .
Mostre que qualquer aplicação fi é mensurável de (X, A) em (Y, θ).
3. Sendo T uma σ-álgebra sobre X para a qual toda e qualquer função fi seja mensurável de (X, T)
em (Y, θ), mostre que se tem então A ⊆ T.
4. No caso particular em que se tem Y = R e θ a σ-álgebra boreliana respectiva (para a topologia usual, é claro), se tivermos ainda E ⊆ P(X) e a famı́lia F = (1E )E∈E , então ter-se-à
necessariamente: A(F, θ) = σ(E).
5. Suponha que (X, τ1 ) e (Y, τ2 ) são dois espaços topológicos, e que B1 e B2 são as respectivas
σ-álgebras Borelianas. Mostre que se F é a famı́lia de todas as aplicações contı́nuas de (X, τ1 )
em (Y, τ2 ), então temos:
A(F, B2 ) ⊆ B1 .
6. Suponha, para terminar, que τ1 é uma topologia definida a partir de uma métrica. Mostre que
então é certo que:
A(F, B2 ) = B1 .
Exercı́cio 83. [7]. Chamaremos recta acabada ao conjunto R que se obtem juntando a R, o conjunto
dos números reais ou recta real, dois objectos distintos, (que não sejam números reais), +∞ e −∞
(respectivamente mais infinito e menos infinito), de tal forma que:
81
Capı́tulo 2
1. A relação de ordem total usual sobre R, <, se possa estender a uma relação de ordem total <
sobre R, de modo a que se verifiquem as propriedades:
∀x ∈ R − ∞<x< + ∞
∀x, y ∈ R x ≤ y ⇔ x≤y .
2. As operações definidas na estrutura de corpo usual de R, admitem extensões naturais para as quais
valem as seguintes regras de cálculo:
∀x ∈ R (±∞) + (±∞) = x + (±∞) = (±∞) + x = ±∞
(±∞)(±∞) = +∞ (±∞)(∓∞) = −∞
∀x ∈ R x(±∞) = (±∞)x = ±∞ , 0 , ±∞ se x > 0 , x = 0 , x < 0 , resp.
3. Consideramos definida sobre R uma topologia τu que faz de (R, τu ) um espaço topológico compacto no qual (R, τu ) se injecta com uma imagem densa. ( Ver curso de topologia ou considerar
esta afirmação como um exercı́cio).
[3]
Atenção : Dado que não definiremos (±∞) + (∓∞) não estará nunca em questão uma estrutura
algébrica sobre a recta acabada que prolongue a de grupo aditivo comutativo que habitualmente conhecemos
sobre a recta real.
Considere a famı́lia de partes de R definida por:
A = {B, B ∪ {+∞}, B ∪ {−∞}, B ∪ {−∞, +∞} : B ∈ B} ,
onde B é a σ-álgebra de Borel sobre a recta real.
1. Mostre que A é uma σ-álgebra sobre R.
2. Suponha que E é uma famı́lia de partes tal que σ(E) = B. Considere A1 , a σ-álgebra sobre R,
gerada por E ∪ {{+∞}}. Mostre que se tivermos R ∈ A1 então temos certamente que : A1 = A.
Indicação : Para a alı́nea 2, use um exercı́cio anterior. O exercı́cio que vai seguir-se descreve
precisamente a estrutura da σ-álgebra de Borel, associada à topologia usual, sobre a recta acabada.
[3]
Exercı́cio 84. [7]. Com as mesmas notações do exercı́cio anterior, mostre que:
1. A σ-álgebra A sobre R, é gerada pela classe:
{]a, +∞[: a ∈ R} ∪ {{+∞}} .
2. A σ-álgebra A sobre R, é também gerada pela classe:
{]a, +∞] : a ∈ R} .
3. A σ-álgebra A sobre R, é gerada pela classe:
{]a, b[: a, b ∈ R a < b} ∪ {{+∞}} .
4. A σ-álgebra A sobre R, é gerada pela classe:
B(R) ∪ {{+∞}} ∪ {{−∞}} .
5. B(R) = {B ∈ A : B ⊆ R} .
6. B(R) é a σ-álgebra traço de A em R.
7. A σ-álgebra A é a elevação de B a R, isto é:
A = {B ⊆ R : B ∩ R ∈ B(R)} .
Indicação : Utilize os resultados obtidos nos exercı́cios anteriores. O exercı́cio seguinte descrevenos precisamente a topologia usual da recta acabada e a relação desta com a topologia usual da recta
real.
82
Medida, Probabilidade e Acontecimentos
[3]
Exercı́cio 85. [7]. Considere a famı́lia de partes de R definida por:
τ (R) = {ω, ω ∪ [−∞, x[, ω∪]y, +∞], ω ∪ [−∞, x[∪]y, +∞] : ω ∈ τu x, y ∈ R} ,
em que τu é a topologia usual da recta real. Seja B(R) a σ-álgebra gerada pela famı́lia τ (R).
1. Mostre que A = B(R). Onde A é a σ-álgebra definida no exercı́cio anterior.
2. Mostre que a topologia usual sobre a recta real é o traço na recta real da topologia τ (R), topologia
esta que é exactamente a topologia usual sobre a recta acabada que mencionámos na definição.
Exercı́cio 86. Seja S um conjunto e D uma famı́lia de subconjuntos de S. Diz-se que D é um d-sistema [2]
sobre S se:
1. S ∈ D;
2. Se A, B ∈ D e A ⊆ B então B\A ∈ D;
3. Se An ∈ D e An ↑ A, então A ∈ D.
Mostre que uma famı́lia Σ de subconjuntos de S é uma σ-álgebra se e só se Σ for um π-sistema e um
d-sistema.
Exercı́cio 87. Supondo que C é uma famı́lia de subconjuntos de S, defina-se d(C) como sendo a in- [2]
tersecção de todos os d-sistemas que contêm C. Isto é, d(C) é o mais pequeno d-sistema que contém
C.
Suponha que I é um π-sistema.
1. Seja D1 := {B ∈ d(I) : B ∩ C ∈ d(I), ∀C ∈ I}. Verifique que D1 = d(I).
2. Defina-se o conjunto D2 := {A ∈ d(I) : B ∩ A ∈ d(I), ∀B ∈ d(I)}. Prove que D2 = d(I).
3. Mostre que d(I) = σ(I). E observe que qualquer d-sistema que contenha um π-sistema, contém a
σ-álgebra gerada por esse π-sistema.
2.7.1
Medidas Positivas
Exercı́cio 88. Seja (X, F) um espaço mensurável, e a ∈ X fixo. Defina-se δa : F → R̄ por:
1 se a ∈ F
∀F ∈ F δa (F ) =
.
0 se a 6∈ F
[1]
Mostre que δa é uma medida sobre (X, F).
Exercı́cio 89. Seja (X, 2X ) o espaço mensurável maximal sobre o conjunto infinito X. Seja a aplicação [1]
µ definida em 2X (e tomando valores em R), por: µ(A) = card(A) se o subconjunto A de X fôr um
conjunto finito e µ(A) = +∞ se se produzir a hipótese contrária. Mostre que µ é uma medida sobre
(X, 2X ) a que chamaremos medida de contagem.
Indicação : Distinga o caso em que a união da famı́lia de partes duas a duas disjuntas é finita, do
caso em que não é.
Exercı́cio 90. Seja X um conjunto infinito e seja R a seguinte famı́lia de subconjuntos de X: A ∈ R [1]
se e só se A é finito ou Ac é finito. Seja µ a seguinte função em R: µ(A) = 0 se A é finito, e µ(A) = 1
se Ac é finito. Será R anel? E será µ medida?
83
Capı́tulo 2
Exercı́cio 91.
1. Seja X um conjunto infinito e R a familia de todos os subconjuntos numeráveis [1]
de X. Será R um σ-anel?
2. Seja µ uma medida em R. Mostre que existe uma função f : X −→ [0, ∞[ tal que
X
µ(A) =
f (x)
x∈A
para todo A ∈ R.
[2]
Exercı́cio 92. Seja X a recta real e R = RLeb . (Ou seja, uniões finitas de intervalos.) Dado A ∈ R
seja µ(A) = 1 se, para algum ε positivo, A contêm o intervalo ]0, ε[. Caso contrário µ(A) = 0. Mostre
que µ é uma função aditiva mas não é σ-aditiva.
[1]
Exercı́cio 93. Considere a aplicação f definida na σ-álgebra do espaço mensurável sobre os inteiros
não negativos (N, 2N ) e tal que se o subconjunto A fôr finito se tenha :
f (A) =
X
n∈A
1
,
(n + 1)2
e se o subconjunto A fôr infinito se tenha f (A) = +∞. Diga se a aplicação assim definida é uma
medida. Caso a resposta seja negativa sugira uma modificação da definição de modo a que a aplicação
seja mesmo uma medida.
Indicação : Relembre os resultados conhecidos sobre as séries de Riemann.
[2]
Exercı́cio 94. Seja A a σ-álgebra sobre X, conjunto não numerável, gerada pelos sub-conjuntos
singulares. Seja µ a função de conjuntos definida em A pela correspondência µ(A) = 0 quando A
é um conjunto numerável e por µ(A) = 1, quando se produzir a eventualidade contrária em A isto é
quando fôr o complementar de A numerável. Mostre que o trio (X, A, µ) é um espaço de medida.
Indicação : Distinga o caso em que a união da famı́lia numerável de conjuntos dois a dois disjuntos
é numerável, do caso em que não é.
[1]
Exercı́cio 95. Considere o espaço de Borel (R, B(R)). Admita que existe uma e uma só medida λ
sobre B(R) verificando as condições seguintes:
λ([0, 1]) = 1 e ∀x ∈ R ∀B ∈ B(R) λ(x + B) = λ(B) .
Chamaremos a esta medida a medida de Borel da recta real. Mostre então o seguinte:
1. ∀n ≥ 1 λ([0, n1 [) ≤
1
n
2. Qualquer subconjunto numerável de R é boreliano e tem medida nula.
3. Para qualquer intervalo (a, b) da recta real (aberto, fechado ou semi-aberto) tem-se que: λ((a, b)) =
b − a.
4. λ(Q) = 0 , λ(R) = +∞ , λ(R − Q) = +∞.
5. λ é uma medida σ-finita.
Indicação : É suficiente aplicar as definições e as propriedades fundamentais desta medida tal
como se encontram formuladas no enunciado. Recomendamos vivamente ao leitor a leitura das primeiras
quatro páginas de [10] onde está redigida a construção da medida de Borel no plano segundo Kolmogoroff.
Uma construção geral das medidas de Borel (Lebesgue) será estudada no fim do segundo capı́tulo do livro
[9].
84
Medida, Probabilidade e Acontecimentos
[2]
Exercı́cio 96. Seja F uma função contı́nua à direita e monótona crescente definida na recta real. Se
A é um intervalo de extremidades a e b, seja
µF (A) = F (b) − F (a+ ) .
Mais geralmente, se A é uma união disjunta de intervalos
A=
N
[
Ai
i=1
P
seja µF (A) = N
i=1 µF (Ai ). Mostre que µF é uma medida no anel gerado pelos intervalos que por vezes
se representa por RLeb ; ou seja, prove que é σ-aditiva.
√ 2
Observação 19. Se tomarmos para F a primitiva da função 1/ 2π e−x /2 , µF é designada por medida Gaussiana.
Exercı́cio 97. Considere o espaço de probabilidade (Ω, F, P) e defina-se a aplicação X de Ω em R [1]
constantemente igual a a. Mostre que X é uma variável aleatória.
Exercı́cio 98. Sejam (Ω, F, P) um espaço de probabilidade e Y uma aplicação de Ω em R mensurável e [1]
que nunca se anula. Mostre que 1/Y é uma variável aleatória relativamente ao mesmo espaço.
Exercı́cio 99. Dê um exemplo de uma função f definida de um conjunto S em R que não seja mensurável [1]
mas tal que |f | e f 2 o sejam.
Exercı́cio 100. [8] Determine a distribuição da variável aleatória X que apenas toma os valores 0 e [1]
1, supondo que toma o valor 1 com probabilidade p ∈ [0, 1] (diz-se que X segue uma distribuição de
Bernoulli de parâmetro p).
[2]
Exercı́cio 101. Seja FX a função de distribuição da variável aleatória X. Mostre que:
1. limx→+∞ FX (x) = 1;
2. limx→−∞ FX (x) = 0;
3. FX é contı́nua à direita.
Exercı́cio 102.
1. Seja A um subconjunto numerável de R. Diz-se que a densidade de A está bem [2]
definida se o limite
µL {A ∩ [−T, T ]}
D(A) = lim
T →∞
2T
existe. Se o limite existe, esta expressão é chamada de densidade de A. Consegue dar um exemplo
de um conjunto A mensurável cuja densidade não esteja definida?
2. Mostre que, se A1 e A2 têm densidades bem definidas e são disjuntos, então A1 ∪A2 tem densidade
bem definida e
D(A1 ∪ A2 ) = D(A1 ) + D(A2 ).
3. Mostre que existem conjuntos A e Ai , i = 1, 2, ... com densidades bem definidas tais que A =
∪∞
i=1 Ai (com os Ai disjuntos) mas
X
D(A) 6=
D(Ai ).
85
Capı́tulo 2
Exercı́cio 103. Seja X um conjunto, R um σ-anel de subconjuntos de X, e µ1 e µ2 medidas em R. [2]
Seja L a familia dos conjuntos A ∈ R para os quais µ1 (A) = µ2 (A). Assuma-se que X ∈ R e que
µ1 (X) = µ2 (X) < ∞. Mostre que L tem as seguintes propriedades:
1. X ∈ L.
2. Se A, B ∈ L e B ⊆ A, então A \ B ∈ L.
3. Se Ai ∈ L i = 1, 2, ... e A = ∪∞
i=1 Ai (união disjunta) então A ∈ L.
Observação 20. A familia L com as propriedades listadas é chamada de sistema-λ.
[2]
Exercı́cio 104. Seja C o conjunto de todos os números pertencentes ao intervalo [0, 1], que podem ser
escritos na forma
∞
X
ω=
ak 3−k
k=1
com ak = 0, 2. Mostre que C não é numerável. (C é o conjunto de Cantor).
Indicação : Use o processo da diagonal de Cantor.
[3]
Exercı́cio 105. O objectivo deste exercı́cio é mostrar que o Conjunto Triádico de Cantor é um
boreliano não numerável, cuja medida de Borel é nula. Considere em [0, 1] o conjunto C definido pela
intersecção da famı́lia (Cn )n∈N∗ em que se tem para os dois primeiros elementos da famı́lia:
2
1
2 1
2 7
8
1
∪
, 1 C2 = 0,
∪
,
∪
,
∪
,1 .
C1 = 0,
3
3
9
9 3
3 9
9
1. Descreva o elemento genérico da famı́lia (Cn )n∈N∗ , e represente gráficamente as primeiras três
etapas da construção do conjunto de Cantor.
2. Mostre que C é um fechado para a topologia usual (e consequentemente, é um boreliano).
3. Mostre que a medida de Borel ( λ) do conjunto C é nula.
4. Mostre rápidamente que:
(
C=
N
x ∈ [0, 1] : ∃(xn )n∈N ∈ {0, 2} x =
n=+∞
X
n=0
xn
3n+1
)
,
concluindo em seguida que C é equipotente a R.
Indicação : Para mostrar a alı́nea 3 use o facto de C ser a intersecção de conjuntos cuja medida de
Borel pode controlar. Para mostrar 4 atente na representação gráfica das primeiras etapas de construcão
de C.
[2]
Exercı́cio 106. Dada uma qualquer familia C de subconjuntos de um conjunto X, mostre que existe um
σ-anel Rσ minimal contendo C.
[1]
Exercı́cio 107. Seja c ∈ Rn . Dado um qualquer subconjunto A de Rn , seja
A + c = {ω ∈ R n; w − c ∈ A} .
Prove que, se A é mensurável, então A + c é mensurável e
µL (A + c) = µL (A) .
(2.2)
Indicação : Primeiro, prove a igualdade para multi-intervalos. De seguida, mostre que na equação
(2.2) as medidas exteriores são iguais.
86
Medida, Probabilidade e Acontecimentos
[1]
Exercı́cio 108. Seja f : R −→ R a aplicação linear x −→ ax + b, com a e b constantes e a > 0. Mostre
que, se A é mensurável então f (A) é mensurável e
µL (f (A)) = aµL (A).
[1]
Exercı́cio 109. Seja f : Rm −→ Rn uma aplicação contı́nua. Mostre que, se A é um subconjunto
Boreliano de Rn , então f −1 (A) é um subconjunto Boreliano de Rm . Defina-se
µf (A) = µL (f −1 (A)).
Mostre que µf é uma medida sobre os subconjuntos de Borel de Rn .
Exercı́cio 110. Seja X um conjunto, F uma σ-álgebra de subconjuntos de X, e µ uma medida de [1]
probabilidade em F. Seja A1 , A2 , A3 , ... uma sequência de subconjuntos de X pertencentes a F.
1. Mostre que, se A1 ⊆ A2 ⊆ A3 ..., então
∞
[
µ
!
Ai
= lim µ(Ai ).
i→∞
i=1
2. Mostre que, se A1 ⊇ A2 ⊇ A3 ..., então
µ
∞
\
!
Ai
i=1
= lim µ(Ai ).
i→∞
Exercı́cio 111. Seja X um conjunto, F uma σ-álgebra de subconjuntos de X, e µ uma medida de [2]
probabilidade em F. Mostre que, se A1 , A2 , A3 , ... é uma sequência de subconjuntos de X pertencentes a
F, então
µ(lim inf An ) ≤ lim inf µ(An ) ≤ lim sup µ(An ) ≤ µ(lim sup An ).
Exercı́cio 112. Para o passeio aleatório com pausas, prove que com probabilidade 1 há infinitas pausas. [3]
(Use o lema de Borel-Cantelli).
Exercı́cio 113. Mostre que, se µ1 , . . . , µn são medidas sobre um espaço mensurável (Ω, F) e a1 , . . . , an [1]
são números reais não negativos, então a função λ definida para E ∈ F por
λ(E) =
n
X
aj µj (E),
j=1
é uma medida sobre (Ω, F).
Exercı́cio 114. Seja (µn ) uma sucessão de medidas sobre um espaço mensurável (Ω, F) com µn (Ω) = 1. [1]
Mostre que, se λ for definida por
λ(E) =
∞
X
1
µ (E),
n n
2
n=1
E ∈ F,
então λ é uma medida sobre (Ω, F) e λ(Ω) = 1 .
87
Capı́tulo 2
Exercı́cio 115. Pretende-se demonstrar o resultado seguinte. Sejam S um conjunto, I um π-sistema [2]
sobre S e Σ := σ(I). Supondo que µ1 e µ2 são medidas sobre (S, Σ) tais que µ1 (S) = µ2 (S) < ∞ e
µ1 = µ2 em I, então
µ1 = µ2 em Σ.
Indicação: Mostre que o conjunto D := {F ∈ Σ : µ1 (F ) = µ2 (F )} é um d-sistema.
Exercı́cio 116. Seja (Ω, F, P) um espaço de probabilidade e considerem-se I1 , I2 , I3 três π-sistemas [3]
sobre Ω tais que, para k = 1, 2, 3
Ik ⊆ F e Ω ∈ Ik
Prove que se
P[I1 ∩ I2 ∩ I3 ] = P[I1 ] × P[I2 ] × P[I3 ]
∀Ik ∈ Ik
(k = 1, 2, 3),
então σ(I1 ), σ(I2 ), σ(I3 ) são independentes. Porque se exigiu que Ω pertencesse a Ik ?
[3]
Exercı́cio 117. Seja (X, A, µ) um espaço de medida. Seja D(µ) a famı́lia das partes µ-desprezáveis.
Considere a famı́lia de partes de X definida por:
e = {A ∪ D : A ∈ A , D ∈ D(µ)} .
A
e é uma σ-álgebra sobre X, contendo a σ-álgebra A.
1. Mostre que A
e (para o qual, obviamente se tem A ∈ A e ainda
e = A ∪ D um elemento qualquer de A
2. Seja A
e Mostre que
e = µ(A) como uma aplicação de conjuntos sobre A.
D ∈ D(µ). Defina-se então µ
e(A)
e
então µ
e é uma medida sobre a σ-álgebra A.
3. Mostre que a medida µ
e coincide com a medida µ quando restringida à σ-álgebra A.
e
4. Mostre que (X, A, µ
e) é um espaço de medida completo.
e µ
Indicação : Ver o livro [9] na página 28. Ao espaço de medida (X, A,
e) assim construı́do a partir do
espaço de medida (X, A, µ) chamaremos o completado de (X, A, µ). Ao espaço de medida completado
de um espaço de Borel de um espaço euclidiano munido com a topologia usual chamaremos espaço de
Lebesgue.
[3]
Exercı́cio 118 (O espaço de Borel na recta real não é completo.). Considere o espaço de Borel
] λ).
e
(X, B(R), λ) e o seu completado o espaço de Lebesgue (X, B(R),
Mostre então que a σ-álgebra
do espaço de medida completado, (a σ-álgebra de Lebesgue), é equipotente à famı́lia de todos os subconjuntos da recta real 2R . Admitindo que B(R) é equipotente a R conclua que o espaço de Borel da recta
real não é completo.
Indicação : Construa uma bijecção adequada usando a representação genérica de um elemento da
σ-álgebra completada.
[3]
Exercı́cio 119. Considere o espaço de Lebesgue associado ao espaço de Borel da recta real. ( Mesmas
notações que no exercı́cio anterior). Mostre que então se tem:
] x+B
]
e ∈ B(R)
e ∈ B(R)
1. ∀x ∈ R ∀B
] λ(x
e + B)
e B)
e ∈ B(R)
e = λ(
e
2. ∀x ∈ R ∀B
Indicação : Utilize as propriedades já conhecidas do espaço de medida de Borel e do seu completado.
[3]
Exercı́cio 120 (Existência de conjuntos que não são mensuráveis à Lebesgue.). Seja (R, B(R), λ) o
] λ),
e o espaço de Lebesgue da recta real.
espaço de Borel da recta real e o seu espaço completado (R, B(R),
R
]
e
Vamos mostrar que B(R) 6= 2 , isto é que existe um subconjunto da recta real que não é λ-mensurável.
88
Medida, Probabilidade e Acontecimentos
1. Mostre que a relação R definida sobre [0, 1[ por:
xRy ⇔ x − y ∈ Q ,
é uma relação de equivalência.
2. Mostre que é possı́vel construir um conjunto E ”escolhendo”um representante em cada classe de
equivalência da relação R.
3. Mostre que a famı́lia (E + q)q∈Q∩[−1,1] forma uma cobertura de [0, 1[, em que os elementos são
dois a dois disjuntos. Mostre ainda que:
[
[0, 1[⊂
(E + q) ⊂ [−1, 2] .
q∈Q∩[−1,1]
e
Conclua que E não pode ser λ-mensurável.
Indicação : Relembre a formulação do axioma da escolha e use as propriedades fundamentais da
medida de Lebesgue.
Exercı́cio 121. Seja µ uma medida sobre um espaço mensurável (Ω, F) e A um conjunto fixo de F. [1]
Mostre que, a função λ definida para E ∈ F por λ(E) = µ(A ∩ E), é uma medida sobre (Ω, F).
Exercı́cio 122 (Completação de um espaço de medida). Suponha que (S, Σ, µ) é um espaço de medida. [2]
Defina-se a famı́lia N de subconjuntos de S como sendo N := {N ∈ P(S) : ∃Z ∈ Σ tal que N ⊆ Z e µ(Z) = 0}.
Considere-se agora que para qualquer subconjunto F de S se diz que F ∈ Σ∗ se ∃E, G ∈ Σ tais que
E ⊆ F ⊆ G e µ(G\E) = 0.
1. Mostre que Σ∗ é uma σ-álgebra sobre S.
2. Verifique que Σ∗ = σ(Σ, N).
3. Definindo para F ∈ Σ∗ , que µ∗ (F ) = µ(E) = µ(G) verifique que (S, Σ∗ , µ∗ ) é um espaço de
medida (trata-se da completação de (S, Σ, µ)).
Observe que no novo espaço de medida (S, Σ∗ , µ∗ ), qualquer N ∈ N é mensurável (N ∈ Σ∗ ) e tem medida
nula (µ∗ (N ) = 0).
Bibliografia
[1] Robert G. Bartle. The elements of integration and Lebesgue measure. Wiley Classics
Library. John Wiley & Sons Inc., New York, 1995. Containing a corrected reprint
of the 1966 original [ıt The elements of integration, Wiley, New York; MR0200398
(34 #293)], A Wiley-Interscience Publication.
[2] Pedro Bruno Teodoro Braumann. Teoria da Medida e da Probabilidade, Parte Iálgebra de Conjuntos. Fundação Calouste Gulbenkian, 1987.
[3] Jean Genet. Mesure et Intégration. Librairie Vuibert, Paris, 1976.
[4] Paul R. Halmos. Measure Theory. D. Van Nostrand Company, Inc., New York, N.
Y., 1950.
[5] A. Kirillov and A. Gvichiani. Théorèmes et problèmes d’analyse fonctionnelle.
“Mir”, Moscow, 1982. Translated from the Russian by Djilali Embarek.
89
Capı́tulo 2
[6] A. N. Kolmogorov and S. V. Fomin. Elements of the theory of functions and functional analysis. Dover Publications, 1999.
[7] Etiènne Ménard. Mesure et Intégration, Notes pour Travaux Dirigés avec rappels
de cours. Université de Rouen, 1979.
[8] Paulo Eduardo de Oliveira. Exercı́cios de Teoria das Probabilidades. Coimbra,
1998–1999.
[9] Walter Rudin. Real and complex analysis. McGraw-Hill Book Co., New York, third
edition, 1987.
[10] Jorge Salazar. Medida e Integração. FCT-UNL, Monte de Caparica, 1991. Notas
para a disciplina de Análise Matemática V, Departamento de Matemática.
[11] David Williams. Probability with martingales. Cambridge Mathematical Textbooks.
Cambridge University Press, Cambridge, 1991.
[12] L. C. Young. Lectures on the calculus of variations and optimal control theory.
Chelsea Pub Co, 1980.
90
Capı́tulo 3
As Funções Mensuráveis
You state ... that a function, too, can act as the indeterminate element.
This I formerly believed, but now this view seems doubtful to me
because of the following contradiction. Let w be the predicate: to be a
predicate that cannot be predicated of itself. Can w be predicated of
itself?
Russell (1902) Letter to Frege in Jean van Heijenoort, From Frege to
Godel: A Source Book in Mathematical Logic, 1879–1931 Harvard
University Press, Cambridge, MA, 1967, 3rd printing 1976, página 124.
3.1
Introdução
Relembremos que no modelo de Kolmogorov para as probablidades, um espaço de probabilidade (Ω, F, P), isto é, um espaço de medida em que P[Ω] = 1, pode ser interpretado
da seguinte forma.
• O conjunto Ω representa o conjunto dos estados do universo e cada ω ∈ Ω é uma
das realizações do fenómeno que estamos a estudar. Cada realização corresponde
assim a um dos estados do mundo.
• A álgebra-σ, F representa o conjunto de todos os acontecimentos relevantes no
fenómeno em estudo.
• A medida de probabilidade P indica-nos, para cada acontecimento E ∈ F, o grau de
confiança que temos na realização desse acontecimento associando-lhe P[E] ∈ [0, 1].
Um dado acontecimento E ∈ F é suposto conhecido se para qualquer estado do universo
ω ∈ Ω é possı́vel distinguir entre
ω ∈ E ou ω ∈
/E,
isto é se o acontecimento E teve ou não teve lugar.
91
Capı́tulo 3
Uma experiência aleatória assume muitas vezes a forma de uma aplicação X : Ω −→
Θ em que Θ é um dado conjunto com interesse. Em muitos casos esse conjunto é o
conjunto dos números reais mas não necessariamente; pense-se na escolha de um vector
abstracto ao acaso em que Θ seria, naturalmente, um dado espaço vectorial. A uma tal
aplicação damos usualmente o nome de variável aleatória. Assim, se considerarmos a
experiência de escolher um número ao acaso no intervalo [0, 1], teremos naturalmente
Θ = [0, 1]. Ainda no contexto dessa experiência, podemos considerar o jogo seguinte. Se
o número saı́do pertencer a [0, 1/2[ ganhamos uma unidade monetária e se pertencer a
[1/2, 1] não ganhamos nada. O jogo pode ser representado por f (X) em que f : [0, 1] −→
{0, 1} está definida por:
(
1 se x ∈ [0, 1/2[
f (x) =
0 se x ∈ [1/2, 1] .
O ganho esperado com este jogo, usualmente representado por E[f (X)], pode ser calculado se suposermos, com toda a naturalidade, que a distribuição de X é a lei uniforme
sobre [0, 1]. Com efeito, relembrando o estudado na disciplina de Probabilidades e Estatı́stica e observando que apesar da função f ser descontı́nua no ponto x = 1/2 o integral
de Riemann impróprio pode ser calculado da forma usual, tem-se que:
Z
E[f (X)] =
1
Z
f (x)dx =
0
1/2
Z
1
f (x)dx +
0
Z
f (x)dx =
1/2
1/2
dx + 0 =
0
1
.
2
Suponhamos agora o jogo seguinte. Se o número saı́do for racional, isto é, pertencer a
Q ∩ [0, 1] ganhamos uma unidade monetária e no caso contrário não ganhamos nada.
Agora neste caso, o jogo pode ser representado D(X) em que D : [0, 1] −→ {0, 1} está
definida por:
(
1 se x ∈ Q ∩ [0, 1]
D(x) =
0 se x ∈ (R \ Q) ∩ [0, 1] .
A determinação do ganho esperado, E[D(X)], com este jogo é impossı́vel no quadro do
integral de Riemann, utilizado no jogo anterior, uma vez que terı́amos de calcular
Z
E[D(X)] =
1
D(x)dx .
0
Acontece que D é a famosa função de Dirichlet que é descontı́nua em cada ponto do seu
domı́nio devido à densidade dos números racionais e dos números irracionais em [0, 1].
Pela teoria do integral de Riemann, usando as somas de Darboux inferiores e superiores
N
N
para uma qualquer sequência de partições 0 = xN
0 < x1 < · · · xN +1 = 1, terı́amos que
92
As Funções Mensuráveis
se D fosse Riemann integrável de integral
0=
lim
N →+∞
N
max0≤i≤N |xN
i+1 −xi |→0
Z
=
R1
0
N
X
i=0
D(x)dx ∈ R:
!
inf
N
ξ∈[xN
i ,xi+1 ]
N
D(ξ) (xN
i+1 − xi ) =
1
D(x)dx =
0
=
lim
N →+∞
N
max0≤i≤N |xN
i+1 −xi |→0
N
X
i=0
!
sup
N
ξ∈[xN
i ,xi+1 ]
N
D(ξ) (xN
i+1 − xi ) = 1
uma vez que pela densidade dos racionais nos reais se verifica sempre inf ξ∈[xi ,xi+1 ] D(ξ) =
0 e pela densidade dos irracionais nos reais se verifica sempre supξ∈[xi ,xi+1 ] D(ξ) = 1. No
entanto, pelo que conhecemos dos argumentos de cardinalidade, isto é, #Q = ℵ0 e
#(R \ Q) = #R, é natural pensar que a probabilidade de tirar um racional ao acaso em
[0, 1] seja zero e que a probabilidade de escolher um irracional ao acaso em [0, 1] seja igual
a um, pelo que um resultado desejável para o ganho esperado neste segundo jogo seria
E[D(X)] = 0. Podemos pois concluir que o quadro das funções contı́nuas e do integral
de Riemann é insuficiente para tratar de forma aceitável até pequenos problemas como
o colocado com este segundo jogo.
No que vai seguir-se introduziremos a classe das funções mensuráveis que engloba,
sob certas condições naturais, a classe das funções contı́nuas. No contexto da classe das
funções mensuráveis é possı́vel definir um integral mais abrangente que o integral de
Riemann, o integral de Lebesgue, e que, exceptuando o caso dos integrais impróprios de
Riemann, generalisa completamente o integral de Riemann das funções reais de variável
real.
3.2
As funções mensuráveis
Começamos por estudar um caso particular de funções mensuráveis com especial relevo tanto para a Análise Matemática como para as Probabilidades. Uma propriedade
importante que observaremos no contexto deste caso particular motivará a definição de
aplicaccão mensurável no caso em que o contradomı́nio é um qualquer espaço mensurável.
No quadro do modelo de Kolmogorov uma variável aleatória é apenas uma aplicação
mensurável. Tal como já observámos uma variável aleatória não tem necessariamente que
tomar valores reais, uma vez que considerar um objecto qualquer de um dado conjunto
ao acaso, torna natural considerar aplicações com valores nesse conjunto. Para podermos
definir a lei probabilı́stica que reje essa experiência é imprescindı́vel que essa aplicação
seja mensurável.
93
Capı́tulo 3
3.2.1
As funções mensuráveis: a recta acabada
Vamos considerar primeiramente as funções com valores em R, a recta acabada. Relembramos1 que
R = [−∞, +∞] = R ∪ {+∞} ∪ {−∞}
onde +∞ e −∞ são dois objectos distintos de qualquer número real2 e tais que com a
relação de ordem ≤R definida por


x ≤R y
∀x, y ∈ R x ≤R y ⇔ x = −∞ ou x ∈ R


y = +∞ ou y ∈ R
(R, ≤R ) é um conjunto totalmente
mento. Convenciona-se ainda que:


x +R y
∀x, y ∈ R x +R y = −∞


+∞
se x, y ∈ R
e y = +∞
e x = −∞ ,
ordenado com primeiro (−∞) e último (+∞) ele-
se x, y ∈ R
se (x = −∞ e y ∈ R) ou (y = −∞ e x ∈ R)
se (x = +∞ e y ∈ R) ou (y = +∞ e x ∈ R) ,
tendo-se naturalmente que x+R = y +R x.
Convenciona-se também que:


x ×R y se x, y ∈ R



0
se (x = 0 e y ∈ R) ou (y = 0 e x ∈ R)
∀x, y ∈ R x ×R y =
−∞
se (x = −∞ e y >R 0) ou (x = +∞ e y <R 0)



+∞
se (x = +∞ e y >R 0) ou (x = −∞ e y <R 0) ,
tendo-se naturalmente que x ×R y = y ×R x.
Observação 21 (Muito importante). Note-se que não foi atribuı́do qualquer sentido às
expressões (+∞) + (−∞) ou (−∞) + (+∞). Com efeito, também se assume que
∀x, y, z ∈ R, z >R 0 x ≤R y ⇐ (x ×R z) ≤R (y ×R z).
e que para x, y, z ∈ R sempre que x +R z e y +R z estão bem definidos se tem
x ≤R y ⇐ (x +R z) ≤R (y +R z).
Relembremos que estas propriedades prolongam em R a compatibilidade respectivamente, do produto e da adição, com a relação de ordem usual em R. Esta compatibilidade faz de (R, ≤) um corpo ordenado que adicionalmente é arquimedeano, completo
e no qual todo o subconjunto majorado tem supremo. Com estas propriedades (R, ≤) é
1
2
94
Vejam-se os exercı́cios das aulas práticas 51 a 53 sobre este assunto.
Pode considerar-se, por exemplo, −∞ := {R} e +∞ := {{R}}.
As Funções Mensuráveis
único a menos de um isomorfismo. Se se pretender prolongar as operações subjacentes à
X
estrutura de corpo, a R então é fácil ver que não há forma de definir −∞+∞ mantendo
X
os axiomas relativos à estrutura de espaço vectorial. Com efeito, seja f ∈ R e α ∈ R+ .
Tem-se que de acordo com estes axiomas,
∀x ∈ X f (x) ∈ R ⇒ (f (x) − α · f (x) = (1 − α) · f (x)) ,
independentemente do valor de α. Suponhamos, agora, que para x0 ∈ X se tem que
f (x0 ) = +∞. Como vale, para a < 0, pelas regras adoptadas no cálculo integral de
Lebesgue, que a · +∞ = −∞ tem-se que:
(
(1 − α) · f (x) = +∞ se 0 < α < 1
(f (x0 ) − α · f (x0 ) = +∞ − ∞) =
(1 − α) · f (x) = −∞ se 1 < α ,
mostrando que +∞ − ∞ não pode ter um resultado único se valer a convenção a · +∞ =
−∞ para a < 0.
Esta indefinição dos sı́mbolos (+∞) + (−∞) ou (−∞) + (+∞) tornará necessárias
algumas precauções adicionais na definição da soma de funções (veja-se a observação 23).
Da mesma forma que em (R, ≤), a recta real com a relação de ordem usual, pode-se
definir em (R, ≤R ) a famı́lia de intervalos
I = {]x, y[, [−∞, y[, ]x, +∞] : x, y ∈ R}
Por definição, τR topologia usual sobre R é a topologia gerada pela famı́lia I. Pode
mostrar-se que (R, τR ) é um espaço topológico compacto. Pode verificar-se que sendo τR
a topologia usual sobre R se tem
τR = {ω, ω ∪ {−∞}, ω ∪ {+∞}, ω ∪ {−∞, +∞} : ω ∈ τR }
o que tem como consequência imediata que o traço desta topologia τR sobre R coincide
com τR e que, em consequência, a injecção canónica de (R, τR ) em (R, τR ) é contı́nua;
por isso (R, τR ) é uma compactificação de (R, τR ).
O espaço de Borel (R, B(R)) é tal que, por definição:
B(R) = σ(τR ) .
Pode mostrar-se que sendo B(R) a álgebra-σ de Borel sobre R:
B(R) = {B, B ∪ {−∞}, B ∪ {+∞}, B ∪ {−∞, +∞} : B ∈ B(R)} .
(3.1)
Para o que vai seguir-se, o leitor poderá numa primeira leitura admitir a fórmula 3.1.
Sempre que não haja risco de confusão usar-se-ão os sinais habituais (>, ≥, <, ≤) relativos à relação de ordem usual sobre R, em vez dos sinais correspondentes (>R , ≥R , <R , ≤R )
para a relação de ordem sobre R. Um exemplo deste uso é dado na Proposição 16.
95
Capı́tulo 3
Definição 22. Seja (X, A) um espaço de medida. A função f : X −→ R é
mensurável se e só se:
∀α ∈ R {x ∈ X : f (x) > α} = f −1 (]α, +∞]) ∈ A .
(3.2)
Observação 22. A definição de função mensurável depende do espaço de medida tal como
se pode constatar imediatamente na expressão 3.2. É óbvio que se A = P(X) todas as
funções são mensuráveis. É fácil verificar que se X 6= ∅ e A = {∅, X} as únicas funções
mensuráveis são as funções constantes.
Exemplo 7 (Um exemplo fundamental). Seja A ⊂ X e IA a função indicatriz de A
definida por:
(
1 se x ∈ A
∀x ∈ X IA (x) =
(3.3)
0 se x ∈
/A.
Então IA é mensurável se e só se A ∈ A. Com efeito, dado α ∈ R verifica-se imediatemente que:


X se α < 0
{x ∈ X : IA (x) > α} = A se 0 ≤ α < 1


∅
se 1 ≤ α ,
logo se A ∈ A tem-se que para qualquer α ∈ R vale {x ∈ X : IA (x) > α} ∈ A.
Reciprocamente, se se verificar a condição 3.2 então, para α = 1/2,
A = {x ∈ X : IA (x) > 1/2} ∈ A .
O resultado seguinte mostra que na definição 3.2 o sinal particular >, da relação de
ordem sobre R usado para definir o conjunto, não é relevante podendo ser substituı́do
por um qualquer dos três outros ≥, <, ≤.
Proposição 16. A função f : X −→ R é mensurável, de (X, A) em (R, B(R))
se e só se se verificar uma qualquer das quatro propriedades equivalentes seguintes.
(1) ∀α ∈ R {x ∈ X : f (x) > α} ∈ A.
(2) ∀α ∈ R {x ∈ X : f (x) ≥ α} ∈ A.
(3) ∀α ∈ R {x ∈ X : f (x) < α} ∈ A.
(4) ∀α ∈ R {x ∈ X : f (x) ≤ α} ∈ A.
Demonstração. Basta demonstrar a equivalência das quatro condições entre si dado que
a primeira condição é exactamente a condição que define a mensurabilidade de uma
função com valores em R. Observe-se primeiramente que (1) ⇔ (4) e que (2) ⇔ (3)
96
As Funções Mensuráveis
por passagem ao complementar dado que A é uma álgebra-σ. Suponhamos agora (1).
Note-se que:
{x ∈ X : f (x) ≥ α} = f −1 ([α, +∞])
e que como se verifica imediatamente,
[α, +∞] =
+∞
\
]α −
n=1
1
, +∞] .
n
Com efeito a inclusão ⊆ resulta de se ter:
∀n ≥ 1 [α, +∞] ⊂]α −
1
, +∞] .
n
A inclusão no sentido contrário comprova-se observando que se
∀n ≥ 1 α −
1
< x ≤ +∞ ,
n
passando ao limite quando n −→ +∞ virá que α ≤ x ≤ +∞.
Pelas propriedades da imagem recı́proca de uma função na igualdade em (a), tem-se
que:
+∞
\
1
−1
{x ∈ X : f (x) ≥ α} = f ([α, +∞]) =(a)
f −1 (]α − , +∞]) ∈(b) A
n
n=1
Note-se que (b) se justifica dado que por (1), para cada n ≥ 1 se tem f −1 (]α − n1 , +∞]) ∈
A e como A é uma álgebra-σ a intersecção numerável de elementos de A pertence a A.
Demonstrámos assim que (1) ⇒ (2). Mostremos agora que (2) ⇒ (1). Para esse efeito
suponhamos que se verifica (2), observemos que:
{x ∈ X : f (x) > α} = f −1 (]α, +∞])
e que como se verifica imediatemente:
]α, +∞] =
+∞
[
[α +
n=1
1
, +∞] .
n
Com efeito, a inclusão ⊆ resulta de se ter para qualquer x ∈]α, +∞] isto é, tal que α < x,
que existe n0 ∈ N tal que α + 1/n0 ≤ x. A inclusão no sentido contrário é imediata dado
que
1
∀n ≥ 1 [α + , +∞] ⊂]α, +∞] .
n
Mais uma vez, pelas propriedades da imagem recı́proca de uma função na igualdade em
(c), tem-se que:
{x ∈ X : f (x) > α} = f
−1
(]α, +∞]) =(c)
+∞
[
f −1 ([α +
n=1
1
, +∞]) ∈(d) A
n
em que (d) tem uma justificação semelhante à de (b) acima.
97
Capı́tulo 3
Tal como com as funções contı́nuas a soma e o produto de funções mensuráveis
com valores na recta acabada são funcções mensuráveis. No entanto, algum cuidado é
necessário dado não ser possı́vel definir em geral a soma de funções com valores em R.
Proposição 17. Sejam f, g : X −→ R funções mensuráveis.
(i) Se para todo o x ∈ X se tiver que f (x) + g(x) está bem definido como um
ponto em R então, f + g é mensurável.
(ii) Para qualquer α ∈ R a função αf está bem definida e é mensurável.
(iii) As funções max(f, g) e min(f, g) são mensuráveis.
(iv) Se f, g ≥ 0 então a função f × g é mensurável.
Demonstração. Suponhamos que para todo o x ∈ X, f (x)+g(x) está bem definido como
um ponto em R o que significa que nunca se produz a situação em que f (x) + g(x) =
+∞ − (+∞) ou f (x) + g(x) = −∞ + (+∞). Sendo certo que para qualquer α ∈ R fixo:
[
{x ∈ X : (f + g)(x) > α} =
({x ∈ X : f (x) > r} ∩ {x ∈ X : g(x) > α − r}) (3.4)
r∈Q
temos que f + g é mensurável uma vez o conjunto {f + g > α}, à esquerda na fórmula,
pertence a A. Com efeito, pela representação à direita na fórmula esse conjunto é união
numerável de conjuntos que, pelas hipóteses feitas sobre f e g, pertencem a A. Para
demonstrarmos a fórmula 3.4 consideremos primeiro os casos em que f ou g tomam
valores infinitos. Se f (x) = −∞ ou g(x) = −∞ (ou ambos) então f (x) + g(x) = −∞,
x∈
/ {f +g > α} e para qualquer r ∈ Q se tem também que x ∈
/ {f > r}∩{g > α −r}. Se
f (x) = +∞ ou g(x) = +∞ (ou ambos) então f (x) + g(x) = +∞, para qualquer α ∈ R
tem-se x ∈ {f + g > α}; se ambos f (x) = +∞ e g(x) = +∞ tem-se que para qualquer
r racional x ∈ {f > r} ∩ {g > α − r}; se, por exemplo, for f (x) = +∞ e g(x) ∈ R então
para qualquer r ∈ Q tal que r < g(x) − α tem-se que x ∈ {f > r} ∩ {g > α − r}.
Consideremos agora apenas os pontos x ∈ X tal que f (x), g(x) ∈ R. Se para um
dado x ∈ X se verificar que para r ∈ Q se tem f (x) > r e g(x) > α − r então, tem-se
necessáriamente f (x) + g(x) > r + (α − r) = α. Para a inclusão no sentido contrário
observe-se que se para x ∈ X se tiver f (x) > α − g(x) então 3 exite um racional r ∈ Q
tal que f (x) > r > α − g(x). Para esse racional r tem-se, obviamente, f (x) > r e
g(x) > α − r.
A demonstração da proposição (ii) é imediata usando a proposição 16 no caso em
que α 6= 0. Quando α = 0 tem-se que αf ≡ 0 e, naturalmente, toda a função constante
é mensurável.
Para justificarmos a afirmação feita em (iii) note-se que
{x ∈ X : max(f, g) > α} = {x ∈ X : f > α} ∩ {x ∈ X : g > α}
3
98
Pela densidade dos racionais em R.
As Funções Mensuráveis
e que também se tem
{x ∈ X : min(f, g) < α} = {x ∈ X : f < α} ∪ {x ∈ X : g < α} .
pelo que a proposição 16 nos permite concluir.
A demonstração da proposição (iv) faz-se utilizando a fórmula
{x ∈ X : (f ×g)(x) > α} =
[
r∈Q\{0}
{x ∈ X : f (x) > r} ∩ {x ∈ X : g(x) >
α } , (3.5)
r
representação que tem uma justificação semelhante à que demos para a fórmula 3.4.
Uma eventualidade de aplicação da proposição anterior e que se produz com frequência
é a que é descrita na seguinte observação.
Observação 23. Sejam f, g : X −→ R funções mensuráveis finitas µ quase certamente.
Então pode redefinir-se f e g sobre um conjunto de medida nula e, em consequência,
pode definir-se a soma de f com g sobre X de forma a que f + g seja uma função
mensurável.
Com efeito, existe Xf de medida nula tal que sobre o complementar f toma valores
reais e do mesmo modo existe Xg de medida nula tal que sobre o complementar g toma
valores reais. Sobre Xfc ∩ Xgc a soma de f com g está definida naturalmente como soma
de funções tomando valores reais. Para x ∈ Xf ∪ Xg que é um conjunto de medida nula,
podemos definir, por exemplo, f (x) + g(x) = +∞; tal pode corresponder a redefinir, por
exemplo, f = +∞ e g = 0 sobre Xf ∪ Xg .
Note-se que a soma de funções assim definida não corresponde a uma lei de comX
posição interna sobre R uma vez que há uma infinidade de formas de associar ao par
X 2
X
(f, g) ∈ (R ) uma função f + g ∈ R .
Um outro exemplo de aplicação da proposição anterior é a que respeita à decomposição fundamental de uma função como diferença de duas funções positivas de tal
forma que a função inicial é mensurável se e só se as funções da decomposição o forem.
Para melhor entender as noções expostas na proposição seguinte o leitor é convidado a
representar graficamente, num ou dois exemplos, as noções aı́ introduzidas.
99
Capı́tulo 3
Proposição 18. Seja f : X −→ R uma função e seja por definição f + a parte
positiva de f dada por:
∀x ∈ Xf + (x) = max(f, 0)
(3.6)
f − a parte negativa de f dada por:
∀x ∈ Xf − (x) = max(−f, 0)
(3.7)
Então temos que:
(i) f + ≥ 0 e f − ≥ 0;
(ii) Tem-se a seguinte decomposição de f na diferença entre a parte positiva
e a parte negativa:
f = f + − f − , | f |= f + + f − ;
(iii) As partes positiva e negativa f + e f − são mensuráveis se e só se f for
mensurável;
(iv) Tem-se ainda as expressões para os elementos da decomposição de f :
f+ =
| f | +f
| f | −f
, f− =
.
2
2
Demonstração. A verificação de (i) é imediata e decorre das definições de f + e f − .
Verifiquemos a propriedade (ii).


f (x) = max(f (x), 0) − 0
+
−
(f − f )(x) = f (x) = −(−f (x)) = 0 − max(−f (x), 0)


f (x) = 0
se f (x) > 0
se f (x) < 0
se f (x) = 0
Da mesma forma temos que:


| f (x) |= max(f (x), 0) + 0
+
−
(f + f )(x) = | f (x) |= −f (x) = 0 + max(−f (x), 0)


| f (x) |= 0
se f (x) > 0
se f (x) < 0
se f (x) = 0
A proposição (iii) é uma consequência de (iii) na proposição 17, observando que se
f (x) = f + (x) = +∞ então f − (x) = 0 e ainda f (x) = f − (x) = −∞ então f + (x) = 0.
A proposição (iv) decorre de (iii) observando que | f (x) | +f e | f (x) | −f estão sempre
bem definidos.
100
As Funções Mensuráveis
A proposição 17 (iv) pode agora ser generalizada ao caso geral mostrando-se que o
produto de duas quais quer funções mensuráveis com valores na recta acabada é mensurável.
Proposição 19. Sejam f, g : X −→ R funções mensuráveis. Então a função
f × g é mensurável.
Demonstração. Decompondo f = f + − f − e g = g + − g − tem-se que f g = f + g + −
f + g − − f − g + + f − g − . Como, pela proposição 17 (iv), cada uma das parcelas da decomposição anterior é mensurável tem-se que f g é mensurável por aplicação sucessiva
da proposição 17 (ii) e da proposição 17 (i).
O teorema seguinte fornece-nos uma caracterização geral da mensurabilidade das
funções com valores na recta acabada. Esta caracterização é importante para a definição
de função mensurável tendo por domı́nio e contradomı́nio espaços mensuráveis quaisquer.
Teorema 10. Uma função f : X −→ R é mensurável se e só se:
∀B ∈ B(R) f −1 (B) ∈ A .
(3.8)
Observação 24. A condição expressa na fórmula 3.8 pode representar-se de forma equivalente pela notação:
f −1 (B(R)) ⊂ A .
Demonstração. A condição é claramente suficiente para que f seja mensurável uma vez
que se se verificar a fórmula 3.8 temos em consequência de se verificar que para qualquer
α ∈ R, se tem ]α, +∞] ∈ I ⊂ τR ⊂ B(R),
{x ∈ X : f (x) > α} = f −1 (]α, +∞]) ∈ A .
Para verificarmos que a condição é necessária note-se se f for mensurável, admitindo
que B(R) = σ(I) e que
∀I ∈ I f −1 (I) ∈ A ,
isto é, que f −1 (I) ⊂ A então, por um resultado conhecido
apêndice, que justifica a igualdade em (a) temos que:
4
que figura no lema 3 no
f −1 (B(R)) = f −1 (σ(I)) =(a) σ(f −1 (I)) ⊂ A .
uma vez que se f −1 (I) ⊂ A então σ(f −1 (I)) ⊂ A. Verifiquemos agora as hipóteses feitas
no inı́cio desta demonstração.
4
Vejam-se também os exercı́cios das aulas práticas.
101
Capı́tulo 3
Uma primeira aplicação deste teorema é a seguinte forma expedita de verificação do
carácter mensurável de uma função com valores na recta acabada.
Corolário 3. Uma função f : X −→ R é mensurável se e só se:
1. ∀B ∈ B(R) f −1 (B) ∈ A e
2. f −1 ({−∞}), f −1 ({+∞}), f −1 ({−∞, +∞}) ∈ A.
Demonstração. Este resultado é uma consequência de verificação imediata do teorema 10
e da forma geral de um boreliano da recta acabada tal como nos é dado pela fórmula 3.1.
3.2.2
As funções mensuráveis: caso geral
O teorema 10 torna agora natural a definição seguinte que vale para quaisquer espaços
mensuráveis.
Definição 23. Sejam (X, A) e (Y, B) espaços mensuráveis e f : X −→ Y uma
aplicação de X em Y . A aplicação f é mensurável de (X, A) em (Y, B) se e
só se:
f −1 (B) ⊂ A
(3.9)
isto é, de forma equivalente, se e só se:
∀B ∈ B f −1 (B) ∈ A
(3.10)
Uma primeira consequência desta definição é o teorema importante seguinte.
Teorema 11. Sejam (X, A), (Y, B) e (Z, C) espaços mensuráveis, f : X −→ Y
uma aplicação mensurável de (X, A) em (Y, B) e g : Y −→ Z uma aplicação
mensurável de (Y, B) em (Z, C). Então g ◦ f : X −→ Z, a composição de g com
f , é uma aplicação mensurável de (X, A) em (Z, C).
Demonstração. A demonstração por verificação da condição 3.10 na definição 23, é simples. Seja C ∈ C.
(g ◦ f )−1 (C) = f −1 (g −1 (C)).
Como g é mensurável g −1 (C) ∈ B e como f é mensurável f −1 (g −1 (C)) ∈ A, pelo que se
verifica que (g ◦ f )−1 (C) ∈ A para qualquer C ∈ C.
Observação 25. A definição 23 e o teorema fundamental 11 que nos diz, grosso modo, que
a composição de aplicações mensuráveis é mensurável, mostra que, no sentido da teoria
das categorias5 , na categoria dos espaços mensuráveis os morfismos são as aplicações
5
102
Veja-se [][].
As Funções Mensuráveis
mensuráveis. Esta situação deve ser posta em paralelo com outras situações anteriormente estudadas. Assim, na categoria dos espaços vectoriais os morfismos são as
aplicações lineares (a composição de aplicações lineares é linear) e na categoria dos
espaços topológicos os morfismos são as aplicações contı́nuas (a composição de aplicações
contı́nuas é uma aplicação contı́nua).
Uma segunda consequência desta definição geral de mensurabilidade é a que se pode
exprimir grosseiramente na forma: as aplicações contı́nuas são mensuráveis. Observe o
leitor que neste caso esta formulação grosseira da ideia pode induzir em erro dado que
a continuidade se refere às topologias do domı́nio e do contradomı́nio e que a mensurabilidade se refere às álgebras-σ do domı́nio e do contradomı́nio, sendo de esperar que
haja uma qualquer relação especial entre as topologias e as álgebras-σ para o resultado
admitir uma formulação rigorosa.
Teorema 12. Sejam (X, τX ), (Y, τY ) dois espaços topológicos e f : X −→ Y
uma aplicação contı́nua de de (X, τX ) em (Y, τY ). Então f , é mensurável de de
(X, B(τX )) em (Y, B(τY )) em que B(τX )) = σ(τX ) é a álgebra-σ de Borel sobre
X associada a τX e B(τY )) = σ(τY ) é a álgebra-σ de Borel sobre Y associada
a τY .
Demonstração. Seja ω ∈ τY um qualquer aberto de Y . Pela continuidade de f sabemos
que f −1 (ω) ∈ τX , isto é:
f −1 (τY ) ⊆ τX .
Em consequência:
f −1 (B(τY )) =(a) f −1 (σ(τY )) =(b) σ(f −1 (τY )) ⊆(c) σ(τX ) = B(τX ) ,
com as seguintes justificações.
(a) Pela definição da álgebra-σ de Borel sobre Y associada à topologia τY .
(b) Pelo lema 3.
(c) Pelo resultado que nos diz que se a famı́lia de subconjuntos E está contida na
famı́lia de subconjuntos F, então a álgebra-σ gerada por E tem que estar contida
na álgebra-σ gerada por F.
Observação 26 (Advertência suplementar). Tal como referimos antes, é agora óbvio que a
propriedade continuidade implica a mensurabilidade exige uma condição adicional sobre
as álgebras-σ em presença. Essa condição pode ser a seguinte: as álgebras-σ serem as
álgebras-σ de Borel associadas às topologias relativamente às quais as aplicações são
contı́nuas. Naturalmente a propriedade permanece válida se a álgebra-σ do domı́nio
103
Capı́tulo 3
for maior 6 que a álgebra-σ de Borel do domı́nio 7 e se a álgebra-σ do contradomı́nio
for menor 8 que a álgebra-σ de Borel do contradomı́nio. Sem uma qualquer condição
adicional a propriedade continuidade implica a mensurabilidade é falsa em geral como
se pode constatar com um contra-exemplo que o leitor facilmente construirá.
3.2.3
Exemplos adicionais relevantes de funções mensuráveis
O teorema 12 permite-nos enriquecer imediatamente o universo das funções mensuráveis
à Borel com as funções contı́nuas definidas sobre Rn com valores em Rm .
Corolário 4. Seja f : Rn −→ Rm uma aplicação contı́nua relativamente às
topologias usuais de Rn e Rm , respectivamente, τun e τum . Então f é mensurável
de (Rn , B(Rn )) em (Rm , B(Rm )).
Demonstração. É uma simples consequência do teorema 12 observando que B(Rn ) =
σ(τun ) e B(Rm ) = σ(τum ).
O resultado seguinte diz-nos que todo o limite pontual de funções mensuráveis é um
exemplo adicional de função mensurável. Para além disso diferencia, notavelmente, as
funcões mensuráveis com valores reais das funções contı́nuas. Para esclarecer esta ideia
relembremos um exemplo dado pela sucessão de funções (fn )n≥1 definidas em [0, 1] por:
(
(−nx + 1) para 0 ≤ x ≤ 1/n
fn (x) =
0
para 1/n ≤ x
É um simples exercı́cio de Análise Matemática verificar que para cada n ≥ 1, a função
fn é contı́nua de [0, 1] em [0, 1]. Dado que para x = 0 e para qualquer n ≥ 1 se tem
fn (0) = 1 pode concluir-se que
f (0) := lim fn (0) = 1 .
n→+∞
Para qualquer x ∈]0, 1] tem-se que para n ≥ n0 > 1/x
f (x) := lim fn (x) = 0 .
n→+∞
Assim sendo, a sucessão (fn )n≥1 converge pontualmente para f : [0, 1] −→ [0, 1] definida
por:
(
1 se x = 0
f (x) =
0 se x ∈]0, 1] ,
6
No sentido da inclusão.
Um exemplo é a álgebra-σ de Lebesgue, no caso de funções definidas sobre Rn . A álgebra-σ de
Lebesgue é a completada da álgebra-σ de Borel pelo que contem todos os borelianos e adicionalmente
todos os conjuntos desprezáveis relativamente à medida de Lebesgue. Veja-se a este propósito o exercı́cio
relativo à completação de um espaço de medida e a construção das medidas de Lebesgue-Stieltjes por
aplicação do teorema de Carathéodory.
8
Também no sentido da inclusão.
7
104
As Funções Mensuráveis
que, como se constata imediatamente, não é contı́nua em [0, 1]. À vista deste exemplo
podemos afirmar que o limite pontual de funções contı́nuas não é, em geral, uma função
contı́nua 9 .
Teorema 13. Seja (X, A) um espaço mensurável e (fn )n∈N uma sucessão de
funções mensuráveis de (X, A) em (R, B(R)) convergente pontualmente para
f : X −→ R. Então f é mensurável de (X, A) em (R, B(R)).
Observação 27. Antes de demonstrarmos este teorema relembremos duas noções importantes de Análise. Dada uma sucessão de números da recta acabada (an )n∈N podemos
associar-lhe o maior e o menor valor de aderência da sucessão em R dados respectivamente por
lim sup an = inf sup am e lim inf an = sup inf am .
(3.11)
n→+∞
n→+∞
n∈N m≥n
n∈N m≥n
É sabido que a sucessão converge se e só se
lim sup an = lim inf an
n→+∞
n→+∞
e que nesse caso, isto é, se a sucessão converge o limite é dado por:
lim an := lim sup an = lim inf an
n→+∞
n→+∞
n→+∞
Consideremos agora uma sucessão de funções (fn )n∈N tais que para cada n ∈ N se tenha:
fn : X −→ R. Pode-se definir pontualmente as funções lim supn→+∞ fn e lim inf n→+∞ fn
por:
∀x ∈ X
lim sup fn (x) = lim sup (fn (x)) e lim inf fn (x) = lim inf (fn (x))
n→+∞
n→+∞
n→+∞
n→+∞
e fica claro então que a sucessão (fn )n∈N converge pontualmente para a função f :
X −→ R se e só se:
lim sup fn = lim inf fn = f ,
(3.12)
n→+∞
n→+∞
isto é, se e só se:
∀x ∈ X lim sup (fn (x)) = f (x) = lim inf (fn (x)) .
n→+∞
n→+∞
Demonstração. Atendendo à observação anterior, em particular à fórmula 3.12, é óbvio
que o teorema ficará demonstrado se demonstrarmos que são mensuráveis as funções
lim supn→+∞ fn e lim inf n→+∞ fn . Observemos que se definirmos as sucessões de funções
(gn )n∈N e (hn )n∈N por
∀x ∈ X gn (x) := sup fm (x) e hn (x) := inf fm (x) ,
m≥n
9
m≥n
É sabido que o limite uniforme de funçõpes contı́nuas é uma função contı́nua.
105
Capı́tulo 3
então cada uma destas funções é uma função mensurável. É uma consequência da
proposição 16 dado que, como se verifica imediatamente testando as inclusões no dois
sentidos, para qualquer α ∈ R:
{x ∈ X : gn (x) ≤ α} = {x ∈ X : sup fm (x) ≤ α} =
m≥n
+∞
\
{x ∈ X : fm (x) ≤ α}
m=n
e
{x ∈ X : hn (x) < α} = {x ∈ X : inf fm (x) < α} =
m≥n
+∞
[
{x ∈ X : fm (x) < α} .
m=n
Para finalizar basta agora ver que:
lim sup fn = inf gn e lim inf fn = sup hn ,
n→+∞
n≥N
n→+∞
n≥N
o que com uma justificação semelhante à usada para demosntrar o carácter mensurável
de gn e hn permite concluir imediatamente uma vez que se f é limite pontual da sucessão
(fn )n∈N , então f é igual a uma função mensurável.
3.2.4
As funções com valores reais
Poder-se-ia ter iniciado o estudo das funções mensuráveis considerando primeiramente o
caso das funções tomando valores em R e desenvolvendo o estudo de forma semelhante
ao apresentado aqui para as funções com valores em R. Em nosso entender o teorema 13
mostra que o quadro natural é o que prosseguimos nesta apresentação uma vez que, em
geral, o limite pontual de uma sucessão convergente de funções tomando valores reais é,
quando existe, uma função tomando valores na recta acabada.
As funções com valores reais enquadram-se imediatamente no estudo até agora desenvolvido se observarmos o seguinte. Seja jR a injecção canónica de R em R. Por definição
é a aplicação jR : R −→ R que a cada x ∈ R associa jR (x) = x ∈ R. É uma aplicação
contı́nua se o domı́nio e o contradomı́nio estiverem munidos das respectivas topologias
usuais e é uma aplicação mensurável se se o domı́nio e o contradomı́nio estiverem munidos das respectivas álgebras-σ de Borel associadas às topologias usuais. Considere-se a
seguinte observação fundamental.
∀C ⊂ R jR−1 (C) = {x ∈ R : jR (x) = x ∈ C} = C ∩ R .
(3.13)
A cada função f : X −→ R pode associar-se de forma unı́voca f˜ : X −→ R definida por:
f˜ := jR ◦ f .
O seguinte resultado é agora natural.
Proposição 20. A função f : X −→ R é mensurável de (X, A) em (R, B(R))
se só se f˜ := jR ◦ f for mensurável de (X, A) em (R, B(R))
106
As Funções Mensuráveis
Demonstração. Com efeito se f é mensurável o teorema 11 relativo à composição de
aplicações mensuráveis garante-nos que f˜ é mensurável. No outro sentido verificação é
imediata recorrendo à observação 3.13. Com efeito, suponha-se agora f˜ mensurável e
seja B ∈ B(R). Então, observando que B ∈ B(R),
f −1 (B) = f −1 (B ∩ R) = f −1 (jR−1 (B)) = (jR ◦ f )−1 (B) = f˜−1 (B) ∈ A ,
tal como se pretendia.
Temos agora uma proposição equivalente à proposição 16 que pode ser usada para
definir a mensurabilidade das funções tomando valores reais.
Corolário 5. A função f : X −→ R é mensurável se e só se se verificar uma
qualquer das quatro propriedades equivalentes seguintes.
(1) ∀α ∈ R {x ∈ X : f (x) > α} = f −1 (]α, +∞[) ∈ A.
(2) ∀α ∈ R {x ∈ X : f (x) ≥ α}f −1 ([α, +∞[) ∈ A.
(3) ∀α ∈ R {x ∈ X : f (x) < α} = f −1 (] − ∞α[) ∈ A.
(4) ∀α ∈ R {x ∈ X : f (x) ≤ α} = f −1 (] − ∞α]) ∈ A.
Demonstração. Esta proposição é consequência imediata da proposição 20 e da proposição 16 demonstrada para a recta acabada, se observarmos, por exemplo para (1)
que se f for mensurável:
f −1 (]α, +∞[) = f −1 (]α, +∞] ∩ R) = f −1 (jR−1 (]α, +∞])) = f˜−1 (]α, +∞]) ∈ A , (3.14)
uma vez que f˜ também é mensurável. Reciprocamente se se verifica a condição (1), pela
fórmula 3.14 tem-se que f˜ é mensurável pelo que f também é mensurável.
3.2.5
As funções simples
As funções simples são um análogo, para a teoria do integral de Lebesgue, das funções
em escada para o integral de Riemann. Toda a função de variável real, contı́nua sobre
R, é limite uniforme de funções em escada e essa propriedade é crucial para a definição
do integral de Riemann. Na subsecção seguinte ver-se-á que toda a função mensurável
é limite pontual de funções simples. Esta propriedade permitiu a Lebesgue a definição
do seu integral de forma muito natural.
Definição 24. Uma função simples s : X −→ R é uma função que toma
apenas um número finito de valores.
Vamos pôr em evidência uma forma muito útil de representar uma função simples.
Suponhamos que s toma N ∈ N \ {0} valores distintos entre si e considere-se então
107
Capı́tulo 3
s(X) = {α1 , α2 , . . . , αN } o conjunto desses valores distintos que s toma. Consideremos
agora, por definição, para cada i ∈ {1, . . . , N }:
Ai := {x ∈ X : s(x) = αi }
isto é, o conjunto dos pontos de X em que s toma exactamente o valor αi . Observe-se
que dado que considerámos que os valores que s toma são todos distintos, a famı́lia
(Ai )1≤i≤N forma uma partição de X, ou seja conforme se verifica sem dificuldade:
∀i, j ∈ {1, . . . N } i 6= j ⇒ Ai ∩ Aj = ∅ e
N
[
Ai = X .
i=1
Verifica-se imediatamente que usando as funções indicatrizes se tem que:
s=
N
X
αi IAi ,
(3.15)
i=1
isto é, no caso em que s(X) ⊂ R, s pode representar-se como uma combinação linear
de funções indicatrizes. Reciprocamente, qualquer função que se represente como na
fórmula 3.15 e não fazendo qualquer hipótese sobre os conjuntos (Ai )1≤i≤N toma no
máximo 2N valores, pelo que é uma função simples.
P
Definição 25. Uma função simples dada por s = N
i=1 αi IAi diz-se numa das
suas representações canónicas se e só se se verificam as seguintes condições:
(i) Os valores {α1 , . . . , αN } são distintos dois a dois.
(ii) Os conjuntos (Ai )1≤i≤N formam uma partição de X.
Observação 28. Duas representações canónicas de uma função simples diferem apenas
pela ordenação dos valores que a função toma. Existe apenas uma representação canónica
com a propriedade destes valores {α1 , . . . , αN } estarem ordenados por ordem crescente.
Note-se ainda que existe
PN uma infinidade de representações distintas de uma função simples na forma s =
i=1 βi IBi quando não são feitas as hipóteses (i) e (ii) sobre os
valores {β1 , . . . , βN } e sobre os conjuntos (Bi )1≤i≤N . Um exemplo fácil de verificar para
esta situação é o seguinte:
I[0,1] = I[0,1/2] + I[1/2,1] − I{1/2} .
Tratamos
PN seguidamente a questão natural da mensurabilidade das funções simples.
Seja s = i=1 βi IBi em que para qualquer i ∈ {1, . . . N } se tem βi ∈ R e Bi ∈ A. Se
suposermos que
Bi+ ∩ Bi− = ∅
sendo para i+ , i− ∈ {1, . . . , N }, αi+ = +∞ e αi− = −∞, temos que s é mensurável
por aplicação das proposições 17 e 19. No entanto, podemos ter uma função simples
108
As Funções Mensuráveis
P
s= N
i=1 βi IBi mensurável sem que os conjuntos (Bi )1≤i≤N da sua representação sejam
mensuráveis. Um exemplo dessa situação é-nos dado pela função seguinte. Seja A ∈ A
eB∈
/ A mas tal que B ⊂ A. Então se for s = IA temos que, pelo exemplo 7 relativo às
funções indicatrizes, s é mensurável mas:
s = IB + IA\B ,
em que os dois conjuntos da decomposição B e A \ B não são mensuráveis. A proposição
seguinte ultrapassa este problema.
PN
Proposição 21. Seja s =
i=1 αi IAi dada numa das suas representações
canónicas. Então s é mensurável se e só se:
∀i ∈ {1, . . . , N } Ai ∈ A .
(3.16)
Demonstração. Pelo que atrás ficou dito basta mostrar que a condição 3.16 é necessária.
Suponhamos, então, s mensurável dada na sua representação canónica. Dado que para
cada i ∈ {1, . . . , N } se tem {αi } ∈ B(R), tem-se também que:
Ai = {x ∈ X : s(x) = αi } = s−1 ({αi }) ∈ A ,
pelo que a condição 3.16 se verifica.
Notação 2. Para designar uma função f : X −→ Θ mensurável de (X, A) em (Θ, G)
escrevemos
f ∈ M((X, A), (Θ, G)) ,
e sempre que não haja risco de confusão sobre as estruturas mensuráveis em presença
escrevemos f ∈ M. Note-se que por vezes se usa a notação:
f ∈ mA
para as funções mensuráveis tomando valores reais. Para uma função mensurável f não
negativa, isto é, tal que f ≥ 0 escrevemos f ∈ M+ ((X, A), (R, B(R)) ou f ∈ M+ . Para
designar uma função simples s : X −→ R mensurável escrevemos
s ∈ mS((X, A), (R, B(R)) ,
ou se não houver dúvidas quanto aos espaços mensuráveis em presença, s ∈ mS e, para
uma tal função verificando adicionalmente s ≥ 0, escrevemos s ∈ mS+ . Dado que
qualquer combinação linear de funções tomando um número finito de valores é ainda
uma função tomando um número finito de valores tem-se que S((X, A), (R, B(R)) é um
subespaço vectorial de RX quando este se encontra munido das operações usuais de
adição e multiplicação por um escalar de R.
109
Capı́tulo 3
3.2.6
O teorema de Lebesgue
O teorema fundamental desta secção dá-nos uma caracterização das funções mensuráveis
positivas como as que pertencem ao fecho, relativamente à topologia da convergência
simples, do cone das funções simples mensuráveis positivas com valores reais.
Teorema 14. Seja f ∈ M+ ((X, A), (R, B(R)) e por definição seja a sucessão
dada por:
n −1
n2
X
k
I
k+1 (x)+n I{y∈X:f (y)≥n} (x) .
k
2n {y∈X: 2n ≤f (y)< 2n }
k=0
(3.17)
Então (sn )n≥1 é uma sucessão crescente de funções simples mensuráveis com
valores reais não negativos que converge pontualmente para f sobre X.
∀n ≥ 1 ∀x ∈ X sn (x) :=
Demonstração. Cada uma das funções sn é simples pois toma os n2n valores reais não
negativos do conjunto:
1
n2n − 1
0, n , . . . ,
.
2
2n
O carácter mensurável de cada uma das funções sn decorre de cada uma delas ser representada por uma combinação linear de funções indicatrizes de conjuntos que, pela
proposição 16, são mensuráveis dado que f é mensurável. Com efeito tem-se,
k
k+1
k
k+1
≤f <
= f≥ n \ f≥
∈ A e {f (y) ≥ n} ∈ A .
2n
2n
2
2n
Para demonstrar a convergência pontual da sucessão para f consideremos dois casos.
• Primeiramente x ∈ X tal que f (x) = +∞. Neste caso, para qualquer n ≥ 1 tem-se
que sn (x) = n pelo que limn→+∞ sn (x) = +∞ = f (x).
• Suponha-se agora x ∈ X tal que f (x) < +∞. Seja nx ∈ N tal que f (x) < nx . Para
n ≥ nx e dado que os intervalos
[0,
1
1 2
k k+1
n2n − 1
[,
[
,
[,
·
·
·
,
[
,
[,
·
·
·
,
[
, n[
2n 2n 2n
2n 2n
2n
formam uma partição de [0, n[ temos que:
∃!kx ∈ {0, 1, . . . , n2n − 1} f (x) ∈ [
kx kx + 1
,
[
2n
2n
o que tem como consequência que sn (x) = kx /2n e, portanto,
| sn (x) − f (x) |≤
kx + 1 kx
1
− n = n ,
n
2
2
2
(3.18)
o que implica, finalmente limn→+∞, n≥nx | sn (x) − f (x) |= 0 tal como querı́amos.
110
As Funções Mensuráveis
Verifiquemos agora o carácter monótono da sucessão (sn )n≥1 . Para esse efeito vamos
distinguir três casos.
• Seja x ∈ X tal que f (x) ≥ n + 1. Então tem-se que sn (x) ≤ n e sn+1 (x) = n + 1
logo tem-se que sn (x) ≤ sn+1 (x).
• Seja x ∈ X tal que n ≤ f (x) < n + 1. Pela definição vem imediatamente sn (x) = n
e usando a partição de [0, n + 1[ em que os intervalos têm amplitude 1/2n+1 tem-se
que:
n+1
n2
(n + 1)2n+1
f (x) ∈
,
2n+1
2n+1
pelo que sn+1 (x) = n e logo sn (x) ≤ sn+1 (x).
• Por último seja x ∈ X tal que f (x) < n. Considere-se de novo, com a mesma
justificação que acima, o único kx ∈ {0, 1, . . . , n2n − 1} tal que
kx kx + 1
f (x) ∈ n ,
.
2
2n
Pela definição de sn tem-se que sn (x) = kx /2n . Passando à partição de [0, n + 1[
com os intervalos de amplitude 1/2n+1 que é necessário considerar apra determinar
sn+1 tem-se que:
kx kx + 1
2kx 2kx + 1
2kx + 1 2kx + 2
f (x) ∈ n ,
= n+1 , n+1
∪
, n+1
,
2
2n
2
2
2n
2
o que mostra que sn+1 (x) = 2kx /2n+1 caso f (x) ∈ [2kx /2n+1 , 2kx + 1/2n+1 [ ou
sn+1 (x) = (2kx + 1)/2n+1 no caso de se ter f (x) ∈ [(2kx + 1)/2n+1 , (2kx + 2)/2n+1 [.
Tem-se pois em resumo:
(k
2kx +1
x
x
se f (x) ∈ h 22k
n+1 , 2n+1
kx
2n = sn+1 (x)
h
sn (x) = n ≤ kx
(2kx +1) (2kx +2)
1
2
+
=
s
(x)
se
f
(x)
∈
,
n+1
2n
2n+1
2n+1
2n+1
o que mostra a relação anunciada.
A observação cuidadosa da demonstração anterior mostra-nos que se tem o seguinte.
Corolário 6. Com as mesmas notações e condições do teorema de Lebesgue
anterior, se adicionalmente f for limitada então a sucessão (sn )n≥1 converge
uniformemente para f .
Demonstração. Com efeito se para M ∈ R+ se tiver que para qualquer x ∈ X vale
f (x) ≤ M , então para n ≥ M tem-se de acordo com o que se obteve na fórmula 3.18 na
demonstração acima que
1
sup | sn (x) − f (x) |≤ n
2
x∈X
pelo que a convergência é uniforme tal como foi anunciado.
111
Capı́tulo 3
3.2.7
Variáveis aleatórias
Borrowing an image from Plato, we might say that [probability]
distributions have a daemonic nature: they come simultaneously from
celestial objects (the abstract theory of measure spaces) and terrestrial
objects (analysis on R).
Paul Malliavin, Integration and Probability, Springer Verlag, New
York, 1995, página 229.
O modelo de Kolmogorov para a teoria das probabilidades descreve os espaço de
probabilidade como um espaço mensurável sobre o qual está definida uma medida com
massa total unitária e as variáveis aleatórias como funções mensuráveis sobre este espaço.
No entanto, há uma diferença fundamental entre a prática da análise matemática e a
prática da teoria das probabilidades. É que, regra geral, na teoria das probabilidades e
suas aplicações, nomeadamente à Estatı́stica, as variáveis aleatórias são supostamente
conhecidas pela sua função de distribuição ou lei de probabilidade e não, da forma usual
em análise que consiste na definição do domı́nio, do contradomı́nio e da correspondência
que a um elemento do domı́nio associa um elemento do contradomı́nio. Tal deve-se a
que, regra geral o domı́nio de definição de uma variável aleatória, que é o espaço de probabilidade, não é conhecido explicitamente. Com efeito, regra geral, em probabilidades
há interesse em determinar a probabilidade de acontecimentos sem que se necessite, por
isso, de definir o que são as realizações elementares de um tal acontecimento.
Assim, por exemplo, qunado se diz que uma dada caracterı́stica numérica X de uma
população tem lei normal N(m, σ), quer dizer-se que
∀x ∈ R Probabilidade[X ≤ x] = √
1
2πσ 2
Z
x
exp(−
−∞
(t − m)2
)dt ,
2σ 2
sem que tenha sido necessário definir o domı́nio de X e a regra ou expressão analı́tica
que representa X.
Nesta subsecção consideramos que o espaço de medida subjacente ao nosso estudo
das funções mensuráveis é um espaço de probabilidade (Ω, F, P). Seja (Θ, B) um espaço
mensurável.
Definição 26. Uma variável aleatória X : Ω −→ Θ é uma aplicação mensurável de (Ω, F) em (Θ, G).
Tal como se referiu acima, na teoria das Probabilidades e suas aplicações acontece
frequentemente que o conjunto Ω, domı́nio de definição da variável aleatória X, não está
definido concretamente. Em consequência, em geral, não faz sentido presumir que a
variável aleatória X é descrita por uma correspondência que a ω ∈ Ω associa X(ω) ∈ Θ.
Para descrever X considera-se a distribuição dos valores que X toma em Θ. Tal como
veremos em seguida, esta distribuição é concretizada por uma medida de probabilidade
sobre o contradomı́nio da variável aleatória.
112
As Funções Mensuráveis
Definição 27. Seja X : Ω −→ Θ uma variável aleatória. A lei de probabilidade de X, que representamos por LX , é definida por:
∀G ∈ G LX [G] = P[X −1 (G)] .
(3.19)
Proposição 22. A lei de probabilidade da variável aleatória X é uma medida
de probabilidade sobre (Θ, G).
Demonstração. A verificação é imediata uma vez que:
• LX [∅] = P[X −1 (∅)] = P[∅] = 0.
• Sendo (Gn )n∈N uma sucessão de elementos de G dois a dois disjuntos tem-se, pelas
propriedades usuais da imagem recı́proca e por P ser uma medida de probabilidade,
que:
" +∞
#
"
#
" +∞
# +∞
+∞
[
[
[
X −1
−1
LX
Gn = P X (
Gn ) = P
X (Gn ) =
P X −1 (Gn )
n=0
n=0
=
+∞
X
n=0
n=0
LX [Gn ] .
n=0
• LX [Θ] = P[X −1 (Θ)] = P[Ω] = 1,
Estão assim verificados os axiomas de medida de probabilidade.
Exemplo 8 (Variáveis discretas). Seja X : Ω −→ Θ uma variável aleatória tomando como
valores os elementos de uma sucessão (αn )n∈N de tal forma que
∀n ∈ N P[X = αn ] = pn > 0 ,
+∞
X
pn = 1 .
n=0
Verifica-se imediatamente que a lei de X é dada por:
LX =
+∞
X
pn δαn ,
(3.20)
n=0
em que para a ∈ Θ, δa é a medida de Dirac concentrada em a. Com efeito, a fórmula 3.20
define uma medida de probabilidade e para qualquer elemento da álgebra-σ do contradomı́nio de X, seja G ∈ G:
" +∞
#
[
LX (G) = P[X −1 (G)] = P[{ω ∈ Ω : X(ω) ∈ G}] = P
{ω ∈ Ω : X(ω) = αn ∈ G} =
n=0
=
X
αn ∈G
pn =
+∞
X
pn δαn (G) .
n=0
113
Capı́tulo 3
Fica assim estabelecido que as variáveis aleatórias discretas com valores num espaço
mensurável qualquer, têm como leis as medidas de probabilidade que se escrevem como
combinações convexas (generalizadas) de medidas de Dirac.
Observação 29. São exemplos de variáveis aleatórias discretas tomando valores reais, as
variáveis com lei de Bernoulli, com lei binomial, com lei binomial negativa, etc.
Uma forma alternativa de conhecer a lei de probabilidade de uma variável aleatória
tomando valores reais é a que é dada pela função de distribuição da variável aleatória.
Definição 28. Seja X : Ω −→ R uma variável aleatória. A função de distribuição FX : R −→ [0, 1] da variável aleatória X é definida por:
∀x ∈ R FX (x) = P[X ≤ x] = P[X −1 (] − ∞, x])] = LX (] − ∞, x]) .
(3.21)
As funções de distribuição gozam das seguintes propriedades caracterı́sticas.
Proposição 23. Seja FX uma função de distribuição de uma variável aleatória
X. Então:
(i) limx→−∞ FX (x) = 0 e limx→+∞ FX (x) = 1
(ii) A função FX é contı́nua à direita em todo o ponto do seu domı́nio, isto é,
∀x ∈ R limh→0, h>0 FX (x + h) = FX (x) .
(iii) A função FX é não-decrescente, isto é:
∀x, y ∈ R x ≤ y ⇒ FX (x) ≤ FX (y) .
Demonstração. Estas propriedades resultam da igualdade entre o membro mais à direita
e o membro mais à esquerda na fórmula 3.21 e da observação seguinte, que formulamos
de forma pouco rigorosa. Sobre R podemos usar sucessões para as questões de limites e
continuidade. Assim para (i) seja (xn )n∈N ↓ −∞.
" +∞
#
\
lim FX (x) = lim FX (xn ) = lim LX (] − ∞, xn ]) =(a) LX
] − ∞, xn ] =
x→−∞
n→+∞
n→+∞
n=0
= LX [∅] = 0 ,
em que (a) decorre da monotonia da medida de probabilidade LX , para sucessões decrescentes, dado que se tem: (] − ∞, xn ])n∈N ↓ ∅. Sendo (xn )n∈N ↑ +∞ tem-se, da mesma
forma, que:
" +∞
#
[
lim FX (x) = lim FX (xn ) = lim LX (] − ∞, xn ]) =(b) LX
] − ∞, xn ] =
x→+∞
n→+∞
n→+∞
= LX [] − ∞, +∞[] = 0 ,
114
n=0
As Funções Mensuráveis
em que (b) decorre da monotonia da medida de probabilidade LX , para sucessões crescentes, dado que se tem: (] − ∞, xn ])n∈N ↑] − ∞, +∞[. Para (ii), seja (hn )n∈N ↓ 0 uma
sucessão de números positivos. Tem-se que:
lim
h→0, h>0
FX (x + h) = lim FX (x + hn ) = lim LX (] − ∞, x + hn ]) =(c)
n→+∞
n→+∞
" +∞
#
\
= LX
] − ∞, x + hn ] = LX [] − ∞, x]] = FX (x) ,
n=0
em que, tal como acima, (c) resulta da monotonia da medida de probabilidade LX , para
sucessões decrescentes, dado que se tem: (] − ∞, x + hn ])n∈N ↓] − ∞, x]. Finalmente,
para (iii), observe-se que pela monotonia da medida LX :
x ≤ y ⇒] − ∞, x] ⊆] − ∞, y] ⇒ LX (] − ∞, x]) ≤ LX (] − ∞, y]) ⇔ FX (x) ≤ FX (y) ,
tal como querı́amos demonstrar.
Observação 30. Pode demonstrar-se 10 que se F : R −→ [0, 1] verificar as propriedades
(i),(ii) e (iii) da proposição 23 acima, então existe um espaço de probabilidade e uma
variável aleatória X sobre esse espaço de probabilidade tal que, sendo FX a função de
distribuição de probabilidade de X, se tem FX ≡ F .
É evidente a partir da fórmula 3.21 da definição acima que se a lei LX de uma
variável aleatória for dada então, a função de distribuição de probabilidade de X também
é conhecida. O resultado seguinte mostra que a função de distribuição caracteriza a lei
de probabilidade de uma variável aleatória.
Proposição 24. Seja X uma variável aleatória com função de distribuição de
probabilidade FX . Então FX determina completamente LX a lei de probabilidade de X.
Demonstração. Considere-se J o π-sistema de intervalos de R, dado por:
J = {] − ∞, a] : a ∈ R} .
e verificando, como é sabido, σ(J) = B(R). Seja L : J −→ [0, 1] definida por:
∀J =] − ∞, a] ∈ J , L(] − ∞, a]) := FX (a) .
Por uma demonstração semelhante à já exposta na cosntrução das medidas de LebesgueStieltjes, pode mostrar-se que existe uma medida L de probabilidade sobre B(R) que
estende a função L definida sobre J. Uma vez que L e LX coincidem sobre J o teorema de
unicidade mostra que coincidem sobre B(R), pelo que a função de distribuição determina
a lei de probabilidade.
10
Veja-se o capı́tulo sobre a construção das medidas de Borel-Stieltjes.
115
Capı́tulo 3
Exemplo 9 (Variáveis contı́nuas). Este exemplo será generalizado depois de estudado o
integral de Lebesgue. Seja X : Ω −→ R uma variável aleatória tal que existe uma função
contı́nua e Riemann integável, fX definida e não negativa sobre R, verificando
Z
+∞
fX (u)du = 1
−∞
É imediato verificar, usando apenas as propriedades do integral de Riemann das funções
contı́nuas, que a função F definida por
Z x
∀x ∈ R F (x) =
fX (u)du.
−∞
satisfaz todas as propriedades caracterı́sticas de uma função de distribuição pelo faz
sentido considerar FX ≡ F . Pela proposição 24 a lei de probabilidade de X fica completamente determinada dizendo-se então que X tem uma lei contı́nua. A verdadeira
justificação desta designação aparecerá com o teorema de Lebesgue Radon-Nicodym.
Observação 31. São exemplos de variáveis aleatórias contı́nuas, as variáveis com lei de
normal, com lei gama, com lei qui-quadrado, etc, em suma as que, segundo a designação
do curso de Probabilidades e Estatı́stica, admitem densidade.
3.3
Apêndice
Para maior comodidade do leitor apresentamos seguidamente o seguinte resultado importante 11 que foi utilizado pelo menos duas vezes no decurso da exposição. Este
resultado mostra-nos que há duas formas equivalentes de transportar uma álgebra-σ por
imagem recı́proca por meio de uma aplicação quando se conhece uma famı́lia geradora
da álgebra-σ: ou antes de gerar a álgebra-σ ou depois.
Lema 3. Seja f : X −→ Y e E ⊂ P(Y ) Então:
f −1 (σ(E)) = σ(f −1 (E)) .
(3.22)
Demonstração. Tem-se que sendo σ(E) uma álgebra-σ, f −1 (σ(E)) a sua imagem recı́proca
por uma qualquer aplicação f é também uma álgebra-σ, pelo que:
E ⊂ σ(E) ⇒ f −1 (E) ⊂ f −1 (σ(E)) ⇒ σ(f −1 (E)) ⊂ f −1 (σ(E))
Para a demonstração da inclusão no outro sentido considere-se:
G := {F ⊂ Y : f −1 (F ) ∈ σ(f −1 (E))}
11
116
Veja-se para uma referência a este resultado a obra de Laurent Schwartz [4][p. 42].
As Funções Mensuráveis
É fácil verificar que G é uma álgebra-σ. É evidente que E ⊂ G uma vez que se E ∈ E
então f −1 (E) ∈ f −1 (E) ⊂ σ(f −1 (E)). Em consequência, σ(E) ⊂ G o que pela definição
de G significa:
∀F ∈ σ(E) f −1 (F ) ∈ σ(f −1 (E))
ou ainda de forma equivalente:
f −1 (σ(E)) ⊂ σ(f −1 (E))
ou seja, precisamente, a inclusão no sentido que faltava verificar.
3.4
Exercı́cios
[1]
Exercı́cio 123. Considere a função f : R → R definida por

 −1, x ≤ −1
x,
−1 ≤ x ≤ 1
f (x) =

3,
x>1
1. Estude f quanto à continuidade.
2. Considerendo em R os borelianos, indique, justificando, se f é mensurável.
Exercı́cio 124. Considere a função f : R → R definida por

x≥1
 1,
1
1
,
≤x<
f (x) =
n+1
 n+1
0,
x≤0
[1]
1
, ∀n
n
≥1
1. Justifique se f é contı́nua em R. Indique os pontos onde f não é contı́nua.
2. Prove que f é mensurável.
Exercı́cio 125. Seja (X, P(X)) o espaço mensurável discreto. Indique, justificando, quais as funções [1]
f : X → R que são mensuráveis. Seja (X, {∅, X}) o espaço mensurável grosseiro. Indique, justificando,
quais as funções f : X → R que são mensuráveis.
Exercı́cio 126. Consideremos em N a σ-álgebra gerada por: {{i ∈ N : i = 2n, n ∈ N}}. Justifique se [1]
as funções f : N → R definidas por
1. f (n) =
1
,
n+1
2.
f (n) =
n2 ,
−3,
n par
n ı́mpar
,
3. f (n) = (−1)n ,
são mensuráveis.
Exercı́cio 127. Considere a sequência de funções fn (x) = xn , x ∈ [0, 1[. Justifique se (fn )n∈N é uma [1]
sequência dePfunções mensuráveis. Prove, de duas formas distintas, que a função f : [0, 1[→ R definida
por f (x) = n≥0 fn (x) é mensurável.
117
Capı́tulo 3
Exercı́cio 128. Seja (X, F) um espaço mensurável e f, g : X → R duas funções mensuráveis. Mostre [1]
que, para todo a ∈ R, {x ∈ X : f (x) = a} é um conjunto mensurável. Conclua que {x ∈ X : f (x) = g(x)}
é um conjunto mensurável. Será {x ∈ X : f (x) 6= g(x)} também um conjunto mensurável? Justifique.
Exercı́cio 129. Dado um conjunto X e subconjuntos A1 , A2 , ..., seja
A+ = lim sup An =
∞
\
[
An
k=1 n≥k
e
A− = lim inf An =
∞ \
[
An .
k=1 n≥k
Sejam f+ , f− e fn as funções indicatrizes dos conjuntos A+ , A− e An , respectivamente. Prove que
f+ = lim sup fn e f− = lim inf fn .
[2]
Exercı́cio 130.
1. Seja X um conjunto, F uma σ-álgebra de subconjuntos de X. Uma aplicação
f = (f1 , ..., fn ) de X em Rn diz-se mensurável se cada uma das funções coordenadas, fi , é
mensurável. Mostre que f é mensurável se, e só se f −1 (B) ∈ F para todo o Boreliano B ⊆ Rn .
2. Seja g uma função real definida em Rn . Então g diz-se Borel-mensurável se, para todo o real a,
o conjunto {x ∈ Rn ; g(x) > a} é um conjunto de Borel. Prove que, se f : X −→ Rn é mensurável
e g : Rn −→ R é Borel-mensurável, então g ◦ f : X −→ R é mensurável.
[2]
Exercı́cio 131.
1. Seja fn : X −→ R, n = 1, 2, ...,
uma sequência de funções mensuráveis. Mostre
que o conjunto x ∈ X : (fn (x))n∈N∗ converge é mensurável.
n
o
P
Rn (ω)
2. Da alı́nea anterior deduza que o conjunto ω ∈]0, 1] : ∞
é convergente é um subconn=1
n
junto mensurável de ]0, 1].
[3]
Exercı́cio 132. Seja f : R → R uma função monótona crescente. Mostre que f é mensurável.
[3]
Exercı́cio 133. Seja (X, F, µ) um espaço de medida com a seguinte propriedade:
∀B ∈ F, A ⊆ X (A ⊆ B ∧ µ(B) = 0) ⇒ (A ∈ F ∈ ∧µ(A) = 0) .
Duas funções f e g dizem-se iguais µ-quase por toda a parte se f = g, excepto num conjunto contido
num conjunto mensurável com medida zero. Considere duas funções f e g definidas em X. Mostre que
se f = g µ-quase por toda a parte e f é mensurável, então g também é mensurável.
[2]
Exercı́cio 134. Uma função f : R −→ R diz-se semi-contı́nua superiormente em x se:
∀ε > 0 ∃δ > 0 : |x − y| < δ ⇒ f (y) < f (x) + ε
Seja f semi-contı́nua superiormente em todos os pontos de R
1. Mostre que, para todo o a ∈ R, o conjunto {x ∈ R : f (x) < a} é aberto. Conclua que f é
mensurável de R em R
2. Enuncie e demosntre um resultado do mesmo tipo para f semi-contı́nua inferiormente.
118
[2]
As Funções Mensuráveis
[3]
Exercı́cio 135. Seja (X, F, µ) um espaço de medida com µ(X) < ∞. Uma sequência de funções
mensuráveis fn : X −→ R n = 1, 2, ..., diz-se que converge em medida para zero se para todo o ε > 0
lim µ{x ∈ X; |fn (x)| > ε} = 0.
n→∞
1. Prove que, se fn converge pontualmente para zero excepto num conjunto de medida nula, então
fn converge em medida para zero.
2. Mostre que o recı́proco da alinea anterior não é válido.
Indicação : Para a alı́nea 2) considere, X = [0, 1], F os Borelianos de [0, 1], e µ a medida de
Lebesgue. Sejam A1 = [0, 21 ], A2 = [ 12 , 1], A3 = [0, 14 ], A4 = [ 14 , 42 ], A5 = [ 24 , 34 ], A6 = [ 34 , 1] A7 =
[0, 81 ], A8 = [ 81 , 28 ], e assim sucessivamente. Seja fn a função indicatriz de An .)
0
Exercı́cio 136. [2]. Seja um espaço mensurável, (X, A), um subconjunto X de X e a σ-álgebra [3]
0
0
0
traço de A sobre X , a que chamaremos A . Mostre que a injecção canónica jX 0 definida em X
0
0
0
e tomando valores em X é mensurável de (X , A ) em (X, A). Mostre que a σ-álgebra A é a mais
0
pequena σ-álgebra sobre X para a qual a injecção canónica jX 0 é mensurável.
Exercı́cio 137. Seja (X, A) um espaço mensurável e Y um conjunto, uma aplicação f ∈ Y X , uma [2]
famı́lia de partes de Y a que chamaremos F e finalmente a σ-álgebra gerada por F a que chamaremos
D. Mostre que f é mensurável de (X, A) em (Y, D) se e só se tivermos f −1 (F) ⊆ A.
Indicação : É uma consequência imediata das definições.
0
0
Exercı́cio 138. Sejam dois espaços topológicos (X, τ ) e (X , τ ) que consideraremos munidos das [3]
0
0
respectivas σ-álgebras borelianas B e B . Mostre que se uma aplicação f ∈ (X )X é contı́nua de
0
0
0
0
(X, τ ) em (X , τ ) então a aplicação é mensurável de (X, B) e (X , B ). Que é que pode dizer sobre a
proposição recı́proca?
Indicação : Relembre as definições de continuidade (global) e de mensurabilidade.
Exercı́cio 139. Considere o espaço X = [0, 1[ munido da topologia usual, B a σ-álgebra de Borel [2]
respectiva, e A a σ-álgebra formada pelos subconjuntos de X que ou são numeráveis ou têm o complementar numerável. Mostre que a aplicação identidade de X é contı́nua de (X, τu ) nele próprio mas
que não é mensurável de (X, A) em (X, B).
Indicação : A σ-álgebra A foi estudada num exercı́cio anterior. É a σ-álgebra gerada pela famı́lia
dos conjuntos singulares.
Exercı́cio 140. Dê condições necessárias e suficientes para que uma aplicação de um espaço mensurável [2]
(X, A) em R (respectivamente R) munido(s) da σ-álgebra Boreliana usual, seja mensurável.
Indicação : Consulte o livro [3] na página 13.
Exercı́cio 141. Enuncie e demonstre o resultado relativo à mensurabilidade da aplicação composta.
Indicação : Veja o livro ([3]) na página 10.
[2]
Exercı́cio 142. Sejam dois espaços mensuráveis (X, A) e (Y, C), uma aplicação f ∈ Y X , mensurável [3]
0
0
do primeiro espaço no segundo, um subconjunto X de X e A a σ-álgebra traço da σ-álgebra A no
0
0
subconjunto X . Mostre que f/X 0 , a restrição da aplicação em questão, ao conjunto X , é mensurável
0
0
de (X , A ) em (Y, C).
Indicação : Utilize o resultado sobre a composição de funções mensuráveis aplicado à função f e
à injecção canónica.
119
Capı́tulo 3
0
Exercı́cio 143. Sejam dois espaços mensuráveis (X, A) e (Y, C), um subconjunto Y de Y , a σ- [3]
0
0
0
álgebra traço de C no subconjunto Y , que denotaremos por C e duas aplicações g ∈ (Y )X e uma
outra f ∈ Y X tais que:
∀x ∈ X f (x) = g(x) .
0
0
Mostre que f é mensurável de (X, A) em (Y, C) se e só se g fôr mensurável de (X, A) em (Y , C ).
Formule o resultado particular correspondente à recta real e à recta acabada.
Indicação : Simples consequência das definições.
[3]
Exercı́cio 144. Sejam dois espaços mensuráveis (X, A) e (Y, C). Seja (Xn )n∈N uma sucessão de
elementos da σ-álgebra A tal que se tenha X = ∪n∈N Xn . Considere uma aplicação f de X em Y e
para cada n em N a σ-álgebra An , obtida pelo traço de A em Xn . Mostre que f é mensurável de
(X, A) em (Y, C) se e só se para cada n em N a restrição de f a Xn , a que chamaremos f/Xn , fôr
mensurável de (Xn , An ) no espaço (Y, C)
Indicação : Consequência de resultados anteriores.
[2]
Exercı́cio 145. Seja f uma aplicação mensurável de (X, A) em R (este último espaço munido da
σ-álgebra de Borel). Seja A > 0, mostre que a aplicação fA (a que chamaremos a truncatura de f
ao nı́vel A) definida de X em R por:

se |f (x)| ≤ A
 fA (x) = f (x)
fA (x) = A
se f (x) > A

fA (x) = −A
se f (x) < −A
é mensurável de (X, A) em R.
Indicação : Exprima a aplicação truncada em função da aplicação dada usando uma partição do
domı́nio e as funções indicatrizes associadas a essa partição.
[2]
Exercı́cio 146. Sejam (X, A) um espaço mensurável e duas aplicações f e g, mensuráveis de (X, A)
em R. Mostre que os conjuntos seguintes são mensuráveis.
{x ∈ X : f (x) < g(x)}; {x ∈ X : f (x) ≤ g(x)}; {x ∈ X : f (x) = g(x)} .
Indicação : Use o critério fundamental de mensurabilidade para as funções tomando valores reais.
[3]
Exercı́cio 147. Mostre que a aplicação definida de R nele próprio, que ao ponto x associa o ponto
αx ( em que α ∈ R), é mensuravel, quando ambos o domı́nio e o espaço imagem , estão munidos com
a σ-álgebra de Borel.
Indicação : Há que distinguir os casos em que o factor da acção multiplicativa generalizada é finito,
dos casos em que não é.
[3]
Exercı́cio 148. Sejam um espaço mensurável (X, A) e duas aplicações mensuráveis f e g, do espaço
dado na recta acabada. Mostre que são mensuráveis as seguintes aplicações:
αf (α ∈ R)
f ×g
|f |
f + g sempre que tenha sentido
sup(f, g) e inf(f, g)
f + e f − (resp. as partes positiva e negativa de f )
Indicação : Consulte o livro ([3]) para uma proposição semelhante só que formulada para a recta
real.
120
As Funções Mensuráveis
[1]
Exercı́cio 149. Mostre que uma aplicação arbitrária tomando valores na recta real (ou na recta acabada)
é mensurável se e só se as suas partes positiva e negativa o forem. Mostre ainda que é possı́vel ter |f |
mensurável, sem que a aplicação f seja mensurável.
Indicação : Consulte a referência [1] para uma ideia sobre esta observação.
[1]
Exercı́cio 150. Seja (fn )n∈N uma sucessão de aplicações mensuráveis tomando valores em R. Mostre
que então as aplicações definidas a seguir são também mensuráveis:
sup fn ;
inf n∈N fn
n∈N
lim sup fn ;
lim inf n∈N fn
n∈N
Mostre finalmente que se a sucessão converge simplesmente ou pontualmente para uma dada função,
então essa função é mensurável.
Exercı́cio 151. Mostre (usando o critério de mensurabilidade para as funções tomando valores reais) [1]
que uma função indicatriz de um conjunto é mensurável sse o conjunto que é indicado é mensurável.
Indicação : Baseie-se na demonstração dada na aula.
Exercı́cio 152. Mostre que o transladado de um boreliano da recta real é ainda um boreliano da recta [2]
real. O que é que pode dizer relativamente aos transladados dos borelianos da recta acabada?
Indicação : Descreva o transladado, como imagem recı́proca por uma aplicação mensurável, de um
conjunto mensurável.
Exercı́cio 153. Mostre que toda a função mensurável limitada tomando valores reais, é limite uniforme [2]
de uma sucessão de funções simples.
Indicação : Estude atentamente a demonstração dada na aula para o resultado correspondente no
livro [3] (teorema 1.17).
Exercı́cio 154. Seja X uma variável aleatória mensurável no espaço de probabilidade (Ω, F, P). Mostre [2]
que se for π(R) = {] − ∞, x] : x ∈ R}, então:
σ(X) = σ(X −1 (π(R))) .
Exercı́cio 155. Seja (X, A) um espaço mensurável e (fn )n∈N uma sucessão de funções mensuráveis [3]
de X em R. Mostre que:
1. {x ∈ X : (fn (x))n∈N converge} ∈ A .
2. Seja f uma função mensurável definida em X e tomando valores em R. Mostre que o conjunto
seguinte é mensurável:
x ∈ X : lim fn (x) = f (x) .
n∈N
Indicação : Inspire-se da demonstração de um resultado semelhante dado na aula.
Exercı́cio 156. Seja (X, A, µ) um espaço mensurável não completo.
[3]
1. Mostre que existe uma sucessão (fn )n∈N de aplicações mensuráveis de X em R e uma outra
aplicação de X em R, não mensurável tais que a sucessão (fn )n∈N converge µ-quase por toda
a parte, para a função f .
121
Capı́tulo 3
2. Suponha agora que o espaço (X, A, µ) é completo e que a sucessão (fn )n∈N , converge µ-qptp
para a função f . Mostre que então se tem que f é mensurável.
Indicação : Relembre a definição de uma propriedade válida µ-quase por toda a parte e a definição
de espaço mensurável completo.
e µ
Exercı́cio 157. Seja (X, A, µ) um espaço mensurável e (X, A,
e), o espaço completado deste. Mostre [3]
e
que dada uma aplicação f do espaço X na recta acabada R, que seja A-mensurável,
existe então uma
aplicação g do espaço X na recta acabada R que é A-mensurável e tal que f (x) = g(x) mas só µ-qptp
Indicação : Construa a aplicação g de forma a que esta verifique o que é pretendido.
Bibliografia
[1] Jean Genet. Mesure et Intégration. Librairie Vuibert, Paris, 1976.
[2] Etiènne Ménard. Mesure et Intégration, Notes pour Travaux Dirigés avec rappels de
cours. Université de Rouen, 1979.
[3] Walter Rudin. Real and complex analysis. McGraw-Hill Book Co., New York, third
edition, 1987.
[4] Laurent Schwartz. Analyse II. Hermann, Paris, 1993.
122
Capı́tulo 4
A Integração à Lebesgue
Rb
In Chapter 7, he [Lebesgue] reveals his new idea. To define a f (x)dx,
he does not follow Newton, Leibnitz, Cauchy, and Riemann who
partitioned the x-axis between a and b. Instead, he lets l be the
infimum of the values of f , L the supremum, and then partitions the
y-axis between l and L. That is, instead of cutting the area using a
finite number of vertical cuts, he takes a finite number of horizontal
cuts.
David M. Bressoud A Radical Approach to Lebesgue’s Theory of
Integration. Cambridge University Press, Cambridge, 2008, página 132.
4.1
O Integral de Lebesgue
Fixamo-nos como objectivo para esta parte da exposição a definição de uma noção de
integral que supere algumas das limitações encontradas com o integral de Riemann.
Vamos proceder por etapas. Definimos primeiro o integral para as funções indicatrizes de conjuntos mensuráveis, depois para funções simples, seguidamente para funções
mensuráveis positivas e por último para as funções integráveis que será a classe mais alargada na qual o integral fará sentido. Em cada uma destas etapas assegurar-nos-emos da
coerência das definições apresentadas.
Como motivação, considere-se o espaço de Borel ([a, b], B([a, b])) munido da medida
de Lebesgue λ e f = I[a,b] a função indicatriz do intervalo fechado limitado [a, b]. Sabemos
Rb
que o integral de Riemann de f , que representamos por a f (x)dx, pode ser calculado
dando como resultado:
Z b
f (x)dx = b − a .
a
Observemos que,sendo λ a medida de Lebesgue, b−a = λ([a, Rb]) pelo que se pretendermos
definir o integral de Lebesgue de f , que representamos por R f dλ, de forma a coincidir
123
Capı́tulo 4
neste caso simples com o integral de Riemann, deveremos ter:
Z
Z b
f dλ =
f (x)dx = b − a = λ([a, b]) .
a
R
Esta observação torna natural a próxima definição. De ora em diante, salvo indicação
expressa em contrário, consideramos subjacente à nossa exposição um espaço de medida
(X, A, µ).
Definição 29. Seja A ∈ A. Então,
(i) O integral de Lebesgue de IA considerada como função de X em R,
relativamente à medida µ é dado por:
Z
IA dµ = µ(A) .
(4.1)
X
(ii) Para qualquer α ∈ R, o integral de Lebesgue de αIA considerada como
função de X em R, relativamente à medida µ é dado por:
Z
αIA dµ = α × µ(A) ,
(4.2)
X
com a convenção que 0 × +∞ = +∞ × 0 = 0 × −∞ = −∞ × 0 = 0 sempre
que uma destas quatro eventualidades se produzir na expressão α × µ(A).
Em toda a nossa exposição seguinte valerá esta convenção.
Observação 32. A fórmula de definição 4.2 é justificada se pretendermos que se verifique
genericamente, isto é de momento sem quaisquer hipóteses adicionais, a fórmula:
Z
Z
αf dλ = α ×
f dλ .
X
X
Definição 30. Seja s : X −→ R uma função simples mensurável não negativa
dada numa das suas representações canónicas por:
s=
N
X
αi IAi ,
i=1
sendo {α1 , . . . , αN } o conjunto dos N valores distintos que toma s e para cada
1 ≤ i ≤ N , Ai = s−1 ({αi }). O integral de Lebesgue de s relativamente à
medida µ é dado por:
Z
N
X
s dµ =
αi µ(Ai ) ,
(4.3)
X
124
i=1
A Integração à Lebesgue
Observação 33. O integral de Lebesgue de uma função simples mensurável e não negativa, dada numa qualquer das suas representações canónicas, está sempre bem definido
+
como um elemento de R . Não depende da representação canónica usada na fórmula 4.3,
representação que está associada a uma dada indexação do conjunto dos valores que s,
uma vez que a soma no membro à direita nessa fórmula é comutativa.
A proposição seguinte permitir-nos-á mostrar que o integral de Lebesgue definido,
até agora, para as funções simples mensuráveis positivas opera linearmente.
Proposição 25. Sejam s e r duas funções simples mensuráveis positivas.
Então:
Z
Z
Z
r dµ .
(4.4)
s dµ +
(s + r) dµ =
X
X
X
Demonstração. Consideremos para cada uma das funções s e r representações canónicas
para as quais sabemos que os integrais estão definidos sem ambiguidade.
s=
N
X
αi IAi e r =
i=1
M
X
βi IBj .
j=1
M
Dado que se tem X = ∪N
i=1 Ai = ∪j=1 Bj podemos obter novas representações de s e r
como combinação linear de indicatrizes de conjuntos dois a dois disjuntos da seguinte
forma.
s=
N
X
i=1
αi IAi ∩X =
N
X
αi IAi ∩(∪M
j=1 Bj )
=
i=1
N
X
i=1
αi
M
X
IAi ∩Bj =
j=1
N X
M
X
αi IAi ∩Bj
i=1 j=1
e procedendo do mesmo modo tem-se também
r=
N X
M
X
βj IAi ∩Bj ,
i=1 j=1
pelo que naturalmente podemos escrever sem qualquer ambiguidade:
s+r =
N X
M
X
(αi + βj ) IAi ∩Bj .
i=1 j=1
A representação acima de s + r é dada como combinação linear de indicatrizes de elementos de A que tomados em conjunto formam uma partição de X. Para obtermos,
a partir dessa representação, a representação canónica necessitamos apenas eliminar os
eventuais valores idênticos na famı́lia (αi + βj )1≤i≤N,1≤j≤M . Sejam então {γ1 , . . . , γP }
os valores distintos que s + r toma e seja por definição:
∀k ∈ {1, . . . , P } Ik := {(i, j) ∈ {1, . . . , N } × {1, . . . , M } : αi + βj = γk }
125
Capı́tulo 4
Podemos agora agrupar todos os elementos da famı́lia a famı́lia (αi + βj )1≤i≤N,1≤j≤M
sobre os quais s + r toma o mesmo valor e temos então por definição:
[
(Ai ∩ Bj ) .
∀k ∈ {1, . . . , P } Ck :=
(i,j)∈Ik
Obtem-se assim uma das representações canónicas de s + r:
X
s+r =
(αi + βj ) IAi ∩Bj =
(i,j)∈{1,...,N }×{1,...,M }
=
P
X
k=1

X
(αi + βj ) IAi ∩Bj =
k=1 (i,j)∈Ik

γk 
P
X
X
IAi ∩(Bj )  =
P
X
γk ICk .
k=1
(i,j)∈Ik
Podemos agora aplicar a definição 30 na igualdade mais à direita e na igualdade mais à
esquerda para obtermos o resultado pretendido na seguinte cadeia de igualdades.


Z
P
P
X
X
[
(s + r)dµ =
γk µ(Ck ) =(a) =
γk µ 
(Ai ∩ Bj ) =(b)
X
k=1
=
P
X
k=1
=
k=1
X
γk
(i,j)∈Ik
µ(Ai ∩ Bj ) =(c)
P
X
X
(αi + βj ) µ(Ai ∩ Bj ) =(d)
k=1 (i,j)∈Ik
(i,j)∈Ik
N X
M
X
(αi + βj ) µ(Ai ∩ Bj ) =
i=1 j=1
=
N X
M
X
αi µ(Ai ∩ Bj ) +
i=1 j=1
=
N
X
N
X
βj µ(Ai ∩ Bj ) =
i=1 j=1

αi µ Ai ∩ (
i=1
=
N X
M
X
M
[
j=1
αi µ(Ai ) +
i=1
M
X

Bj ) +
M
X
βj µ (
j=1
N
[
Ai ) ∩ Bj
i=1
Z
βj µ(Bj ) =
j=1
!
Z
sdµ +
X
rdµ .
X
com as seguintes justificações.
(a) Pela definição dos conjuntos Ck .
(b) Dado que os conjuntos (Ai ∩ Bj )(i,j)∈Ik são dois a dois disjuntos.
(c) Uma vez que para (i, j) ∈ Ik se tem que γk = αi + βj .
(d) Dado que somar sobre ∪Pk=1 Ik = {(i, j) : 1 ≤ i ≤ N, 1 ≤ j ≤ M }.
126
=
A Integração à Lebesgue
Este resultado tem como consequência que o integral de uma função simples não
depende da representação da função como combinação linear de indicatrizes que tomamos
para efectuar o cálculo do integral.
Corolário 7. Seja s uma função simples mensurável positiva dada numa qualquer das suas representações
s=
Q
X
δl IDl ,
l=1
com, para cada l ∈ {1, . . . Q}, Dl ∈ A. Então tem-se que:
Z
s dµ =
X
Q
X
δj µ(Dl ) .
l=1
Demonstração. Dado que para cada l ∈ {1, . . . Q} se tem que δl IDl é uma função simples
mensurável positiva, tem-se o resultado pretendido aplicando a proposição 25 anterior.
Observação 34. Sublinhamos a importância da hipótese da mensurabilidade dos conjuntos que intervêem nas representações das funções simples. Sem essa hipótese não seria
possı́vel determinar as medidas desses conjuntos e, em consequência, o integral da função
simples não poderia ser definido. Essa hipótese tem como consequência que as funções
simples para as quais se define o integral de Lebesgue são necessariamente mensuráveis.
Veremos seguidamente que esta observação se mantêm para todas as funções para as
quais o integral de Lebesgue está definido.
O resultado seguinte mostra-nos uma propriedade importante do integral de Lebesgue
das funções simples, a propriedade da monotonia do integral relativamente à função a
integrar.
Proposição 26. Sejam s, r ∈ S+ ((X, A), (R, B(R)) tais que s ≤ r. Então:
Z
Z
s dµ ≤
r dµ .
X
X
Demonstração. Sejam duas representações canónicas de r e s, respectivamente:
s=
N
X
i=1
αi IAi e r =
M
X
βi IBj .
j=1
127
Capı́tulo 4
Da mesma forma que na demonstração da proposição 25, temos que s e r admitem ainda
as seguintes representações canónicas:
s=
N X
M
X
αi IAi ∩Bj e r =
i=1 j=1
N X
M
X
βj IAi ∩Bj .
i=1 j=1
A hipótese s ≤ r implica que:
∀x ∈ Ai ∩ Bj s(x) = αi ≤ βj = r(x) .
Em consequência do corolário 7:
Z
Z
N X
M
N X
M
X
X
s dµ =
αi µ(Ai ∩ Bj ) ≤
r dµ ,
βi µ(Ai ∩ Bj ) =
X
i=1 j=1
i=1 j=1
X
tal como querı́amos demonstrar.
Apresentamos agora a definição de integral de uma função mensurável positiva.
Como se verá, esta definição decorre naturalmente da possibilidade de aproximar uma
qualquer destas funções por uma sucessão de funções simples mensuráveis.
Definição 31. Seja f ∈ M+ ((X, A), (R, B(R)) qualquer. Por definição tem-se
que o integral de Lebesgue de f sobre X é dado por:
Z
Z
+
+
f dµ = sup
s dµ : s ∈ S , s ≤ f ∈ R .
(4.5)
X
X
O integral de Lebesgue de f sobre um conjunto mensurável E qualquer
é dado por:
Z
Z
f dµ =
f IE dµ .
(4.6)
E
X
Observação 35. Note-se que no membro à direita da fórmula 4.5 o integral utilizado é o
dado pela definição 30. Para f uma função simples mensurável positiva temos então duas
definições. A que foi dada inicialmente, isto é a definição 30 e esta outra, a definição 31.
Os integrais dados por estas duas definições coincidem mostrando que as definições são
coerentes. Com efeito, seja para maior facilidade na apresentação da argumentação
R (3)
R (2)
e apenas no âmbito desta observação, X o integral segundo a definição 30 e X o
integral segundo a definição 31. Como f ∈ S+ e naturalmente f ≤ f tem-se que:
(Z
)
Z (2)
Z (3)
(2)
+
f dµ ≤
f dµ = sup
s dµ : s ∈ S , s ≤ f .
X
X
X
Seja agora uma qualquer s ∈ S+ com s ≤ f . Pela proposição 26 relativa à monotonia
tem-se que:
Z (2)
Z (2)
s dµ ≤
f dµ
X
128
X
A Integração à Lebesgue
e em consequência
Z
(Z
(3)
(2)
s dµ : s ∈ S , s ≤ f
+
f dµ = sup
X
)
Z
(2)
≤
X
f dµ ,
X
ficando assim estabelecida a coerência, para as funções simples, desta nova definição com
a definição anterior.
A proposição 26 admite agora uma extensão natural a todas as funções mensuráveis
positivas. Tem-se ainda um resultado imediato sobre a monotonia do integral relativamente ao domı́nio de integração.
Proposição 27. Sejam f, g ∈ M+ ((X, A), (R, B(R)) tais que f ≤ g. Então:
Z
Z
f dµ ≤
g dµ .
X
X
Sejam e E, F ∈ A tais que E ⊆ F . Então:
Z
Z
f dµ ≤
f dµ .
E
F
Demonstração. Seja uma qualquer s ∈ S+ tal que s ≤ f . Então, a hipótese feita sobre
g implica que s ≤ g pelo que:
Z
Z
Z
Z
+
+
f dµ = sup
s dµ : s ∈ S , s ≤ f ≤ sup
s dµ : s ∈ S , s ≤ g =
g dµ .
X
X
X
X
o que demonstra o primeiro resultado enunciado. O segundo resultado enunciado decorre
do primeiro observando que as hipóteses sobre os domı́nios de integração E e F implicam
que f IE ≤ f IF , pelo que:
Z
Z
Z
Z
f dµ =
f IE dµ ≤
f IF dµ =
f dµ ,
E
X
X
F
como pretendı́amos.
Observação 36. A definição dada na fórmula 4.6 tem uma consequência muito importante
que importa agora realçar. Seja f ∈ M+ qualquer.
Z
∀E ∈ A µ(E) = 0 ⇒
f dµ = 0 ,
(4.7)
E
isto é, o integral de uma qualquer função mensurável sobre um conjunto de medida nula
é nulo. Note-se que se f = IA então pela fórmula 4.6 e pela definição de integral de uma
função simples:
Z
Z
Z
Z
f dµ =
IA dµ =
IA IE dµ =
IA∩E dµ = µ(A ∩ E) ≤ µ(E) = 0 .
E
E
X
X
129
Capı́tulo 4
P
Suponhamos agora que f é uma função simples mensurável, isto é, f = N
i=1 αi IAi em
que para i ∈ {1, 2, . . . , N } se tem αi ∈ R e Ai ∈ A. Tem-se então com a convenção
0 × +∞ = 0:
!
Z
X
Z
Z
N
N
N
X
X
f dµ =
αi IAi dµ =
IAi dµ =
αi
αi × µ(Ai ∩ E) = 0 .
E
E
i=1
i=1
E
i=1
Finalmente, seja f mensurável positiva qualquer. Então tem-se, dado o que já verificámos
para as funções simples, que:
Z
Z
Z
f dµ = sup{ s dµ : s ∈ S, s ≤ f } = sup{ sIE dµ : s ∈ S, s ≤ f } = 0 .
E
X
E
Veremos a seguir que a propriedade expressa na fórmula 4.7 é válida para qualquer
função mensurável.
Tal como ficou patente na fórmula 4.5, o integral de uma função mensurável positiva
pode valer +∞. Para total coerência com a teoria do integral de Riemann e para permitir
a eventual definição do integral para uma função mensurável não necessariamente não
negativa vamos nomear as funções mensuráveis positivas cujo integral pertence a R+ ,
isto é, é finito como: funções integráveis. Depois definiremos as funções integráveis e o
integral respectivo para funções mensuráveis com valores em R.
Definição 32. Uma função f ∈ M+ é integrável se e só se:
Z
f dµ < +∞ .
(4.8)
X
Uma função f ∈ M((X, A), (R, B(R)), não necessariamente não negativa, é
integrável se e só se f + , a sua parte positiva, e f − , a sua parte negativa, o
forem e com essas condições,
Z
Z
Z
+
f dµ :=
f dµ −
f − dµ ,
(4.9)
X
X
X
isto é, o integral de uma função integrável é dado pela diferença dos integrais
das sua parte positiva e negativa.
Observação 37. Note-se que o integral de uma função mensurável qualquer, quando
existe, é um número real.
Proposição 28. Seja f integrável. Então (−f ) é integrável e
Z
Z
(−f ) dµ = −
f dµ .
X
130
X
A Integração à Lebesgue
Demonstração. É uma aplicação das definições observando que (−f )+ = max(−f, 0) =
f − e (−f )− = max(−(−f ), 0) = f + .
Notação 3. Os conjuntos de funções integráveis têm propriedades importantes que
constituem um dos argumentos mais fortes justificando a importância do integral de Lebesgue. Com efeito, após passagem o cociente por meio de uma relação de equivalência
adequada, podem ser munidos de uma estrutura de espaços vectoriais normados completos constituindo assim os exemplos mais importantes de espaços de Banach. Assim
para p ∈ [1, +∞[ definimos
Z
p
p
L ((X, A, µ), (R, B(R)) := f : X −→ R : f ∈ M ,
| f | dµ < +∞ ,
(4.10)
X
o conjunto das funções de potência p-ésima integrável. Sempre que não haja dúvidas
sobre os espaços de medida subjacentes representamos estesconjuntos por Lp . Estes
conjuntos serão objecto de um estudo aprofundado num capı́tulo especial. De particular
relevo para o que vai seguir-se é o conjunto L1 das funções integráveis.
4.2
Propriedades importantes do integral de Lebesgue
There is a delicious irony here. Riemann introduced his definition of
the integral for the purpose of understanding how discontinuous a
function could be and still be integrable. It appeared that it could be
very discontinuous, having discontinuities at all rational numbers. In
fact, there are Riemann integrable functions with discontinuities at the
points in a set of cardinality c. Now that we are finally putting the
Riemann integral behind us, we get the answer that Riemann was
seeking. A Riemann integrable function is always a very continuous
function. It is continuous almost everywhere. A function that is
discontinuous only at the rational numbers is not very discontinuous.
The Lebesgue integral enables us to handle truly discontinuous
functions. Dirichlet’s function, the characteristic function of the
rationals, was created to show that a function could be so
discontinuous that it would make no sense to talk about its integral.
This was considered a function beyond the pale. Yet, as we shall see,
the Lebesgue integral of this function has a simple and natural
meaning. Over any interval, the integral of this function is the measure
of the set of rationals in that interval, which is zero.
David M. Bressoud A Radical Approach to Lebesgue’s Theory of
Integration. Cambridge University Press, Cambridge, 2008, página 167.
No teorema seguinte apresentamos dois resultados da maior importância no contexto
do integral de Lebesgue. O primeiro enunciado no teorema diz-nos que uma função integrável relativamente a uma dada medida só pode tomar o valor +∞ num conjunto
desprezável relativamente a essa medida. O segundo enunciado deve ser posto em paralelo com o resultado correspondente para o integral de Riemann que nos diz que toda
131
Capı́tulo 4
a função de sinal constante, contı́nua e com integral de Riemann nulo sobre um intervalo fechado limitado, é nula sobre esse intervalo. Com efeito, na versão deste resultado
para o integral de Lebesgue que apresentamos seguidamente, enfraquece-se a hipótese
de continuidade, passando a exigir-se a mensurabilidade. A conclusão também sai mais
fraca, uma vez que a função é apenas não nula num conjunto desprezável relativamente
à medida com a qual é calculado o integral e não identicamente nula, como no caso do
integral de Riemann.
Teorema 15. Seja f ∈ M+ ((X, A), (R, B(R)).
1. Se f , mensurável não negativa for, integrável relativamente à medida µ
então f é finita quase por toda a parte relativamente à medida µ.
Z
0≤
f dµ < ∞ ⇒ (f < +∞ µ − qptp) .
X
2. Se f , mensurável de sinal constante, tiver integral nulo relativamente à
medida µ então f é nula quase por toda a parte relativamente à medida
µ.
Z
(f ≥ 0) e 0 ≤
f dµ = 0 ⇒ (f = 0 µ − qptp) .
X
3. Se f , mensurável qualquer, tiver integral nulo sobre todo o conjunto mensurável E, relativamente à medida µ então f é nula quase por toda a parte
relativamente à medida µ.
Z
∀f ∈ M((X, A), (R, B(R))
∀E ∈ A
f dµ = 0 ⇒ (f = 0 µ − qptp) .
E
Demonstração. Para demonstrarmos 1, seja por definição para cada n ≥ 1:
An := {x ∈ X : f (x) ≤ n}
e observe-se que An ⊆ An+1 pelo que (Acn )n≥1 forma uma sucessão decrescente de conjuntos tal que:
+∞
\
Acn = {x ∈ X : f (x) = +∞} .
n=1
Em consequência da proposição 27 sobre a monotonia do integral relativamente à função
integranda e ao domı́nio de integração, temos que para cada n ≥ 1:
Z
Z
Z
+∞ >
f dµ ≤
f dµ ≥ n
dµ = n µ(Acn ) .
Acn
X
Acn
Temos pois que para n ≥ 1:
µ(Acn )
132
1
≤
n
Z
f dµ −→ 0 ,
X
A Integração à Lebesgue
pelo que pela monotonia da medida relativamente às sucessões decrescentes, temos:
!
+∞
\
µ({f = +∞}) = µ
Acn = lim µ(Acn ) = 0 .
n→+∞
n=1
A demonstração de 2 segue uma argumentação semelhante considerando para cada n ≥ 1:
Bn := {x ∈ X : f (x) >
1
},
n
observando-se imediatamente que Bn ⊂ Bn+1 , isto é, (Bn )n≥1 é uma sucessão crescente
e
+∞
[
Bn = {x ∈ X : f (x) > 0} .
n=1
Pela monotonia do integral relativamente à função integraanda e ao domı́nio de integração tem-se que:
Z
Z
Z
1
1
f dµ ≥
dµ = µ(Bn ) ≥ 0 ,
f dµ ≥
0=
n Bn
n
Bn
X
pelo que µ(Bn ) = 0 para n ≥ 1 . Em consequência da subaditividade da medida temos
que:
! +∞
+∞
[
X
µ({f > 0}) = µ
Bn =
µ(Bn ) = 0 ,
n=1
n=1
tal como querı́amos demonstrar. Para demonstrarmos 3 consideremos E = {f ≥ 0} ∈ A
e observe-se que f + = f + IE e f − IE = 0 pelo que pela definição de integral de uma
função integrável:
Z
Z
Z
Z
Z
+
−
0=
f dµ =
f IE dµ =
f IE dµ −
f IE dµ =
f + dµ − 0 .
E
X
X
X
X
Aplicando o segundo enunciado à função f + temos que f + = 0 µ-qptp. Se repetirmos o
argumento com E = {f ≤ 0} ∈ A vamos concluir que f − = 0 µ-qptp pelo que finalmente
f = f + − f − = 0 µ-qptp tal como pretendı́amos.
4.3
Os teoremas de convergência
Uma das razões que explica o sucesso da teoria do integral de Lebesgue, aferido por
uma utilização quase universal desta em todas as áreas da Matemática desde a Análise
Funcional e a Análise Matemática com as Equações Diferenciais ordinárias, estocásticas
e às derivadas parciais, até às Probabilidades e à Estatı́stica passando pelos Sistemas
Dinâmicos, são os teoremas de convergência. Genericamente, sem qualquer pretensão de
rigor matemático, um teorema de convergência é um resultado em que se estabelecem
condições para que se tenha:
Z
Z
lim
fn =
lim fn ,
n→+∞
n→+∞
133
Capı́tulo 4
quando (fn )n∈N é uma sucessão de funções para a qual esta fórmula faça sentido. No caso
do integral de Riemann, se a sucessão (fn )n∈N for uma sucessão de funções contı́nuas
convergente uniformemente, sobre um intervalo fechado limitado [a, b], para uma dada
função limite tem-se que:
Z b
Z b
lim fn (x) dx .
lim
fn (x) dx =
a n→+∞
n→+∞ a
Num conjunto muito significativo de situações interessantes não se tem a convergência
uniforme da sucessão de funções para a função limite mas apenas a convergência pontual,
pelo que o resultado que citámos para o integral de Riemann é inútil. O integral de
Lebesgue ultrapassa a limitação de exigir a convergência uniforme com condições que
têm, na prática, um âmbito muito mais vasto de aplicação e são muito mais fáceis de
verificar que a convergência uniforme.
Num primeiro tipo de resultados temos como condições essenciais que a sucessão de
funções mensuráveis não negativas seja também monótona.
Teorema 16 (Convergência monótona de Lebesgue). Seja (fn )n∈N ∈
(M+ )N uma sucessão crescente de funções, isto é, tal que:
∀x ∈ X ∀n ∈ N 0 ≤ fn (x) ≤ fn+1 (x) .
Então:
Z
lim
n→N
fn dµ
Z =
X
lim fn
X
n→N
dµ .
Demonstração. Seja então (fn )n∈N uma sucessão crescente de funções mensuráveis positivas, sobre o espaço de medida (X, A, µ), convergindo pontualmente para f . Tem-se
em virtude da monotonia do integral relativamente à função integranda que:
Z
Z
lim
fn dµ ≤
f dµ ,
n→+∞ X
X
pelo que a desigualdade num sentido fica desde já verificada.
Considere-se agora ϕ uma função simples mensurável positiva. Tem-se que λ, definida
por
Z
λ(E) :=
ϕ dµ,
E
para qualquer E ∈ A é uma medida. Defina-se agora:
An := {x ∈ X : fn (x) ≥ αϕ(x)}
com 0 < α < 1 e ϕ ∈ S+ tal que ϕ ≤ f . Verifica-se que
Z
Z
Z
αϕ dµ ≤
fn dµ ≤
fn dµ .
An
134
An
X
A Integração à Lebesgue
Podemos agora verificar que:
Z
Z
ϕ dµ = lim
ϕ dµ
X
Z
e que então
n→+∞ A
n
Z
ϕ dµ ≤ lim
α
X
n→+∞ X
fn dµ .
Conclue-se que
Z
Z
f dµ ≤ lim
n→+∞ X
X
fn dµ .
tal como se pretendia no enunciado.
Como primeira consequência do teorema de convergência monótona de Lebesgue
temos o resultado seguinte que mostra que sobre o cone das funções mensuráveis não
negativas o operador que a uma função associa o seu integral de Lebesgue é linear.
Proposição 29. Sejam f, g ∈ M+ e α ∈ R.
R
R
R
1. X (f + g) dµ = X f dµ + X g dµ .
R
R
2. X αf dµ = α X f dµ .
Demonstração. Para verificarmos a primeira das afirmações feitas no enunciado seja
(sn )n∈N (respectivamente (rn )n∈N ) uma sucessão crescente de funções simples mensuráveis não negativas convergente pontualmente para f (respectivamente para g) que,
pelo teorema de Lebesgue, sabemos existir. É óbvio que (sn + rn )n∈N é também uma sucessão crescente de funções simples mensuráveis não negativas convergente pontualmente
para f + g pelo que por aplicações consecutivas do teorema da convergência monótona
de Lebesgue (em (a) e (c)) e da aditividade do integral no quadro das funções simples
dada pela proposição 25 (em (b)), temos que:
Z
Z
Z
(f + g) dµ =
lim (sn + rn ) dµ =(a) lim
(sn + rn ) dµ =(b)
n→+∞ X
X
X n→+∞
Z
Z
Z
Z
= lim
sn dµ +
rn dµ = lim
sn dµ + lim
rn dµ =(c)
n→+∞
n→+∞ X
n→+∞ X
X
X
Z
Z
Z
Z
=
lim sn dµ +
lim rn dµ =
f dµ +
g dµ .
X n→+∞
X n→+∞
X
X
Para demonstrar 2 basta observar que (αsn )n∈N é uma sucessão crescente de funções
simples mensuráveis não negativas convergente pontualmente para f pelo que se tem
por novas aplicações do teorema da convergêcia monótona de Lebesgue (em (a) e (c)),
Z
Z
Z
Z
αf dµ =
lim αsn dµ =(a) lim
αsn dµ = lim α
sn dµ =
n→+∞
n→+∞
X
X n→+∞
X
X
Z
Z
= α lim
sn dµ =(b) α
lim sn dµ ,
n→+∞ X
X n→+∞
ficando assim demonstrado o resultado.
135
Capı́tulo 4
A proposição 29 admite uma extensão imediata ao conjunto das funções integráveis.
Corolário 8. Sejam f, g ∈ L1 e α ∈ R. Então:
R
R
• X αf dµ = α X f dµ.
R
R
R
• X (f + g) dµ = X f dµ + X g dµ.
Demonstração. Seja α > 0. Então temos que (αf )+ = αf + e (αf )− = αf − pelo que
Z
Z
Z
Z
Z
Z
−
−
+
+
f dµ .
f dµ = α
αf dµ = α
f dµ −
αf dµ −
αf dµ =
X
X
X
X
X
X
Para α = 0 e α < 0 averificação do primeiro
enunciado faz-se de foema semelhante.
R
Veriiquemos o segundo
enunciado. Como X | f | dµ < +∞ temos que f < +∞ µ-qptp.
R
Do mesmo modo X | g | dµ < +∞ pelo que g < +∞ µ-qptp. Em consequência,
existe X1 ∈ A tal que sobre X1 a soma de f com g está univocamente definida e tal que
µ(X1c ) = 0. Considere-se umas quaisquer redefinições de f e de g sobre X1c . Então, uma
vez que é nulo o integral de qualquer função mensurável sobre o conjunto de medida
nula X1c , temos que:
Z
Z
Z
Z
Z
Z
(f + g) dµ =
(f + g) dµ =(a)
f dµ +
g dµ =
f dµ + g dµ
X
X1
X1
X1
X
sendo que a igualdade em (a) é justificada naturalmente com a decomposição f = f + −f −
e g = g + − g − pelo que f + g = (f + + g + ) − (f − + g − ) com as funções não negativas
integráveis f + + g + e f − + g − .
A proposição seguinte estabelece uma diferença significativa entre o integral de Lebesgue e o integral de Riemann impróprio para funções definidas num domı́nio não limitado
de R.
Proposição 30. Uma dada função f : X −→ R é integrável se e só se | f | o
for.
R
R
Demonstração. Se f é integrável então X f + dµ < +∞ e X f − dµ < +∞ pelo que
como | f |= f + + f − temos que pela proposição 29:
Z
Z
Z
+
| f | dµ =
f dµ +
f − dµ < ∞ .
X
X
X
Suponhamos agora | f | integrável. Por aplicação da proposição 27, dado que f + ≤| f |
e f + ≤| f | tem-se:
Z
Z
Z
Z
f + dµ ≤
| f | dµ < ∞ e
f − dµ ≤
| f | dµ < ∞ .
X
tal como querı́amos.
136
X
X
X
A Integração à Lebesgue
Observação 38. Vamos esclarecer com um exemplo o sentido do comentário que fizémos
acima como introdução à proposição 30. A função f : [0, +∞] −→ R dada por
(
sin x
se x 6= 0
x
f (x) =
1
se x = 0
é contı́nua e limitada sobre o seu domı́nio de definição. Em consequência, tanto o integral
de f como o integral à Riemann de | f | existem sobre qualquer intervalo fechado limitado
de [0, +∞]. Um exercı́cio de Análise Matemática simples mostra que:
+∞
Z
M
Z
f (x) dx =
0
lim
M →+∞ 0
+∞
X
sin x
dx =
(−1)k
x
Z
k=0
(k+1)π
kπ
| sin x |
dx < +∞ ,
x
sendo que a expressão à direita converge por ser uma série alternada cujo módulo do
R (k+1)π |sin x|
dx)k∈N que decresce para zero. Um argumento
termo geral é a sucessão ( kπ
x
do mesmo tipo mostra que:
Z
+∞
0
+∞
M
Z
| f (x) | dx =
lim
M →+∞ 0
X 1
sin x
| dx ≥
= +∞ .
x
2k + 1
|
k=0
Fica assim posto em evidência um exemplo de uma função f cujo integral impróprio
de Riemann existe em R, ou seja é Riemann integrável, mas tal que | f | não tem o
correspondente integral impróprio de Riemann em R.
O teorema da convergência monótona de Lebesgue admite uma versão para sucessões
decrescentes que exige uma hipótese adicional. Esta hipótese implica que todos os elementos da sucessão são integráveis a partir de uma certa ordem pelo que o resultado
obtido não tem um âmbito tão alargado de aplicação como o do teorema para sucessões
crescentes.
Proposição 31. Seja (gn )n∈N ∈ (mM+ )N tal que:
(i) A sucessão é decrescente isto é:
∀n ∈ N gn ≥ gn+1 ≥ 0 .
(ii) Pelo menos uma das funções da sucessão é integrável, isto é:
Z
∃n0 ∈ N
gn0 dµ < +∞ .
(4.11)
X
Então tem-se que:
Z
lim
n→+∞
gn dµ
X
Z =
lim gn
X
n→+∞
dµ .
(4.12)
137
Capı́tulo 4
Demonstração. Considere-se a sucessão (fn )n∈N definida para n ≥ n0 + 1 por fn = gn0 −
gn . É imediato verificar que se trata de uma sucessão crescente de funções mensuráveis
não negativas e integráveis, pelo que se pode aplicar o teorema da convergência monótona
de Lebesgue para se obter:
Z
Z
Z
Z
lim gn dµ .
gn0 dµ −
gn dµ =
gn0 dµ − lim
n→+∞
X
X n→+∞
X
X
R
dado que X gn0 dµ ∈ R pode simplificar-se esta expressão obtendo-se o resultado pretendido na fórmula 4.12.
Exemplo 10. Sem a hipótese adicional 4.11 acima a proposição é falsa em geral. Com
efeito, seja em (R, B(R), λ) a sucessão de funções dada para cada n ∈ N por I]n,+∞[ .
Tem-se naturalmente que I]n,+∞[ ≥ I]n+1,+∞[ uma vez que ]n + 1, +∞[⊂]n, +∞[, pelo
que a sucessão é decrescente. Tem-se também que:
Z
I]n,+∞[ dλ = λ(]n, +∞[) = +∞
∀n ∈ N
R
No entanto, tem-se que a sucessão converge pontualmente para a função nula, como se
verifica imediatamente, pelo que:
Z
Z lim
I]n,+∞[ dλ = +∞ =
6 0=
lim I]n,+∞[ dλ ,
n→+∞
R
R
n→+∞
o que justifica a afirmação feita.
O teorema da convergência monótona tem ainda uma aplicação importante às séries
de funções com termos não negativos.
Teorema 17. Seja (fn )n∈N ∈ (M+ )N . Então
+∞ Z
X
n=0
X
fn dµ
Z
=
X
+∞
X
!
fn
dµ .
n=0
Demonstração.
Se considerarmos a sucessão (SN )N ∈N das somas parciais da série de
P
funções ( n≥0 fn ) definida por:
∀N ∈ N SN :=
+∞
X
fn ,
n=0
temos que para qualquer N ∈ N se tem SN +1 − SN = fN +1 ≥ 0 pelo que sucessão
das somas parciais é não decrescente. Em consequência, pelo teorema da convergência
monótona de Lebesgue:
!
Z
X
Z Z
+∞
+∞ Z
X
fn dµ =
lim SN dµ = lim
SN dµ =
fn dµ ,
X
n=0
X
N →+∞
exactamente como pretendı́amos.
138
N →+∞
X
n=0
X
A Integração à Lebesgue
Lema 4 (Lema de Fatou). Seja (fn )n∈N ∈ (M+ )N uma sucessão de funções
mensuráveis positivas quaisquer. Então:
Z
Z fn dµ .
lim inf fn dµ ≤ lim inf
n→N
n→N
X
X
Demonstração. Consideremos a sucessão (gn )n∈N definida por gn := inf m≥n fm , tal como
na demonstração do teorema relativo ao carácter mensurável do limite de uma sucessão
de funções mensurávei. Trata-se de uma sucessão crescente de funções mensuráveis não
negativas, como se verifica facilmente uma vez que,
∀x ∈ X gn (x) = inf{fn (x), fn+1 (x), fn+2 (x), . . . } ≤ gn+1 (x) = inf{fn+1 (x), fn+2 (x), . . . } ,
tendo-se ainda que:
lim gn (x) = lim
n→+∞
n→+∞
inf fm (x) = sup inf fm (x) = lim inf fm (x) .
m≥n
n∈N
n→+∞
m≥n
Dado que para cada n ∈ N e m ≥ n se tem gn ≤ fm vem, em virtude da monotonia do
integral relativamente à função integranda :
Z
Z
gn dµ ≤
fm dµ ,
X
X
pelo que uma vez que a desigualdade acima vale para qualquer m ≥ n:
Z
Z
gn dµ ≤ inf
m≥n
X
fm dµ
,
X
Por aplicação do teorema da convergência monótona de Lebesgue (em (a)) vem finalmente:
Z
Z
Z Z
lim inf fn dµ =
lim gn dµ =(a) lim
gn dµ ≤ lim inf
fm dµ .
X
n→N
X n→+∞
n→+∞
Como a sucessão de termo geral inf m≥n
R
Z
lim
inf
n→+∞ m≥n
X
fm dµ é crescente temos que
fm dµ
X
X
n→+∞ m≥n
X
Z
= lim inf
n→N
fn dµ
,
X
pelo que a demonstração do lema de Fatou fica assim concluı́da.
No caso de existir uma função integrável que domine os elementos da sucessão tem-se
uma versão do lema de Fatou aplicável aos limites superiores.
139
Capı́tulo 4
Lema 5 (Lema de Fatou inverso). Seja (X, A, µ), seja (fn )n∈N ∈ (M+ )N
uma sucessão de funções mensuráveis positivas quaisquer tais que:
∃g ∈ L1 ∀n ∈ N fn ≤ g .
Tem-se então que:
Z
fn dµ
lim sup
n→N
Z lim sup fn dµ .
≤
n→N
X
X
Demonstração. Se aplicarmos o lema de Fatou à sucessão (g − fn )n∈N temos dado que
todas as funções da sucessão são integráveis que:
Z
Z Z Z
(g − fn ) dµ =
lim inf (g − fn ) dµ ≤ lim inf
lim sup fn dµ =
g dµ −
n→N
n→N
n→N
X
X
X
X
Z
Z
=
g dµ − lim sup
fn dµ ,
n→N
X
X
o que nos permite concluir como querı́amos.
O teorema seguinte é um dos mais importantes da teoria do integral de Lebesgue,
tanto pelo carácter quase universal da sua aplicação como por ser um bom exemplo de
um resultado com consequências fortes mas com um conjunto de hipóteses não demasiadamente restritivas.
Teorema 18 (Convergência dominada de Lebesgue). Seja (fn )n∈N uma
sucessão de funções mensuráveis tais que:
(i) A sucessão (fn )n∈N é dominada por uma função não negativa integrável,
isto é:
∃g ∈ L1 , g ≥ 0 ∀n ∈ N ∀x ∈ X | fn (x) |≤ g(x) .
(4.13)
(ii) A sucessão (fn )n∈N converge pontualmente sobre X para uma função f .
Então f é integrável e
Z
Z
f dµ = lim
X
n→+∞ X
fn dµ .
(4.14)
Demonstração. Mostremos primeiramente que f é integrável. Com efeito, em virtude
da sucessão ser dominada por g, tem-se para todo o x ∈ X e para todo o n ∈ N que:
| f (x) |=| f (x) − fn (x) + fn (x) |≤| f (x) − fn (x) | + | fn (x) |≤| f (x) − fn (x) | +g(x) ,
140
A Integração à Lebesgue
pelo que passando ao limite virá | f (x) |≤ g(x) uma vez que pela segunda hipótese,
lim | f (x) − fn (x) |= 0 .
n→+∞
Em consequência da monotonia do integral relativamente à função integranda, | f (x) |∈
L1 e por isso f é integrável. Vamos mostrar seguidamente a segunda tese usando para
o efeito o lema de Fatou. Usando a majoração obtida para f tem-se para todo o x ∈ X
e para todo o n ∈ N que:
| f (x) − fn (x) |≤| f (x) | + | fn (x) |≤ 2g(x) .
pelo que 2g− | f − fn |≥ 0, podendo-se pois aplicar à sucessão com este termo geral o
lema de Fatou que nos dá:
Z
Z
lim inf (2g− | f − fn |) dµ ≤ lim inf
2g− | f − fn | dµ .
X n→+∞
n→+∞
X
Observando que lim inf n→+∞ (−an ) = − lim supn→+∞ an
Z
Z
Z
Z
2g dµ −
lim sup | f − fn | dµ ≤
2g dµ − lim sup
X
X n→+∞
n→+∞
X
| f − fn | dµ
,
X
pelo que se tem, em consequência de ser lim supn→+∞ | f − fn |= 0,
Z
Z
0=
lim sup | f − fn | dµ ≥ lim sup
| f − fn | dµ .
X n→+∞
n→+∞
X
Para concluir, observe-se que para uma sucessão com valores não negativos na recta
acabada se tem sempre 0 ≤ lim inf n→+∞ an ≤ limn→+∞ an ≤ lim supn→+∞ an , pelo que:
Z
Z
Z
fn dµ ≤ lim
| f − fn | dµ = 0 ,
lim f dµ −
n→+∞
X
X
n→+∞ X
tal como querı́amos demonstrar.
No quadro do integral de Lebesgue a situação com as séries de funções mensuráveis
é particularmente simples como demonstra o resultado seguinte.
Teorema 19. Seja (fn )n∈N ∈ (mM)N tal que
+∞ Z
X
| fn | dµ < +∞ .
n=0 X
P
Então ( +∞
n=0 fn ) a série de termo geral dado pela sucessão, converge pontualmente salvo talvez num conjunto de medida nula e tem-se:
!
Z
+∞ Z
+∞
X
X
fn dµ =
fn dµ .
n=0
X
X
n=0
141
Capı́tulo 4
Demonstração. Note-se que, pelo teorema 17 se tem
Z X
+∞
| fn | dµ =
X n=0
+∞ Z
X
| fn | dµ < +∞ ,
n=0 X
Em consequência do enunciado 1 do teorema 15, tem-se que:
+∞
X
| fn |< +∞ µ − qptp ,
n=0
isto é a série de termo geral (fn )n∈N converge absolutamente e, por isso, pontualmente
salvo num conjunto desprezável
PN relativamente à medida µ. Se considerarmos a sucessão
das somas parciais SN := n=0 fn tem-se que:
| SN |≤
N
X
| fn |≤
n=0
+∞
X
| fn |< +∞ µ − qptp ,
n=0
pelo que pelo teorema da convergência dominada de Lebesgue:
!
Z
X
Z
Z +∞
+∞ Z
X
fn dµ =
lim SN dµ = lim
SN dµ =
fn dµ .
X
n=0
X
n→+∞
n→+∞
X
n=0
X
tal como querı́amos verificar.
4.4
Esperança matemática
Nesta secção vamos particularizar os resultados sobre o integral de Lebesgue aos espaços
de probabilidade. Seja então um tal espaço (Ω, F, P).
É conhecido que para a definição de quantidades básicas associadas a variáveis
aleatórias - esperança, variância, covariância - é necessária a imposição prévia de condições
de integrabilidade às variáveis aleatórias. Relembremos as seguintes definições.
Seja (Ω, A, P) um espaço de probabilidade e X : Ω −→ R uma variável aleatória.
Definição 33. Se X for P integrável, a esperança matemática de X, representada por E[X] é definida por:
Z
E[X] :=
X(ω)dP(ω) .
Ω
Se X 2 for P integrável, a variância de X, representada por V[X] é definida por
V[X] := E[X 2 ] − E[X]2 .
Seja Y : Ω −→ R uma outra variável aleatória.
142
A Integração à Lebesgue
Definição 34. Se X 2 e Y 2 forem P integráveis, a covariância de X e Y ,
representada por C[X, Y ] é definida por
C[X, Y ] := E[(X − E[X])(Y − E[Y ])] .
Observação 39. A esperança matemática é a representação matemática do conceito de
valor médio de uma variável aleatória. Esta identificação entre o integral e o valor médio
apareceu como totalmente natural no contexto do integral de Riemann. Com efeito, em
virtude do teorema do valor médio do cálculo integral, se f for contı́nua sobre o intervalo
fechado limitado [a, b] existe c ∈ [a, b] tal que
1
f (c) =
b−a
Z
b
Z
f (x)dx =
a
b
f (x)
a
dx
.
b−a
No contexto da teoria das probabilidades, a igualdade entre o termo mais à esquerda e
o termo mais à direita pode ser interpretado como exprimindo que c é o valor médio da
variável aleatória f relativamente à probabilidade uniforme sobre [a, b].
O leitor é convidado a efectuar a tradução imediata para propriedades da esperança
matemática de todos os resultados até agora obtidos para o integral de Lebesgue.
O resultado seguinte mostra como calcular a esperança matemática a partir da lei
de probabilidade. Este resultado é de uso constante na teoria das probabilidades e mais
uma vez mostra como o conhecimento da correspondência que define uma dada variável
aleatória não é essencial para a determinação da respectiva esperança matemática se se
conhecer a lei de probabilidade dessa variável aleatória.
Teorema 20. Seja X : Ω −→ R uma variável aleatória de lei LX . Seja f :
R −→ R mensurável positiva. Então:
Z
Z
f (x) dLX (x) .
(4.15)
E[f (X)] =
f (X) dP =
Ω
R
Uma dada função f : R −→ R é integrável relativamente à medida LX se e só se
f (X) for integrável relativamente à medida P e nessas condições vale também
a fórmula 4.15.
Demonstração. Nesta demonstração faz-se uso de uma metodologia de utilização constante no contexto da integração à Lebesgue para verificar uma dada propriedade, tipicamente uma propriedade envolvendo um integral. Um primeiro exemplo desta metodologia foi dado na observação 36. Esta metodologia consiste em demonstrar o que se
pretende nos quatro passos fundamentais seguintes.
(1) Mostrar a propriedade para as funções indicatrizes.
(2) Por linearidade estender a demonstração às funções simples.
143
Capı́tulo 4
(3) Utilizando a densidade de S+ em M+ e o teorema da convergência nmonótona de
Lebesgue demonstrar a propriedade para as funções mensuráveis positivas.
(4) Finalmente mostrar a propriedade para as funções integráveis usando a decomposição da função nas suas partes positivas e negativas.
Suponhamos então f = IB com B ∈ B(R). A igualdade na fórmula 4.15 resulta de se
ter
(
1 se X(ω) ∈ B isto é, se ω ∈ X −1 (B)
f (X)(ω) = IB (X)(ω) =
0 se X(ω) ∈
/ B isto é, se ω ∈
/ X −1 (B)
pelo que IB (X) = IX −1 (B) e em consequência:
Z
Z
Z
−1
f (X) dP =
IX −1 (B) dP = P[X (B)] = LX (B) =
IB (x) dLX (x) .
Ω
Ω
R
PN
Seja agora uma função simples s =
i=1 βi IBi com αi ∈ R e Bi ∈ B(R), para i ∈
{1, . . . , N }. Então por linearidade e usando o já verifiicado para as funções indicatrizes:
!
Z
X
Z
Z
Z
N
N
N
X
X
s(X) dP =
βi IBi (X) dP =
IBi dLX =
βi
IBi (X) dP =
βi
Ω
Ω
i=1
N
X
Z
=
R
Ω
i=1
!
βi IBi
i=1
R
Z
dLX =
s dLX .
R
i=1
No terceiro passo, como indicado acima, seja f ∈ mM+ . Pelo teorema de Lebesgue
considere-se (sn )n∈N uma sucessão crescente de funções simples mensuráveis não negativas convergindo pontualmente para f sobre R. Verifica-se imediatamente que (sn (X))n∈N
é uma sucessão crescente de funções simples mensuráveis positivas definidas sobre Ω.
Pelo teorema da convergência monótona de Lebesgue e pelo que já verificámos para as
funções simples:
Z
Z
Z
Z f (X) dP =
lim sn (X) dP = lim
sn (X) dP = lim
sn dLX =
n→+∞
n→+∞
Ω
Ω n→+∞
Ω
R
Z Z
=
lim sn dLX =
f dLX .
n→+∞
R
R
Seja então f integrável relativamente à medida LX , Então tem-se que:
Z
Z
+
f dLX < +∞ e
f − dLX < +∞ .
R
R
Podemos aplicar o que já vimos para as funções mesnuráveis não negativas quaisquer,
observando que f (X)+ = f + (X) e f (X)− = f − (X) e obtemos:
Z
Z
Z
Z
Z
Z
+
−
+
−
f (X) dP =
f (X) dP −
f (X) dP =
f dLX −
f dLX =
f dLX ,
Ω
Ω
Ω
tal como foi afirmado no enunciado.
144
R
R
R
A Integração à Lebesgue
Observação 40. O integral de Lebesgue-Stieltjes de uma dada função h - suficientemente
integrável - relativamente à função FX pode ser definido por:
Z
Z
h(x)dFX (x) :=
h(x)dµFX (x) ,
R
R
sendo o último integral um integral de Lebesgue usual em que µFX é a medida associada
à função crescente FX da forma seguinte.
Para complementos de informação sobre este assunto veja-se a construção das medidas de Borel-Stieltjes.
Definição 35. Seja a famı́lia de partes de R dada por S = Se = {]a, b] : a, b ∈
R a ≤ b}. Tem-se então que, o conjunto C(S) - denominado o anel gerado por
S - é o conjunto das uniões finitas de elementos de S.
Proposição 32. C(S) é o conjunto de uniões finitas de elementos de S dois a
dois disjuntos.
Demonstração. Exercı́cio.
Proposição 33. Seja F uma função real de variável real, monótona crescente,
continua à direita e verificando limx→+∞ F (x) = 1 e limx→−∞ F (x) = 0. Seja
o conjunto S = Se = {]a, b] : a, b ∈ R a ≤ b} e, por definição considere-se:
µF (]a, b]) = µ(]a, b]) := F (b) − F (a+ ) = F (b) − F (a).
Então, tem-se que:
1. µ é uma função de conjuntos aditiva sobre S.
2. µ é uma função de conjuntos aditiva-σ sobre S.
Demonstração. Exercı́cio.
Proposição 34. Com as notações da definição 35 e das proposições 32 e 33
verifica-se que:
Z
Z
E[h(X)] =
h(x)dLX (x) =
h(x)dµFX (x) .
R
R
Demonstração. Exercı́cio.
145
Capı́tulo 4
Exercı́cios
4.4.1
Integração de Funções Mensuráveis Positivas
Nesta secção estudar-se-ão as principais propriedades da integração das funções mensuráveis positivas relativamente a uma medida positiva sobre um espaço mensurável. Os
exercı́cios, que na sua maior parte são muito simples, deverão ser atentamente estudados com o fim de possibilitar ao leitor, uma familiarização com as ideias mais simples da
teoria da integração relativa a uma medida positiva.
Notações: Denotaremos o conjunto das funções mensuráveis de (X, A, µ) na recta
real (ou na recta acabada) por M((X, A, µ), R) e o conjunto das funções mensuráveis
positivas, definidas sobre (X, A, µ), por M((X, A, µ), R+ ) ou mais simplesmente por
M+ .
[1]
Exercı́cio 158. Seja I o intervalo unitário. Mostre que
Z
1
x dλ = ,
2
I
usando apenas propriedades do integral de Lebesgue.
[1]
Exercı́cio 159. Seja J o intervalo 1 ≤ x < ∞. Mostre que
Z
1
dλ = ∞,
J x
usando apenas propriedades do integral de Lebesgue.
[2]
Exercı́cio 160. Seja (X, F, µ) um espaço de probabilidade. Seja f : X −→ [0, +∞[ uma variável
aleatória. O integral
Z
E=
f dµ
X
é chamado de valor esperado de f e o integral
Z
V=
(f − E)2 dµ
X
é chamado de variância de f . Mostre que, se a variância de f é pequena, f desvia-se do seu valor
esperado com pequena probabilidade. Explicite, mostrando que a probabilidade de f desviar-se ε de E-ou
seja,
µ ({x ∈ X; |f (x) − E| > ε})
é menor ou igual a ε−2 V . Poderá aplicar a desigualdade de Markov.
[2]
Exercı́cio 161. Considerando o espaço de medida (]0, 1], B(]0, 1]), λ) e Rk (x) (a k-ésima função de
Rademacher), calcule, utilizando as notações do exercı́cio 160, E [Rk ] e V [Rk ].
[2]
Exercı́cio 162. Seja J um subintervalo finito da recta real e f : J −→ R uma função simples que
toma os valores c1 , ..., cn . A função f é chamada de função em escada se f −1 (ci ) é uma união finita
de intervalos para cada i. Mostre que o integral de Lebesgue e o integral de Riemann coincidem para
funções em escada.
146
A Integração à Lebesgue
[1]
Exercı́cio 163. Seja Hn o número de ”caras” que ocorre nas primeiras n provas de uma sequência de
Bernoulli. Calcule o seu valor esperado e a sua variância.
[2]
Exercı́cio 164. Considere a série ”aleatória”
1
1
1
± ± ± ...
2
4
8
1±
em que a atribuição do sinal + ou − ao n-ésimo termo da série é decidido através do lançamento de
uma moeda. Calcule o seu valor esperado e a sua variância.
Exercı́cio 165. Seja (X, F, µ) um espaço de medida e f uma função mensurável não negativa. Para [3]
todo o a ∈]0, ∞[, seja
φ(a) = µ({x ∈ X; f (x) > a}).
R
k
Suponha que X f dµ < ∞, k > 0. Mostre que existe uma constante C > 0 tal que
φ(a) ≤ Ca−k .
Isto é, mostre que, quando a → +∞, φ tende para zero pelo menos tão rapidamente quanto a−k .
Exercı́cio 166. Seja J um subintervalo finito da recta real e f : J −→ R uma função em escada. Dada [1]
uma função simples s : J −→ R e um número positivo ε, mostre que existe uma função em escada f tal
que
Z
|s − f |dµL < ε.
J
Indicação : Mostre que, se A é um subconjunto mensurável de J, então existe uma união finita de
intervalos B tal que d(A, B) = µ(S(A, B)) < ε. De seguida, prove a igualdade pretendida para o caso
em que s = IA . Prossiga.
Exercı́cio 167. Considere o espaço de medida (X, 2X , δa ) em que δa é a medida de Dirac no ponto [1]
a, sendo este um ponto escolhido do conjunto X. Seja f ∈ M+ . Mostre que então:
Z
f dδa = f (a) .
X
Indicação : Relembre a definição da medida de Dirac e de integral de uma função mensurável
positiva.
Exercı́cio 168. Seja N, 2N , µc ) o espaço mensurável em que µc é a medida de contagem sobre o [1]
espaço mensurável maximal sobre o conjunto dos inteiros positivos. Mostre que para f ∈ M+ se tem:
p=+∞
Z
f dµc =
N
X
f (p) .
p=0
Indicação : Proceda como no exercı́cio anterior.
Exercı́cio 169. Seja (X, A, µ) um espaço de medida. Mostre que existe uma aplicação φ, única, [1]
definida em M+ e tomando valores em R+ verificando as seguintes propriedades:
1. ∀f, g ∈ M+ ∀α ∈ R+ φ(αf ) = αφ(f ) e φ(f + g) = φ(f ) + φ(g) .
2. ∀(fn )n∈N ∈ MN+ fn ≤ fn+1 ⇔ limn→+∞ φ(fn ) = φ(limn→+∞ fn ) .
3. ∀A ∈ A φ(1A ) = µ(A) .
147
Capı́tulo 4
Indicação : Relembre a observação feita na aula a quando da definição de integral de uma função
simples e relacione-a com a terceira propriedade que deseja obter para a função φ. Este exercı́cio mostranos em particular que as propriedades de ”linearidade”e de ”continuidade”do integral asseguram-lhe uma
”unicidade”.
Exercı́cio 170. Seja (X, A, µ) um espaço de medida e f e g duas funções mensuráveis positivas. [2]
Têm-se então as propriedades fundamentais seguintes:
R
1. f (x) = 0 µ − qptp ⇔ X f dµ = 0 .
R
2. X f dµ < +∞ ⇒ f < +∞ µ − qptp .
R
R
3. f (x) = g(x) µ − qptp ⇒ X f dµ = X g dµ .
Indicação : Para demonstrar a segunda propriedade escreva o conjunto em que a função é finita
como a união numerável de conjuntos mensuráveis cuja medida possa controlar.
[1]
Exercı́cio 171. Seja (X, F, µ) um espaço de medida. Mostre que:
1. ∀f, g ∈ mF+ , ∀E ∈ F se tem
Z
Z
f ≤g⇒
f dµ ≤
g dµ;
E
2. Se f =
então
Pm
j=1
E
βj 1Aj for uma função simples mensurável positiva na sua representação canónica,
m
X
;
s dµ : s ∈ S+
f
Z
βj µ(Aj ∩ E) = sup
E
j=1
3. ∀f ∈ mF+ , ∀E, F ∈ F se tem que
Z
Z
f dµ ≤
E⊆F ⇒
f dµ.
F
E
[3]
Exercı́cio 172. Seja (fn )N ∈N uma sucessão crescente de funções mensuráveis positivas, sobre o espaço
de medida (X, F, µ), convergindo (pontualmente) para f .
1. Mostre que
Z
Z
fn dµ ≤
lim
n→+∞
f dµ.
X
X
2. Seja ϕ uma função simples mensurável positiva. Mostre que λ, definido por
Z
λ(E) =
ϕ IE dµ,
∀E ∈ F
X
é uma medida.
3. Seja An = {x ∈ X : fn (x) ≥ αϕ(x)} com 0 < α < 1 e ϕ ∈ S+
f . Verifique que
Z
Z
Z
αϕ dµ ≤
fn dµ ≤
fn dµ.
An
4. Mostre que
Z
An
X
Z
ϕ dµ = lim
X
n→+∞
Z
ϕ dµ
e que então
An
X
5. Conclua que
Z
Z
f dµ ≤ lim
X
148
n→+∞
fn dµ.
X
Z
ϕ dµ ≤ lim
α
n→+∞
fn dµ.
X
A Integração à Lebesgue
[2]
Exercı́cio 173. Considere o espaço de medida (X, F, µ) e mostre que ∀f, g ∈ mF+ , ∀E ∈ F, ∀c ≥ 0,
Z
Z
Z
(f + g) dµ =
f dµ +
g dµ
E
e
E
E
Z
Z
(cf ) dµ = c
f dµ .
E
E
Exercı́cio 174. Seja X
P= {x1 , x2 , ...} um conjunto numerável. Seja P1 , P2 , ... uma sequência de números [2]
não negativos tal que ∞
n=1
P Pn = 1. Considere o espaço de medida (X, P(X), µ), com µ definida para
cada A ⊆ X por µ(A) = xi ∈A Pi . Mostre que qualquer função f : X → R é (X, P(X)) mensurável e
prove que, para f não negativa,
Z
∞
X
f dµ =
Pn f (xn )
X
n=1
Exercı́cio 175. Seja f : R → [0, ∞) mensurável. Dado a ∈ R, seja fa (x) = f (x − a). Mostre que fa é [2]
mensurável e que
Z
Z
fa dλ = f dλ .
Indicação : Ver exercicio 92.
Exercı́cio 176. Seja (X, F, µ) um espaço de medida e A e B subconjuntos mensuráveis de X. Mostre [1]
que, se µ(∆(A, B)) = 0, então, para toda a função mensurável não negativa f ,
Z
Z
f dµ =
f dµ.
B
A
Exercı́cio 177. Seja (X,
R A, µ) um espaço de medida e f e g duas funções mensuráveis positivas tais [3]
que f ≤ g e ainda que X g dµ < +∞. Mostre então que sempre que f − g tem sentido se tem também:
Z
Z
Z
(f − g) dµ =
f dµ −
g dµ .
X
X
X
Indicação : Mostre que ambos os membros da igualdade existem e depois que são iguais, usando o
procedimento habitual.
[1]
Exercı́cio 178 (Aplicação do teorema da convergência monótona de Lebesgue). .
1. Seja (fn )n∈N uma sucessão decrescente de funções
R mensuráveis positivas admitindo como limite
a função (mensurável) f . Suponha que se tem X f1 dµ < +∞. Mostre que então se tem:
Z
Z
fn dµ =
f dµ .
lim
n→+∞
X
X
2. Sendo (R, B(R), λ) o espaço de medida de Borel e a sucessão de aplicações definidas na recta
+
real e tomando valores em R definida para n inteiro positivo por fn = 1[n,+∞[ , mostre que a
sucessão assim definida é uma sucessão decrescente de funções mensuráveis mas que se tem:
Z
Z
lim fn dµ 6= lim
fn dµ .
X n→+∞
n→+∞
X
Indicação : Para a alı́nea 1) reduza o problema a uma situação já estudada. Para a alı́nea seguinte
calcule simplesmente cada um dos termos da desigualdade.
149
Capı́tulo 4
[1]
Exercı́cio 179 (Sobre o lema de Fatou). .
1. Para (fn )n∈N uma sucessão arbitrária de funções mensuráveis positivas tem-se sempre que:
Z
Z
fn dµ .
lim inf fn dµ ≤ lim inf
X n→+∞
n→+∞
X
2. Sendo (R, B(R), λ) o espaço de medida de Borel e a sucessão de aplicações definidas na recta real
+
e tomando valores em R definida para n inteiro positivo por fn = n1]0, 1 ] . Mostre que:
n
Z
Z
fn dµ .
lim inf fn dµ < lim inf
X n→+∞
n→+∞
X
Indicação : Para a primeira alı́nea reveja simplesmente a demonstração do livro [3] e para a segunda
calcule ambos os termos da desigualdade.
[1]
Exercı́cio 180. Verifique o lema de Fatou inverso para funções: se (fn )n∈N for uma sucessão de funções
mensuráveis
positivas em (X, F), tal que existe uma função g ∈ mF+ para a qual se tem fn ≤ g ∀n ∈ N
R
e X g dµ < +∞, então
Z
Z
(lim sup fn ) dµ ≥ lim sup
X
n→∞
n→∞
fn dµ.
X
Indicação : Aplique o lema de Fatou à sucessão (g − fn )n∈N .
[2]
Exercı́cio 181. Considere um espaço de medida (X, A, µ) e uma função mensurável positiva cujo
integral sobre X relativamente à medida µ seja finito.
1. Mostre que se definir o conjunto mensurável An = {x ∈ X : f (x) ≥ n}, para cada n inteiro,
então tem obrigatóriamente limn→+∞ nµ(An ) = 0.
2. Mostre que existe um espaço de medida, a que chamará (X, A, µ) e uma função mensurável
positiva
sobre esse espaço, tais que se tenha limn→+∞ nµ(An ) = 0 e contudo se tenha também
R
f
dµ
=
+∞.
X
Indicação : Pense nos espaços conhecidos procurando saber porque é que a recı́proca não vai valer,
em geral.
4.4.2
Integração de Funções Mensuráveis Quaisquer
Nesta secção vamos considerar funções mensuráveis sem fazer nenhuma restrição aos
eventuais valores negativos (finitos ou infinitos) que essas funções possam tomar. A
extensão da noção de integral de uma destas funções relativamente a uma medida finita (que é feita preservando a ”linearidade”do integral e usando as partes positiva e
negativa da função) vai obrigar a precauções suplementares com o fim de se evitarem
indeterminações.
[1]
Exercı́cio 182. Considere a função f : R → R definida por

−3, −6 ≤ x ≤ −4




−2 ≤ x ≤ −1
 2,
− 21 , − 13 ≤ x ≤ 1
f (x) =


2≤x≤3
 3,


0,
x < −6, x > 3 x ∈] − 4, −2[∪] − 1, −1/3[∪]1, 2[
150
A Integração à Lebesgue
Será f uma função simples? No caso de a resposta ser afirmativa, escreva-a na sua forma canônica.
Estude f quanto á mensurabilidade. No caso de f ser mensurável, calcule, recorrendo à definição de
integral de uma função mensurável qualquer,
Z
f (x) dλ.
R
Exercı́cio 183. Diga, justificando, se a seguinte afirmação é verdadeira: A combinação linear finita [1]
de funções mensuráveis é uma função mensurável. Se todas essas funções forem integráveis,
o integral da combinação linear é a combinação linear dos integrais. O valor desta proposição
ainda será mesmo se retirar-mos a condição de finitude da combinação linear? Justifique dando um
exemplo no espaço de medida (N, P(N), µc ).
Exercı́cio 184. No espaço de medida (N, P(N), µc ), indique quais os valores de α, com α parâmetro [1]
real, a função fα : N → R definida por
α
f (n) =
(n + 1)α
é µc integrável.
Exercı́cio 185. Seja (X, F, µ) um espaço de medida. Seja s uma função simples mensurável. Mostre [1]
que se A ∈ F é tal que µ(A) = 0, então
Z
s dµ = 0.
A
Conclua que se s1 e s2 são duas funções simples mensuráveis integráveis tais que s1 = s2 µ-qptp, então
Z
Z
s1 dµ =
s2 dµ.
X
X
n
Exercı́cio 186.
P Seja fn :] − 1, 1[→ R uma sucessão de funções mensuráveis definida por fn (x) = x . [1]
Seja f (x) = n≥0 fn (x). Determine
Z
f (x)dλ
[0,1/2]
por dois métodos distintos.
Exercı́cio 187. Seja um espaço de medida (X, A, µ) e uma função µ-integrável f . Mostre que então [2]
se tem:
Z
f = 0 µ − qptp ⇔ ∀A ∈ A
f dµ = 0 .
A
Indicação : Relembre a definição de integral de uma função mensurável qualquer dada como a
diferença dos integrais da respectivas partes positiva e negativa, quando estes existem.
Exercı́cio 188. Seja um espaço de medida (X, A, µ) e uma função µ-integrável f .
[3]
1. Considere a sucessão (fn )n∈N , formada pelas truncadas de f , isto é definida por:
∀n ∈ N ∀x ∈ X fn (x) = |f (x)|I{|f |≤n} + nI{|f |>n} .
Mostre que a sucessão (fn )n∈N é uma sucessão crescente de funções mensuráveis positivas que
converge pontualmente para |f |.
151
Capı́tulo 4
2. Conclua a seguinte propriedade de ”continuidade do integral relativamente ao domı́nio de integração”:
Z
∀ε > 0 ∃δ = δ(ε, f ) > 0 ∀A ∈ A µ(A) ≤ δ ⇒
|f | dµ < ε .
A
Indicação : Para a demonstração da propriedade de continuidade deverá tomar em conta que o
controle sobre o valor do integral, depende não só da extensão do domı́nio de integração mas também da
possibilidade da função tomar aı́ valores ”muito grandes”.
[1]
Exercı́cio 189. Seja (X, A, µ) um espaço de medida e considere uma sucessão (An )n∈N de subconjuntos
de X mensuráveis, tais que se tenha ainda:
n=+∞
X
µ(An ) < +∞ .
n=0
Mostre que o conjunto dos pontos de X que pertencem a uma infinidade de elementos da sucessão é um
conjunto µ-desprezável, isto é que:


n=+∞
\ p=+∞
[
Ap  = 0 .
µ
n=0
p≥n
Indicação : Este enunciado é a parte de Borel do célebre lema de Borel-Cantelli, resultado que é
de uso constante em teoria das probabilidades. Para a demonstração basta utilizar a sub-aditividade da
medida.
[2]
Exercı́cio 190. Seja (X, A, µ) um espaço de medida e considere uma sucessão (fn )n∈N de funções
mensuráveis tomando valores na recta real e verificando:
n=+∞
X Z
|fn | dµ < +∞ .
X
n=0
1. Mostre que nestas condições a série (
um conjunto µ-desprezável.
P
fn ) é absolutamente convergente sobre X a menos de
2. Mostre que a função f definida sobre X e tomando valores reais representada na forma seguinte
para um ponto x arbitrário do domı́nio:
f (x) =
n=+∞
X
fn (x)I{x:P fn (x) cv.} + 0 × I{x:P fn (x) cv.}c ,
n=0
é uma função mensurável.
3. Mostre finalmente que f integrável e que o seu integral relativamente à medida µ é dado por:
Z
f dµ =
X
n=+∞
X Z
n=0
fn dµ .
X
Indicação : Inspire-se na demonstração de um resultado semelhante no livro [3], na página 29.
[3]
Exercı́cio 191. Seja (X, A, µ) um espaço de medida finito. Considere uma sucessão (fn )n∈N de funções
mensuráveis limitadas tomando valores na recta real e definidas em X e convergindo uniformemente
para uma função f ∈ RX .
1. Mostre que cada um dos elementos da sucessão é uma função integrável.
152
A Integração à Lebesgue
2. Mostre que f é limitada. Mostre que de facto f é integrável e que o seu integral relativamente à
medida µ é dado por:
Z
Z
fn dµ .
f dµ = lim
n→+∞
X
X
Indicação : Utilize a majoração evidente que decorre da convergência uniforme e do facto do espaço
de medida em questão ser finito.
e µ
Exercı́cio 192. Seja (X, A, µ) um espaço de medida e seja (X, A,
e) o seu completado.
[3]
1. Seja f uma função mensurável definida em (X, A) e tomando valores no segmento positivo da
e R+ ) e que se tem ainda:
recta acabada. Mostre que f ∈ M(X, A,
Z
Z
f de
µ.
f dµ =
X
X
2. Seja f uma função µ-integrável. Mostre que então essa função é µ
e-integável e que se tem tal
como na alı́nea anterior:
Z
Z
f dµ =
f de
µ.
X
X
3. Seja fe uma função µ
e-integrável. Mostre que existe uma função tomando valores reais e µintegrável verificando ainda:
Z
Z
Z
|f − fe| de
µ=0 e
f dµ =
f de
µ.
X
X
X
Indicação : Relembre as definições envolvidas e as técnicas básicas para a verificação que uma
função de um tipo dado é integrável.
Exercı́cio 193. Seja (X, A, µ) um espaço de medida e seja E um subconjunto de X. Considere a [3]
σ-álgebra AE , o ”traço”da σ-álgebra dada no sub-conjunto e a µE medida induzida pela medida dada
nessa σ-álgebra, que é definida por:
∀B ∈ AE µE (B) = µ(B ∩ E) .
1. Mostre que o trio (X, AE , µ)E é um espaço de medida.
2. Sendo f ∈ M+ , mostre que a restrição de f ao subconjunto E é mensurável (obviamente de
+
(X, AE ) em R munido da σ-álgebra de Borel habitual) e que se tem:
Z
Z
f IE dµ =
f|E dµE .
X
E
Indicação : São verificações triviais.
Exercı́cio 194. Seja X = [0, 1], RF = B([0, 1]) e µ = Leb. Defina-se fn := n 1]0,1/n[ . Mostre que [2]
limn→∞ fn = 0, ∀x ∈ X, mas que X fn dµ = 1 ∀n. Desenhe o gráfico de g := supn |fn | e mostre que
g∈
/ L1 (X, F, µ).
153
Capı́tulo 4
4.4.3
Teoremas de Convergência. Aplicações.
Exercı́cio 195. Considere a sucessão de funções fn : R → R definida por
[1]
1
fn (x) = −e n I[0,1− 1 ] (x) ,
n
para n ≥ 2. Represente graficamente f2 , f3 e f4 . Justifique
R se fn é umaRsucessão de funções mensuráveis.
Determine f (x) = limn→+∞ fn (x). Calcule limn→+∞ R fn (x) dλ e R f (x) dλ. Utilize o teorema de
convergência adequado para justificar que
Z
Z
f (x) dλ.
fn (x) dλ =
lim
n→+∞
R
R
Exercı́cio 196. Considere a sucessão de funções fn : R → R definida por
1
fn (x) = 1 − n I[0,1[ (x) ,
2
para n ≥ 1. Represente graficamente f1 , f2 e f3 . Justifique
R se fn é umaRsucessão de funções mensuráveis.
Determine f (x) = limn→+∞ fn (x). Calcule limn→+∞ R fn (x) dλ e R f (x) dλ. Utilize o teorema de
convergência adequado para justificar que
Z
Z
fn (x) dλ =
f (x) dλ.
lim
n→+∞
[1]
R
R
Exercı́cio 197. Use o teorema de convergência monótona de Lebesgue para obter a demonstração da
seguinte proposição:
Seja (X, F, µ) um espaço de medida. Seja fn : X → [0, +∞[ uma sequência de funções mensuráveis tal
que f1 ≥ f2 ≥ · · · ≥ 0 e
Z
f1 dµ 6= +∞.
X
Seja f (x) = limn→+∞ fn (x). Mostre que
Z
Z
lim
n→+∞
fn (x) dµ =
R
f (x) dµ .
R
[1]
Exercı́cio 198. Enuncie o teorema da convergência monótona de Lebesgue, o Lema de Fatou, o e o
teorema da convergência dominada de Lebesgue, no espaço de probabilidade (Ω, F, P).
[2]
Exercı́cio 199. Seja (fn )n∈N uma sucessão de funções mensuráveis em (X, F, µ), tais que:
i) (fn )n∈N converge (pontualmente) para f ;
ii) Existe g ∈ L1 (X, F, µ) positiva e tal que ∀x ∈ X, ∀n ∈ N, |fn (x)| ≤ g(x).
1. Verifique que f é integrável;
2. Mostre que
Z
Z
f dµ =
X
Z
( lim fn ) dµ = lim
X n→∞
n→∞
fn dµ.
X
Indicação
: Para
R
R a alı́nea 2) aplique o Lema de Fatou inverso à sucessão (|fn − f |)n∈N e tenha em
conta que | h dµ| ≤ |h| dµ, ∀h ∈ L1 .
[3]
Exercı́cio 200. Seja (R, B(R), λ) o espaço mensurável de Lebesgue e seja para cada x número real, o
inteiro definido por: [x] = inf{n ∈ N : |x| < n}.
154
[1]
A Integração à Lebesgue
1. Represente gráficamente a função real de variável real definida por:
(
log([x]) x 6= 0
∀x ∈ R f (x) =
0
no caso contrário.
2. Mostre que f é mensurável (obviamente do espaço de Borel, no espaço de Borel).
R
3. Calcule para n ∈ N∗ o integral : [−n,+n] f dλ.
Indicação : Reconheça o teorema de convergência que deve aplicar neste caso e oriente a redacção
da solução nesse sentido.
Exercı́cio 201. Seja (R, B(R), λ) o espaço mensurável de Lebesgue e seja para cada x número real, o [3]
inteiro definido por: [x] = inf{n ∈ N : |x| < n} − 1
1. Represente gráficamente a função real de variável real definida por:
∀x ∈ R g(x) = exp(−[x]) ,
mostrando em seguida que é uma função mensurável (relativamente aos espaços adequados).
R
f dλ
R
2. Calcule o integral :
Indicação : É uma variante do anterior.
Exercı́cio 202. Seja (X, F, µ) um espaço de medida e f uma função mensurável, limitada e não negativa. [2]
Mostre que
Z
n2n X
k−1
k−1
k
f dµ = lim
µ
x
∈
X;
≤
f
(x)
<
.
n→∞
2n
2n
2n
X
k=1
1
RExercı́cio 203. Seja fn : R −→ R definida por n I]0,n[ . Mostre que fn −→ 0 uniformemente mas [1]
fn dµL = 1. Porque é que este exemplo não é um contra-exemplo ao teorema da convergência dominada
de Lebesgue?
R
R
Exercı́cio 204. Compare lim inf fn dµL com lim inf fn dµL para a sequência dada no exercicio ante- [2]
rior. Poderá a desigualdade no lema de Fatou ser substituida por uma igualdade?
Exercı́cio 205. Seja (X, F, µ) um
R espaço de medida. Uma função mensurável f : X −→ R diz-se [2]
integrável em média quadrática se f 2 dµ < ∞. Mostre que, se µ(X) < ∞, toda a função integrável em
média quadrática é integrável.
Indicação : Considere separadamente o integral de |f | sobre o conjunto onde |f | ≥ 1 e o integral
sobre o conjunto onde |f | < 1.
Exercı́cio 206. Seja (X, F, µ) um espaço de medida. Uma sequência de funções mensuráveis fn : X −→ [2]
R n = 1, 2, ..., diz-se que converge em medida para zero se para todo o ε > 0
µ{x ∈ X; |fn (x)| > ε} −→ 0
quando n −→ ∞. Mostre que, se
recı́proco não é válido.
R
|fn |dµ −→ 0, então fn converge para zero em medida. Mostre que o
Exercı́cio 207. Seja (X, F, µ) um espaço de medida e f e g funções mensuráveis. Mostre que, se f é [2]
integrável e g é limitada e mensurável, então f g é integrável.
155
Capı́tulo 4
Exercı́cio 208. Considere o espaço de probabilidade (Ω, F, P) e a sucessão de variáveis aleatórias [2]
(Xn )n∈N convergente pontualmente para X. Mostre que, se existir uma constante k para a qual |Xn (ω)| ≤
k, ∀(n, ω), então limn→+∞ E[|Xn − X|] = 0.
Exercı́cio 209 (Integração relativamente à medida imagem). . Seja (X, A, µ) um espaço mensurado [3]
e seja (Y, B) um outro espaço mensurável. Considere F ∈ Y X uma aplicação mensurável do primeiro
espaço no segundo e a função de conjuntos µF , definida por:
∀B ∈ B µF (B) = µ(F −1 (B) .
1. (Y, B, µF ) é um espaço de medida.
2. Para B elemento da σ-álgebra B, tem-se que: 1B ◦ F = 1F −1 (B) .
3. Para uma função f mensurável de (Y, B) no espaço de Lebesgue (R, B(R)) tem-se que:
Z
Z
f ◦ F dµ .
f dµF =
X
Y
4. No caso particular em F ∈ NZ é a aplicação mensurável relativamente às σ-álgebras maximais e
definida por F (n) = n2 , para cada inteiro relativo, em que a medida µ é a medida de contagem
sobre (Z, 2Z ), calcule o seguinte integral, para p, q inteiros positivos não nulos.
Z
log(n)1[p,p+q] (n) dµF (n) .
N
Indicação : Para a solução da alı́nea 3) deverá seguir o método geral para provar um resultado
relativo às funções mensuráveis. Começar com as funções caracterı́sticas, passar para as funções simples
e seguidamente para as funções mensuráveis positivas terminando com as funções mensuráveis quaisquer.
[3]
Exercı́cio 210. Seja (X, F, µ) um espaço de medida e f uma função mensurável não negativa. Para
A ∈ F seja
Z
µf (A) =
f dµ.
A
Mostre que µf é uma medida em F. Mais, mostre que, se g é uma função mensurável não negativa,
então para E ∈ F
Z
Z
gdµf =
gf dµ.
E
E
[1]
Exercı́cio 211. [2] Calcule as esperanças matemáticas das variáveis aleatórias definidas nos exercı́cios
100, e 255.
[1]
Exercı́cio 212. [2] Calcule a média da variável aleatória X cuja distribuição verifica P (X = k) =
θk
e−θ , onde θ > 0 (diz-se que X segue uma distribuição de Poisson de parâmetro θ).
k!
[1]
Exercı́cio 213. Seja X uma variável aleatória com lei de Bernoulli, sendo P[X = 1] = p e P[X = 0] =
1 − p.
1. Determine PX (B), com B ∈ B(R).
2. Considere a medida de Dirac, no espaço (R, P(R), δa ). Escreva PX (B) como combinação linear
de medidas de Dirac.
156
A Integração à Lebesgue
3. Seja f ∈ L1 (R, P(R), δa ), mostre que
Z
f (x) dδa (x) = f (a).
R
4. Mostre que E[h(x)] = ph(1) + (1 − p)h(0).
5. Calcule o valor médio, a variância e o momento de ordem n em relação à origem da variável
aleatória X.
[1]
Exercı́cio 214. Seja X uma variável aleatória tal que
∀B ∈ B(R)
Z
P[X ∈ B] =
f (x) dx.
B
1. Mostre que FX , a função de distribuição de X, é derivável em R e que ∀x ∈ R,
F 0 (x) = f (x).
2. Seja h : R −→ R tal que E[|h(x)|] < +∞. Mostre que
Z
h(x) f (x) dx.
E[h(X)] =
R
3. Considere a variável aleatória X com lei normal reduzida. Calcule E[X].
Exercı́cio 215. Seja (X, A, µ) um espaço de medida, E um subconjunto mensurável e uma função [3]
definida em X e tomando valores reais, que verifique ainda:
Z
∃a ≥ 1 ∀x ∈ X f (x) ≥ a e
f dµ < +∞ .
E
∗
1/n
1. Mostre que se tem: ∀n ∈ N f
≤ f.
R 1/n
2. Conclua que: limn→+∞ E f
dµ = µ(E).
Indicação : Reconheça de que teorema de convergência é, este exercı́cio, uma aplicação.
Exercı́cio 216. [2] Sejam Xn variáveis aleatórias reais tais que existe uma variável aleatória real Y
integrável e que verifica |Xn (ω)| ≤ Y (ω), n ∈ N, q.c.. Prove que
Z
lim sup
|Xn | dP = 0.
c→+∞ n∈N
[2]
{|Xn |>c}
Exercı́cio 217 (Transformada de Fourier de uma Medida Positiva). . Seja µ uma medida positiva [3]
sobre o espaço de Borel (R, B(R)) de massa total igual à unidade (isto é uma probabilidade). Considere
a correspondência complexa de variável real definida por:
Z
∀t ∈ R ψµ (t) =
eitx dµ(x) .
R
1. Mostre que ψµ é de facto uma função bem definida e que se tem para todo o t número real
|ψµ (t)| ≤ 1.
2. Mostre que ψµ é uma função contı́nua do seu argumento, mostrando que:
∀t ∈ R ∀(sn )n∈N ∈ RN
lim sn = t ⇒ lim ψµ (sn ) = ψµ (t) .
n→+∞
n→+∞
3. Mostre que ψµ (0) = 1 e que a função ψµ é de tipo positivo, isto é, que se tem para qualquer
inteiro estritamente positivo n:
X X
∀t1 , t2 , ..., tn ∈ R ∀a1 , a2 , ..., an ∈ C
ψµ (ti − tj )ai aj ≥ 0 .
1≤i≤n 1≤j≤n
157
Capı́tulo 4
Indicação : Para a alı́nea 2), use um teorema de convergência adequado.
Exercı́cio 218. A função caracterı́stica de uma variável aleatória X é uma aplicação ϕX : R −→ C [1]
definida por:
ϕX (t) = E[eitX ] .
1. Verifique que eitX é integrável.
2. Verifique as seguintes propriedades
(a) ϕX (0) = 1.
(b) |ϕX (t)| ≤ 1,
∀t ∈ R.
(c) ϕX (t) é contı́nua em R.
(d) ϕ−X (t) = ϕX (t),
∀t ∈ R.
(e) ϕaX+b (t) = eibt ϕX (at).
[1]
Exercı́cio 219. Sejam Xi , com i = 1, . . . , n, variáveis aleatórias independentes com lei normal reduzida.
1. Determine ϕXi (t).
2. Determine ϕX , atendendo a que
h
i
ϕX (t1 , t2 , . . . , tn ) = E ei(t1 X1 +t2 X2 +...+tn Xn )
[2]
com
X = (X1 , X2 , . . . , Xn ).
Exercı́cio 220. Seja ϕ a função caracterı́stica da variável aleatória X com função de distribuição F .
Mostre que:
1. Se F é contı́nua nos pontos a e b (a < b) então
F (b) − F (a) = lim
T →+∞
1
2π
Z
T
−T
eita − eitb
ϕ(t) dt.
it
2. Se ϕ(t) for integrável então X tem função densidade f (x) = F 0 (x) contı́nua e
Z
1
f (x) =
e−itx ϕ(t) dt.
2π R
[3]
Exercı́cio 221 (Cálculo de um integral). .
1. Mostre que para qualquer par (x, t) pertencente ao produto cartesiano do intervalo [−1, 1] pelo
intervalo [0, +∞[, vale:
n=+∞
X n
sen(t)
=
x sen(t)e−(n+1)t .
t
e −x
n=0
2. Mostre que vale:
n=N
X
2t
n
−(n+1)t ∀t > 0 ∀N ∈ N x sen(t)e
.
≤ t
e
−x
n=0
3. Mostre, utilizando o que precede, que a função f (t) =
R +∞
de Borel nele próprio e calcule: 0 f (t) dt.
sen(t)
et −x
é mensurável do espaço mensurável
Indicação : Relembre o resultado sobre a soma dos termos de uma progressão geométrica e procure
obter as majorações indicadas após o que terá que utilizar um teorema de convergência.
158
A Integração à Lebesgue
[3]
Exercı́cio 222. Uma igualdade entre funções de distribuição Considere (X, A, µ) um espaço
mensurado e uma aplicação f , definida em X tomando valores nos inteiros positivos e integrável.
Sejam ainda por definição:
F (t) = µ({x ∈ X : f (x) > t}) e G(t) = µ({x ∈ X : f (x) ≥ t}) .
1. Mostre se para n inteiro positivo fôr Mn = µ({x ∈ X : f (x) = n}), então será também:
Z
f (x) dµ(x) =
n=+∞
X
X
nMn .
n=0
2. Conclua então que:
Z
f (x) dµ(x) =
n=+∞
X
X
F (n) =
n=+∞
X
n=0
G(n) .
n=0
Indicação : Explicite o integral a calcular usando o facto da função em questão ser a valores
inteiros.
Exercı́cio 223 (Uma desigualdade ”de tipo Markov”). . Seja (X, A, µ) um espaço mensurado, r e ε [3]
números reais estritamente positivos. Seja ainda uma função real de variável em X mensurável (nas
σ-álgebras adequadas). Mostre que se tem então:
Z
1
µ({x ∈ X : |f (x)| > ε}) ≤ r
|f (x)|r dµ(x) .
ε X
Indicação : Decomponha o integral correspondente ao primeiro termo da desigualdade em duas
partes, uma das quais desprezará.
Exercı́cio 224. Seja X : Ω → Rn uma variável aleatória tal que:
∃p ∈ ]0, +∞[
[1]
E[|X|p ] < ∞.
Mostre a desigualdade de Chebychev
P[|X| ≥ λ] ≤
1
E[|X|p ]
λp
∀λ > 0.
[2]
Exercı́cio 225. Suponha que existe K > 0 tal que
M = E[exp (k|X|)] < ∞.
Mostre que
E[|X| ≥ λ] ≤ M e−Kλ
∀ ≥ 0.
[1]
Exercı́cio 226. Sejam X, Y : Ω → R duas variáveis aleatórias independentes. Mostre que:
E[XY ] = E[X] · E[Y ]
Exercı́cio 227. Seja (X, A, µ) um espaço de medida. Sejam p, q ∈ [1, +∞] , considerados como expo- [3]
entes conjugados. Seja a sucessão (fn )n∈N ( respectivamente (gn )n∈N ) de elementos de Lp (X, A, µ) (
respectivamente de Lq (X, A, µ) ) convergente para f ∈ Lp ( respectivamente para g ∈ Lq ). Mostre que:
lim ||fn gn − f g||1 = 0 .
n→+∞
159
Capı́tulo 4
Exercı́cio 228. Considere o espaço de Lebesgue ([0, 1], L, λ) e para cada x ∈ [0, 1] seja a representação [3]
decimal de x dada por:
n=+∞
X an (x)
x=
, an (x) ∈ {0, 1, 2, ..., 9} .
10n+1
n=0
(Consideraremos sempre, para o caso de um número racional, a representação decimal que não termina.)
1. Mostre que, para cada n inteiro positivo ou nulo, a correspondência que a x em [0, 1], associa
an (x) número real em {0, 1, 2, ..., 9}, define uma aplicação.
2. Mostre que, para cada n inteiro positivo ou nulo, a aplicação an (x) é mensurável (relativamente
às σ-álgebras adequadas).
R
3. Determine [0,1] an (x)dλ(x), para cada n inteiro positivo ou nulo.
[3]
Exercı́cio 229. Seja (X, A, µ)um espaço de medida σ-finito e seja g uma aplicação de X em R ”Borel”mensurável e verificando:
∀f ∈ L1 (X, A, µ) f g ∈ L1 (X, A, µ) .
Mostre que:
∃a ∈ R µ{x ∈ X : |g(x)| > a} = 0 .
[3]
Exercı́cio 230. Considere o espaço mensurado de Lebesgue (R, L, λ) e seja (gn )n∈N a sucessão de
funções reais de duas variáveis reais definidas por:
(−1)n (xy)n
−x2
∀x ∈ R ∀y ∈ R gn (x, y) = √
exp
.
2
2π (2n)!
1. Admitindo que:
Z
x2n exp
∀n ∈ N
R
−x2
2
dλ(x) =
√
2π
(2n)!
,
2n n!
e relembrando os resultados conhecidos sobre a função exponencial (domı́nio, contradomı́nio, continuidade, desenvolvimento em série de potências. etc), mostre que:
∀y ∈ R
∞ Z
X
n=0
|gn (x, y)|dλ(x) < +∞ .
R
2. Conclua que:
1
∀y ∈ R √
2π
[3]
Z
exp
R
−x2
2
exp(ixy)dλ(x) = exp
−y 2
2
Exercı́cio 231. Seja (X, A, µ) um espaço de medida finito. Seja (νn )n∈N , uma sucessão crescente de
medidas positivas finitas sobre (X, A), cada uma delas absolutamente contı́nua relativamente à medida
µ. Suponha que para cada A ∈ A, o limite seguinte:
ν(A) = lim νn (A) ,
n→+∞
existe no conjunto dos números reais R.
1. Mostre que a aplicação definida por ν é uma medida sobre (X, A).
2. Mostre que:
∀n ∈ N ∃hn ∈ L1 (X, A, µ) ∀A ∈ A νn (A) =
Z
hn dµ .
A
3. Mostre que a sucessão (hn )n∈N é crescente.
160
A Integração à Lebesgue
4. Mostre que a medida ν é absolutamente contı́nua relativamente a µ.
[3]
Exercı́cio 232. Seja (X, A, µ) um espaço de medida. Considere (An )n∈N , uma sucessão crescente de
subconjuntos mensuráveis tais que se tenha :
[
X=
An .
n∈N
Seja f uma função numérica Borel-mensurável, verificando a condição:
Z
∃c > 0 ∀n ∈ N
|f |dµ < c .
An
Mostre que então temos f ∈ L (X, A, µ).
1
Exercı́cio 233. Considere para n inteiro positivo, Rn como o espaço das funções numéricas contı́nuas [3]
definidas no espaço topológico compacto ({1, ..., n}, P({1, ..., n})). Seja λ uma forma linear positiva
definida sobre Rn . Mostre que existe µ, uma medida positiva única definida no espaço mensurável
({1, ..., n}, P({1, ..., n})), tal que:
∀X = (x1 , x2 , ..., xn ) ∈ Rn λ(X) = µ(1)x1 + µ(2)x2 + ... + µ(n)xn .
Exercı́cio 234. Considere o espaço mensurado de Lebesgue ([0, 1], L, λ) e para x ∈ [0, 1] seja uma das [3]
representações decimais de x dada por:
x=
n=+∞
X
n=0
an
, an ∈ {0, 1, 2, ..., 9} .
10n+1
1. Seja para cada n inteiro não negativo, o conjunto An , definido por:
An = {x ∈ [0, 1] : ∀p ∈ {0, 1, ..., n} ap 6= 2} .
Descreva sucintamente A0 e A1 .
9 n+1
2. Mostre que: ∀n ∈ N, λ(An ) ≤ ( 10
)
3. Mostre que: λ(lim sup An ) = 0.
Exercı́cio 235. Sejam (X, A) ainda (Y, B) dois espaços mensuráveis e f uma aplicação de X em Y (não [3]
necessariamente mensurável). Seja (An )n∈N uma partição de X, formada por conjuntos A-mensuráveis
e seja:
∀n ∈ A An = σ((Ap )p∈{0,1,2,...,n} ) .
1. Mostre que (An )n∈N é uma sucessão crescente de sub-σ-álgebras de A.
p=n
2. Seja para cada n inteiro fn , a restrição de f a ∪p=0
Ap . Mostre que se f fôr mensurável de (X, A)
em (Y, B), então fn é mensurável de (X, An ) em (Y, B), para cada n inteiro.
3. Suponha que para cada n inteiro se tem fn mensurável de (X, An ) em (Y, B). Mostre que então
f é mensurável de (X, A) em (Y, B).
Indicação : Mostre que ∀x ∈ X f (x) = limn→∞ fn (x).
Exercı́cio 236. Seja (X, P(X), µ)um espaço de medida que verifica ainda:
[3]
∀x ∈ X µ({x}) = 1 .
Considere uma função numérica f integrável relativamente à medida µ.
161
Capı́tulo 4
1. Mostre que se f fôr uma função indicatriz de um dado conjunto A, então o conjunto A é necessariamente finito.
2. Mostre que no caso geral, o suporte de f é necessariamente uma sucessão (xi )i∈N de elementos
de X.
R
P
P
3. Mostre que então X f dµ = i∈N f (xi ) e que se tem ainda i∈N |f (xi )| < +∞.
Exercı́cio 237. Considere o espaço de Lebesgue ([0, 1], L, λ) e para cada n inteiro não nulo a função [3]
definida por:
∀x ∈ [0, 1] fn (x) = xn−1 − 2x2n−1 .
R
R 1 Pn=+∞
P
1
f (x)dλ(x) .
1. Calcule e compare 0
fn (x) dλ(x) e ainda n=+∞
n=1
n=1
0 n
2. Explique os resultados obtidos (usando os teoremas de convergência que estudou).
[3]
Exercı́cio 238. Prove que se G é uma variável aleatória com distribuição normal N (0, 1), então, para
x > 0,
Z ∞
1 2
1 2
1
1
P[G > x] = √
e− 2 y dy ≤ √ e− 2 x .
2π x
x 2π
Sejam X1 , X2 , ... uma sequência de variáveis aleatórias independentes com distribuição N (0, 1). Prove
que, co probabilidade 1, L ≥ 1, onde
p
L = lim sup(Xn / 2 log n).
Seja Sn = X1 + ... + Xn . Recorde que Sn /sqrtn tem distribuição N (0, 1). Prove que
h
n
oi
p
P lim inf |Sn | < 2 n log n = 1.
Observação 41. Note que este resultado implica a lei Forte: P[Sn /n → 0] = 1.
Comentários à Bibliografia
Para os leitores que desejem uma perspectiva cultural abrangente sobre as teorias da
medida e da integração o livro de David Bressoud [1] é leitura indispensável.
Bibliografia
[1] David M. Bressoud. A radical approach to Lebesgue’s theory of integration. MAA
Textbooks. Cambridge University Press, Cambridge, 2008.
[2] Paulo Eduardo de Oliveira. Exercı́cios de Teoria das Probabilidades. Coimbra, 1998–
1999.
[3] Walter Rudin. Real and complex analysis. McGraw-Hill Book Co., New York, third
edition, 1987.
162
Capı́tulo 5
Os Espaços de Funções
Integráveis
5.1
Introdução
Neste texto procuramos esclarecer as propriedades essenciais dos conjuntos de funções integráveis. Estes conjuntos dão origem aos exemplos fundamentais de espaços de Banach.
Preparamos também o leitor de forma a podermos abordar com bases sólidas a teoria
das martingalas. Estes processos estocásticos a tempo discreto são modelos naturais
da evolução do preços dos activos nos mercados financeiros. A teoria das martingalas
necessita uma aproximação rigorosa e completa das noções ligadas às probabilidades
condicionais e este tipo de aproximação, às probabilidades e esperanças condicionais,
só pode ser feito rigorosamente no quadro que detalhamos seguidamente. São leituras
aconselhadas para as matérias que vamos expor os textos [4] e [3].
5.2
Os conjuntos de funções integráveis Lp
Seja (X, A, µ) um espaço de medida qualquer. Nesta secção estudaremos os conjuntos de
funções, definidas em X e tomando valores em R = [−∞, +∞] cujos módulos levantados
a potências de ordem p ∈ [1, +∞[ são funções integráveis.
Notações
O conjunto das funções f : X −→ [−∞, +∞] que são mensuráveis de (X, A) em
(R, B(R)) representar-se-á por M((X, A), (R, B(R))) ou por M(A), ou ainda por M
quando não seja de recear uma qualquer ambiguidade relativamente aos espaços mensuráveis em discussão.
163
Capı́tulo 5
Definição 36. Seja para p ∈ [1, +∞[, o conjunto Lp (X, A, µ), que salvo qualquer receio de ambiguidade também representaremos no que vai seguir-se por
Lp , definido por:
Z
p
p
| f | dµ < +∞ .
L (X, A, µ) = f ∈ M(A) :
X
Uma primeira observação que é possı́vel fazer é que se α ∈ R e f ∈ Lp então como
se tem
Z
Z
Z
| f |p dµ < +∞ ,
| α |p | f |p dµ =| α |p
| αf |p dµ =
X
X
X
Lp .
vale também que αf ∈
Naturalmente, põe-se a questão de saber se para duas
funções pertencentes a Lp a soma ainda pertence a Lp . Observemos, primeiramente, que
há uma dificuldade em definir a soma ponto a ponto para tais duas funções quaisquer.
Com efeito, tem-se que:
Z
f ∈ Lp ⇒
| f |p dµ < +∞ ⇒| f |< +∞ µ-qptp .
X
o mesmo acontecendo para g ∈ Lp . Em consequência, f + g está definida salvo, talvez,
num conjunto de medida nula, isto é:
0
0
0
∃X ⊂ X, X ∈ A, µ((X )c ) = 0
0
∀x ∈ X (f + g)(x) = f (x) + g(x) ,
0
não podendo nós garantir que sobre (X )c seja possı́vel definir de uma só maneira f + g.
Representaremos seguidamente por f +̃g a adição de duas funções salvo, talvez, num
conjunto de medida nula.
Proposição 35. Seja p ∈ [1, +∞[ e f, g ∈ Lp . Então f +̃g ∈ Lp .
Demonstração. Observe-se que dado que a função F (x) = xp é convexa (veja-se a
secção 5.5) tem-se que para a, b ∈]0, +∞[
a b
(a + b)p = 2p ( + )p ≤ 2p−1 (ap + bp )
2 2
Em consequência
Z
Z
Z
p
p−1
p
p
| f +̃g | dµ ≤ 2
(| f | + | g | )dµ =
X
X
X
p
Z
| f | dµ +
| g |p dµ < +∞ ,
X
tal como se pretendia.
Pode pois constatar-se que Lp , não sendo um espaço vectorial de funções, dado que
não é possı́vel definir a adição pontual de funções tem, no entanto, as propriedades
164
Os Espaços de Funções Integráveis
usuais desejadas de estabilidade relativamente à operação +̃ a que abusivamente chamaremos adição e à multiplicação por um escalar. Veremos, adiante, como ultrapassar
esta dificuldade passando ao cociente.
A proposição que apresentamos
a seguir mostra, em particular, que a correspondênR
p
p
cia que a f ∈ L associa X | f | dµ, que já sabemos ser uma aplicação homogénea de
grau p é, também, subaditiva e subaditiva-σ.
Proposição 36 (Desigualdades de Hölder e Minkowski). Sejam p, q ∈]1, +∞[
expoentes conjugados, isto é, tais que:
1 1
+ =1.
p q
Então tem-se para f, g funções mensuráveis positivas quaisquer que:
1. Desigualdade de Hölder
Z
Z
p
f g dµ ≤
X
1/p Z
1/q
q
f dµ
g dµ
X
X
2. Desigualdade de Minkowski
Z
p
(f + g) dµ
X
1/p
Z
≤
p
1/p
f dµ
Z
+
X
p
1/p
g dµ
X
Demonstração. A primeira desiguldade é consequência das duas desigualdades que formulamos seguidamente, válidas para α, β > 0 com α + β = 1 e a, b ≥ 0. Primeiramente,
temos:
aα · bβ ≤ α a + β b .
Com efeito, sendo a função F (x) = − ln x uma função convexa tem-se que, como α+β = 1
− ln(α a + β b) ≤ −α ln a − β ln b .
Donde resulta a desigualdade pretendida, aplicando a exponencial aos dois membros
desta desigualdade. Em segundo lugar e, em consequência, temos para p, q expoentes
conjugados:
1
1
a · b ≤ ap + bq ,
p
q
que resulta de aplicar a primeira desigualdade com a = up , b = v q , α = 1/p e β = 1/q.
Para demosnstrarmos as desigualdades enunciadas na proposição podemos sempre supor
que
1/q
Z
1/p
Z
f p dµ
< +∞ e 0 <
g q dµ
< +∞ .
0<
X
X
165
Capı́tulo 5
Se tal não acontecer as desigualdades são trivialmente verdadeiras não havendo nada a
demonstrar. Com esta primeira hipótese é agora possı́vel supor que se tem:
Z
1/p
p
Z
f dµ
= 1 e, também,
X
1/q
q
g dµ
=1
X
1/p
R
Com efeito, se tal não acontecer podemos sempre substituir f por f˜ := f / X f p dµ
1/p
R
e g por g̃ := g/ X g p dµ
, estando f˜ e g̃ bem definidas dado que os integrais nos
denominadores respectivos são números reais estritamente positivos. Sob esta hipótese,
dado que então, como já anteriormente observáramos, f e g são finitas salvo talvez num
conjunto de medida nula, podemos escrever que:
f ·g ≤
1 p 1 q
f + g µ − qptp ,
p
q
pelo que integrando sobre X se tem, tendo em conta as hipóteses sobre p, q, f e g:
Z
f gdµ ≤
X
1
p
Z
f p dµ
1/p
+
X
1
q
Z
g q dµ
1/q
Z
f p dµ
=1=
X
1/p Z
X
g q dµ
1/q
,
X
isto é, a desigualdade de Hölder. Para a demonstração da desigualdade de Minkowski
observemos, primeiramente, que a soma f + g está sempre bem definida como função
mensurável com valores em [0, +∞]. Seguidamente, apliquemos a desigualdade de Hölder
observando que p e q = p/(p − 1) são expoentes conjugados tendo-se então a cadeia de
desigualdades e igualdades seguinte.
Z
Z
(f + g)p dµ =
(f + g) · (f + g)p−1 dµ =
X
Z
ZX
p−1
f · (f + g) dµ +
g · (f + g)p−1 dµ ≤(a)
=
X
X
Z
≤
1/p Z
1/q
p
q(p−1)
f dµ
·
(f + g)
dµ
+
X
X
1/p Z
1/q
p
q(p−1)
g dµ
·
(f + g)
dµ
=
X
X
Z
Z
! Z
Z
+
1/p
p
=
f dµ
1/p
p
+
X
g dµ
X
·
p)
(f + g) dµ
1
q
.
X
Em resumo obteve-se que:
Z
X
(f + g)p dµ ≤
Z
X
f p dµ
1/p
Z
+
X
1/p ! Z
1
q
p)
p
(f + g) dµ
g dµ
·
;
X
R
ou ainda, observando que no caso trivial em que X (f + g)p dµ = 0 não há nada a
demonstrar dado que a desiguladade de Minkowski é, nessas condições, trivialmente
166
Os Espaços de Funções Integráveis
verdadeira,
Z
(f + g)p dµ
1− 1
Z
q
f p dµ
≤
1/p
Z
g p dµ
+
X
X
1/p !
.
X
Com efeito, pode-se dividir, ambos os membros da desigualdade anterior pelo factor à
direita no termo da direita desta desigualdade. Uma vez que em virtude da definição de q
se tem que 1 − 1/q = 1/p e, portanto, o expoente do membro à esquerda na desigualdade
é 1/p, a desigualdade anterior é, exactamente, a desigualdade de Minkowski.
A propriedade seguinte estende a desigualdade de Minkowski às séries de funções
mensuráveis positivas.
Proposição 37 (Subaditividade-σ ou convexidade numerável). Seja (fn )n∈N∗
uma sucessão de funções mensuráveis positivas. Então tem-se que:
Z
X
+∞
X
!1/p
!p
fn
+∞ Z
X
≤
dµ
n=1
fnp dµ
1/p
.
X
n=1
Demonstração. Trata-se
P de uma aplicação imediata do lema de Fatou à série de funções
mensurv́eis positivas ( +∞
n=1 fn ), série que é convergente em [0, +∞]. Com efeito, tem-se
que:
Z
X
+∞
X
!1/p
!p
fn
dµ
N
X
Z
=
n=1
lim inf
X N →+∞
N
X
lim inf
N →+∞ X
N
X
= lim inf
≤ lim inf
N →+∞
X
!1/p
!p
fn
dµ
=
!1/p
!p
fn
≤(b)
dµ
n=1
N Z
X
n=1
≤(a)
dµ
n=1
Z
N →+∞
fn
n=1
Z
≤
!1/p
!p
X
fnp dµ
1/p
=
+∞ Z
X
n=1
fnp dµ
1/p
X
Note-se que desigualdade indicadas com (a) e (b) se devem respectivamente ao lema de
Fatou e à desigualdade de Minkowski.
Seguidamente, vamos explorar algumas propriedades, a que chamaremos propriedades métricas dos conjuntos de funções integráveis. Definimos agora uma noção de convergência em tudo semelhante às noções de convergência usuais para funções, por exemplo a convergência uniforme de sucessões de funções.
167
Capı́tulo 5
Definição 37. Seja para p ∈ [1, +∞[, (fn )n∈N∗ uma sucessão de funções pertencentes ao conjunto Lp . Diremos que esta sucessão converge em Lp se e só
se:
Z
1/p
p
∗
∗
p
∃f ∈ L ∀ > 0 ∃N ∈ N ∀n ∈ N n ≥ N ⇒
| f − fn | dµ
≤.
X
Tal como no caso da convergência uniforme de funções importa, por vezes, demonstrar que uma sucessão de funções converge sem que conheçamos o limite desta sucessão.
Tal como no caso da convergência uniforme podemos utilizar, na convergência em Lp , o
critério de Cauchy que explicitamos a seguir.
Definição 38. Seja para p ∈ [1, +∞[, (fn )n∈N∗ uma sucessão de funções pertencentes ao conjunto Lp . Diremos que esta sucessão é uma sucessão de Cauchy
em Lp se e só se:
Z
1/p
∗
∗
p
∀ > 0 ∃N ∈ N ∀n, m ∈ N n, m ≥ N ⇒
| fm − fn | dµ
≤.
X
Tal como nos casos já estudados, é fácil mostrar que se uma sucessão converge em
Lp então é de Cauchy em Lp . A recı́proca também é verdadeira pondo em evidência o
carácter completo dos conjuntos Lp .
Teorema 21. Seja para p ∈ [1, +∞[, (fn )n∈N∗ uma sucessão de Cauchy em Lp . Então
a sucessão converge em Lp .
Demonstração. A definição de sucessão de Cauchy em Lp , mostra que é possı́vel escolher
uma sucessão de inteiros (nk )k∈N∗ tal que se verifique.
Z
1/p
1
∗
p
∀k ∈ N
| fnk+1 −̃fnk |
≤ k+1 .
2
X
k+1
∗
Com efeito, basta apenas aplicar
P+∞a definição à sucessão de valores (1/2 )k∈N . Considerese agora a série de funções ( k=1 (fnk+1 −̃fnk )). Esta série converge absolutamente, salvo
talvez sobre um conjunto de medida nula, em consequência da subaditividade-σ vista na
proposição 37. Com efeito,
!p !1/p +∞ Z
1/p
Z
+∞
X
X
p
| fnk+1 −̃fnk | dµ
≤
| fnk+1 −̃fnk | dµ
≤
X
k=1
≤
k=1
+∞
X
k=1
X
1
< +∞ .
2k+1
Pode-se pois concluir que:
+∞
X
k=1
168
| fnk+1 −̃fnk |< +∞ µ − qptp .
(5.1)
Os Espaços de Funções Integráveis
Seja agora a função f definida µ-quase por toda a parte, por:
f = fn1
+∞
X
+
(fnk+1 −̃fnk ) .
k=1
É claro que f ∈ Lp , uma vez que em consequência da majoração obtida na cadeia de
desigualdades 5.1 e, dado que fn1 ∈ Lp , se tem que:
Z
p
1/p
| f | dµ
Z
p
1/p
| fn1 | dµ
≤
X
Z
1/p
p
| fnk+1 −̃fnk | dµ
+
< +∞ .
X
X
Para terminar é suficiente constatar que a sucessão (fn )n∈N∗ converge para a função f .
Com efeito, em resultado de uma aplicação da desigualdade de Minkowski, tem-se que
dado > 0, N ∈ N∗ , dado pela condição de Cauchy e suficientemente grande, de tal
forma que se r, s, K ∈ N∗ se r, s, K ≥ N se tenha que
Z
1/p
p
| fr −̃fs | dµ
X
+∞
X
≤
e também
2
k=K
1
2k+1
≤
.
2
Então para n inteiro tal que n ≥ N , tem-se que, em consequência da propriedade de
subaditividade-σ já referida,
Z
p
1/p
| f −̃fn | dµ
Z
≤
p
1/p
| f −̃fnK | dµ
X
Z
≤
X
!1/p
| fnk+1 −̃fnk |p dµ
X
k=K
1/p
| fnK −̃fn | dµ
+
X
+∞ Z
X
p
+ ≤
2
+∞
X
k=K
≤
1
2k+1
!
+
≤
2
R
o que mostra que limn→+∞ ( X | f −̃fn |p dµ)1/p = 0, tal como se pretendia
5.2.1
O conjunto das funções de quadrado integrável
Um caso notável destes conjuntos Lp de funções integráveis obtem-se quando p = 2. Os
conjuntos L2 podem ser munidos de uma estrutura álgébrica e topológica semelhante
à de um espaço euclideano. Com efeito considere-se a correspondência que a f e g em
L2 (X, A, µ) associa, por definição,
Z
f, g =
f · g dµ ,
X
denominada, abusivamente, forma bilinear. Pela desigualdade de Hölder, uma vez que
com p = q = 2 vem (1/p) + (1/q) = 1 se tem que
Z
Z
| f · g | dµ ≤
X
X
| f |2 dµ
1/2 Z
·
| g |2 dµ
1/2
< +∞ ,
(5.2)
X
169
Capı́tulo 5
em consequência da condição imposta a f e a g, trata-se de uma aplicação bem definida
que goza das propriedades constantes na proposição seguinte.
Proposição 38. Sejam f, fa , g, ga , ∈ L2 (X, A, µ), α, β ∈ R. Então tem-se que:
1. A forma é positiva
f, g ≥ 0
2. A forma é simétrica
f, g = g, f 3. A forma é bilinear
αf +̃βfa , g = α f, g +β fa , g e
f, αg +̃βga = α f, g +β f, ga .
4. A forma é semidefinida positiva
f, f = 0 ⇒ f = 0µ − qptp .
5. Desigualdade de Cauchy-Schwarz
1/2 Z
| f | dµ
·
Z
2
| f, g |≤
X
2
| g | dµ
1/2
.
X
Demonstração. Para demonstrar as primeiras quatro propriedades desta proposição basta
aplicar as propriedades do integral de Lebesgue. A quinta propriedade resulta da desigualdade trivial seguinte:
Z
Z
| f, g |=|
f · g dµ |≤
| f · g | dµ .
X
X
e da aplicação da desigualdade de Hölder tal como foi feito acima na fórmula 5.2.
Observação 42. As propriedades postas em evidência na proposição anterior são em tudo
semelhantes às propriedades dum produto interno. Observe-se que em consequência
des√
tas propriedades tem-se que a correspondência que a f ∈ L2 (X, A, µ) associa f, f tem quase todas as propriedades de uma norma. Justifica-se pois a notação seguinte:
p
|| f ||L2 := f, f =
fórmula nos será útil a seguir.
170
Z
2
| f | dµ
X
1/2
(5.3)
Os Espaços de Funções Integráveis
Continuando com a analogia com os espaços euclideanos temos a seguinte resultado
que tem uma interpretação geométrica sugestiva, tal como é sabido.
Proposição 39 (Igualdade do paralelogramo). Sejam f, g, ∈ L2 (X, A, µ).
Então com as notações da fórmula 5.3 tem-se que:
|| f +̃g ||2L2 + || f −̃g ||2L2 = 2 || f ||2L2 +2 || g ||2L2 .
Demonstração. A demonstração é imediata utilizando as propriedades da forma bilinear
usada para definir a aplicação || · ||L2 . Com efeito tem-se que
|| f +̃g ||2L2 + || f −̃g ||2L2 = f +̃g, f +̃g + f −̃g, f −̃g =
= f, f + g, g +2 f, g +
+ f, f + g, g −2 f, g =
= 2 || f ||2L2 +2 || g ||2L2 ,
com as justificações evidentes.
É possı́vel prosseguir com a analogia entre os espaços euclideanos usuais e o conjunto L2 e demonstrar, para este conjunto, um resultado relativo à existência de uma
projecção ortogonal. A existência em L2 de uma forma bilinear com propriedades semelhantes às de um produto interno de um espaço euclideano usual, permite-nos definir
a noção de funções ortogonais neste conjunto.
Definição 39. Sejam f, g ∈ L2 . Dizemos que f e g são ortogonais em L2 , o
que representamos por f ⊥ g se e só se f, g = 0.
Teorema 22. Seja K ⊂ L2 um conjunto que consideraremos munido das duas operações
- a multiplicação por um escalar e a adição salvo talvez num conjunto de medida nula
- definidas em L2 . Suponhamos que K fechado para a convergência em L2 , isto é,
que toda a sucessão de elementos de K, convergente em L2 , converge para uma função
pertencente a K. Então, verificam-se as propriedades seguintes.
1. Suponhamos que K é um conjunto convexo para as duas operações referidas isto
é que:
∀f, g ∈ K ∀α, β ∈ [0, 1] α + β = 1 ⇒ αf +̃βg ∈ K.
Então, tem-se que:
∀f ∈ L2
∃f ⊥ ∈ K
|| f −̃f ⊥ ||L2 = ∆ := inf{|| f −̃h ||L2 : h ∈ K} .
171
Capı́tulo 5
2. Suponhamos que K é um conjunto estável para as duas operações referidas isto é
que:
∀f, g ∈ K ∀α, β ∈ R αf +̃βg ∈ K .
Então, tem-se que:
∀h ∈ K f −̃f ⊥ ⊥ h .
Se K for um conjunto estável para as duas operações referidas, caso em também é
convexo para essas operações, tem-se ainda que para qualquer g ∈ K, se g verificar a
condição 1 ou, a condição equivalente 2, então g = f ⊥ salvo talvez num conjunto de
medida nula.
Demonstração. A definição de ∆ como ı́nfimo de um conjunto de números reais implica
que existe uma sucessão (fn )n∈KN∗ de elementos de K, tal que se verifica a condição:
limn→+∞ || f −̃fn ||L2 = 0. Por aplicação da igualdade do paralelogramo tem-se para
quaisquer m, n ∈ N∗ que:
1
1
|| f −̃fm ||2L2 + || f −̃fn ||2L2 = 2 || f −̃ (fm +̃fn ) ||2L2 +2 || (fn −̃fm ) ||2L2 .
2
2
Com efeito, basta considerar que, na igualdade do paralelogramo, se pretende que seja:
h+̃g = f −̃fm e h−̃g = f −̃fn ,
de onde resulta, imediatamente que - por adição membro a membro - h = f −̃1/2(fm +̃fn )
e depois - por substituição que - g = 1/2(fn −̃fm ). É fácil verificar, agora, que a sucessão
(fn )n∈KN∗ é uma sucessão de Cauchy em L2 . Com efeito, uma vez que K é convexo
tem-se que 1/2(fm +̃fn ) ∈ K pelo que
1
|| f −̃ (fm +̃fn ) ||2L2 ≥ ∆ .
2
Em consequência, vem que:
|| (fn −̃fm ) ||2L2 ≤|| f −̃fm ||2L2 + || f −̃fn ||2L2 −2∆
(5.4)
o que mostra que limn,m→+∞ || (fn −̃fm ) ||2L2 = 0 uma vez que por construção se tem
limn→+∞ || f −̃fn ||2L2 = 0 e ainda que limm→+∞ || f −̃fm ||2L2 = 0. Como L2 é completo
tem-se que a sucessão (fn )n∈KN∗ converge em L2 para uma função f ⊥ ∈ L2 . Como K
é fechado para a convergência em L2 e os elementos da sucessão convergente (fn )n∈KN∗
pertencem a K, podemos concluir que f ⊥ ∈ K. Observe-se ainda que:
|| f −̃f ⊥ ||L2 = lim || f −̃fm ||L2 = ∆ ,
n→+∞
pelo que fica demostrada a proposição 1 do enunciado. Para demonstrarmos a segunda
proposição, observe-se que vale a primeira proposição uma vez que se K é estável então,
172
Os Espaços de Funções Integráveis
obviamente K é convexo pelo que se tem para qualquer t ∈ R e g ∈ K, tendo em conta
a estimativa dada pela fórmula 5.4 que:
∆2 ≤|| f −̃f ⊥ −̃t · g ||2L2 = ∆2 − 2t f −̃f ⊥ −̃, g +t2 || f −̃f ⊥ −̃t · g ||L2
o que pode ainda escrever-se para g ∈ K fixo e t ∈ R qualquer:
0 ≤ −2t f −̃f ⊥ −̃, g +t2 || f −̃f ⊥ −̃t · g ||L2 .
Tal só pode acontecer se for f −̃f ⊥ −̃, g = 0 isto é se for f −̃f ⊥ ⊥ g tal como
querı́amos demonstrar.
5.3
O espaço de Banach Lp ((X, A, µ))
O conjunto de funções cujos módulos têm potência p ∈ [1, +∞[ integrável assemelha-se,
pelas propriedades que temos vindo a explorar, a um espaço vectorial normado completo de funções. No entanto, essa descrição é abusiva uma vez que como também já
observámos não é possı́vel definir uma adição ponto a ponto de funções em Lp . Além
deste primeiro obstáculo, e ainda que fosse possı́vel definir a adição ponto a ponto de
funções,
a aplicação que a uma dada função f ∈ Lp associa o número real positivo
R
p
( X | f | dµ)1/p verifica apenas
Z
(
p
1/p
| f | dµ)
= 0 ⇒ (f = 0 µ − qptp) .
X
Ora,
para que essa aplicação seja uma norma que, no caso de se verificar
R é necessário
p
1/p
= 0, venha como consequência f ≡ 0 em que 0 é o elemento neutro
( X | f | dµ)
para a adição de funções, isto é, a função identicamente igual a zero. Há forma de
ultrapassar estas dificuldades utilizando como processo a identificação de funções iguais
salvo talvez num conjunto de medida nula. A identificação faz-se considerando classes
de equivalência para uma relação de equivalência adequadamente definida, tal como
veremos seguidamente.
Proposição 40. Seja a relação R definida em Lp por:
∀f, g ∈ Lp f Rg ⇔ f = gµ-qptp .
Então R é uma relação de equivalência sobre Lp .
Demonstração. É uma verificação imediata que apenas tem que ter em conta as propriedades elementares dos conjuntos de medida nula.
173
Capı́tulo 5
Definição 40. Seja Lp ((X, A, µ)), conjunto também representado por Lp sempre que não haja perigo de confusão sobre o espaço de medida subjacente, o
conjunto cociente de Lp pela relação R, isto é:
Lp ((X, A, µ)) := Lp ((X, A, µ))/R = {[f ]R : [f ]R = {g ∈ Lp : f = g µ-qptp }, f ∈ Lp } ,
em que [f ]R representa a classe de f para a relação R.
É agora fácil ver que podemos construir assim um espaço vectorial de classes de
funcções integráveis.
Proposição 41. (Lp ((X, A, µ)), +, ×) é um espaço vectorial sobre R sendo as
operações definidas por:
[f ]R + [g]R = [f +̃g]R e α × [f ]R = [αf ]R
para f, g ∈ Lp e α ∈ R.
Demonstração. Trata-se de verificar que são válidas as propriedades de definição dos
espaços vectoriais recorrendo à definição de [f ]R com sendo o conjunto dado por {g ∈
Lp : f = g µ-qptp } e às propriedades elementares dos conjuntos de medida nula.
Observação 43. É notável que se tenha obtido deste modo um espaço vectorial funcional
em que os elementos não são funções mas sim classes de funções para a igualdade salvo,
talvez, num conjunto de medida nula. Adiante veremos que se trata de um espaço
vectorial normado completo. É comum assumir que as propriedades, para este espaço
vectorial funcional, que veremos a seguir se aplicam ao conjunto de funções Lp que lhe
deu origem, sem que tal seja rigorosamente verdade.
A desigualdade de Minkowski permite agora mostrar que o espaço vectorial Lp pode
ser munido de uma norma.
Proposição 42. Seja a aplicação || · ||p definida em Lp e tomando valores em
[0, +∞[ dada por:
p
∀[f ]R ∈ L
Z
|| [f ]R ||p :=
p
1/p
| f | dµ
,
X
é uma norma sobre Lp .
Demonstração. Observe-se que se [f ]R = 0 então tem-se que f = 0 salvo, talvez, num
conjunto de medida nula pelo que se tem que:
Z
|| [f ]R ||p =
174
X
| f |p dµ
1/p
=0
(5.5)
Os Espaços de Funções Integráveis
Reciprocamente, tem-se que se se verificar a fórmula 5.5 então virá que f = 0 µ − qptp
pelo que [f ]R = 0. Quanto à segunda propriedade das normas que devemos verificar
tem-se trivialmente que:
Z
1/p
Z
1/p
p
p
|| α[f ]R ||p =|| [α · f ]R ||p =
| α · f | dµ
=| α |
| f | dµ
=
X
X
=| α | · || [f ]R ||p
Para a terceira propriedades das normas que temos de verificar, note-se que pela desigualdade de Minkowski se tem que:
Z
1/p
p
|| [f ]R + [g]R ||p =|| [f +̃g]R ||p =
| f +̃g | dµ
≤
X
Z
p
≤
1/p
| f | dµ
Z
1/p
| g | dµ
+
X
p
X
=|| [f ]R ||p + || [g]R ||p ,
pelo que a desigualdade triangular para a norma é trivialmente verificada.
Teorema 23. O espaço vectorial normado (Lp ((X, A, µ)), || · ||p ) é um espaço de Banach.
Demonstração. Seja ([fn ])n∈N∗ uma sucessão de Cauchy de elementos de Lp . Então a
sucessão de representantes das classes de equivaência que formam essa sucessão, (fn )n∈N∗
é uma sucessão de Cauchy em Lp . Pelo resultado que assegura o carácter completo de
Lp , existe f ∈ Lp tal que a sucessão (fn )n∈N∗ converge para f em Lp , isto é tal que:
Z
1/p
p
lim
| fn −̃f | dµ
=0
n→+∞
X
É óbvio que [f ] ∈ Lp e é fácil mostrar que a sucessão ([fn ])n∈N∗ converge para [f ] em
Lp . Com efeito, dado que:
Z
1/p
p
|| [fn ] − [f ] ||p =|| [fn −̃f ] ||p =
| fn −̃f | dµ
X
tem-se que limn→+∞ || [fn ] − [f ] ||p = 0.
Observação 44. De agora em diante, não faremos qualquer distinção quanto à notação
entre as funções elementos de Lp - que representávamos genericamente por f - e as classes
de funções elementos de Lp - que representávamos por [f ]R ou [f ]. Tal foi feito, no que
atrás ficou exposto, apenas com o propósito pedagógico de sublinhar as diferenças e,
sobretudo, as semelhanças entre estes dois conjuntos. Fica claro que sempre que temos
uma função de Lp podemos considerar a classe das funções de Lp que lhe são iguais
salvo, talvez, num conjunto de medida nula e que, sempre que temos um elemento de
Lp , uma classe de funções de Lp iguais entre si salvo talvez num conjunto de medida
nula, podemos considerar um elemento da classe como representante desta.
175
Capı́tulo 5
O espaço com produto interno L2 ((X, A, µ))
5.4
O estudo feito para o conjunto L2 pode transcrever-se agora para o espaço L2 podendo
pois mostrar-se que este espaço é um espaço de Hilbert, isto é, um espaço com produto
interno completo para a métrica associada.
5.5
Conjuntos e funções convexas; a desigualdade de Jensen
Nesta secção vamos detalhar algumas noções relevantes relativas às funções convexas.
Na ausência de diferenciabilidade a hipótese de convexidade permite preservar muitas
das propriedades que são úteis em problemas de optimização. Veremos adiante na subsecção 5.5.2 que as funções convexas de variável real são de grande utilidade dado estarem
na origem de desigualdades importantes como a desigualdade de Jensen. O detalhe com
que apresentamos estas noções deriva de pretendermos que este primeiro contacto seja
muito motivador para estudos futuros.
5.5.1
As funções convexas com domı́nio em Rn
Começamos por estudar as noções de conjunto convexo e de função convexa no caso
multidimensional.
Definição 41. Sejam x, y ∈ Rn . O segmento de extremidades x e y é por
definição:
[| x, y |] := {z ∈ Rn : ∃λ ∈ [0, 1] z = λy + (1 − λ)x} .
Observação 45. Se x, y ∈ R e x ≤ y, então [| x, y |] = [x, y] = {z ∈ R : x ≤ z ≤ y}, isto
é, o segmento é o intervalo de R com as extremidades correspondentes. Com efeito, se
z ∈ [| x, y |] então, uma verificação algébrica imediata mostra que:
z=
y−z
y−x
x+
z−x
y−x
y
com
y−z
y−x
≥0,
z−x
y−x
≥0,
y−z
y−x
+
z−x
y−x
=1
pelo que z ∈ [| x, y |]. Reciprocamente, se z ∈ [| x, y |], então para λ ∈ [0, 1] tem-se que
z = λy + (1 − λ)x = x + λ(y − x) pelo que, como (y − x) ≥ 0, se tem que x ≤ z ≤ y, isto
é, z ∈ [| x, y |].
O resultado seguinte mostra que para representar um segmento qualquer no plano,
é suficiente saber desenhar um intervalo.
176
Os Espaços de Funções Integráveis
Exercı́cio 239. Mostre que todo o segmento em Rn é a imagem por um deslocamento
de um intervalo de R.
1
A definição seguinte descreve uma propriedade geométrica com conteúdo intuitivo
fácil de apreender. O leitor é desde já convidado a representar graficamente e no plano
esta definição.
Definição 42. Um conjunto C ⊆ Rn é convexo se e só se
∀x, y ∈ C [| x, y |] ⊂ C ,
isto é, se dados dois quaisquer pontos de C o segmento que os une está contido
em C.
Apresentamos seguidamente alguns exemplos fundamentais de conjuntos convexos.
Exemplo 11. Dada uma qualquer norma k·k sobre Rn , as bolas aberta e fechada de centro
a (0, r) e B f (0, r)) são conjuntos convexos.
em zero e raio r > 0 (respectivamente Bk·k
k·k
Tal resulta de se verificar, por exemplo para a bola fechada,
kλy + (1 − λ)xk ≤ kλyk + k(1 − λ)xk ≤ λr + (1 − λ)r = r .
Exemplo 12. Sendo C ⊆ Rn um conjunto convexo, então para qualquer a ∈ Rn , tem-se
que C + a := {c + a : c ∈ C} é um convexo, isto é todo o transladado de um convexo
é convexo. A verificação é simples como se pode ver já a seguir. Sejam x, y ∈ C + a,
podendo escrever-se pela definição x = cx +a, y = cy +a com cx , cy ∈ C. Para z ∈ [| x, y |]
tem-se que:
z = λ(cy + a) + (1 − λ)(cx + a) = (λcy + (1 − λ)cx ) + a = cz + a ,
com cz = λcy + (1 − λ)cx ∈ C uma vez que C é convexo.
Exemplo 13. É uma consequência dos dois exemplos anteriores que,para uma qualquer
norma sobre Rn , qualquer bola (aberta ou fechada) é um conjunto convexo. Com efeito,
para uma qualquer bola aberta verifica-se imediatamente que:
a
a
Bk·k
(b, r) = b + Bk·k
(0, r) ,
sendo verdade uma fórmula semelhante para qualquer bola fechada como o leitor verificará facilmente.
Enquanto que a reunião de conjuntos convexos não é, em geral, um conjunto convexo,
veja-se no exemplo 14 o caso da reunião de dois pontos em Rn , o mesmo não acontece
com a intersecção de conjuntos convexos.
1
Relembre que todo o deslocamento no plano se pode representar como a composição de uma transladação, de uma rotação e de uma dilatação.
177
Capı́tulo 5
Proposição 43. Seja (Ci )i∈I uma famı́lia qualquer de conjuntos convexos.
Então
\
Ci
i∈I
é convexo, isto é, toda a intersecção de conjuntos convexos é um conjunto convexo.
Demonstração. É uma simples consequência das definições que se verifica sem qualquer
dificuldade.
Esta proposição tem uma consequência importante. Dado um conjunto arbitrário
existe sempre um convexo que o contêm e que é o menor com essa propriedade.
Definição 43. Dado A ⊆ Rn o envólucro convexo de A, representado por
con(A) é por definição
\
con(A) =
C,
C convexo, A⊂C
isto é, é o menor conjunto convexo que contem A.
Observação 46. Observe-se que Rn ∈ {C convexo, A ⊂ C} 6= ∅, pelo que con(A) está
sempre bem definido. Note-se ainda que, pela proposição anterior, con(A) é um conjunto convexo. Por outro lado, todo o conjunto convexo contendo A tem que conter a
intersecção de todos os convexos que contêm A, pelo que con(A) é o menor conjunto
convexo contendo A.
Exemplo 14. Note-se que se A = {x, y} ⊂ Rn então A não é convexo. Tem-se ainda que
con(A) = [| x, y |]. Com efeito, tem-se obviamente que A = {x, y} ⊂ [| x, y |]. Verifica-se
ainda que [| x, y |] é convexo. De facto, se z1 , z2 ∈ [| x, y |] com z1 = µ1 y + (1 − µ1 )x e
z2 = µ2 y + (1 − µ2 )x, sendo µ1 , µ2 ∈ [0, 1], tem-se que para λ ∈ [0, 1]:
λz2 + (1 − λ)z1 = (λµ1 + (1 − λ)µ2 )y + (λ(1 − µ1 ) + (1 − λ)(1 − µ2 ))x ∈ [| x, y |] ,
uma vez que se α := λµ1 + (1 − λ)µ2 vem que α ∈ [0, 1] e como
(λµ1 + (1 − λ)µ2 ) + (λ(1 − µ1 ) + (1 − λ)(1 − µ2 )) = 1
vem também 1 − α = λ(1 − µ1 ) + (1 − λ)(1 − µ2 ).
Os conjuntos convexos permitem agora definir as funções convexas. O leitor é aconselhado a representar graficamente exemplos das duas noções introduzidas na definição
seguinte.
178
Os Espaços de Funções Integráveis
Definição 44. Seja C ⊆ Rn um convexo e f : C −→ R.
1. O epı́grafo de f é por definição o conjunto
Eπ (f ) := {(x, y) ∈ C × R : f (x) ≤ y} ;
2. A função f é convexa se e só se Eπ (f ) for um conjunto convexo.
Exemplo 15. As funções lineares e as aplicações afins são convexas. Note-se que se uma
aplicação f : Rn −→ R for linear então:
∃a1 , . . . , an ∈ R ∀x = (x1 , . . . , xn ) f (x) = a1 x1 + · · · + an xn .
(5.6)
Com efeito sendo δij o delta de Kronecker dado por
(
1 se i = j
j
δi =
0 se i =
6 j,
se considerarmos (e1 , . . . , en ), a base canónica de Rn em que os vectores da base são
dados por ei = (δi1 , δi2 , . . . , δin ) então vem necessariamente ai = f (ei ). Ora, uma função
com a forma dada pela fórmula 5.6 é necessariamente convexa pela definição 44 uma vez
que:
f (λy + (1 − λ)x) = λf (y) + (1 − λ)f (x) .
O mesmo se passa com uma função afim que é a soma de uma função convexa com um
vector fixo.
O teorema seguinte fornece uma caracterização das funções convexas que muitas
vezes aparece dada como a definição inicial de função convexa.
Teorema 24 (Caracterização fundamental das funções convexas). Seja C ⊆ Rn um
convexo e f : C −→ R. A uma função f é convexa se e só se:
∀x, y ∈ C ∀λ ∈ [0, 1] f (λy + (1 − λ)x) ≤ λf (y) + (1 − λ)f (x) .
Demonstração. Sejam então (x1 , y1 ), (x2 , y2 ) ∈ Eπ (f ) e λ ∈ [0, 1]. Observando que se
tem λ(x1 , y1 ) + (1 − λ)(x2 , y2 ) = (λx1 + (1 − λ)x2 , λy1 + (1 − λ)y2 ), vem por f ser convexa
que
f (λx1 + (1 − λ)x2 ) ≤ λf (x1 ) + (1 − λ)f (x2 ) ≤ λy1 + (1 − λ)y2 ,
pelo que λ(x1 , y1 ) + (1 − λ)(x2 , y2 ) ∈ Eπ (f ) e este conjunto é convexo. Suponhamos
agora que Eπ (f ) é convexo e seja λ ∈ [0, 1]. Como (x1 , f (x1 )), (x2 , f (x2 )) ∈ Eπ (f ) virá
que λ(x1 , f (x1 )) + (1 − λ)(x2 , f (x2 )) ∈ Eπ (f ) pelo que
f (λx2 + (1 − λ)x1 ) ≤ λf (x2 ) + (1 − λ)f (x1 ) ,
tal como se pretendia.
179
Capı́tulo 5
Um primeiro resultado interessante sobre as funções convexas é o que garante que
as funções convexas definidas num conjunto aberto são funções contı́nuas nesse conjunto
aberto.
Teorema 25. Seja C ⊂ Rn um convexo e f : C −→ R uma função convexa. Então f é
contı́nua em C.
Demonstração. Uma demonstração interessante deste resultado pode ser vista em
http://planetmath.org/?op=getobj&from=objects&id=731
Muitos outros resultados interessantes sobre as funções convexas estudadas nesta
secção, as funções de várias variáveis, poderiam ser aqui apresentados. No entanto, dado
o nosso objectivo imediato, que é o de contextualizar a apresentação da desigualdade de
Jensen, vamos passar imediatamente ao estudo das funções convexas definidas em R.
5.5.2
As funções convexas com domı́nio em R
No que vai seguir vamos debruçar-nos com mais detalhe no caso particular das funções
convexas de variável real. Para melhor apreensão das ideias subjacentes à noção de
função convexa atente-se na representação gráfica seguinte2 .
Figura 5.1: O gráfico de uma função convexa de variável real
Considerem-se os pontos do plano definidos por:
Px = (x, f (x)) , Pu = (u, f (u)) , Py = (y, f (y)) ,
2
180
Veja-se o endereço seguinte: http://en.wikipedia.org/wiki/Convex function .
Os Espaços de Funções Integráveis
em que u = tx + (1 − t)y. Designaremos pela justaposição sobrelinhada 3 de dois pontos
a recta que passa por esses dois pontos. Assim, Px Pu designa a recta que passa por Px
e Pu . Pelo teorema 24 a função é convexa se para cada u ∈ [| x, y |] se verificar que Pu
está abaixo de Px Py (confirme-se esta afirmação na figura 5.1).
Uma primeira observação que esclarece o carácter geométrico da noção de convexidade é a que é formulada na proposição seguinte.
Proposição 44. Seja dada uma função f : [a, b] −→ R. Então, para a < x <
u < y < b com u = tx + (1 − t)y, são equivalentes as seguintes afirmações.
1. O ponto Pu está abaixo da recta Px Py (isto é, f é convexa).
2. O declive de Px Py é inferior ou igual ao declive de Pu Py .
3. O declive de Px Pu é inferior ou igual ao declive de Px Py .
Demonstração. Nesta demonstração usaremos a caracterização de função convexa que
nos é dada pelo teorema 24. A recta Px Py pode representar-se por:
f (x) +
f (y) − f (x)
(u − x) ,
y−x
pelo que a primeira afirmação se pode representar como
f (u) ≤ f (x) +
f (y) − f (x)
(u − x) .
y−x
Com as hipóteses feitas sobre x, u, y, esta expressão é obviamente equivalente a
f (u) − f (x)
f (y) − f (x)
≤
,
u−x
y−x
ou seja, verificando-se assim a equivalência entre a primeira e a segunda afirmação. Para
a equivalência entre a primeira e terceira afirmação pode observar-se que a recta Px Py
também se pode representar por:
f (y) +
f (y) − f (x)
(u − y) ,
y−x
pelo que a primeira afirmação se pode representar como
f (u) ≤ f (y) +
f (y) − f (x)
(u − y) .
y−x
Esta expressão é equivalente a
f (y) − f (x)
f (y) − f (u)
≤
,
y−x
y−u
o que representa a terceira afirmação.
3
Usamos a palavra sobrelinhada por oposição a sublinhada.
181
Capı́tulo 5
Esta proposição pode ser reformulada num segundo critério de convexidade para
funções de variável real.
Corolário 9 (Caracterização das funções convexas). Seja dada uma função f : [a, b] −→
R. Então f é convexa se e só se:
∀a < x < u < y < b
f (u) − f (x)
f (y) − f (x)
f (y) − f (u)
≤
≤
.
u−x
y−x
y−u
(5.7)
Uma consequência notável desta caracterização é dada por este outro critério de
convexidade
Teorema 26 (Critério do declive crescente). Seja ∅ =
6 I ⊂ R um intervalo. Uma função
f : I −→ R é convexa se só se para cada x ∈ I a função sx : I \ {x} −→ R denominada
função declive em x, que a u ∈ I \ {x} associa
sx (u) :=
f (u) − f (x)
u−x
for crescente.
Demonstração. Pela fórmula 5.7 tem-se que se
∀u < y sx (u) =
f (y) − f (x)
f (u) − f (x)
≤
= sx (y)
u−x
y−x
se e śo se f for convexa.
A fórmula 5.7 tem ainda uma outra consequência notável que é a existência e a
monotonia das derivadas laterais.
Definição 45. Seja f uma função real de variável real. Diremos que f admite
D f (x) como sua derivada lateral esquerda no ponto x se existir o limite
dado por:
D f (x) = lim
u↑x
f (u) − f (x)
f (u) − f (x)
= lim sx (u) = sup sx (u) = sup
, (5.8)
u↑x
u−x
u−x
u<x
u<x
e dizemos que f admite D+ f (x) como sua derivada lateral direita no ponto
x se existir o limite dado por:
D+ f (x) = lim
u↓x
f (u) − f (x)
f (u) − f (x)
= lim sx (u) = inf sx (u) = inf
. (5.9)
x<u
x<u
u↓x
u−x
u−x
Teorema 27. Seja ∅ =
6 I ⊂ R um intervalo aberto e f : I −→ R uma função convexa.
Então, para cada x ∈ I existem D f (x) e D+ f (x) e verifica-se ainda:
D f (x) ≤ D+ f (x) .
182
Os Espaços de Funções Integráveis
0
0
Demonstração. Com efeito para u, xu ∈ I se se verificar u < x < u virá, pelo critério
0
0
do declive crescente, que sx (u) ≤ sx (u ) pelo que, por exemplo, para qualquer u fixo
0
D f (x) = sup sx (u) ≤ sx (u )
u<x
0
e fazendo em seguida variar u ,
0
D f (x) = sup sx (u) ≤ inf 0 sx (u ) = D+ f (x) .
u<x
x<u
Este teorema tem como consequência imediata o facto de toda a função convexa de
variável real se localmente Lipchitziana no interior do seu domı́nio de definição e, em
consequência, ser uma função contı́nua no interior do seu domı́nio de definição.
Teorema 28. Seja ∅ =
6 I ⊂ R um intervalo e f : I −→ R uma função convexa. Então:
0
0
∀∅ =
6 [a, b] ⊆ I ∃L > 0 ∀x, x ∈ [a, b]
0
| f (x) − f (x ) |≤ L | x − x | .
0
Demonstração. Para a e b como no enunciado e para a ≤ x < x ≤ b tem-se, em
consequência dos teoremas 26 e 27 que:
f (u) − f (a)
f (x) − f (a)
f (a) − f (x)
≤
≤ sup
= D f (x) ≤
a<u
u−a
x−a
a−x
a<x
D+ f (a) = inf
0
0
f (x ) − f (x)
f (u) − f (x)
f (u) − f (x )
≤
≤ sup
=
0
x<u
u−x
x −x
u − x0
u<x0
≤ D+ f (x) = inf
0
0
f (u) − f (x )
f (b) − f (x )
= D f (x ) ≤ D+ f (x ) = inf
≤
≤
0
0
u−x
b − x0
x <u
f (u) − f (b)
≤ sup
= D f (b) .
u−b
u<b
0
(5.10)
0
A cadeia de desigualdades 5.10 mostra que se verifica
0
0
0
0
−[f (x ) − f (x)] ≤ −D+ f (a)[x − x] e f (x ) − f (x) ≤ D f (b)[x − x] ,
pelo que, definindo L := max (−D+ f (a), D f (b)), tem-se o resultado formulado.
Os resultados anteriores permitem ainda formular critérios muito importantes de
convexidade.
Teorema 29. Seja ∅ 6= I ⊂ R um intervalo e f : I −→ R uma função tal que D+ f ou
D f sejam crescentes. Então f é convexa.
183
Capı́tulo 5
0
0
Demonstração. Seja x < t < u < t < y. Observe-se que se tem para u < t < y que:
0
D+ f (t ) = inf
v>t
f (v) − f (t)
f (y) − f (u)
≤
v−t
y−u
Em consequência, tem-se então que:
f (u) − f (x)
f (u) − f (t)
f (u) − f (t)
≤
≤ inf
= D+ f (t) ≤ sup D+ f (t) ≤
t<u
u−x
u−t
u−t
t∈]x,u[
0
f (v) − f (t)
f (y) − f (u)
≤
,
v>t
v−t
y−u
≤ inf D+ f (t) ≤ D+ f (t ) = inf
t∈]u,y[
sendo a desigualdade entre a primeira e a segunda linha resultante da aplicação da
hipótese. Verifica-se assim o critério do declive crescente que é condição suficiente para
que f seja convexa.
Corolário 10. Seja ∅ 6= I ⊂ R um intervalo aberto e f : I −→ R uma função derivável
0
sobre I tal que a derivada f seja crescente sobre I. Então f é convexa.
0
Demonstração. Se f for derivável em I então para qualquer x ∈ I tem-se que f =
D+ f (x) = D f (x) pelo que o corol’ario resulta do teorema 29.
Corolário 11. Seja ∅ 6= I ⊂ R um intervalo aberto e f : I −→ R uma função duas
00
vezes derivável sobre I tal que a segunda derivada f seja não negativa sobre I. Então
f é convexa.
Demonstração. Se f for duas vezes derivável em I com a segunda derivada não negativa
então a primeira derivada é crescente pelo que este corolário resulta do corolário anterior.
O resultado seguinte (veja-se o exerı̧cio 13.34 em [1, p. 202] ou o texto em [3, p. 62])
é da maior importância e admite uma generalização às dimensões superiores baseada nos
◦
teoremas de separação. Dado um intervalo I representaremos por I o interior de I.
Teorema 30. Seja ∅ =
6 I ⊂ R um intervalo e f : I −→ R uma função convexa. Então:
◦
◦
∀u ∈I ∃β ∈ R ∀x ∈I
f (x) ≥ β(x − u) + f (u) ,
isto é, existe um número β tal que a linha recta com declive β que passa pelo ponto
Pu = (u, f (u)) está sempre abaixo do gráfico de f .
Demonstração. Em resultado da caracterização dada pela fórmula 5.7 tem-se para a <
x < u < y < b que
f (u) − f (x)
f (u) − f (x)
f (y) − f (x)
f (y) − f (u)
≤ β = sup
≤
≤
,
u−x
u−x
y−x
y−u
a<x<u
pelo que, por um lado se verifica para a < x < u que −f (x) ≤ β(u − x) − f (u) ou seja
f (x) ≥ β(x − u) + f (u) e por outro para u < y < b que f (y) ≥ β(y − u) + f (u), podendo
pois concluir-se como pretendido.
184
Os Espaços de Funções Integráveis
Este resultado tem como consequência uma outra caracterização importante e muito
útil das funções convexas (veja-se [2, p. 18 e 19]), a saber, que toda a função convexa
num intervalo aberto é o envólucro superior das funções afins que a minoram.
Teorema 31. Seja A o conjunto das funções afins de variável real isto é:
A = {f : R −→ R : ∃a, b ∈ R ∀x ∈ R f (x) = ax + b} .
Seja ∅ =
6 I ⊂ R um intervalo aberto e f : I −→ R uma função. Seja f : I −→ R definida
por:
∀x ∈ I f (x) = sup {h(x) : h ∈ A , h ≤ f } .
Tem-se então que:
1. f minora f , isto é, f ≤ f .
2. f é convexa.
3. f = f se e só se f for convexa.
Demonstração. A primeira propriedade é óbvia pela definição. A segunda propriedade
decorre de se verificar para uma qualquer famı́lia de funções (fi )i∈J que
\
Eπ (sup fi ) =
Eπ (fi )
i∈J
i∈J
e de ser óbvio que toda a função afim é uma função convexa. Pelo que, se as funções
(fi )i∈J forem convexas então supi∈J fi também é uma vez que o seu epı́grafo se representa
como uma intersecção de conjuntos convexos que, já sabemos, é um conjunto convexo
pela proposição 43. Para a terceira propriedade note-se que se se verificar f = f então
pela segunda propriedade f é convexa. Suponha-se então que f é convexa. Para u ∈ I,
seja hu (x) = β(x − u) + f (u) a função afim que minora f e que passa pelo ponto
Pu = (u, f (u)) com declive β, dada pelo teorema 30. Tem-se que
f (u) = hu (u) ≤ sup {h(u) : h ∈ A , h ≤ f } = f (u) ≤ f (u) ,
tal como querı́amos demonstrar.
5.5.3
A desigualdade de Jensen
A desigualdade que apresentamos seguidamente é da maior importância no estudo das
funções integráveis.
Teorema 32. Seja (X, A, µ) um espaço de medida finito, isto é tal que, µ(X) < +∞.
Seja ∅ 6= I ⊂ R um intervalo, φ : I −→ R uma função convexa e f : X −→ R uma
aplicação tal que f (X) ⊆ I, f ∈ L1 e φ ◦ f ∈ L1 . Então:
Z
Z
1
1
φ
f dµ ≤
φ ◦ f dµ .
µ(X) X
µ(X) X
185
Capı́tulo 5
R
◦
Demonstração. Seja u = (1/µ(X)) X f dµ. Então u ∈I . Com efeito, sendo I o fecho
(ou aderência) de I é imediato verificar que u ∈ I uma vez que f (X) ⊂ I. Se u ∈ I \ I
◦
◦
então, uma vez que I é um intervalo tem-se que I=I pelo que u pertence à fronteira
de I. Suponhamos f não constante µ quase certamente sobre I e que I = (a, b), isto
é, que I é o intervalo de extremidades a e b. Suponhamos por um instante que u = a,
por exemplo. Consideremos uma aplicação afim h tal que h(u) = 0 e ainda tal que
h(x) ≥ 0 para x ∈ I. Uns segundos de reflexão, com o auxı́lio a uma representação
gráfica, mostram que uma tal aplicação h existe não sendo necessário recorrer a nenhum
teorema especial sobre os conjuntos convexos. Tem-se então, dado que h é afim, que:
Z
1
h(u) =
h(f ) dµ = 0
µ(X) X
pelo que h(f ) = 0 µ quase certamente sobre I, isto é, f (x) ∈ h−1 ({0}) ∩ I µ quase
certamente sobre I. O que mostra que f (x) = u quase certamente sobre I uma vez que
h−1 ({0}) = u por construção de h. Como suposémos que f era não constante µ quase
certamente sobre I temos uma contradição pelo que u não pode pertencer à fronteira de
I. Seja agora β dado pelo teorema 30 tal que
∀v ∈ I φ(v) ≥ β(v − u) + φ(u) ,
o que pode escrever-se, considerando v = f (x) ∈ I para qualquer x ∈ X,
φ(f (x)) ≥ β(f (x) − u) + φ(u) .
Integrando esta desigualdade tem-se que:
Z
Z
1
β
φ(f ) dµ ≥
(f (x) − u) dµ + φ(u) = φ(u) ,
µ(X) X
µ(X) X
ou seja, exactamente, a desigualdade de Jensen.
5.6
Uma construção dos espaços Lp
Esta secção, que pode ser omitida numa primeira leitura, é motivada pela necessidade
de definir rigorosamente os espaços vectoriais de funções integráveis. No texto principal
considerámos Lp ⊂ [−∞, +∞]R , isto é, considerámos Lp como subconjunto do conjunto
das funções definidas em X tomando valores em R = [−∞, +∞]. Enquanto que RX , o
conjunto das aplicações de X em R pode ser considerado como munido de uma estrutura
natural de espaço vectorial sobre R, tal não acontece com [−∞, +∞]X de modo a que
se possa considerar RX como subespaço vectorial de [−∞, +∞]X . Tal acontece, em
particular, devido às dificuldades em definir o resultado da adição −∞ + ∞. Com efeito,
consideremos a adição usual de funções e a multiplicação usual de uma função por um
escalar definidas seguidamente.
∀f, g ∈ RX ∀α ∈ R (f + g)(x) = f (x) + g(x) , (α · f )(x) = α · f (x) .
186
(5.11)
Os Espaços de Funções Integráveis
Se se pretender prolongar estas operações a [−∞, +∞]X então é fácil ver que não há
forma de definir −∞ + ∞ mantendo os axiomas relativos à estrutura de espaço vectorial.
Com efeito, seja F ∈ [−∞, +∞]X , α ∈ R+ . Então tem-se que de acordo com estes
axiomas,
∀x ∈ X F (x) ∈ R ⇒ (F (x) − α · F (x)) = (1 − α) · F (x) ,
independentemente do valor de α. Suponhamos, agora, que para x0 ∈ X se tem que
F (x0 ) = +∞. Como vale, pelas regras adoptadoas no cálculo integral de Lebesgue, para
a < 0 que a · +∞ = −∞ tem-se que:
(
(1 − α) · F (x) = +∞ se 0 < α < 1
(F (x0 ) − α · F (x0 )) = +∞ − ∞ =
(1 − α) · F (x) = −∞ se 1 < α ,
mostrando que +∞ − ∞ não pode ter um resultado único se valer a convenção a · +∞ =
−∞ para a < 0. Em consequência, o conjunto Lp não pode ser considerado como
subespaço vectorial de um espaço vectorial de funções com as operações usuais definidas
pelo conjunto de fórmulas acima 5.11. Propomos seguidamente uma forma de definir os
conjuntos Lp de forma a que estes possam ser munidos de estruturas naturais de espaços
vectoriais sobre R.
Definição 46. Seja (X, A, µ) um espaço de medida completo. O conjunto
X(µ)
R
é definido por:
R
X(µ)
:= {(f, Xf ) : Xf ∈ A, µ((Xf )c ) = 0, f : Xf → R} .
(5.12)
X(µ)
Os elementos de R
são pares em que o elemento da esquerda é uma função,
tomando valores reais, tendo como domı́nio o elemento da direita do par, elemento este
que é um conjunto de medida plena no espaço de medida (X, A, µ). Podem, agora,
definir-se as operações subjacentes à estrutura de espaço vectorial.
Observação 47. A hipótese do espaço de medida ser completo permitirá considerar AXf ,
a álgebra-σ natural sobre Xf .
187
Capı́tulo 5
Proposição 45. No conjunto R
forma
∀(f, Xf ), (g, Xg ) ∈ R
X(µ)
X(µ)
˜ definidas da seguinte
as operações +̃, ˜· e ×
(f, Xf )+̃(g, Xg ) := (f + g, Xf ∩ Xg ) ,
(5.13)
˜ Xg ) := (f × g, Xf ∩ Xg ) ,
(f, Xf )×(g,
(5.14)
que denominaremos adição,
∀(f, Xf ), (g, Xg ) ∈ R
X(µ)
que denominaremos produto e
∀(f, Xf ) ∈ R
X(µ)
α ∈ R α˜·(f, Xf ) := (α · f, Xf ) ,
(5.15)
a que chamaremos multiplicação por um escalar são respectivamente duas leis de
X(µ)
, +̃,˜·) é um
composição interna e ume lei de composição externa tais que (R
X(µ)
˜ é um grupo multiplicativo.
− {0}, ×)
espaço vectorial sobre R e tais que (R
Demonstração. A demonstração é uma verificação simples que usa o facto que a intersecção de dois conjuntos de medida plena é ainda um conjunto de medida plena.
Estamos em condições de definir os espaçõs de funções integráveis que para distinguir dos conjunstos de funções integráveis definidos anteriormente denotaremos por
Lp♠ (X, A, µ).
Definição 47. Para p ∈ [1, +∞[ tem-se que
(
Z
X(µ)
p
:
L♠ (X, A, µ) := (f, Xf ) ∈ R
)
| f |p dµ < +∞
(5.16)
Xf
De forma equivalente ao que fizémos para os conjuntos de funções integráveis Lp
temos a seguinte proposição:
Proposição 46. Lp♠ (X, A, µ) é um subespaço vectorial de R
X(µ)
.
Demonstração. Sejam (f, Xf ), (g, Xg ) ∈ Lp♠ . A desigualdade (a + b)P ≤ 2p−1 (ap + bp )
mostra-nos que:
Z
Z
| (f, Xf )+̃(g, Xg ) |p dµ =
| f + g |p dµ ≤
Xf ∩Xg
≤2
p−1
Z
p
Z
| f | dµ +
Xf
!
p
| g | dµ
< +∞ .
Xg
Dado que a multiplicação não põe qualquer problema tem-se o resultado enunciado.
188
Os Espaços de Funções Integráveis
Pode agora definir-se uma semi-norma sobre estes espaços de funções integráveis.
Lp♠ .
Proposição 47. A aplicação que a f ∈ Lp♠ associa
|| (f, Xf )
||♠
p=
!1/p
Z
p
| f | dµ
.
Xf
é uma seminorma sobre Lp♠ .
Demonstração. A homogeneidade de grau um é de trivial verificação.
!1/p
Z
|| α˜·(f, Xf ) ||♠
p=
=| α | · || (f, Xf ) ||♠
p .
| f |p dµ
| α |p ·
Xf
Em consequência da desigualdade de Hölder:
|| (f, Xf )+̃(g, Xg )
||♠
p
!1/p
Z
p
=
| f + g | dµ
≤
!1/p
Z
Xf ∩Xg
Z
p
| f | dµ
≤
| g | dµ
+
!1/p
p
| f | dµ
=
=(a)
Xf ∩Xg
Xf ∩Xg
Z
!1/p
p
Z
| g | dµ
+
Xf
!1/p
p
=
Xg
♠
=|| (f, Xf ) ||♠
p + || (g, Xg ) ||p
em que a igualdade em (a) resulta de tanto Xf ∩ Xg como Xf ou Xg serem conjuntos
de medida plena. Por outro lado, se se verificar
!1/p
Z
|| (f, Xf ) ||♠
p=
| f |p dµ
=0,
Xf
tem-se que f = 0 µ-quase por toda a parte em Xf tal como já se tinha visto.
Tal como no caso já estudado anterirmente tem-se que os espaços de funções integráveis são completos.
Teorema 33. O espaço Lp♠ é um espaço seminormado completo.
Demonstração. Seja (fn , Xfn )n∈N∗ uma sucessão de Cauchy em Lp♠ . Seja, por definição
0
X =
+∞
\
Xfn .
n=1
189
Capı́tulo 5
Então (fn IX 0 )n∈N∗ é uma sucessão de Cauchy no conjunto Lp . Dado que este conjunto
é completo seja f ∈ Lp dado como limite em Lp da sucessão (fn IX 0 )n∈N∗ . É fácil ver
0
agora que a sucessão (fn , Xfn )n∈N∗ converge para (f, X ) ∈ Lp♠ . Com efeito, tem-se que:
0
lim || (fn , Xfn ) − (f, X )
n→+∞
||♠
p
!1/p
Z
= lim
n→+∞
p
Xfn ∩X 0
Z
= lim
n→+∞
X0
0
| fn − f | dµ
| fn − f |p dµ
=(a)
1/p
=0
0
em que a igualdade em (a) resulta de tanto Xfn ∩X como X serem conjuntos de medida
plena.
Para concluir esta construção dos espaços de funções integráveis é suficiente mostrar
que se pode obter os espaços de Banach de classes de funções integráveis. Note-se apenas
que vale o seguinte resultado:
Proposição 48. Para cada [f ] ∈ Lp existe (h, Xh ) ∈ Lp♠ tal que
h · IXh ∈ [f ] .
Para cada (h, Xh ) ∈ Lp♠ existe [f ] ∈ Lp tal que
h · IXh ∈ [f ] .
Demonstração. Seja h um representante de [f ], a classe de de f em Lp . Então, tal como
já observámos h é finita µ-quase por toda a parte, isto é, como o espaço de medida é
completo e o conjunto (Xh )c := {x ∈ X : h(x) ∈ {−∞, +∞}} é µ desprezável logo
mensurável, (h, Xh ) ∈ Lp♠ . Seja (h, Xh ) ∈ Lp♠ . É claro que h · IXh está bem definida
sobre X e que h · IXh ∈ Lp , pelo que pondo f := h · IXh se tem o resultado afirmado.
Para o efeito, pode considerar-se a relação R♠ definida em Lp♠ por:
(f, Xf )R♠ (g, Xg ) ⇔ f · IXf = g · IXg µ − qptp em X .
Note-se que, com esta definição, dois elementos de Lp♠ estão em relação se as funções
de cada um deles coincidem sobre um conjunto de medida plena. É imediato verificar
que R♠ é uma relação de equivalência sobre Lp♠ . Podemos definir o conjunto cociente
Lp♠ /R♠ e o resultado natural que temos de verificar é o que está explicitado no seguinte
teorema.
Teorema 34. Com as notações introduzidas anteriormente, tem-se que Lp♠ := Lp♠ /R♠
é isomorfo a Lp .
190
Os Espaços de Funções Integráveis
Demonstração. O resultado enunciado decorre da proposição 48. Com efeito seja [(f, Xf )]♠
um elemento de Lp♠ . Tem-se que:
[(f, Xf )]♠ = (g, Xg ) : f · IXf = g · IXg µ − qptp .
Considere-se φ a aplicação que a [(f, Xf )]♠ associa [f · IXf ] ∈ Lp . Esta aplicação está
bem definida uma vez que se (g, Xg ) ∈ [(f, Xf )]♠ então f · IXf = g · IXg µ qptp pelo
que em Lp se tem [f · IXf ] = [g · IXg ] pelo que as imagens por φ não dependem do
representante da classe. A aplicação φ é obviamente sobrejectiva uma vez que se for
[f ] ∈ Lp , pela proposição, existe (h, Xh ) ∈ Lp♠ tal que h · IXh ∈ [f ]. Em consequência
φ([(h, Xh )]♠ ) = [h · IXh ] = [f ]. Verifiquemos que φ é injectiva. Considerem-se [(f, Xf )]♠
e [(g, Xg )]♠ tais que φ([(f, Xf )]♠ ) = φ([(g, Xg )]♠ ) isto é tais que [f ·IXf ] = [g·IXg ] em Lp .
Então, pela definição de Lp , tem-se que f ·IXf = g ·IXg µ-qptp pelo que (f, Xf )R♠ (g, Xg )
o que quer dizer que [(f, Xf )]♠ = [(g, Xg )]♠ .
5.7
Exercı́cios
Exercı́cio 240. Seja (X, A, µ) um espaço de medida. Considere uma sub-σ-álgebra A1 da σ-álgebra [3]
dada e a medida µ1 obtida como a restrição da medida inicial à sub-σ-álgebra.
1. Mostre que (X, A1 , µ1 ) é um espaço de medida.
2. Seja f uma aplicação Borel-mensurável de (X, A1 ) em R. Mostre que então f é também Borelmensurável de (X, A) em R
3. Suponha que a função f da alı́nea anterior, é integrável relativamente à medida µ1 . Mostre que
então, a função f é integrável relativamente à medida µ e que se tem:
Z
Z
f dµ =
f dµ1
X
X
4. Mostre que se tem ainda:
∀p ∈ [1, +∞] Lp (X, A1 , µ1 ) ⊂ Lp (X, A, µ).
Exercı́cio 241.
[3]
1. Mostre que:
p
∀a, b ∈ R ∀p ≥ 1 |a − b| ≤ 2
p−1
p
p
(|a| + |b| ) .
2. Seja (X, A, µ) um espaço de medida e sejam para p ≥ 1 fixo, f e g dois elementos do espaço
Lp (X, A, µ). Mostre que se tem: |f − g|p ∈ L1 (X, A, µ).
3. Mostre que:
∀a > 0 µ[|f − g| ≥ a] ≤
1
ap
Z
|f − g|p dµ .
X
Indicação :Em 1), para p ≥ 0 a função f (x) = xp é convexa de R∗+ em R∗+ .
Exercı́cio 242. Seja (K, τ ) um espaço topológico compacto seja B, a σ-álgebra de Borel do espaço [3]
topológico em questão e seja µ uma medida sobre (K, B). Considere g uma função contı́nua de (K, τ )
em (R, τu ). Mostre que então g pertence ao espaço L∞ (K, B, µ) e que se tem:
||g||∞ = sup |g(x)|
x∈X
191
Capı́tulo 5
Exercı́cio 243. Seja (X, A, µ) um espaço de medida finito tal que µ(X) = 1. Seja B uma sub-σ-álgebra [3]
de A.
1. Mostre que se pode ter f mensurável de (X, A) em R (relativamente à σ-álgebra de ”Borel”) sem
que f seja mensurável de (X, B) em R (relativamente à σ-álgebra de ”Borel”).
2. Seja f uma função numérica integrável e positiva. Seja νB a medida positiva definida sobre (X, B)
por:
Z
f dµ .
∀B ∈ B νB (B) =
B
Mostre que existe uma função positiva E+ [f | B] que é B-mensurável tal que:
Z
E+ [f | B]dµ
∀B ∈ B νB (B) =
B
3. Seja f ∈ L (X, A, µ). Mostre, considerando as partes positiva e negativa da função f , que existe
uma função E[f | B] que é B-mensurável e que satisfaz a condição seguinte:
Z
Z
∀B ∈ B
f dµ =
E[f | B]dµ
1
B
B
4. Seja M (B) o sub-espaço vectorial do espaço de ”Hilbert”L2 (X, A, µ), constituı́do pelos elementos
deste qu são B-mensuráveis. Mostre que a aplicação que a f ∈ L2 (X, A, µ) associa E[f | B], é um
projector ortogonal de L2 (X, A, µ) sobre M (B).
Bibliografia
[1] Edwin Hewitt and Karl Stromberg. Real and abstract analysis. A modern treatment
of the theory of functions of a real variable. Springer-Verlag, New York, 1965.
[2] Robert R. Phelps. Lectures on Choquet’s theorem. D. Van Nostrand Co., Inc.,
Princeton, N.J.-Toronto, Ont.-London, 1966.
[3] Walter Rudin. Real and complex analysis. McGraw-Hill Book Co., New York, third
edition, 1987.
[4] David Williams. Probability with martingales. Cambridge Mathematical Textbooks.
Cambridge University Press, Cambridge, 1991.
192
Capı́tulo 6
As leis dos grandes números
6.1
Introdução
Neste capı́tulo estudamos algumas formulações da lei dos grandes números. Uma lei
dos grandes números dá o comportamento limite, ou assimptótico, de uma média de observações aleatórias. As leis dos grandes números, de que já vimos exemplos no capı́tulo
dedicado à representação diádica de um número real, podem ser usadas para justiifcar
a interpretação frequentista das probabilidades.
O estudo do comportamento assimptótico necessita, naturalmente, da introdução de
noções de limite. Consoante o limite seja tomado em probablidade ou quase certamente
assim teremos uma lei dos grandes números fraca ou forte, respectivamente.
6.2
A convergência em probabilidade
Relembremos a noção de convergência em probabilidade introduzida anteriormente bem
como algumas das propriedades essenciais desta noção que importa conhecer. Seja
(Ω, A, P) um espaço de probabilidade.
Definição 48. Uma sucessão de variáveis aleatórias (Xn )n∈N tomando valores
em R converge em probabilidade para X se e só se:
∀ > 0
lim P[| Xn − X |> ] = 0 .
n→+∞
Observação 48. Esta noção de convergência diz-nos que escolhido um tamanho para as
vizinhanças em R, dado por , a probabilidade do acontecimento formado pelos pontos
ω ∈ Ω tais que Xn (ω) fica fora do intervalo [X(ω) − , X(ω) + ] tende para zero quando
n tende para infinito. A definição pode estender-se imediatamente a variáveis aleatórias
tomando valores em Rn bastando substituir a distância em R, dada pelo valor absoluto,
pela distância em Rn dada por uma qualquer norma, por exemplo, a norma Euclideana.
193
Capı́tulo 6
Notação 4. Para uma sucessão satisfazendo a definição anterior escreve-se:
pr.
Xn −−−−−→ X .
n→+∞
Naturalmente que há relações entre as diferentes formas de convergência estudadas.
Um primeiro resultado é que se houver convergência em L1 então há convergência
em probabilidade. Relembre que havendo convergência em L1 não tem que haver convergência quase certa. O melhor que se pode afirmar é que é possı́vel extrair uma
subsucesão convergindo quase certamente.
Proposição 49. Seja (Xn )n∈N convergindo em L1 para X. Então, (Xn )n∈N
converge em probabilidade para X, isto é:
pr.
L1
Xn −−−−−→ X ⇒ Xn −−−−−→ X .
n→+∞
n→+∞
Demonstração. A hipótese implica que limn→+∞ E[|Xn − X|] = 0. Pela desigualdade de
Tchebycheff tem-se que, para qualquer > 0
E[|Xn − X| > ] ≤
E[|Xn − X|]
pelo que o resultado anunciado decorre imediatamente.
Um segundo resultado relaciona a convergência em probabilidade com a convergência
quase certa.
Proposição 50. Seja (Xn )n∈N convergindo quase certamente para X. Então,
(Xn )n∈N converge em probabilidade para X, isto é:
q.c.
pr.
Xn −−−−−→ X ⇒ Xn −−−−−→ X .
n→+∞
n→+∞
Demonstração. A hipótese pode ser expressa escrevendo que há convergência pontual
da sucessão de funções mensuráveis (Xn )n∈N para a função mensurável X salvo, talvez,
0
num conjunto de probabilidade nula. Ou seja, tem-se para um dado Ω :
0
0
Ω := ω ∈ Ω : lim inf Xn (ω) = X(ω) = lim sup Xn (ω) ∈ A, P[Ω ] = 1 .
n→+∞
n→+∞
Seja > 0 fixo. Pela definição;
0
∀ω ∈ Ω ∃n ∈ N ∀m ≥ n |Xn (ω) − Xn (ω)| ≤ 194
As leis dos grandes números
o que implica.
0
Ω ⊂
[ \
{|Xm − Xn | ≤ } = lim inf {|Xm − Xn | ≤ } ,
n→+∞
n∈N m≥n
ou passando aos complementares pelas leis de Morgan,
0
lim sup {|Xm − Xn | > } ⊂ (Ω )c .
n→+∞
Em consequência do lema de Fatou inverso (veja-se a página 205) pode afirmar-se que:
0 ≤ lim inf P [{|Xm − Xn | > }] ≤ lim sup P [{|Xm − Xn | > }] ≤
n→+∞
n→+∞
0
≤ P lim sup {|Xm − Xn | > } ≤ P[(Ω )c ] = 0 ,
n→+∞
o que implica que limn→+∞ P[{|Xm − Xn | > }] = 0, tal como pretendı́amos demonstrar.
Observação 49. A recı́proca não é verdadeira, isto é, uma sucessão de variáveis aleatórias
pode ser convergente em probabilidade e não ser convergente quase certamente. Com
efeito, considere-se uma sucessão de variáveis aleatórias independentes (Xn )n∈N verificando:
1
1
∀n ∈ N P[Xn = 1] =
, P[Xn = 0] = 1 − .
n
n
É imediato verificar que a sucessão converge para X ≡ 0 em probabilidade, uma vez
que para > 0 se tem que P[| Xn |> ] = P[Xn = 1] = 1/n. Para verificarmos
que a sucessão indicada não converge quase certamente apliquemos o lema de BorelCantelli (veja-se adiante na página 204). Observe-se que se, para n ∈ N considerarmos o
acontecimento An := {X
= 1} tem-seP
que (An )n∈N é uma sucessão de acontecimentos
Pn+∞
independentes tal que n=1 P[An ] = +∞
n=1 (1/n) = +∞. Por Borel-Cantelli deduz-se
que P[lim supn→+∞ An ] = 1 ou seja:

P
+∞
\

[

Am  = P 
n=1 m≥n
+∞
\

[
{Xm = 1} = 1 .
n=1 m≥n
0
0
0
Quer isto dizer que se pode considerar Ω ∈ A tal que P[Ω\Ω ] = 0 e tal que para ω ∈ Ω ,
se tem ω ∈ ∩+∞
n=1 ∪m≥n {Xm = 1}, ou ainda:
0
∀ω ∈ Ω ∀n ∈ N ∃mn (ω) ≥ n Xmn (ω) = 1 ,
existindo assim uma subsucessão (Xmn (ω))n∈N de (Xn (ω))n∈N que admite 1 como limite. Do mesmo modo, considerando os acontecimentos definidos para cada n ∈ N por
Bn := {Xn = 0} se pode inferir a existência de uma outra subsucessão (Xln (ω))n∈N de
195
Capı́tulo 6
00
00
(Xn (ω))n∈N que admite 0 como limite para ω ∈ Ω ∈ A e tal que P[Ω\Ω ] = 0. Suponhamos que a sucessão de variáveis aleatórias (Xmn )n∈N era convergente P quase certamente.
0
00
Então, para cada ω pertencente a um conjunto de probabilidade plena (no caso Ω ∩ Ω ,
por exemplo) verificar-se-ia que a sucessão (Xmn (ω))n∈N seria uma sucessão numérica
convergente. Mas isso é impossı́vel porque uma sucessão numérica convergente não pode
admitir duas subsucessões numéricas distintas (no caso, (Xmn (ω))n∈N e (Xln (ω))n∈N )
para dois números distintos (no caso, 1 e 0, respectivamente).
As propriedades enunciadas na proposição seguinte são importantes por permitirem
operacionalizar a noção de convergência em probabilidade mas, sobretudo, porque as
respectivas demostrações familiarizam o leitor com o modo de tratar os conjuntos que
aparecem nas questões relacionadas com a convergência em probabilidade.
Proposição 51. Sejam (Xn )n∈N e (Yn )n∈N convergindo em probabilidade para
duas variáveis aleatórias X e Y , respectivamente, variáveis finitas P quase certamente. Seja ϕ : R −→ R uma função contı́nua. Então:
1. A sucessão (Xn + Yn )n∈N converge em probabilidade para X + Y .
2. A sucessão (ϕ(Xn ))n∈N converge em probabilidade para ϕ(X).
3. A sucessão (Xn · Yn )n∈N converge em probabilidade para X · Y .
Demonstração. A primeira propriedade resulta de uma observação simples. Considere-se
0
0
0
0
Ω ∈ A tal que, sobre Ω X e Y são finitas e P[Ω ] = 1. Como, para cada ω ∈ Ω ,
|(Xn (ω) + Yn (ω)) − (X(ω) + Y (ω)| ≤ |Xn (ω) − X(ω)| + |Yn (ω) − Y (ω)| ,
tem-se que para qualquer > 0 que
n
o n
o
0
0
ω ∈ Ω : |Xn (ω) − X(ω)| ≤
∩ ω ∈ Ω : |Yn (ω) − Y (ω)| ≤
⊂
2
2
n
o
0
⊂ ω ∈ Ω : |(Xn (ω) + Yn (ω)) − (X(ω) + Y (ω))| ≤ ,
0
pelo que, pelas leis de Morgan, pela subaditividade da medida e pela condição sobre Ω ,
se tem que:
P [|(Xn + Yn ) − (X + Y )| > ] ≤ P [|Xn − X| > ] + P [|Yn − Y | > ] ,
desigualdade que implica o resultado anunciado. A segunda propriedade é muito importante. Para maior simplicidade da demonstração que vai seguir-se supomos que X
toma valores em R, sendo assim finita P quase certamente. A tı́tulo de exercı́cio, o leitor deverá redigir a demostração no caso geral do enunciado. Formulamos primeiro um
pequeno resultado técnico que mostra que no caso em que X é finita quase certamente,
o conjunto em que X não é limitada tem uma probabilidade arbitrariamente pequena.
196
As leis dos grandes números
Lema 6. Sendo X finita P quase certamente verifica-se que:
∀δ > 0 ∃Aδ > 0 P[|X| > Aδ ] ≤ δ ,
(6.1)
Demonstração. É suficiente considerar os conjuntos Bn := {|X| ≥ n} para n ∈ N.
Verifica-se imediatamente que a sucessão (Bn )n∈N é uma sucessão decrescente de conjuntos mensuráveis pelo que:
lim Bn =
n→+∞
+∞
\
Bn = {|X| = +∞} .
n=1
Em consequência, pela popriedade de continuidade inferior da medida:
0 = P[|X| = +∞] = P lim Bn = lim P [Bn ] = lim P [|X| ≥ n] .
n→+∞
n→+∞
n→+∞
A igualdade entre o primeiro e o último termo desta cadeia de igualdades garante o
resultado enunciado no lema.
Fixe-se δ > 0. Vamos mostrar limn→+∞ P [|ϕ(Xn ) − ϕ(X)| > δ] = 0. Seja agora
> 0 qualquer e A/2 > 0 dado pela fórmula 6.1 do lema acima. Considere-se o intervalo
fechado limitado [−2A/2 , 2A/2 ]. A restrição de ϕ, função contı́nua, a este compacto é
uniformemente contı́nua pelo que:
∃η > 0, η ≤ A/2 ∀x, y ∈ [−2A/2 , 2A/2 ] |x − y| ≤ η ⇒ |ϕ(x) − ϕ(y)| ≤ δ .
Em consequência de se ter,
∀ω ∈ Ω ||Xn (ω)| − |X(ω)|| ≤ |Xn (ω) − X(ω)| ,
vem que para |X(ω)| ≤ A/2 que |Xn (ω)| ≤ |X(ω)| + η ≤ 2A/2 e por isso verifica-se que
{|X| ≤ A/2 } ∩ {|Xn − X| ≤ η} ⊂ {|ϕ(Xn ) − ϕ(X)| ≤ δ} ,
ou seja, pelas leis de Morgan que
{|ϕ(Xn ) − ϕ(X)| > δ} ⊂ {|X| > A/2 } ∪ {|Xn − X| > η} .
Considere-se agora n0 ∈ N tal que para n ≥ n0 se verifica que P[|Xn − X| > η] ≤ /2.
Vem então que para n ≥ n0
P [|ϕ(Xn ) − ϕ(X)| > δ] ≤ P |X| > A/2 + P [|Xn − X| > η] ≤ + = ,
2 2
tal como se pretendia demonstrar. A terceira propriedade resulta de se ter que:
Xn · Yn =
1
(Xn + Yn )2 − Xn2 − Yn2 .
2
e das duas primeiras propriedades demonstradas.
197
Capı́tulo 6
Observação 50. A toda a noção de convergência de sucessões 1 pode fazer-se corresponder
uma noção de fecho e, por isso, uma topologia. Pode mostrar-se que existe uma métrica
d sobre o espaço L das variáveis aleatórias limitadas P quase certamente que torna
metrizável e completa a topologia da convergência em probabilidade, isto é, tal que, (L, d)
é um espaço métrico completo no qual as sucessões de variáveis aleatórias convergentes
são sucessões convergentes em probabilidade. A métrica d pode ser definida por
∀X, Y ∈ L d(X, Y ) = E[|X − Y | ∧ 1] .
Veja-se a refereência [2] sobre este assunto 2 .
6.3
A lei fraca dos grandes números
A lei fraca dos grandes números faz intervir a convergência em probabilidade que é
uma convergência mais fraca que a convergência quase certa uma vez que, de acordo
com a proposição 50 acima, se houver convergência quase certa há convergência em
probabilidade.
Teorema 35 (Lei fraca - convergência em probabilidade). Seja (Xn )n∈N uma
sucessão de variáveis aleatórias iid (independentes
PNe identicamente distribuı́das
2
com X ∈ L . Então se for por definição SN := n=1 Xn tem-se que
1
pr.
SN −−−−−→ E[X]
n→+∞
N
ou o mesmo é dizer que
"
!
#
N
1 X
∀ > 0 lim P Xn − E[X] > = 0 .
n→+∞
N
n=1
Demonstração. Vamos detalhar a demonstração que é uma consequência da desigualdade
de Tchebycheff. Com efeito, note-se primeiramente que:
!
!
N
N
N
1 X
1
1 X
1 X
Xn −
E[X] =
(Xn − E[X]) .
SN − E[X] =
N
N
N
N
n=1
n=1
n=1
Seja agora > 0 qualquer. Pela desigualdade de Tchebycheff tem-se que:

#
" N
!2 
N
X
X
1
1
P SN − E[X] > = P (Xn − E[X]) > N ≤ 2 2 E 
(Xn − E[X])  .
N
N n=1
1
n=1
Satisfazendo um conjunto de propriedades técnicas adequadas.
Pode obter-se em linha no endereço:
http://www.univ-lr.fr/poles/sciences/formations/math/AgregExterneMath/ressources/probabilites/cours.pdf
.
2
198
As leis dos grandes números
Observe-se agora que como pela hipótese de independência se tem cov(Xn , Xm ) = 0
então:



!2 
N
N
X
X
E
(Xn − E[X])  = E 
(Xn − E[X])(Xm − E[X]) =
n=1
=
N
X
n,m=1
E[(Xn − E[X])2 ] + 2
n=1
= N V[X] + 2
N
X
E [(Xn − E[X])(Xm − E[X])] =
n,m=1,n<m
N
X
cov(Xn , Xm ) = N V[X] .
n,m=1,n<m
Em consequência,
1
V[X]
1
−−−−−→ 0 .
P SN − E[X] > ≤ 2 2 · N V[X] =
N
N N 2 N →+∞
tal como querı́amos demonstrar.
6.4
Uma lei forte dos grandes números
Apresentamos seguidamente um enunciado da lei forte dos grandes números sob a
hipótese de existência de momentos de ordem quatro uniformemente limitados para
as variáveis aleatórias da sucessão. O resultado mais geral é o de Kolmogorov que para
sucessões iid exige apenas a existência do primeiro momento.
Teorema 36 (Lei forte - convergência quase certa). Seja (Xn )n∈N uma sucessão
de variáveis aleatórias independentes centradas (E[Xn ] = 0) e tais que:
∃K > 0 ∀n ∈ N E[Xn4 ] ≤ K .
(6.2)
Então, tem-se que:
N
1
1 X
qc.
SN =
Xn −−−−−→ 0 .
n→+∞
N
N
n=1
Demonstração. A demonstração segue as linhas gerais da demosntração efectuada para
a lei forte dos grandes números para sucessões de Bernoulli. Numa primeira observação
aproveita-se o facto das variáveis aleatórias serem independentes e com médias nulas
para obter uma representação simplificada do quarto momento da sucessão das somas
parciais. Tem-se então que:





"N
#
!4 
N
N X
N X
N X
N
N
X
X
X
X
4
E SN
Xk4 +6 E 
Xi Xj 
= E
Xn  = E 
Xi Xj Xk Xl  = E
n=1
i=1 j=1 k=1 l=1
k=1
i,j=1 i<j
199
Capı́tulo 6
dado que, se tem que há 42 = 6 modos de escolher os termos da última soma e para
i, j, k e l distintos se tem que:
E Xi Xj3 = E Xi Xj2 Xk = E [Xi Xj Xk Xl ] = 0 ,
em consequência da independência, de se ter Xj ∈ L4 ⊂ L3 ⊂ L2 ⊂ L1 e do lema da
página 6.6. Observe-se também que pela desigualdade de Jensen se tem para qualquer
j:
2
E Xj2 ≤ E Xj4 ≤ K ,
o que por sua vez, conjuntamente com a independência, também implica que, para
quaisquer i 6= j:
2
E Xi2 Xj2 ≤ E Xi2 E Xj2 ≤ K .
Note-se que há N2 = N (N − 1)/2 formas de escolher os termos da útima soma na
fórmula 6.4, pelo que:
4
E SN
≤ N · K + 3N (N − 1) · K ≤ 3K · N 2 .
Tem-se assim finalmente que:
" +∞ # +∞ +∞
X SN 4
X E S4
X
1
N
E
=
≤ 3K
< +∞ .
4
N
N
N2
N =1
N =1
N =1
P
4
Em consequência, a série +∞
N =1 (SN /N ) converge P quase certamente pelo que o termo
geral converge para zero ou seja pode conscluir-se que, (SN /N ) −−−−−→ 0 tal como se
N →+∞
pretendia.
O corolário seguinte mostra que o que de facto é importante é que as variáveis
aleatórias tenham o mesmo valor médio e não a hipótese de serem centradas.
Corolário 12. Seja (Xn )n∈N uma sucessão de variáveis aleatórias independentes, tais
que
∀n ∈ N E[Xn ] = µ .
verificando a hipótese dada pela fórmula 6.2 do enunciado do teorema. Então tem-se
que:
1
qc.
SN =−−−−−→ µ ,
n→+∞
N
Demonstração. Basta considerar a sucessãom de variáveis aleatórias definidas para cada
n ≥ 1 por Yn = Xn − µ que verifica evidentemente as hipóteses do teoremaa uma vez
que, E[Yn ] = 0 e que pela desigualdade de Minkowski:
E[Yn4 ] = kYn k44 ≤ (kXn k4 + µ)4 ≤ (K + µ)4 ,
podendo assim aplicar-se o resultado anterior.
200
As leis dos grandes números
6.5
Sobre a simulação
Nesta secção apresentamos algumas consequências dos resultados anteriores que interessam aos estudos de simulação aleatória.
Uma questão natural que aparece em consequência das leis dos grandes números é a
determinação da velocidade de convergência das médias amostrais de uma dada variável
aleatória para o valor esperado dessa variável aleatória.
6.5.1
Critério do erro absoluto
Apresentamos primeiramente resultados simples cujas demonstrações recorrem apenas à
desigualdade de Tchebycheff.
Um primeiro caso simples é o das médias amostrais de uma variável aleatória uniforme num intervalo fechado limitado em que se pode mostrar
√ que essa velocidade de
3
convergência, para o critério do erro absoluto é da ordem de N onde N é a dimensão
da amostra.
Como se poderá observar na demonstração, o resultado seguinte subsiste com uma
demonstração semelhante para médias amostrais de uma qualquer variável aleatória
desde que esta seja limitada.
Proposição 52. Seja (Xn )n∈N uma amostra de X variável aleatória com lei
uniforme em [0, 1], isto é, (Xn )n∈N é uma sucessão de variáveis aleatórias independentes e identicamente distribuı́das com X _ U([0, 1]). Então:
#
"
N
1 X
k
∃k > 0 E Xi − E[X] ≤ √ .
(6.3)
N
N
i=1
Demonstração. Considere-se para maior facilidade de leitura que, por definição,
!
N
N
1 X
1 X
WN :=
Xi − E[X] =
(Xi − E[X]) .
N
N
i=1
i=1
Verifica-se imediatamente que E[WN ] = 0 e V[WN ] = V[X]/N . Seja > 0 arbitrário.
Decompondo o integral, temos, pela desigualdade de Tchebycheff que nos garante a
maojoração P[| WN |≥ ] ≤ E[WN2 ]/2 e, uma vez que | WN |≤ 1/2:
Z
Z
E[| WN |] =
| WN | dP +
| WN | dP ≤
{|WN |<}
{|WN |≥}
Z
Z
1
(6.4)
≤
| WN | dP +
dP ≤
2 {|WN |≥}
{|WN |2 <2 }
1 V[X]
1
≤ 2 P[{| WN |2 < 2 }] + P[{| WN |≥ }] ≤ 2 +
.
2
2 N 2
3
Veja-se para uma definição do critério do erro absoluto [, p. 320] (Kloeden).
201
Capı́tulo 6
Uma vez que > 0 é arbitrário, considere-se 2 = η e observe-se que a função
p f (η) = η +
V[X]/(2N η) definida para η ∈]0,p
+∞[ admite um mı́nimo global para η0p= V[X]/(2N
p )
tomando então o valor f (η0 ) = 2V[X]/N . Assim sendo, se for k = 2V[X] = 1/6
tem-se, em consequência da majoração 6.4, que a fórmula 6.5 está verificada.
Uma generalização do resultado anterior dá-nos a velocidade de convergência forte
do método de Monte Carlo para integrais.
Proposição 53 (Método de Monte Carlo). Seja (Xn )n∈N uma amostra de X
variável aleatória com lei uniforme em [0, 1]. Seja φ uma função definida,
mensurável e limitada sobre [0, 1]. Então:
#
"
Z 1
N
1 X
k
∃k > 0 E φ(Xi ) −
φ(x) dx ≤ √ .
(6.5)
N
N
0
i=1
Demonstração. Observe-se primeiramente que pelo resultado de integração relativamente à medida imagem se tem, considerando a medida de Lebesgue λ sobre [0, 1]
que:
Z
φ dλ = E[φ(X)] .
[0,1]
Pode-se pois recomeçar a demostração do resultado anterior considerando
!
N
N
1 X
1 X
ZN :=
φ(Xi ) − E[φ(X)] =
(φ(Xi ) − E[φ(X)]) ,
N
N
i=1
i=1
observando que E[ZN ] = 0 e V[ZN ] = V[φ(X)]/N . Note-se que se for para qualquer
x ∈ [0, 1], φ(x) ≤ K, então tem-se que, | ZN |≤ 2K. Da mesma forma que no resultado
anterior obtem-se que para qualquer η := 2 > 0:
E[| ZN |] ≤ 2 P[{| ZN |2 < 2 }] + 2K
V[φ(X)]
V[φ(X)]
= η + 2K
.
2
N
Nη
Achando o valor mı́nimo do segundo membro, vem finalmente:
p
2 2KV[φ(X)]
√
E[| ZN |] ≤
,
N
tal como pretendido.
6.6
Exercı́cios
Os exercı́cios seguintes exploram variantes simples da leis fortes dos grandes números.
202
As leis dos grandes números
Exercı́cio 244 (Uma lei fraca para variáveis independentes não identicamente distribuı́das). Seja
(Xn )n∈N uma sucessão de variáveis aleatórias independentes (note-se que não necessariamente identicamente distribuı́das) tais que:
∀n ∈ N E[Xn ] = µ ∈ R ; ∃K > 0 ∀n ∈ N V[Xn ] ≤ K .
Mostre que então (SN /N )N ∈N converge em probabilidade para µ.
Exercı́cio 245. Seja uma moeda tal que quando é lançada sai caras, representada por (1), com probabilidade p ∈ [0, 1] e sai coroas, representada por (0), com a probabilidade 1 − p. Seja Xn o resultado de
um qualquer lançamento da moeda ao ar.
1. Mostre que se os lançamentos forem independentes:
"
!
#
N
1 X
∀ > 0 lim P Xn − p > = 0 .
n→+∞
N
n=1
2. Mostre que se os lançamentos forem independentes, com probabilidade um tem-se que:
lim
n→+∞
N
1 X
Xn = p .
N n=1
[3]
Exercı́cio 246. [1] Sejam Xn , n ∈ N, variáveis aleatórias reais e p > 1.
p
P
≤ 1 Pn |Xk |p .
1. Prove que, qualquer que seja n ∈ N, n1 n
k=1 Xk
k=1
n
P+∞
p
1
2. Se k=1 E (|Xk | ) < ∞, prove que P n |X1 + · · · + Xn | > c −→ 0, para todo o c > 0.
3. Suponha agora que as variáveis aleatórias são independentes, de médias nulas e tais que existe
uma constante M > 0 que verifica supk∈N , V ar(Xk ) ≤ M . Prove que, para todos os α > 12 e
c > 0, se tem P n1α |X1 + · · · + Xn | > c −→ 0.
Exercı́cio 247. [1] Para cada n ∈ N fixo, consideremos variáveis aleatórias Xn,1 , . . . , Xn,kn indepen- [3]
dentes e identicamente distribuı́das, onde kn , n ∈ N, é uma sucessão de números naturais não nulos.
Suponhamos que todas as variáveis aleatórias têm médias nulas e que existe uma constante M > 0 tal
que supn,m∈N V ar(Xn,m ) ≤ M . Defina-se Sn = Xn,1 + · · · + Xn,kn . Prove que, dada uma sucessão de
números reais vn , n ∈ N, se kvn2 −→ 0 então, para todo o ε > 0 fixo, P | Svnn | > ε −→ 0.
n
Exercı́cio 248. [1] Sejam Xn , n ∈ N, variáveis aleatórias reais.
P
1. Prove que, se +∞
n=1 P (|Xn | > ε) < +∞ para todo o ε > 0, então Xn −→ 0 quase certamente.
2. Suponhamos que as variáveis aleatórias Xn têm médias nulas, que Cov(Xi , Xj ) = 0 sempre que
i 6= j e que existe uma constante M > 0 tal que supn∈N V ar(Xn ) ≤ M < +∞. Defina-se
Sn = X1 + · · · + Xn .
(a) Prove que
Sn2
n2
−→ 0 quase certamente.
203
[3]
Capı́tulo 6
(b) Defina-se q(n) como o maior quadrado perfeito inferior ou igual a n. Mostre que n − q(n) ≤
√
2 n.
(c) Prove que
(d) Prove que
Sn −Sq(n)
n
Sn
−→ 0
n
−→ 0 quase certamente.
quase certamente.
Exercı́cio 249. Os do fim das folhas de exercı́cios um dos quais, pleo menos, até ficou resolvido.
Apêndice: Resultados complementares
Nesta secção apresentamos alguns resultados importantes que embora sendo essenciais
para uma completa compreensão do texto principal ou já foram estudados antes ou não
tendo sido estudados anteriormente não devem figurar na linha principal de desenvolvimento do texto.
O primeiro resultado é de importância capital nas probabilidades modernas.
Lema 7 (Borel-Cantelli). Seja (An )n∈N uma sucessão de acontecimentos na álgebrasigma A do espaço de probabilidade.
P
P[An ] < +∞ então P[lim supn→+∞ An ] = 1.
1. Se n=+∞
n=1
P
P[An ] = +∞ verifica2. Se os acontecimentos forem independentes então se n=+∞
n=1
se que P[lim supn→+∞ An ] = 0.
Demonstração. A primeira implicação resulta de se ter para qualquer n ∈ N, em consequência da subaditividade que,




\ [
[
X
P[lim sup An ] = P 
Am  ≤ P 
Am  ≤
P[Am ] −−−−−→ 0 ,
n→+∞
n∈N m≥n
m≥n
n→+∞
m≥n
uma vez que o resto de uma série convergente tende para zero. A segunda implicação
é verificada se se mostrar que P[lim inf n→+∞ Acn ] = 0. Ora, sendo p ∈ N qualquer, em
consequência da subaditividade, da independência dos conjuntos, de se ter para x ≥ 0
que 1 − x ≤ e−x que
" n+p
#
X
\
X n+p
Y
X n+p
Y
c
c
c
P[lim inf An ] ≤
P
Am =
P[An ] =
(1 − P[An ]) ≤
n→+∞
n∈N
≤
X
n∈N
m=n
exp(−
n∈N m=n
n+p
X
m=n
n∈N m=n
!
P[An ])
−−−−−→ 0 ,
p→+∞
uma vez que pela hipótese sobre a série se tem que limp→+∞
qualquer n ∈ N.
204
Pn+p
m=n P[An ]
= +∞ para
As leis dos grandes números
O segundo resultado é de grande utilidade na teoria das probabilidades e generaliza
um dos teoremas de convergência, o lema de Fatou.
Lema 8 (Lema de Fatou inverso). Seja (An )n∈N uma sucessão de acontecimentos.
Então:
lim sup P[An ] ≤ P lim sup An .
n→+∞
n→+∞
Demonstração. O lema decorre de duas observações e da continuidade da medida para
sucessões decrescentes. A primeira observação é que a sucssão de conjuntos (∪m≥n An )n∈N
é decrescente e como tal verifica


\ [
[
Am = lim sup An
Am  =
lim 
n→+∞
n∈N m≥n
m≥n
n→+∞
e do mesmo modo a sucessão (supm≥n P[Am ])n∈N é decrescente e por isso se verifica:
sup P[Am ] = inf sup P[Am ] = lim sup P[An ] .
lim
n→+∞
n∈N
m≥n
n→+∞
m≥n
A segunda observação é que pela monotonia das medidas:





[
[
∀n ∈ N ∀m ∈ N P[Am ] ≤ P 
Am  ⇒ ∀n ∈ N sup P[Am ] ≤ P 
Am  .
m≥n
m≥n
m≥n
Em consequência da continuidade da medida para sucessões decrescentes tem-se que:


[
lim sup P[An ] = lim
sup P[Am ] ≤ lim P 
Am  = P lim sup An ,
n→+∞
n→+∞
m≥n
n→+∞
n→+∞
m≥n
tal como querı́amos demonstrar.
Corolário 13. Seja (An )n∈N uma sucessão de acontecimentos na álgebra-sigma A do
espaço de probabilidade. Tem-se então que:
P lim inf An ≤ lim inf P[An ] ≤ lim sup P[An ] ≤ P lim sup An ,
n→+∞
n→+∞
n→+∞
n→+∞
pelo que se limn→+∞ An existe, isto é se, lim inf n→+∞ An = lim supn→+∞ An , então
tem-se que
P lim An = lim P [An ] .
n→+∞
n→+∞
Demonstração. A primeira desigualdade resulta do lema de Fatou para conjuntos, a
segunda das definições de limite superior e de limite inferior de uma sucessão de conjuntos
e a última desigualdade é o lema de Fatou inverso.
205
Capı́tulo 6
O resultado seguinte é uma propriedade que resulta da indepêndencia.
Proposição 54. Sejam X e Y duas variáveis aleatórias integráveis e independentes. Então X · Y ∈ L1 e verifica-se que:
E[X · Y ] = E[X] · E[X] .
(6.6)
Demonstração. Suponhamos inicialmente que X ≥ 0 e Y ≥ 0. Consideremos então
sucessões crescentes de funçõe simples mensuráveis positivas (Xn )n∈N e (Yn )n∈N convergindo quase certamente para X e Y , respectivamente. A existência destas sucessões é
garantida pelo teorema de Lebesgue e pode-se escrever:
Xn :=
n −1
n2
X
k=0
n
n2
−1
X
k
k
1
I
1I k
k+1 + n1I{X≥n} , Yn :=
k+1 + n1I{Y ≥n} .
k
{
≤X<
}
n
2n
2n
2
2n { 2n ≤Y < 2n }
k=0
De forma mais condensada tem-se que para certos conjuntos de inteiros IX e JY :
X
X
Xn =
αkn 1IAnk Yn =
βln 1IBln .
k∈IX
l∈JY
com Ank ∈ σ(X), Bln ∈ σ(Y ), αkn ≥ 0 e βln ≥ 0. A hipótese de independência de X e Y
implica a fórmula 6.6 para as variáveis Xn e Yn . Com efeito tem-se que:



 

X X
X
X
αkn βln 1IAnk 1IBln  =
βln 1IBln  = E 
E[Xn · Yn ] = E 
αkn 1IAnk  · 
=
k∈IX l∈JY
l∈JY
k∈IX
X X
αkn βln E
1IAnk · 1IBln =
=
αkn βln P [Ank
∩
Bln ]
=
=
X X
αkn βln P [Ank ] P [Bln ] =

 
X
1IAnk ∩Bln =
k∈IX l∈JY
k∈IX l∈JY

αkn βln E
k∈IX l∈JY
k∈IX l∈JY
X X
X X
αkn P [Ank ] · 
k∈IX
X
βln P [Bln ] = E[Xn ] · E[Yn ] .
l∈JY
A a fórmula 6.6 para X e Y resulta agora do teorema da convergência monótona de
Lebesgue às sucessões (Xn · Yn )n∈N , (Xn )n∈N e (Yn )n∈N . Com efeito, tem-se que:
E[X·Y ] = lim E[Xn ·Yn ] = lim E[Xn ]·E[Yn ] = lim E[Xn ]· lim E[Yn ] = E[X]·E[Y ] .
n→+∞
n→+∞
n→+∞
n→+∞
Seja agora a decomposição na parte postiva e negativa de X = X + −X − e Y = Y + −Y − ,
variáveis aleatórias integráveis. Tem-se então que
X · Y = (X + − X − ) · (Y + − Y − ) = X + · Y + − X + · Y − − X − · Y + + X − · Y − ,
206
As leis dos grandes números
tendo-se então pelo já demonstrado que:
E[X + · Y + ] = E[X + ] · E[Y + ] < +∞ ,
o mesmo acontecendo para cada um dos restantes três termos da soma acima. Pode-se
pois concluir que X · Y é integrável e que
E[X · Y ] = E[(X + − X − ) · (Y + − Y − )] = E[X + · Y + − X + · Y − − X − · Y + + X − · Y − ] =
= E[X + ] · E[Y + ] − E[X + ] · E[Y − ] − E[X − ] · E[Y + ] + E[X − ] · E[Y − ] =
= E[X + ] · (E[Y + ] − E[Y − ]) − E[X − ] · (E[Y + ] − E[Y − ]) =
= (E[X + ] − E[X − ]) · (E[Y + ] − E[Y − ]) = E[X] · E[Y ] .
uma vez que X e Y são integráveis.
6.7
Resoluções
Resolução:[Exercı́cio 246] Para a primeira alı́nea, aplicamos a desigualdade de Jensen
à função xp para p ∈]1, +∞[ num integral relativo a P, probabilidade uniforme sobre
Nn := {1, . . . , n} que se pode escrever com a medida de contagem dado que P({k}) =
(1/n)µc ({k}) .
n
p Z
n
1 X p Z
1X
p
Xk dP(k) ≤
|Xk | dP(k) =
Xk = |Xk |p .
n
n
Nn
Nn
k=1
k=1
Para a segunda alı́nea, aplica-se a desigualdade de Markov, usa-se a alı́nea anterior
e passa-se ao limite na seguinte fórmula.
#
" n
#
" n
" n
#
+∞
1 X p
1 X 1
1
1X
1 X
p
P
Xk > c ≤ p E Xk ≤ p E
|Xk | ≤
E [|Xk |p ] .
n
n
c
c
n
nc p
k=1
k=1
k=1
k=1
Para a última alı́nea, aplica-se de novo a desigualdade de Markov e usa-se a propriedade da variância da soma ser a soma das variâncias para variáveis aleatórias independentes:
#
"
#
" n
n
X 1 X 1
M
P α
Xk > c ≤ 2α 2 V Xk ≤ 2α−1 2 ,
n n c
n
c
k=1
ficando assim resolvidas as três questões.
k=1
♦
Bibliografia
[1] Paulo Eduardo de Oliveira. Exercı́cios de Teoria das Probabilidades. Coimbra, 1998–
1999.
[2] Frédéric Testard. Calcul Intégral et Probabilités, Cours destiné aux candidats à
lággrégation. Internet, 2006.
207
208
Capı́tulo 7
Independência
7.1
Introdução
A noção de independência torna a teoria das probabilidades uma disciplina distinta da
teoria da medida. Trata-se de um conceito com raı́zes intuitivas muito profundas. Por
outro lado, a formalização matemática deste conceito dá origem a resultados surpreendentes talvez, porque a independência é uma hipótese com grande poder de simplificação.
A hipótese de independência das observações é essencial nos procedimentos estatı́sticos
pois permite a utilização de resultados potentes e com hipóteses muito gerais: leis dos
grandes números e teoremas do limite central.
As alternativas à hipótese de independência não são muitas. A estacionaridade permite, em certos casos, a utilização dos teoremas ergódicos. A propriedade de Markov,
em que o futuro e o passado são independentes dado o presente é talvez o substituto
mais útil da independência.
Convêm sublinhar exemplos e contra-exemplos de observações independentes. De
um modo grosseiro as condições metereológicas diárias no inverno português podem ser
caracterizadas, nos dois casos extremos mais interessantes em, bom tempo (sol, temperatura amena, vento fraco ou moderado) ou mau tempo (chuva ou neve, temperaturas
baixas, vento forte). Temos a ideia de que a probabilidade de que amanhã esteja bom
tempo sabendo que hoje está bom tempo é maior que a probabilidade de que amanhã
esteja mau tempo sabendo que hoje está bom tempo. Do mesmo modo, pensamos que
a probabilidade de que amanhã esteja mau tempo sabendo que hoje está mau tempo é
maior que a probabilidade de que amanhã esteja bom tempo sabendo que hoje está mau
tempo. Assim sendo, as condições metereológicas num dia não são independentes das
condições metereológicas do dia anterior.
Um outro exemplo de observação de variáveis aleatórias que tendemos a não considerar como independentes é a do par em que a primeira coordenada nos dá o cociente
de inteligência do pai e a segunda coordenada o cociente de inteligência do filho. Uns
instantes de reflexão mostram que tanto os factores heriditários como os resultantes da
educação contribuem para esta não independência.
Os exemplos de observações independentes são geralmente tirados dos jogos de azar,
209
Capı́tulo 7
moeda ao ar, dados, cartas, etc uma vez que a independência é assimilada a um dos
factores que contribui para o equilı́brio do jogo.
A exposição que apresentamos neste capı́tulo é baseada em [4] embora possamos
completá-la com outras referências, por exemplo, [3] e [2].
7.2
A noção de independência
Seja (Ω, A, P) um espaço de probabilidade.
Definição 49 (Álgebras - σ independentes). Diremos que as álgebras - σ
G1 , G2 , . . . , Gn , . . . sobre Ω são independentes se e só se:
∀n ∈ N ∀i1 , . . . in distintos P[Gi1 ∩ Gi2 ∩ · · · ∩ Gin ] =
n
Y
P[Gik ] ,
k=1
em que convencionamos que, para qualquer i ∈ N se tem Gi ∈ Gi .
Observação 51. A definição de uma sucessão de famı́lias independentes de subconjuntos
num espaço de probabilidade, não necessariamente álgebras - σ, pode fazer-se exactamente do mesmo modo.
Exercı́cio 250. Sejam G e H duas sub-álgebras - σ de A independentes. Mostre que se A ∈ G ∩ H então
P[A] ∈ {0, 1}.
Definição 50 (Variáveis aleatórias independentes). Diremos que as variáveis
aleatórias X1 , X2 , . . . , Xn , . . . são independentes se e só se as álgebras - σ geradas por essas variáveis aleatórias σ(X1 ), σ(X2 ), . . . , σ(Xn ), . . . o forem.
Observação 52. Dado que para cada i ∈ N se tem que:
σ(Xi ) = Xi−1 (B) : B ∈ B(R)
de acordo com a definição 49. as variáveis aleatórias X1 , X2 , . . . , Xn , . . . são independentes se e só se:
" n
#
n
\
Y
−1
∀n ∈ N ∀i1 , . . . in distintos P
Xik (Bik ) =
P[Xi−1
(Bik )] ,
k
k=1
em que Bik ∈ B(R).
210
k=1
Independência
Definição 51 (Acontecimentos independentes). Diremos que os acontecimentos E1 , E2 , . . . , En , . . . pertencentes à álgebra - σ A são independentes se e só se as álgebras - σ geradas por esses acontecimentos
σ({E1 }), σ({E2 }), . . . , σ({En }), . . . o forem.
Observação 53. Dado que para cada i ∈ N se tem que:
σ({Ei }) = {Ei , Eic , ∅, Ω}
de acordo com a definição 51, os acontecimentos E1 , E2 , . . . , En , . . . são independentes
se e só se:
" n
#
n
\
Y
∀n ∈ N ∀i1 , . . . in distintos P
Fik =
P[Fik ] ,
k=1
em que Fik = Eik ou Fik =
k=1
Eick .
Exercı́cio 251. Mostre que os acontecimentos E1 , E2 , . . . , En , . . . são independentes se e só se as
variáveis aleatórias 1IE1 , 1IE2 , . . . , 1IEn , . . . o forem.
Uma questão natural que se pode colocar é a de verificar a independência. A proposiçãao seguinte é importante para esse efeito.
Proposição 55. Sejam G, H duas sub-ágebras - σ da álgebra A. Sejam I, J
sistemas - π tais que σ(I) = G e σ(J) = H. Então G e H são independentes se
e só se I e J o forem a , isto é, se e só se:
∀I ∈ I, ∀J ∈ J P[I ∩ J] = P[I] · P[J]
a
Aqui usamos a observação 51 para definir sistemas - π independentes.
Uma consequência imediata desta proposição é que é possı́vel recuperar a definição
usual de independência de acontecimentos.
Observação 54. Dado que {Ei } é um sistemas - π que gera σ({Ei }) tem-se que os
acontecimentos E1 , E2 , . . . , En , . . . são independentes se e só se:
" n
#
n
\
Y
∀n ∈ N ∀i1 , . . . in distintos P
Eik =
P[Eik ] ,
k=1
k=1
coincidindo com a definição usual de acontecimentos independentes.
Demonstração. Suponha-se que I e J são independentes. Fixe-se I ∈ I e considere-se
para i = 1, 2 a função de conjuntos µiI definida em H que a H ∈ H associa µ1I (H) =
P[I ∩ H] (respectivamente µ2I (H) = P[I] · P[H]). Verifica-se que µ1I e µ2I são medidas
211
Capı́tulo 7
sobre (Ω, H) com a mesma massa total que coincidem sobre J devido a termos suposto
que I e J são independentes, logo coincidem sobre σ(J) = H.
i definida em G
Fixe-se H ∈ H e considere-se para i = 1, 2 a função de conjuntos νH
1
2
que a G ∈ H associa νH (G) = P[G ∩ H] (respectivamente νH (G) = P[G] · P[H]). Verifica1 e ν 2 são medidas sobre (Ω, G) com a mesma massa total que coincidem sobre
se que νG
G
J, porque como µ1I e µ2I coincidem sobre H tem-se que:
∀I ∈ I ∀H ∈ HP[I ∩ H] = P[I] · P[H] .
1 e ν 2 coincidem sobre σ(I) = G para cada H ∈ H, isto é,
Em consequência, νH
H
∀G ∈ G ∀H ∈ HP[G ∩ H] = P[G] · P[H] ,
ou seja, G e H são independentes. Se G e H forem independentes é óbvio que I e J são
independentes, uma vez que I ⊆ G e J ⊆ H.
Exercı́cio 252. Verifique que para i = 1, 2, se tem que µiI são medidas sobre (Ω, H) tais que µ1I (Ω) =
2
1
i
(Ω).
(Ω) = νH
são medidas sobre (Ω, G) tais que νH
µ2I (Ω). Verifique que para i = 1, 2, se tem que νH
Uma aplicação inesperada da noção de independência é a que decorre do resultado
seguinte.
Proposição 56 (Lemas de Borel-Cantelli). Seja E1 , E2 , . . . , En , . . . uma sucessão de acontecimentos em (Ω, A).
1. Em geral vale:
+∞
X
P[En ] < +∞ ⇒ P[lim sup En ] = 0
n→+∞
n=1
2. Se os acontecimentos forem independentes, então:
+∞
X
P[En ] = +∞ ⇒ P[lim sup En ] = 1
n→+∞
n=1
Demonstração. A demonstração do primeiro lema de Borel-Cantelli é simples. Com
efeito tem-se:




+∞
\ [
[
X
P[lim sup En ] =(a) P 
Em  ≤(b) P 
Em  ≤(c)
P [Em ] −−−−−→ 0
n→+∞
n=1 m≥n
com as seguintes justificações:
212
m≥n
m≥n
n→+∞ (d)
Independência
(a) Pela definição de limite superior de uma sucessão de conjuntos.
(b) Pela monotonia da medida P e dado que para cada n ∈ N se tem que:
+∞
\
[
Em ⊆
n=1 m≥n
[
Em .
m≥n
(c) Pela subaditividade da medida P
P
P
(d) Como por hipótese a série +∞
n=1 P[En ] converge o resto
m≥n P [Em ] tende para
zero.
Para demonstrarmos a segunda proposição do lema de Borel-Cantelli vamos verificar
que, sob a condição enunciada, se tem que
c =0.
P lim sup En
n→+∞
Observe-se que pela independência se tem que para n ≤ m ≤ r


\
Y
Y
c 
c
P
Em
=
P [Em
]=
(1 − P [Em ])
n≤m≤r
n≤m≤r
n≤m≤r
Pela continuidade superior da medida, pode passar-se ao limite quando r tende para
+∞ obtendo-se:


\
Y
c 
P
Em
=
(1 − P [Em ])
n≤m
n≤m
mas dado que para x ≥ 0 se tem 1 − x ≤ e−x1 , vem




\
X
c 
P
Em
≤ exp −
P [Em ] −−−−−→ 0
n≤m
n≤m
n→+∞
uma vez que a série é divergente.
7.3
A lei do 0-1 de Kolmogorov
A lei do 0-1 de Kolmogorov é um dos resultados que por si só justificam o estudo
matemático da noção de independência. Mostra que a independência de uma sucessão
de variáveis aleatórias conjugada com uma condição técnica relativa à posibilidade de
descartar o que se passa num qualquer número finito de termos da sucessão, reduz o
comportamento assimptótico da sucessão ao caso não aleatório.
1
O que se pode ver considerando a série alternada que dá e−x .
213
Capı́tulo 7
Para precisarmos a condição técnica evocada, vamos definir a álgebra - σ de cauda
de uma sucessão de variáveis aleatórias.
Definição 52. Seja X1 , X2 , . . . Xn . . . uma sucessão de variáveis aleatórias.
Seja por definição, para cada n ≥ 1:
Tn = σ(Xn , Xn+1 , . . . ) = σ(σ(Xn ), σ(Xn+1 ), . . . )
A álgebra - σ de cauda da sucessão de variáveis aleatórias X1 , X2 , . . . Xn . . . é
por definição:
∞
\
T=
Tn .
n=1
Exercı́cio 253. Mostre que para uma sucessão de variáveis aleatórias X1 , X2 , . . . Xn . . . se tem, para
cada n ≥ 1:
σ(Xn , Xn+1 , . . . ) = σ(σ(Xn ), σ(Xn+1 ), . . . )
Observação 55. Veja o exercı́cio 259 para exemplos de acontecimentos mensuráveis relativamente à álgebra - σ de cauda.
Teorema 37 (Lei do 0-1 de Kolmogorov). Seja (Xn )n∈N uma sucessão de
variáveis aleatórias independentes. Então a álgebra - σ de cauda T é trivial
relativamente a P, isto é:
1. ∀T ∈ T P[T ] = 0 ∨ P[T ] = 1
2. X ∈ mT ⇒ ∃c ∈ R P[X = c] = 1
Demonstração. A demonstração estrutura-se em seis passos conducentes a mostrar que
a álgebra - σ de cauda é independente dela própria o que permitirá concluir como no
enunciado.
Para cada n ≥ 1
Kn := {{Xi ≤ xi } : 1 ≤ i ≤ n , xi ∈ R ∪ {+∞}}
é um sistema - π e verifica-se que σ(Kn ) =: χn = σ(X1 , . . . Xn ).
Tem-se também que para cada n ≥ 1
Ln := {{Xj ≤ xj } : n + 1 ≤ i ≤ n + r , r ∈ N∗ , xi ∈ R ∪ {+∞}}
é um sistema - π e verifica-se que σ(Ln ) =: Tn = σ(Xn+1 , . . . Xn+r . . . ).
214
Independência
Sendo (Xn )n∈N uma sucessão de variáveis aleatórias independentes, tem-se para cada
n ≥ 1 que Kn e Ln são sistemas - π independentes e logo, pela proposição 56, χn e Tn
são álgebras - σ independentes.
Logo, uma vez que T ⊆ Tn , tem-se que χn e T são álgebras - σ independentes.
Mostremos que χ∞ := σ(X1 , . . . Xn , . . . ) e T são independentes. Teêm-se os factos
seguintes
• χn ⊆ χn+1
• K∞ := ∪+∞
n=1 χn é um sistema - π e σ(K∞ ) = χ∞ .
• K∞ e T são independentes, logo mais uma vez pela proposição 56, σ(K∞ ) = χ∞ e
T são independentes.
Como T ⊆ χ∞ , tem-se que T é independente de T pelo que:
T ∈ T ⇒ P[T ] = P[T ∩ T ] = P[T ] · P[T ]
e necessariamente, P[T ] ∈ {0, 1}.
Seja agora X mensurável relativamente á álgebra - σ de cauda. Tem-se então que
para cada x, P[X ≤ x] ∈ {0, 1}. Seja por definição:
c = sup{x : P[X ≤ x] = 0}
É óbvio que se c = −∞ então P[X = −∞] = 1 e que se c = +∞ então P[X = +∞] = 1.
Suponhamos então c ∈ R. Tem-se que:
" +∞ # X
+∞ [
1
1
P[X < c] = P
X ≤c−
≤
P X ≤c−
=0
n
n
n=1
e ainda
n=1
" +∞ #
\
1
P[X ≤ c] = P
X ≤c+
=1,
n
n=1
e logo, P[X = c] = 1 tal como querı́amos.
Observação 56. No capı́tulo sobre o teorema de Fubini veremos como construir sucessões
de variáveis aleatórias indeendentes com leis dadas.
7.4
Exercı́cios complementares
Exercı́cio 254. Mostre que se as famı́lias E e F forem independentes e se G ⊆ E, então as famı́lias G e
F são independentes.
Exercı́cio 255. [1] Sejam X1 , . . . , Xn variáveis aleatórias independentes todas seguindo distribuição de [1]
Bernoulli de parâmetro p. Determinar a distribuição de Y = X1 + · · · + Xn (diz-se que X segue uma
distribuição binomial de parâmetros (n, p)).
215
Capı́tulo 7
Exercı́cio 256. Seja X um conjunto, F uma σ-álgebra de subconjuntos de X, e µ uma medida de [3]
probabilidade em F. Suponha-se que A1 , A2 , ..., An são conjuntos independentes e pertencentes a F.
1. Mostre que Ac1 , A2 , ..., An são independentes.
2. Seja A um dos seguintes conjuntos A1 ∩A2 , Ac1 ∩Ac2 , A1 ∩Ac2 , ac1 ∩A2 . Mostre que A, A3 , A4 , ..., An
são independentes.
3. Seja Fk a mais pequena sub-algebra de F contendo A1 , ..., Ak . Mostre que se A ∈ Fk então
A, Ak+1 , ..., An são independentes.
4. Seja Fk a mais pequena sub-algebra de F contendo A1 , ..., Ak e Fn−k a mais pequena sub-algebra
0
0
de F contendo Ak+1 , ..., An . Mostre que se A ∈ Fk e A ∈ Fn−k então A e A são independentes.
[2]
Exercı́cio 257.
1. Seja Ai o subconjunto do intervalo ]0, 1] correspondente ao acontecimento ”cara
na i-ésima prova” numa sequência de Bernoulli.Mostre que os Ai são independentes.
2. Seja Bi o subconjunto do intervalo ]0, 1] correspondente ao acontecimento ”cara,coroa,cara na
i, i + 1, i + 2 ésimas provas” numa sequência de Bernoulli.Mostre que B1 , B4 , B7 , B10 , ... são
independentes.
Exercı́cio 258 (Independência e independência dois a dois). Considere dois lançamentos independentes de uma moeda equilibrada ao ar. Como modelo dessa observação seja o espaço de probabilidade
({0, 1}2 , P({0, 1}2 ), P) em que P é a probabilidade uniforme sobre {0, 1}2 . Convencionamos que 1 representa cara e 0 coroa. Considere os acontecimentos:
A1 = {Cara ocorre no primeiro lançamento}
A2 = {Cara ocorre no segundo lançamento}
A3 = {Apenas uma cara ocorre nos lançamentos}.
Mostre que para i = 1, 2, 3 se tem P[Ai ] = 1/2, que para i 6= j P[Ai ∩ Aj ] = 1/4 = P[Ai ] · P[Aj ] e que,
finalmente, P[A1 ∩ A2 ∩ A3 ] = 0. Conclua que os acontecimentos são dois a dois independentes mas que
{A1 , A2 , A3 } não é uma famı́lia de acontecimentos independentes.
Exercı́cio 259. Seja (Xn )n∈N uma sucessão de variáveis aleatórias. Mostre que seguintes conjuntos são
mensuráveis relativamente à álgebra - σ de cauda.
1. {ω ∈ Ω : (Xn (ω))n∈N converge }
P
2. {ω ∈ Ω : n≥1 Xn (ω) converge }
P
3. {ω ∈ Ω : (1/k) kn=1 Xn (ω) converge }
Exercı́cio 260. Seja o espaço de probabilidade (R, B(R), PN ) em que B(R) é a álgebra-σ de Borel e
∀B ∈ B(R) PN (B) =
x2
Z
B
e− 2
√ dx .
2π
Considere a sucessão de variáveis aleatórias (Xn )n≥1 em que para n ≥ 1 e x ∈ R se tem Xn (x) = xn /n.
P
1. Mostre que a série ( n≥1 Xn ) converge em A = [−1, 1[ e diverge em Ac .
2. Mostre que PN (A) > 0 e PN (Ac ) > 0 e conclua que (Xn )n≥1 não é uma sucessão de variáveis
aleatórias independentes no espaço de probabilidade (R, B(R), PN ).
216
Independência
Exercı́cio 261. Seja o espaço de Lebesgue. (]0, 1], L(]0, 1]), λ) e para cada x ∈]0, 1], (αn (x))n∈N o
desenvolvimento binário não degenerado de x, isto é, uma sucessão de números de {0, 1} tal que:
1. ∀n ≥ 1∃m ≥ n αn (x) 6= 0
P+∞
n+1
= x.
2.
n=1 αn (x)/2
Mostre que (αn (x))n∈N é uma sucessão de variáveis aleatórias independentes definidas no espaço de
probabilidade (]0, 1], L(]0, 1]), λ).
∗
Exercı́cio 262. Com as notações do exercı́cio anterior mostre que para qualquer sucessão (cn )n≥1 ∈ RN


X
P
(−1)αn cn converge  ∈ {0, 1}
n≥1
Exercı́cio 263. [1] Consideremos Xn , n ≥ 1, variáveis aleatórias independentes e todas com função [3]
de distribuição F e T1 , T2 variáveis aleatórias com valores nos naturais e independentes da sucessão
Xn , n ≥ 1. Definam-se as variáveis aleatórias Y1 (ω) = XT1 (ω) (ω) e Y2 (ω) = XT2 (ω) (ω).
1. Mostre que se F (min(s, t)) = F (s)F (t) para todos os s, t ∈ R, então a distribuição das variáveis
aleatórias Xn , n ≥ 1, é uma medida de Dirac.
2. Mostre que a função de distribuição de Y1 é F e que a função de distribuição de Y2 é também F .
3. Caracterize a distribuição do vector aleatório (Y1 , Y2 ).
4. Suponha que a distribuição comum às variáveis aleatórias Xn , n ≥ 1, não é uma medida de Dirac.
Mostre que Y1 é independente de Y2 se e só se P (T1 = T2 ) = 0.
Bibliografia
[1] Paulo Eduardo de Oliveira. Exercı́cios de Teoria das Probabilidades. Coimbra, 1998–
1999.
[2] Karl R. Stromberg. Probability for analysts. Chapman & Hall Probability Series.
Chapman & Hall, New York, 1994. Lecture notes prepared by Kuppusamy Ravindran.
[3] Daniel W. Stroock. Probability theory, an analytic view. Cambridge University Press,
Cambridge, 1993.
[4] David Williams. Probability with martingales. Cambridge Mathematical Textbooks.
Cambridge University Press, Cambridge, 1991.
217
218
Capı́tulo 8
A Esperança Condicional
Nota prévia
Estas notas complementam e esclarecem o capı́tulo segundo do livro de texto que adoptámos [1]. A matéria é de importância fundamental pelo que se justifica esta duplicação
de pontos de vista sobre o mesmo assunto. Desenvolvemos estas notas, relativas à noção
de esperança condicional, a partir da exposição magistral de David Williams na obra [7],
completando-a com alguns desenvolvimentos que serão convenientemente referenciados
por altura da respectiva apresentação.
8.1
A informação disponı́vel ou conhecida
No modelo de Kolmogorov para as probabilidades, quando se considera um espaço de
probabilidades (Ω, F, P), temos como interpretação que Ω representa o conjunto das
realizações do fenómeno em estudo, F ⊂ P(Ω) representa a famı́lia dos acontecimentos
possı́veis no modelo e, finalmente, P a medida de probabilidade sobre (Ω, F) que é a
aplicação que a cada acontecimento A ∈ F associa P[A] ∈ [0, 1], representa o grau de
confiança que temos na realização do acontecimento A.
Para representar a evolução da informação disponı́vel ao longo do tempo é, por
vezes, útil considerar que os acontecimentos conhecidos de F são os elementos de G
uma subalgebra-σ de F que relembramos, é a álgebra-σ dos acontecimentos possı́veis no
modelo.
Note-se que um acontecimento A ∈ F é conhecido se, dado ω ∈ Ω, isto é, uma realização do fenómeno, é sempre possı́vel decidir qual das duas proposições contraditórias
ω ∈ A, ou ω ∈
/A,
é verdadeira.
Observe-se também que, sendo os acontecimentos conhecidos representados por G
subalgebra-σ de F, uma variável aleatória X, isto é, uma função de Ω em R mensurável
de (Ω, F) em (R, B(R)), é conhecida, face à informação disponı́vel dada por G, se e só
219
Capı́tulo 8
se X for mensurável de (Ω, G) em (R, B(R)). Com efeito, se X for conhecida, então são
conhecidos os conjuntos X −1 (B) para B ∈ B(R), isto é, para cada B ∈ B(R) é possı́vel
afirmar que X −1 (B) ∈ G. Tal equivale a afirmar que:
σ(X) = {X −1 (B) : B ∈ B(R)} ⊆ G .
Seja então (Ω, F, P) um espaço de probabilidade e X uma variável aleatória sobre
Ω, tomando valores reais. Sendo X de quadrado integrável é sabido que E[X] se pode
interpretar como a melhor estimativa de X, no sentido dos mı́nimos quadrados, independentemente de qualquer informação disponı́vel. Representemos por
E[X | G]
a melhor aproximação de X no sentido dos mı́nimos quadrados, dada a informação
disponı́vel representada por G. Com esta convenção ter-se-á
E[X] = E[X | {∅, Ω}] ,
uma vez que G = {∅, Ω}, subalgebra-σ trivial de F que é constituı́da pelo acontecimento impossı́vel (∅) e pelo acontecimento certo (Ω), representa a informação mı́nima
disponı́vel. Observe-se ainda que, tal como foi dito acima, E[X] é a melhor aproximação
de X no sentido dos mı́nimos quadrados, ou ainda:
|| X − E[X] ||L2 = inf{|| X − Y ||L2 : Y ∈ M(Ω, {∅, Ω})} .
Neste capı́tulo, vamos tentar responder à questão seguinte: qual a melhor estimativa de
X, no sentido dos mı́nimos quadrados, baseada na informação conhecida, representada
por G?
8.2
O teorema de Radon-Nikodym
Seja (X, A) um espaço mensurável e µ e σ duas medidas sobre este espaço.
Definição 53. A medida σ é absolutamente contı́nua relativamente a µ se
e só se:
∀A ∈ A µ(A) = 0 ⇒ ν(A) = 0 .
Nestas condições escreve-se: σ µ.
A proposição seguinte fornece exemplos muito importantes de medidas absolutamente
contı́nuas.
220
A Esperança Condicional
Proposição 57. Seja (Ω, F, P)R um espaço de probabilidade e f uma função
mensurável positiva verificando Ω f · dP = 1. Seja Qf definida por
Z
f dP .
(8.1)
∀A ∈ F Qf [A] =
A
Então Qf é uma medida de probabilidade sobre (Ω, F), absolutamente contı́nua
relativamente a P e tem-se que
Z
Z
g · f dP .
g dQf =
∀g ∈ L1 (Ω, F, P)
Ω
Ω
Definição 54. A qualquer função h, igual a f quase certamente, verificando a
fórmula 8.1, chamamos a densidade de Radon-Nikodym de Qf relativamente
a P e escreve-se:
dQf
dQf = f · P ou também f =
.
dP
Demonstração. A demonstração de que Qf é uma medida faz-se sem qualquer dificuldade
usando o teorema da convergência monótona de Lebesgue.
Exemplo 16. Seja X ∈ N(0, 1), isto é, admitindo uma distribuição normal estandardizada. Tem-se então por definição que:
Z x
t2
1
√ e− 2 dt
FX (x) = P[X ≤ x] =
2π
−∞
√
Sendo fX (x) = 1/ 2π exp(−t2 /2) uma função contı́nua tem-se que a densidade de X
relativamente à medida de Lebesgue é dada por
x2
1
0
FX (x) = √ e− 2 .
2π
É notável que o exemplo dado pela proposição 57 constitua, em condições muito
gerais, o caso tı́pico de medidas absolutamente contı́nuas. Tal é afirmado pelo teorema
de Radon-Nikodym, teorema que usaremos adiante repetidamente.
Teorema 38. Sejam µ e ν duas medidas finitas sobre um espaço mensurável
(X, A). Suponha-se que ν µ. Existe então f ∈ L1+ (X, A, µ) tal que:
Z
∀A ∈ A ν(A) =
f dµ .
A
221
Capı́tulo 8
Demonstração. Considere a medida finita λ := ν + µ. Para qualquer g ∈ L2 (λ) tem-se
em consequência da desigualdade de Cauchy-Schwarz que:
X
| g | dλ ≤
| g | dν ≤
g dν |≤
|
Z
Z
Z
Z
1/2
| g | dλ
X
X
2
·
p
λ(X) < +∞ .
X
R
Pode assim constatar-se que a aplicação φ que a g ∈ L2 (λ) associa X g dν é uma forma
linear limitada sobre L2 (λ), pelo que em consequência dos resultados usuais sobre os
espaços de Hilbert existe h ∈ L2 (λ) tal que:
Z
Z
2
g · h dλ .
g dν =< g, h >L2 (λ) :=
∀g ∈ L (λ) φ(g) :=
X
X
Podemos afirmar que h ≤ 1. Com efeito, considerando para qualquer E mensurável com
λ(E) > 0, a função g = IE na última fórmula, tem-se que:
Z
1
ν(E)
h dλ =
≤1.
λ(E) E
λ(E)
Dado que E é arbitrário com medida positiva vem que necessariamente h ≤ 1. Atendendo
a que λ = ν + µ tem-se então em consequência da penúltima fórmula que:
Z
Z
∀g ∈ L2 (λ)
(1 − h)g dν =
g · h dµ .
(8.2)
X
X
Sendo por definição:
A = {x ∈ X : 0 ≤ f (x) < 1} e B = {x ∈ X : f (x) = 1}
vem em consequência da fórmula 8.2 e considerando g = IB , que:
Z
Z
µ(B) =
h dµ =
(1 − h)dν = 0 .
B
B
Seja agora um conjunto mensurável F qualquer. Observando que sobre o conjunto de
medida µ nula B (e por se ter ν µ de medida ν nula) se tem, em consequência de 8.2
que,
Z
Z
(1 − hn+1 ) dν =
F
h (1 + h + · · · + hn )dµ
F
então, uma vez que sobre B, limn→+∞
função mensurável e integrável, vem:
hn
= 0 e que limn→+∞ h (1 + h + · · · + hn ) =: f
Z
ν(F ) =
f dµ ,
F
tal como querı́amos. Para uma demonstração deste teorema numa formulação muito
geral veja-se por exemplo [6][p. 122].
222
A Esperança Condicional
8.3
Definição e Propriedades
Nesta secção começamos por introduzir a noção de esperança condicional nos espaços
discretos estabelecendo uma ligação com o conceito de probabilidade condicional neste
contexto, noção já anteriormente estudada. Depois de abstrairmos as principais propriedades da noção de esperança condicional desenvolvemos a apresentação do teorema
de Kolmogorov que lida com a existência e unicidade no caso geral. Por último detalhamos algumas propriedades que são de grande utilidade para o cálculo efectivo. Esta
exposição é complementada com um conjunto de exercı́cios práticos que contribuirão se
forem resolvidos conscienciosamente para uma adequada compreensão deste assunto.
8.3.1
Motivação: o caso finito
Seja (Ω, F, P) um espaço de probabilidade e X e Z duas variáveis aleatórias tomando
um número finito de valores i.e.:
X(Ω) = {x1 , . . . xm } e Z(Ω) = {z1 , . . . zn } .
É conhecida a definição seguinte da probabilidade condicional de um acontecimento dado
um outro acontecimento cuja probabilidade seja não nula.
P[X = xi | Z = zj ] :=
P[{X = xi } ∩ {Z = zj }]
.
P[Z = zj ]
Dado que esta probabilidade condicional pode interpretar-se como uma nova probabilidade definida sobre o novo espaço {Z = zi }, é natural definir o valor esperado da variável
aleatória X, relativamente a esta probabilidade.
E[X | Z = zj ] :=
m
X
xi P[X = xi | Z = zj ] .
i=1
Para tornar a expressão anterior independente do ponto zi , pode agora definir-se uma
nova variável aleatória do seguinte modo.
Definição 55. A variável aleatória E[X | Z] é definida por:
E[X | Z] :=
n
X
E[X | Z = zj ] I{Z=zj } .
(8.3)
j=1
Note-se que esta definição diz-nos que se para um ω ∈ Ω fixo, se tem Z(ω) = zj para
um certo j, então verifica-se que:
E[X | Z](ω) := E[X | Z = zj ] .
A variável aleatória assim definida goza de algumas propriedades que passamos a observar detalhadamente.
223
Capı́tulo 8
• Considere-se a álgebra-σ G gerada pela variável aleatória Z. Uma consequência de
um exercı́cio das aulas práticas é que:
[
G = σ(Z) = { {Z = zj } : J ⊆ {1, . . . , n}} .
(8.4)
j∈J
pelo que se torna claro que a expressão (8.3) define uma variável aleatória G mensurável.
• O cálculo formal seguinte, que seria possı́vel justificar desde que as hipóteses
adequadas se encontrassem explicitadas, mostra-nos uma outra propriedade importante verificada pela variável aleatória que definimos acima. Para qualquer
l ∈ {1, . . . , }:
Z
E[X | Z]dP = E[X | Z = zl ] · P[Z = zl ] =
{Z=zl }
=
=
m
X
i=1
m
X
xi
P[{X = xi } ∩ {Z = zl }]
· P[Z = zl ] =
P[Z = zl ]
xi P[{X = xi } ∩ {Z = zl }] =
i=1
Z
=
XdP .
{Z=zl }
Devido à forma geral dos elementos de G, que é possı́vel observar na fórmula (8.4)
e, devido à aditividade do integral relativamente a uma partição do domı́nio de
integração, temos finalmente que:
Z
Z
∀G ∈ G
E[X | Z]dP =
XdP .
(8.5)
G
G
• Uma outra propriedade da variável aleatória E[X | Z] tem uma interpretação
geométrica à qual será dada esclarecimento complementar na demonstração do
teorema de Kolmogorov. Consideremos L2 (Ω, F, P) o espaço das variáveis aleatórias de quadrado integrável, isto é:
Z
2
2
L (Ω, F, P) := X : Ω 7→ [−∞, +∞] :
X dP < +∞ .
Ω
Este espaço pode ser munido de uma forma (bilinear) semi-definida positiva, um
produto interno, do modo seguinte.
Z
2
∀X, Y ∈ L (Ω, F, P) hX, Y i :=
X × Y dP .
Ω
Para este produto interno pode definir-se a noção de ortogonalidade semelhante à
noção de ortogonalidade no espaço euclidiano usual, isto é:
∀X, Y ∈ L2 (Ω, F, P) X ⊥ Y ⇔ hX, Y i = 0
224
A Esperança Condicional
Pode verificar-se que, para G subalgebra-σ de F, se tem que L2 (Ω, G, P) é um
subespaço fechado de L2 (Ω, F, P) e que E[X | Z] é a projecção ortogonal de X ∈
L2 (Ω, F, P) sobre o subespaço L2 (Ω, G, P). Sabe-se, veja-se [7][p. 67], que Z é a
projecção ortogonal de X ∈ L2 (Ω, F, P) sobre L2 (Ω, G, P) se e só se:
∀Y ∈ L2 (Ω, G, P) hX − Z, Y i = 0 .
Ora, a segunda propriedade que analisámos acima e que está condensada na
fórmula 8.5, permite-nos dizer que:
∀G ∈ G hX − E[X | Z], IG i = 0 .
Mas, pela linearidade e pela densidade das funções simples em L2 (Ω, G, P) pode
deduzir-se que:
∀Y ∈ L2 (Ω, G, P) hX − E[X | Z], Y i = 0 ,
pelo que a conclusão anunciada segue.
8.3.2
O caso geral
As duas primeiras propriedades que pudemos observar, acima, no caso de duas variáveis
aleatórias tomando um número finito de valores foram usadas por Kolmogorov1 para definir em condições muito gerais a noção de esperança condicional. A terceira propriedade
sugere a demonstração do resultado fazendo apelo à estrutura geométrica de espaço de
Hilbert das variáveis aleatórias de quadrado integrável.
Teorema 39 (Kolmogorov 1933). Seja (Ω, F, P) um espaço de probabilidade,
X uma variável aleatória integrável e G uma subalgebra-σ de F. Então:
1. Existe então uma variável aleatória Y tal que:
(a) Y é integrável,
(b) Y é mensurável relativamente a G,
(c) Y verifica a seguinte propriedade:
Z
Z
∀G ∈ G
Y dP =
G
XdP .
(8.6)
G
2. Se Y e Ỹ forem duas variáveis aleatórias verificando as três propriedades
da alı́nea 1 acima, então
Y = Ỹ P q.c. .
1
Andrei Nikolaevich Kolmogorov, matemático russo (1903–1987) fundamentou a teoria das probabilidades axiomatizando-a no quadro da teoria da medida. Segundo Hoffman-Jorgensen [3][vol. I, p.
xxxvi] a obra datada de 1933, onde expõe esta fundamentação, foi recebida pelos probabilistas seus contemporâneos quase com euforia. Esta obra foi posteriormente traduzida para a lı́ngua inglesa (veja-se
[4]).
225
Capı́tulo 8
Em consequência do ponto 1 do teorema de Kolmogorov podemos definir a noção de
esperança condicional no caso geral.
Definição 56. Nas condições do teorema de Kolmogorov acima qualquer
variável aleatória que verifique as propriedades do ponto 1 denomina-se uma
versão da esperança condicional de X dada G e representa-se por:
E[X | G] .
Demonstração. A prova da existência para uma dada variável aleatória X em L2 decorre
do teorema que garante a melhor aproximação para ”subespaços”fechados de L2 . Com
efeito, como se verifica que G ⊂ F temos L2 (Ω, G, P) ⊂ L2 (Ω, F, P) podendo mesmo
afirmar que, com as reservas feitas quanto à definição da adição nos espaços de funções
integráveis, L2 (Ω, G, P) é um subespaço vectorial de L2 (Ω, F, P). Em consequência, para
qualquer elemento X de L2 (Ω, F, P) existe Y no espaço L2 (Ω, G, P) tal que:
∀Z ∈ L2 (Ω, G, P) X − Y ⊥ Z .
Como para G ∈ G se tem que IG ∈ L2 (Ω, G, P) temos:
∀G ∈ G
< X − Y, IG >= 0
e como as variáveis X and Y são integráveis temos, de forma equivalente,
Z
Z
XdP =
Y dP .
∀G ∈ G
G
G
Observando que, por ser Y ∈ L2 (Ω, G, P) se tem que Y é G mensurável e é integrável
temos, finalmente, que Y é uma versão da esperança condicional de X dada G. Para a
demonstração do caso geral e da unicidade serão úteis os dois resultados seguintes.
Lema 9. Seja U mensurável positiva e limitada. Então como U ∈ L2 (Ω, G), tem-se que:
E[U | G] ≥ 0.
Demonstração. Seja W uma versão de E[U | G] e suponhamos que P[W < 0] > 0. Como
se verifica facilmente, pelo método usual, que:
{W < 0} =
+∞
[
1
{W < − } ,
n
n=1
em que a famı́lia de acontecimentos ({W < − n1 })n≥1 é crescente, tem-se que
1
0 < P[W < 0] lim ↑ P[W < − ] .
n→+∞
n
Em consequência, existe necessariamente n0 ≥ 1 tal que P[W < − n10 ] > 0 (caso contrário
ter-se-ia P[W < 0] = 0). Dado que U sendo limitada é de quadrado integrável relativamente à medida de probabilidade, o que já se verificou acima para estas variáveis
226
A Esperança Condicional
aleatórias permite-nos concluir usando a condição 8.6 que se verifica a seguinte contradição
Z
Z
1
1
W dP ≤ −
U dP =
P[W < − ] < 0
0≤
n
n
0
0
{W <−1/n0 }
{W <−1/n0 }
uma vez que W ∈ mG e assim sendo {W < −1/n0 } ∈ G.
Este segundo resultado é uma antecipação do resultado de adição das esperanças
condicionais.
Lema 10. Sejam Z1 , Z2 ∈ L2 (Ω, F). Então tem-se que:
E[Z1 + Z2 | G] = E[Z1 | G] + E[Z2 | G] ,
isto é, se W1 e W2 forem, respectivamente, versões de E[Z1 | G] e E[Z2 | G] tem-se que
W1 + W2 é uma versão de E[Z1 + Z2 | G].
Demonstração. Tem-se que W1 + W2 mG dado que a soma de funções mensuráveis relativamente a G é mensurável relativamente a G. Seja agora G ∈ G. A condição expressa
na fórmula 8.6 que usamos na igualdade em (a) para cada uma das funções W1 e W2
permite-nos concluir que:
Z
Z
Z
Z
Z
(W1 + W2 ) dP =
W1 dP +
W2 dP =(a) =
Z1 dP +
Z2 dP =
G
G
G
G
G
Z
= (Z1 + Z2 ) dP ,
G
Podemos então concluir que W1 + W2 verifica também a condição 8.6 pelo que W1 + W2
é uma versão de E[Z1 + Z2 | G].
Seja agora X ∈ L1 . Consideremos X + , a parte positiva de X e associemos-lhe uma
sucessão crescente de variáveis aleatórias limitadas e por isso em L2 , pelo processo de
truncatura habitual:
Xn+ = X + I{X≤n} + nI{X>n} .
Pelo que já demonstrámos anteriormente seja Yn+ uma versão de E[Xn+ | G]. Então
(Yn+ )n≥1 é uma sucessão crescente de variáveis aleatórias positivas em resultado da
aplicação do lema 9 a cada uma das variáveis Xn+ e dos lemas 9 e 10 a cada uma
+
das diferenças Xn+1
− Xn+ . Seja agora:
Y + := lim sup Yn+ ∈ mG .
n→+∞
Por aplicação do teorema da convergência monótona de Lebesgue em (a) e por aplicação
da condição 8.6 da definição das versões da esperança condicional em (b), tem-se que
para qualquer G ∈ G:
Z
Z
Z
Y + dP =
lim Yn+ dP =(a) lim
Yn+ dP =(b)
n→+∞
n→+∞
G
G
G
Z
Z
Z
+
+
= lim
Xn dP =(a)
lim Xn dP =
X + dP
n→+∞ G
G n→+∞
G
227
Capı́tulo 8
Tem-se então que Y + verifica a condição 8.6 que nos garante que Y + é uma variável
aleatória integrável (uma vez que X + é) que é uma versão de E[X + | G]. Procedendo de
igual modo para X − demonstramos o teorema no caso geral.
Verifiquemos agora a unicidade. Seja Y e Ỹ duas versões de E[X | G]. Pretendemos
mostrar que a diferença entre as duas versões é nula quase certamente. Para tal seja por
definição a função positiva e mensurável relativamente a G:
h := (Y − Ỹ )I{Y ≥Ỹ } .
Pela condição 8.6 que se aplica na igualdade indicada com (a) abaixo tem-se que para
qualquer G ∈ G que:
Z
Z
Z
(Y − Ỹ ) dP =
Y dP +
Ỹ dP =(a)
G∩{Y ≥Ỹ }
G∩{Y ≥Ỹ }
G∩{Y ≥Ỹ }
Z
Z
X dP = 0 ,
X dP +
=
G∩{Y ≥Ỹ }
G∩{Y ≥Ỹ }
R
isto é, G h dP = 0 para qualquer G ∈ G. Uma vez que h é não negativa tem-se
que h = 0 quase certamente. Com efeito, com um argumente semelhante ao utilizado
acima na demostração do lema sobre a monotonia do operador associado à esperança
condicional, tem-se que h > 0 = ∪+∞
n=1 {h > 1/n} sendo esta última uma reunião de
elementos de uma famı́lia crescente. Assim sendo, supondo que P[h > 0] > 0, existe
necessariamente um inteiro n0 tal que P[h > 1/n0 ] > 0. Em consequência:
Z
1
0=
h dP ≥
P[h > 1/n0 ] > 0
n0
{h>1/n0 }
o que é absurdo. Podemos recomeçar esta argumentação mas agora com a função:
g := (Ỹ − Y )I{Ỹ ≥Y }
e concluir também que g = 0 quase certamente. Finalmente, como Y − Ỹ = h −
g = 0 quase certamente, temos que as duas versões são iguais quase certamente. Esta
demonstração foi ligeiramente adaptada daquela que se pode encontrar na obra [7][p.
86].
8.3.3
Como calcular esperanças condicionais
Para a determinação das esperanças condicionais num dado caso especı́fico podem usarse como métodos, entre outros, os seguintes:
• O recurso à definição evocada na secção 8.3.1 para o caso em que as variáveis
aleatórias são discretas, isto é tomam um número finito ou infinito numerável de
valores.
• O recurso às densidades das leis das variáveis aleatórias em presença.
228
A Esperança Condicional
• O recurso às propriedades operatórias das esperanças condicionais.
Vamos estudar em detalhe o método evocado no segundo ponto da lista anterior
enquanto que o terceiro método referido será desenvolvido na secção 8.3.5. Relembremos
que se X e Z forem variáveis aleatórias admitindo uma lei conjunta com densidade dada
pela função de duas variáveis fX,Z (x, z) então X (respectivamente Z) admite como
densidade fX (respectivamente fZ ) dada por:
Z
Z
fX,Z (x, z)dx .
fX,Z (x, z)dz respectivamente fZ (z) =
fX (x) =
R
R
Observe-se ainda que se, por exemplo, for fX (x0 ) = 0 então, dado que fX,Z é positiva,
se tem que fX,Z (x0 , z) = 0 quase por toda a parte relativamente à variável z e à medida
de Lebesgue em R.
Teorema 40. Sejam X e Z, variáveis aleatórias admitindo uma lei conjunta
com densidade dada pela função de duas variáveis fX,Z (x, z). Seja h uma
função Borel mensurável tal que:
Z
E[|h(X)|] =
|h(X)|fX (x)dx < +∞ ,
R
onde fX (x) é a densidade da lei marginal da variável X. Então se for g a
função de variável real definida por:
Z
fX,Z (x, z)
g(z) :=
h(x)
I
dx ,
fZ (z) {fZ 6=0}
R
tem-se que g(Z) é uma versão de E[h(X) | σ(Z)].
Demonstração. Pelas propriedades de definição da esperança condicional enunciadas no
teorema de Kolmogorov e dado que por definição se tem que:
σ(Z) := Z −1 (B(R) := {Z −1 (B) : B ∈ B(R} ,
para que seja válida a condição do teorema temos que verificar que:
Z
Z
∀B ∈ B(R)
g(Z)dP =
h(X)dP .
Z −1 (B)
Z −1 (B)
Atendendo a que IZ −1 (B) ≡ IB ◦ Z, é equivalente verificar que:
Z
Z
∀B ∈ B(R)
g(Z) (IB ◦ Z) dP =
h(X) (IB ◦ Z) dP .
Z −1 (B)
Ω
Dado que, por hipótese, as leis das variáveis aleatórias são-nos dadas pelas respectivas
densidades a igualdade entre os integrais pode representar-se de forma equivalente por:
Z
Z Z
∀B ∈ B(R)
g(z) IB (z) fZ (z)dz =
h(x) IB (z)fX,Z (x, z) dxdz .
R
R
R
229
Capı́tulo 8
Finalmente, observando que se para um dado z0 se tiver fZ (z0 ) = 0 então também para
todo o x se verifica que fX,Z (x, z0 ) = 0 salvo num conjunto de medida de Lebesgue nula,
podemos usar no integral, sem lhe alterar o valor, a seguinte igualdade:
fX,Z (x, z) =
fX,Z (x, z)
I
fZ (z) ,
fZ (z) {fZ 6=0}
Em consequência, podemos representar a igualdade de integrais acima pela igualdade
seguinte:
Z
Z
Z
fX,Z (x, z)
fZ (z)
h(x)
fZ (z)g(z)dz =
∀B ∈ B(R)
I
dx dz ,
fZ (z) {fZ 6=0}
B
R
B
o que mostra que a representação para a função g, formulada na hipótese do teorema, é
suficiente para garantir o resultado anunciado.
8.3.4
A esperança condicional E[X | T = y]
A proposição seguinte mostra como representar uma variável aleatória mensurável relativamente a uma álgebra-σ gerada por uma outra variável aleatória.
Proposição 58. Seja (Ω, F, P) um espaço de probabilidade, e Y uma variável
aleatória definida sobre este espaço tomando valores em Rm . Seja agora T uma
outra variável aleatória sobre o mesmo espaço e tomando valores em Rp . Então,
se Y for mensurável relativamente à sigma-álgebra σ(T ), existe uma aplicação
φ de Rp em Rm tal que:
Y = φ(T ) .
Demonstração. Seja Y = IA uma função indicatriz mensurável relativamente a σ(T ),
isto é relativamente à álgebra-σ gerada por T . Tem-se que:
A ∈ σ(T ) ⇒ ∃B ∈ B(Rp ) A = T −1 (B) .
Como se tem que:
(
1
IA (ω) = IT −1 (B) (ω) =
0
se T (ω) ∈ B
se T (ω) ∈
/B
= IB (T (ω)) ,
Definindo Φ = IB tem-se que Y = Φ(T ) tal como se afirmou. Para o caso geral segue-se
o processo sistemático de considerar as funções simples mensuráveis positivas, depois
as funções mensuráveis positivas quaisquer e finalmente as funções mensuráveis de sinal
qualquer.
230
A Esperança Condicional
Vimos anteriormente que dadas duas variáveis aleatórias X e Y tomando valores
em conjuntos finitos, sempre que para y ∈ Y (Ω) se tenha P[Y = y] 6= 0 é possı́vel
definir E[X | T = y] sem qualquer ambiguidade através das probabilidades condicionais
usuais. No caso em que P[Y = y] = 0, o que se verifica, para qualquer y ∈ Y (Ω), para as
variáveis aleatórias Y absolutamente contı́nuas relativamente à medida de Lebesgue, usar
as probabilidades condicionais usuais não é possı́vel. Na proposição seguinte apresentase um método que permite definir as probabilidades condicionais do tipo E[X | T = y]
para variáveis aleatórias gerais, desde que X seja integrável. A proposição estabelece
ainda uma relação útil, para efeitos do cálculo efectivo de esperanças condicionais, entre
E[X | T = y] e E[X | T ].
Proposição 59. Seja (Ω, F, P) um espaço de probabilidade, e X uma variável
aleatória definida sobre este espaço tomando valores em R. Seja agora T uma
outra variável aleatória sobre o mesmo espaço e tomando valores em R. Definese a esperança condicional de X dado que T = t como qualquer função
mensurável φ, de R em R, tal que:
Z
Z
∀B ∈ B(R)
φ(t) dLT (t) =
X dP .
T −1 (B)
B
onde LT é a lei de T . Tem-se então que:
1. A esperança condicional de X dado que T = t existe se X for integrável.
2. Se Y e Ỹ forem duas esperanças condicionais de X dado que T = t então:
Y = Ỹ salvo talvez um conjunto de probabilidade LT nula .
3. Verifica-se ainda que:
E[X | T ] = φ(T ) se φ(t) = E[X | T = t] .
Demonstração. Suponhamos primeiramente X ≥ 0. Considere-se a medida ν definida
para B ∈ B(R) por:
Z
ν(B) =
X dP .
T −1 (B)
A medida ν é finita dado que X é integrável. Verifica-se que ν LT uma vez que se
tem para B ∈ B(R):
LT (B) = P[T −1 (B)] = 0 ⇒ ν(B) = 0.
Pelo teorema de Radon-Nikodym tem-se que existe uma função integrável φ tal que
dν = φ · dLT , isto é tal que:
Z
Z
∀B ∈ B(R)
φ(t) dLT (t) = ν(B) =
X dP ,
B
T −1 (B)
231
Capı́tulo 8
tal como querı́amos. No caso de X não necessariamente com sinal constante aplica-se o
método descrito a X + e X − , obtendo-se φ+ e φ− e define-se naturalmente φ = φ+ − φ− .
Sejam agora Y e Ỹ forem duas esperanças condicionais de X dado que T = t.
Consider-se os borelianos de R dados por B+ := {Y ≥ Ỹ } e por B− := {Y < Ỹ }. Como,
por hipótese se tem que:
Z
Z
Y dLT =
B+
Z
X dP =
T −1 (B+ )
Ỹ dLT ,
B+
vem que:
Z
(Y − Ỹ ) · IB+ dLT = 0 .
R
Dado que (Y − Ỹ ) · IB+ ≥ 0 vem, em consequência que:
(Y − Ỹ ) · IB+ = 0 ,
salvo talvez um conjunto de probabilidade LT nula. Procedendo de modo semelhante
para B− pode obter-se que
(Y − Ỹ ) · IB− = 0 ,
salvo talvez um conjunto de probabilidade LT nula. Finalmente, observando que IR =
IB+ ∪ IB− tem-se, como anunciado, que Y − Ỹ = 0 salvo talvez um conjunto de probabilidade LT nula.
Para confirmarmos a última tese avançada no enunciado observe-se que φ(T ) é
mensurável relativamente a σ(T ) uma vez que se tem para B ∈ B(R), φ(T )−1 (B) =
T −1 (φ−1 (B)). Pode também ver-se que sendo φ a esperança condicional de X dado que
T = t se tem:
Z
Z
Z
φ(T ) dP =
φ(t) dLT (t) =
X dP
∀B ∈ B(R)
T −1 (B)
B
T −1 (B)
uma vez que a igualdade mais à esquerda resulta da integração relativamente à lei de T .
A igualdade entre o membro da esquerda e o membro mais à direita, conjuntamente com
a condição de mensurabilidade garantem que φ(T ) é uma versão da esperança condicional
de X dada T , tal como é afirmado no enunciado.
Exercı́cio 264. (Ver [2]) Seja f(X,Y ) (x, y) = (x + y)1I{0≤x,y≤1 (x, y). Determine E[X | Y ].
Exercı́cio 265. (Ver [2]) Suponha que X, Y e U são três variáveis aleatórias tais que X e U são
independentes, U tem distribuição uniforme sobre [0, 1], X tem a lei de Rayleigh de parâmetro σ, isto é
admite a densidade fX (x) = (x/σ 2 ) exp(−x2 /(2σ 2 ))1Ix≥0 (x) e Y = XU . Determine E[X | Y ].
232
A Esperança Condicional
8.3.5
Propriedades operatórias das esperanças condicionais
Nesta secção apresentamos uma lista das principais propriedades operatórias da noção
de esperança condicional que, tal como já referimos, são de grande utilidade no cálculo
explı́cito. Genericamente denotamos por X uma variável aleatória integrável e por G
uma subalgebra-σ de F. As demonstrações destas propriedades constituem exercı́cios
importantes de reolução fácil constituindo a respectiva redacção parte integrante do
trabalho na disciplina.
Propriedade 1. Se Y for uma versão de E[X | G] temos que E[Y ] = E[X] o que podemos
representar por:
E [E[X | G]] = E[X] .
Propriedade 2. Se X for G mensurável então X é uma versão de E[X | G], o que
podemos representar por
E[X | G] = X .
Propriedade 3. Seja Y1 (respectivamente Y2 ) uma versão de E[X1 | G] (respectivamente
E[X2 | G]). Então, para λ e µ números reais, temos que λY1 + µY2 é uma versão de
E[λX1 + µX2 | G],o que podemos representar por:
E[λX1 + µX2 | G] = λE[X1 | G] + µE[X2 | G] .
Propriedade 4. Se X for uma variável aleatória não negativa (isto é X ≥ 0) então
qualquer versão da esperança condicional de X dada G, é não negativa isto é, com o
abuso de notação convencional:
E[X | G] ≥ 0 .
Propriedade 5 (Convergência Monótona). Seja (Xn )n∈N uma sucessão de variáveis
aleatórias não negativas e crescente, quase certamente, para uma outra variável aleatória
X. Então, se para cada n ∈ N se tiver que Yn é uma versão da esperança condicional de
Xn dada G e, se Y for uma versão da esperança condicional de X dada G, tem-se que
limn→+∞ Yn ↑ Y ou ainda:
lim E[Xn | G] ↑ E[X | G]
n→+∞
Propriedade 6 (Convergência Dominada). Seja (Xn )n∈N uma sucessão de variáveis
aleatórias convergentes, quase certamente, para uma outra variável aleatória X e tais
que para V v. a. não negativa e integrável se tenha:
∀n ∈ N |Xn | ≤ V P q. c. .
Então, se para cada n ∈ N se tiver que Yn é uma versão da esperança condicional de
Xn dada G e, se Y for uma versão da esperança condicional de X dada G, tem-se que
limn→+∞ Yn = Y ou ainda:
lim E[Xn | G] = E[X | G]
n→+∞
233
Capı́tulo 8
Propriedade 7. Se Y for uma variável G mensurável tal que E[|XY |] < +∞ então:
E[X · Y | G] = Y · E[XY | G] .
0
0
Propriedade 8 (Tower law). Se G for uma sub-sigma-álgebra de F tal que G ⊂ G
então:
h
i
h
i
0
0
0
E E[X | G ] | G = E E[X | G] | G = E[X | G ] .
A propriedade seguinte é da maior importância no cálculo das esperanças condicionais, nomeadamente, numa generalização das probabilidades totais.
Propriedade 9 (Veja-se [3] página 452, ou [5] página 350, para enunciados mais gerais
e demonstrações.). Seja f : R2 −→ R uma função mensurável, X e Y variáveis aleatórias
reais tais que E[|f (X, Y )|] < +∞ e ainda E[|f (X, y)|] < +∞ quase certamente relativamente à lei de Y .
• Se X e Y forem independentes
E[f (X, Y ) | Y = y] = E[f (X, y)](=: ϕf (y)) .
• Se Y for G mensurável e se X e G forem independentes então:
E[f (X, Y ) | G] = ϕf (Y ) .
Observação 57. Veja-se agora uma aplicação desta propriedade às somas dePum número
aleatório de parcelas aleatórias. Supunhamos que se pretende calcular E[ N
i=1 Xi ] em
que N, X1 , . . . Xi , . . . são independentes e (Xi )i≥1 são identicamente distribuı́das. Então,
pela tower law ter-se-á que:
"N
#
" "N
##
X
X
E
Xi = E E
Xi | N
.
i=1
i=1
Como pela propriedade 9 se tem que:
"N
#
" n
#
n
X
X
X
E
Xi | N = n = E
Xi =
E [Xi ] = n E [X1 ] .
i=1
i=1
i=1
Daqui resulta, pela propriedade 9 ou também pela proposição 59, que:
"N
#
X
E
Xi | N = N E [X1 ]
i=1
e finalmente que:
E
"N
X
i=1
como seria de esperar.
234
#
Xi = E [N ] E [X1 ] ,
A Esperança Condicional
Demonstração. Seja para B ∈ B(R) a função g(x, y) = f (x, y) × 1IB (y). Tem-se a
seguinte cadeia de igualdades para funções da variável y:
E [g(X, y)] = E [f (X, y)] × 1IB (y) = ϕf (y) × 1IB (y) .
Em consequência temos a seguinte cadeia de igualdades numéricas.
Z
Z
Z
f (X, Y )1IB (Y )dP =
f (X, Y )1IY −1 (B) dP =
f (X, Y )dP =
Ω
Ω
Y −1 (B)
Z
h
i
=
g(X, Y )dP = E [g(X, Y )] = E (E [g(X, y)])y=Y =
Ω
h
i
= E [ϕ(y) × 1IB (y)]y=Y = E [ϕ(Y ) × 1IB (Y )] =
Z
Z
ϕ(Y )dP =
ϕ(Y ) × 1IB (Y )dP =
=
Y −1 (B)
Ω
Z
=
ϕ(y)dLY (y) ,
B
em que a independência entre X e Y permite a iteração do operador esperança matemática que conduz à quinta igualdade. Logo, pela proposição 59 e recordando que
ϕf (y) = E [f (X, y)], tem-se que
ϕf (y) = E [f (X, Y ) | Y = y] .
Suponhamos agora que f = 1IA × 1IB com A, B ∈ B(R). Então teremos a seguinte cadeia
de igualdades quase certas entre variáveis aleatórias:
E[f (X, Y ) | G] = E[1IX∈A × 1IY ∈B | G] = 1IY ∈B × E[1IX∈A | G] = 1IY ∈B × E[1IX∈A ] ,
em que a segunda igualdade decorre de Y ser G mensurável e a terceira igualdade de X
e G serem independentes. Ora, pelas definições, temos que
E[1IX∈A ] × 1IY ∈B = E[1IA (X)] × 1IB (Y ) = [E[1IA (X)] × 1IB (y)]y=Y =
= [E[1IA (X) × 1IB (y)]]y=Y = [E[f (X, y)]]y=Y = ϕf (Y ) ,
o que mostra que a proposição é válida no caso particular de f escolhido. A extensão
ao caso geral é agora imediata. Com efeito se considerarmos:
n
o
C = C ∈ C(R2 ) : E[1IC (X, Y ) | G] = ϕ1IC (Y )
pode mostrar-se que C é um sistema-λ que contem os rectângulos mensuráveis. Como
os rectângulos mensuráveis formam um sistema-π que gera a álgebra-sigma de Borel de
R2 , o teorema de Dynkin mostra que C coincide com esta álgebra-sigma. A passagem às
funções mensuráveis quaisquer obtem-se usando a linearidade das esperança condicional e
o correspondente ao teorema da convergência monótona para as esperanças condicionais.
235
Capı́tulo 8
8.4
Exercı́cios
[1]
Exercı́cio 266. Mostre directamente que:
hX − E[X | Z], E[X | Z]i = 0 .
Exercı́cio 267. Seja A = {A1 , A2 , . . . , Ap }, uma partição de Ω.
1. Mostre que:
(
σ(A) =
)
[
Ai : I ⊂ {1, 2, . . . , p}
.
i∈I
[1]
2. Seja X uma variável aleatória σ(A) mensurável. Mostre que X é constante sobre os conjuntos Ai
para i ∈ {1, 2, . . . , p}.
3. Conclua que para α1 , α2 , . . . , αp ∈ R se tem:
X=
p
X
αi IAi .
i=1
[1]
Exercı́cio 268. Seja (Ω, F, P) um espaço de probabilidade e Y uma variável aleatória real e integrável,
definida sobre este espaço.
1. Sendo B = {∅, Ω} determine E[Y |B].
2. Sendo B ∈ F, tal que 0 < P[B] < 1 e B1 = σ({B}) determine E[Y |B1 ].
3. Sendo X uma variável aleatória real tomando, P quase certamente, dois valores x1 ou x2 , determine E[Y |X].
4. Seja X uma variável aleatória tomando, P quase certamente, um número finito de valores x1 , . . . , xn
tal que para i = 1, . . . , n se tenha que P[X = xi ] 6= 0. Determine E[Y |X].
5. Suponha que a variável aleatória X toma, quase certamente, os seus valores num conjunto numerável {xn : n ∈ N} e ainda que
∀n ∈ N P[X = xn ] 6= 0 .
Determine E[Y |X].
[1]
Exercı́cio 269. Sejam X e Y variáveis aleatórias independentes com distribuições de Poisson de
parâmetros λ e µ, respectivamente.
1. Verifique que X + Y segue uma distribuição de Poisson de parâmetro λ + µ.
2. Calcule a distribuição de X condicionada por Z = X + Y .
3. Calcule E(X|Z).
[1]
Exercı́cio 270. Consideremos um vector aleatório (X, Y ) cuja distribuição é dada pelo quadro seguinte:
Y
X
-1
0
1
-1
0.1
0.15
0.05
0
0.15
0
0.05
1
0
0.1
0
2
0.1
0.2
0.1
Calcule a distribuição de Y condicionada por X = n, n = −1, 0, 1, E[Y |X] e E[Y ].
236
A Esperança Condicional
[1]
Exercı́cio 271. Seja (X, Y ) um vector aleatório com densidade fX,Y (x, y). Mostre que se definirmos
fY (y) por:
Z
fY (y) :=
fX,Y (x, y) dx
R
então considerando:
fX,Y (x, y)
I{fY 6=0} (y) ,
fY (y)
fX|Y =y (x) =
tem-se que se
Z
φ(y) =
xfX|Y =y (x)dx
R
φ(Y ) é uma versão da esperança condicional de X dado Y .
Exercı́cio 272. Seja f(X,Y ) (x, y) = e−y I{0≤x≤y} (x, y) definida em R2 .
[1]
1. Mostre qye f(X,Y ) é de facto uma densidade.
2. Mostre que fX (x) = e−x IR+ (x) e que fY (y) = ye−y IR+ (y).
3. Mostre que fX,Y =y (x) = y1 I[0,y] (x) e mostre que E[X | Y ] =
4. Mostre que fX=x,Y (y) =
−y
e
I
(y)
e−x {0≤x≤y}
Y
2
.
e mostre que E[Y | X] = 1 + X.
Exercı́cio 273. Seja X uma variável aleatória uniformemente distribuı́da em [0, 1] e Y uma variável [1]
aleatória tal que condicionalmente a X = x a lei de Y é uniforme em [0, x].
1. Determine a lei do par (X, Y ) e a lei de Y .
2. Determine E[X | Y ] e E[Y | X].
[1]
Exercı́cio 274. Seja (X, Y ) um vector aleatório com densidade
f (x, y) =
2
I[0,+∞[×[0,+∞[ (x, y) ,
(1 + x + y)3
Calcule E(X|Y ).
[2]
Exercı́cio 275. Seja (Xn )n∈N∗ uma sucessão de variáveis aleatórias iid tais que:
E[X1 ] = µ , V[X1 ] = ν .
Seja N uma variável aleatória inteira independente das variáveis (Xn )n∈N∗ e tal que:
E[N ] = α , V[N ] = β .
Seja ainda:
∀n ∈ N∗ Sn =
n
X
Xk .
k=1
1. Mostre que SN é uma variável aleatória e que:
E[SN |N = n] = E[Sn ] .
2. Deduza do resultado precedente expressões para E[SN ] e para V[SN ].
Exercı́cio 276. Sejam D1 e D2 , duas subálgebras-σ de F. Seja D = σ(D1 ∪ D2 ). Seja Y uma variável [3]
aleatória real e integrável, tal que as σ-álgebras D2 e σ(σ(Y ) ∪ D1 ) sejam independentes.
237
Capı́tulo 8
1. Mostre que E[Y |D] = E[Y |D1 ].
2. Seja (Xn )n∈N∗ uma sucessão de variáveis aleatórias integráveis e iid. Seja ainda:
∀n ∈ N∗ Sn =
n
X
Xk .
k=1
Determine E[X1 |Sn , Sn+1 , . . . ].
Exercı́cio 277. Seja (Xn )n∈N uma sucessão de variáveis aleatórias independentes integráveis tal que [3]
para qualquer n ∈ N se tenha E[Xn ] = µn . Seja para cada n ∈ N Fn := σ(X1 , . . . , Xn ). Mostre que:
1. para k > n se tem que:
"
E
k
X
#
Xj | Fn =
j=1
"
E
k
Y
n
X
j=1
#
Xj | Fn =
j=1
k
X
Xj +
µj e que ,
j=n+1
n
Y
!
Xj
j=1
!
k
Y
µj
.
j=n+1
2. Supondo que as variâncias V[Xn ] = σn2 , existem e são finitas mostre que:

k
X
E
(Xj − µj )
!2

| Fn  =
j=1
j=1
[2]
n
X
(Xj − µj )
!2
+
k
X
σj2 .
j=n+1
Exercı́cio 278. Seja um vector aleatório bidimensional (X, Y ) com distribuição FX,Y (x, y). Seja por
definição:
H(x|y) := P[X ≤ x|Y = y] ,
a função de distribuição condicional de X dada Y . Mostre que:
Z y
FX,Y (x, y) =
H(x|u)FY (du) ,
−∞
onde FY (y) é a função de distribuição marginal de Y .
[3]
Exercı́cio 279. Seja um vector aleatório bidimensional (X, Y ) com distribuição gaussiana não degenerada.
1. Pretendemos mostrar que E[X|Y ] é da forma αY +β. Suponha inicialmente que (X, Y ) é centrado
e reduzido da covariância ρ.
(a) Resolva a questão usando o facto que para quaisquer números reais a e b o vector (X −
(aY + b), Y ) é gaussiano.
(b) Resolva a questão usando a densidade do par (X, Y ).
2. Determine E[X|Y + X] e interprete geometricamente o resultado para (X, Y ) centrado e reduzido.
238
A Esperança Condicional
8.5
Resoluções
Resolução:[ Exercı́cio 264] O método consiste em quatro passos. No primeiro determinase, a partir da lei do par (X, Y ), a lei marginal de Y , obtendo-se a densidade fY (y) =
(1/2) + y1I{0≤y≤1} (y). No segundo, calcula-se a lei condicional de X dado que Y = y
vindo a densidade:
2(x + y)
fX|Y =y (x) =
1I
(x, y) .
1 + 2y {0≤x,y≤1}
Para o terceiro passo calcula-se a esperança condicional de X dado que Y = y por
integração da densidade obtida no segundo passo, isto é:
Z 1
2 + 3y
xfX|Y =y (x)dx =
1I
(y) .
E[X | Y = y] =
3(1
+ 2y) {0≤y≤1}
0
No quarto passo aplica-se a observação da alı́nea 3 da proposição 59 e vem
2 + 3Y
E[X | Y ] =
,
3(1 + 2Y )
obtendo-se assim o pretendido.
♦
Resolução:[ Exercı́cio 265] Para seguir o método indicado para o exercı́cio 264 necessitamos a lei conjunta do par (X, Y ) que não nos é dada directamente. No entanto,
observando que f(X,Y ) (x, y) = f(X) (x)fY |X=x (y), que fY |X=x (y) é a densidade da lei de
Y = xU e que esta é a lei uniforme sobre [0, x] pode prosseguir-se aplicando o método
referido.
♦
Bibliografia
[1] Zdzislaw Brzeźniak and Tomasz Zastawniak. Basic stochastic processes. Springer
Undergraduate Mathematics Series. Springer-Verlag London Ltd., London, 1999. A
course through exercises.
[2] Bruce Hajek. An Exploration of Random Processes for Engineers. Notes for ECE
534, 2009.
[3] J. Hoffmann-Jørgensen. Probability with a view toward statistics. Vol. I. Chapman
& Hall Probability Series. Chapman & Hall, New York, 1994.
[4] A. N. Kolmogorov. Foundations of the Theory of Probability. Chelsea Publishing
Company, New York, N. Y., 1950.
[5] Sidney I. Resnick. A probability path. Birkhäuser Boston Inc., Boston, MA, 1999.
[6] Walter Rudin. Principles of mathematical analysis. McGraw-Hill Book Co., New
York, third edition, 1976. International Series in Pure and Applied Mathematics.
[7] David Williams. Probability with martingales. Cambridge Mathematical Textbooks.
Cambridge University Press, Cambridge, 1991.
239
240
Capı́tulo 9
O Teorema de Fubini, Leis
Conjuntas e a Construção de
Sucessões de Variáveis Aleatórias
Independentes
9.1
Introdução
A teoria dos processos estocásticos nasceu, na sua sofisticada forma actual, com Kolmogorov aquando da publicação do texto Grundbegriffe der Wahrscheinlichkeitrechnung 1 .
Nesta obra que é um marco fundamental da moderna teoria das probabilidades, é possı́vel
encontrar o teorema que garante a existência de processos estocásticos gerais.
No caso dos processos estocásticos a tempo discreto, isto é quando o conjunto dos
ı́ndices é um subconjunto dos inteiros, é possı́vel uma aproximação mais simples que
a que é exigida para o teorema de Kolmogorov. Nestas notas temos como um dos
principais objectivos mostrar que se pode garantir a existência de uma sucessão de
variáveis aleatórias independentes com leis dadas a priori. Para esse efeito usamos,
naturalmente, a noção de espaço produto. Neste contexto um resultado fundamental é
o teorema de Fubini que nos esclarece sob que condições é possı́vel trocar a ordem de
integração em integrais múltiplos.
9.2
A construção dos espaços produto e o teorema de Fubini
Inicialmente iremos construir uma estrutura de espaço mensurável sobre o produto cartesiano de dois (ou mais) conjuntos a partir de estruturas semelhantes dadas sobre os
espaços factores.
1
Originalmente publicada em lı́ngua alemã em 1933 mas disponı́vel em lı́ngua Inglesa com o tı́tulo
Foundations of the Theory of Probability (veja-se [1]).
241
Capı́tulo 9
0
0
0
Considere-se (Ω, A, P) e (Ω , A , P ) dois espaços de probabilidade. O produto carte0
siano de Ω por Ω é dado por:
0
0
0
0
Ω × Ω = {(ω, ω ) : ω ∈ Ω , ω ∈ Ω } .
Considere-se por definição:
0
0
S := {A × A : A ∈ A , A ∈ A0 } .
0
Dado que A e A são semi-anéis, S é também um semi-anel 2 . Seja C(S) a álgebra gerada
0
por S e considere-se a álgebra-σ sobre Ω × Ω definida por:
0
A ⊗ A := σ(C(S)) = σ(S) .
0
0
Por definição, A ⊗ A , é a álgebra-σ produto de A por A . Falta-nos apenas iniciar a
0
definição da medida de probabilidade produto. Para S × S ∈ S isto é tal que S ∈ A e
0
S ∈ A seja por definição:
0
0
0
P ⊗ P [S × S ] := P[S] × P0 [S ] .
(9.1)
Definimos assim a medida produto sobre o semi-anel S. É uma aproximação natural
que se pode justificar heuristicamente com o exemplo seguinte. Este exemplo deve ser
encarado neste ponto da exposição de forma intuitiva e irá ser plenamente justificado
adiante.
Exemplo 17. :
Considere-se
0
0
0
(Ω, A, P) = ([0, 1], B([0, 1], λ) = (Ω , A , P ).
Então naturalmente deveremos ter que

0
2

 Ω × Ω = [0, 1]
B([0, 1]) ⊗ B([0, 1]) = B([0, 1]2 )


λ ⊗ λ(A × B) = área (A × B)
Esta é uma aproximação natural para a construção da noção de área em R2 .
Observação 58. Para melhor se compreender o que vai seguir-se é útil conservar presente
este exemplo.
A proposição seguinte descreve uma propriedade de integrais iterados simples que é
relevante para a definição de medida produto.
0
Proposição 60. Seja S × S ∈ S . Então
Z
Z
0
0
0
IS (ω)
IS 0 (ω )dP (Ω ) dP(ω) =
Ω
Ω0
Z
Z
0
0
0
IS (ω)dP(ω) dP(ω ) = P[S] × P0 [S ].
=
IS 0 (ω )
Ω0
2
242
Ω
Ver Construção das Medidas de Lebesgue-Stieltjes, Medida Integração e Probabilidade 2008-2009.
O Teorema de Fubini, Leis Conjuntas e a Construção de Sucessões de Variáveis
Aleatórias Independentes
Demonstração. Como se tem, por exemplo,
Z
0
0
0
IS 0 (ω )dP0 (ω ) = P0 [S ].
0
Ω
a verificação do resultado é imediata.
Observação 59. Como convenção para o que vai seguir-se consideramos a seguinte notação
para o integral duplo:
Z
0
Ω×Ω0
0
0
IS×S 0 (ω, ω )d(P ⊗ P0 )(ω, ω ) := P ⊗ P0 [S × S ] .
Esta notação faz sentido uma vez que por definição se tem que pela fórmula 9.1:
0
0
P ⊗ P0 [S × S ] = P[S] × P[S ]
e esta última quantidade é o valor comum aos integrais duplos que naturalmente surgiriam como interpretação do membro à esquerda na igualdade.
0
Definição 57. Seja (Sn × Sn )n∈N uma sucessão de elementos em S dois a dois
disjuntos. Defina-se
"
# Z Z
[
0
0
0
0 0
S
0 (ω, ω )dP (ω )
I
dP(ω) =
P⊗P
Sn × Sn =
n∈N Sn ×Sn
Ω
Ω0
n∈N
(9.2)
Z Z
=
0
Ω
IS
Ω
n∈N
0 (ω, ω
Sn ×Sn
0
0
0
)dP(ω) dP (ω ).
Observação 60. Esta definição faz sentido uma vez que
0
0
0
∀(ω, ω ) ∀n ∈ N ISn ×S 0 (ω, ω ) = ISn (ω) · IS 0 (ω )
n
n
e que, dado que os conjuntos são dois a dois disjuntos,
0
∀(ω, ω ) IS
0 (ω, ω
n∈N Sn ×Sn
0
)=
X
0
ISn (ω)IS 0 (ω ) .
n
n∈N
Por aplicações sucessivas do teorema da convergência monótona de Lebesgue (veja-se,
243
Capı́tulo 9
por exemplo, em (a)) tem-se que, por exemplo;
Z Z
0
0
S
0 (ω, ω )dP(ω )
dP(ω) =
I
n∈N Sn ×Sn
Ω
Ω0
!
Z
Z X
0
=
ISn (ω) · IS 0 (ω ) dP(ω) =(a) (b)
=
n
Ω0 n∈N
Ω
Z X
Z
ISn (ω)
Ω0
Ω n∈N
0
0
IS 0 (ω )dP (ω ) · dP(ω) =
n
!
Z
X
=
Ω
=
0
X
0
0
P [Sn ] · ISn (ω) dP(ω) =(a)
n∈N
0
Z
0
P [Sn ] ·
ISn (ω)dP(ω) =
Ω
n∈N
X
0
0
P [Sn ] · P[Sn ].
n∈N
Note-se que parte da justificação para a igualdade assinalada P
acima com (b) é que
0
0
0
0
para ω0 ∈ Ω fixo, a aplicação de Ω em R+ que a ω ∈ Ω associa n∈N ISn (ω0 )IS 0 (ω )
n
é, obviamente, mensurável positiva.
Da mesma forma se poderá concluir que
Z Z
X
0
0
0
0
S
0
I
P[Sn ] · P [Sn ].
Sn ×S (ω, ω )dP(ω) dP (ω) =
Ω0
Ω
n
n∈N
n∈N
ficando então justificada a definição.
0
Proposição 61. A função de conjuntos P ⊗ P definida em S na fórmula 9.1 e
na e na fórmula 9.2 da definição anterior para reuniões disjuntas de elementos
de S é aditiva-σ sobre S .
Demonstração. Com efeito pela definição anterior e pela fórmula 9.1 tem-se que para
0
(Sn × Sn )n∈N sucessão de elementos de S dois a dois disjuntos.
"
#
[
X
X
0
0
0
0
0
0
P⊗P
Sn × Sn =
P[Sn ] · P [Sn ] =
P ⊗ P [Sn × Sn ].
n∈N
n∈N
n∈N
Podemos pois sumariar os resultados obtidos até agora na seguinte forma.
0
Teorema 41. Existe P⊗P uma medida de probabilidade única sobre a álgebra-σ
0
0
produto A ⊗ A , denominada a medida produto de P por P , tal que:
0
0
0
0
0
0
∀A × A ∈ A ⊗ A P ⊗ P [A × A ] = P[A] × P [A ] .
244
(9.3)
O Teorema de Fubini, Leis Conjuntas e a Construção de Sucessões de Variáveis
Aleatórias Independentes
0
Demonstração. Existe uma extensão aditiva-σ de P⊗P verificando a fórmula 9.3 a C(S),
0
por teoremas anteriores e uma extensão a σ(C(S)) = A ⊗ A pelo teorema extensão de
Carathéodory.
0
0
0
Definição 58. : O espaço (Ω × Ω , A ⊗ A , P ⊗ P ) assim construı́do é o espaço
0
0
0
de probabilidade produto de (Ω, A, P) por (Ω , A , P ).
Na secção 9.6, apresentamos sob forma de exercı́cios complementares alguns resultados úteis que permitirão aprofundar e esclarecer o que atrás foi exposto. No exercı́cio 286
0
estabelece-se a ligação entre a álgebra-σ produto A⊗A e as projecções. No exercı́cio 287,
com uma resolução mais delicada, constrói-se a álgebra-σ produto de espaços topológicos.
9.3
O Teorema de Fubini
O teorema de Fubini esclarece-nos sobre condições suficientes gerais que permitem inverter a ordem de integração em integrais múltiplos.
Consideremos para o que vai seguir-se as seguintes notações (Ω, F) é um espaço
mensurável, L(F) é o espaço das funções mensuráveis de (Ω, F) em R, B(R) que são
limitadas.
O teorema da classe monótona
O teorema seguinte é um resultado essencial na em teoria da medida e das probabilidades.
Teorema 42. Seja H um conjunto de funções limitadas de Ω em R tal que:
1. H é um espaço vectorial;
2. IΩ ∈ H;
3. ∀(fn )n∈N ∈ HN fn ≥ 0 , fn ↑ f , f limitada
⇒f ∈H
Então, se para I um dado Π-sistema sobre Ω se verificar que
∀I ∈ I II ∈ H ,
tem-se que L(σ(I)) ⊆ H.
Observação 61. : O teorema da classe monótona é essencial para a construção da medida
produto e para os resultados de integração nos espaços produto. Para a demonstração,
veja-se, por exemplo, [2, p. 205].
Como primeira aplicação deste resultado considere-se o seguinte. Para que um inte0
gral iterado de uma função de duas variáveis Z(ω, ω ), por exemplo,
Z Z
0
0
0
Z(ω, ω ) dP (ω ) dP(ω)
Ω
Ω0
245
Capı́tulo 9
R
0
0
0
faça sentido a aplicação ω 7−→ Ω0 Z(ω, ω ) dP (ω ) deve ser pelo menos mensurável. No
exercı́cio seguinte ver-se-á que de facto assim é sob condição suficientemente gerais.
Exercı́cio 280. Seja
Z
0
H := {Z ∈ m(A ⊗ A ) : ω 7−→
0
0
0
0
Z(ω, ω ) dP (ω ) é mensurável sobre Ω} .
Ω
1. Mostre que H verifica as hipóteses do teorema da classe monótona.
0
0
2. Mostre que para A × A ∈ S se tem IA×A0 ∈ H e conclua que L(A ⊗ A ) ⊂ H.
9.3.1
Secções de uma aplicação e o teorema de Fubini
Seja uma dada aplicação definida num espaço produto, isto é, uma função de duas
variáveis.
0
Z :Ω×Ω →R.
As secções de uma tal aplicação correspondem a considerar as funções de uma só
variável que se obtêm fixando uma das variáveis da função inicialmente dada e fazendo
variar a outra variável.
0
0
Definição 59. Para ω0 ∈ Ω e ω0 ∈ Ω arbitrários mas considerados fixos sejam
as secções Zω0 e Zω0 definidas por:
0
0
∀ω ∈ Ω
0
0
0
Zω0 (ω ) = Z(ω0 , ω )
e
∀ω ∈ Ω Zω0 (ω) = Z(ω, ω00 ) .
0
Dada uma função mensurável Z as secções são naturalmente mensuráveis. Com
efeito, Zω0 pode sempre escrever-se como Z ◦ ψω0 em que ψω0 é a aplicação que a ω
0
0
0
associa (ω, ω0 ). Uma vez que para A × A ∈ S:
0
0
(
A
ψ −1
0 (A × A ) = {ω ∈ Ω : (ω, ω0 ) ∈ A × A } =
ω0
∅
0
0
0
0
se ω0 ∈ A
0
0
se ω0 ∈
/A ,
0
tem-se que a secção Zω0 é mensurável. Uma questão natural é a de saber, sob que
0
condições sobre as secções mensuráveis de uma função Z, a função Z é mensurável.
0
Proposição 62. Seja G a classe das funções de L(A ⊗ A ) tais que:
• ∀ω ∈ Ω Zω ∈ mA
0
• ∀ω ∈ Ω
0
Zω0 ∈ mA
0
Então G ≡ L(A ⊗ A )
246
0
O Teorema de Fubini, Leis Conjuntas e a Construção de Sucessões de Variáveis
Aleatórias Independentes
Demonstração. G verifica as hipóteses do teorema da classe monótona. Sendo
0
0
0
S := {A × A : A ∈ A , A ∈ A }
0
tem-se que se S × S ∈ S então IS×S 0 = IS × IS 0 ∈ G como é fácil verificar. Como
0
0
A ⊗ A = σ(S), pelo teorema da classe monótona tem-se que L(α ⊗ α ) ⊆ G.
Exercı́cio 281. Mostre que G verifica as hipóteses do teorema da classe monótona.
Finalmente, o teorema que constitui o objecto principal desta secção.
0
Teorema 43 (Teorema de Fubini). Seja Z ∈ m(A ⊗ A ) . Então se uma ou
outra das seguintes condições se verifica:
1. Z ≥ 0 ou
2.
Z
Z
Ω0
Z Z
0
|Z|dP dP < +∞ ou
Ω
Ω
Z
Ω×Ω
tem-se que:
Z
Ω×Ω0
0
0
Ω
Ω0
|Z|dP
dP < +∞ ou ainda
|Z|d(P × P0 ) < +∞
Z Z
Z d(P ⊗ P ) =
0
Ω0
0
Z dP
Z
dP =
Ω0
Z
0
Z dP dP .
Ω
0
Demonstração. Seja K a classe das funções Z ∈ L(A ⊗ A ) que verificam:
0
0
0
1. ∀ω ∈ Ω
∀ω ∈ Ω
Zω ∈ mA , Zω0 ∈ mA
0
R R
R
R
0
2. Ω Ω0 ZdP dP = Ω0 Ω ZdP dP
Então K verifica as hipóteses do teorema da classe monótona. É imediato verificar que
0
0
0
0
0
para A ∈ A, A ∈ A se tem que IA×A0 ∈ K. Como S = {A × A : A ∈ A , A ∈ A }
0
0
é tal que σ(S) = A ⊗ A tem-se, pelo teorema da classe monótona que L(A ⊗ A ) ⊆ K
0
e logo L(A ⊗ A ) ≡ K. As condições de definição de K valem agora para as funções
0
simples mensuráveis positivas de A ⊗ A em R uma vez que estas são elementos de
0
L(K⊗K ). Pelo teorema de aproximação usual e pelo teorema da convergência monótona
de Lebesgue, as condições valem para as funções mensuráveis positivas quaisquer pelo
que a conclusão do teorema também. Considerando a parte positiva e a parte negativa
de Z e a segunda hipótese do teorema, a conclusão do teorema verifica-se para as funções
0
com sinal qualquer mensuráveis sobre A ⊗ A e satisfazendo essa segunda hipótese.
247
Capı́tulo 9
Exercı́cio 282. Mostre que K verifica as hipóteses do teorema da classe monótona.
Como primeira aplicação imediata do teorema de Fubini vamos justificar um processo
de cálculo dos momentos de uma variável aleatória usando a função de distribuição dessa
variável aleatória.
Proposição 63. No espaço de probabilidade (Ω, A, P) seja uma variável
aleatória X.
1. Para qualquer s > 0:
Z +∞
Z
E[| X |s ] = s
xs−1 P[| X |≥ x] dx = s
0
+∞
xs−1 P[| X |> x] dx .
0
(9.4)
2. Em consequência verifica-se ainda que:
+∞
X
P[| X |≥ n] ≤ E[| X |] ≤ 1 +
n=1
+∞
X
P[| X |≥ n]
n=1
Demonstração. Sem perda de generalidade podemos supor que X é não negativa. Os
dois integrais à direita na fórmula 9.4 são iguais uma vez que:
#{x ∈ R : P[X = x] > 0} ≤ ℵ0 .
isto é, P[X = x] 6= 0 no máximo para uma infinidade numerável de pontos x ∈ R. Temos
então, se representarmos por LX a lei da variável X e por uma aplicação do teorema de
Fubini em (a) que:
Z
s
s
Z
+∞
Z
s
+∞ Z +∞
s−1
E[X ] =
X P=
x dLX (x) =
s y I[0,x] (y) dy dLX (x) =(a)
Ω
0
0
0
Z +∞ Z +∞
Z +∞ Z
s−1
=(a)
I[0,x] (y) dLX (x) s y
dy =
I{y≤X} dP s y s−1 dy =
0
0
0
Ω
Z +∞
=
sy s−1 P[| X |≥ y] dy .
0
Para obtermos a segunda afirmação da proposição basta observar que aplicando a fórmula
que acabámos de demonstrar para s = 1:
E[X] ≥
+∞ Z
X
n
n=1 n−1
248
P[X ≥ n] dx =
+∞
X
n=1
P[X ≥ n] .
O Teorema de Fubini, Leis Conjuntas e a Construção de Sucessões de Variáveis
Aleatórias Independentes
Para a desigualdade no outro sentido:
+∞ Z n
+∞
X
X
E[X] ≤
P[X ≥ n − 1] dx = 1 +
P[X ≥ n] ,
n=1 n−1
n=1
ficando assim concluı́da a demonstração.
9.4
Lei Conjunta de um Par de Variáveis Aleatórias
Seja (Ω, A, P) um espaço de probabilidade e X, Y variáveis aleatórias de (Ω, A) em
(R, B(R)) . É sabido que o par (X, Y ) pode ser considerado como variável aleatória de
(Ω, A) em (R2 , B(R2 )) uma vez que pelo exercı́cio 287 se verifica:
R2 = R × R e B(R) ⊗ B(R) = B(R2 ) .
Dado que σ ({A × B : A ∈ B(R) , B ∈ B(R)}) = B(R2 ) a lei do par (X, Y ) é a única
medida de probabilidade sobre (R2 , B(R2 )) que verifica, por definição,
∀A, B ∈ B(R2 ) L(X,Y ) (A × B) := P (X, Y )−1 (A × B) = P X −1 (A) ∩ Y −1 (B)
Se X e Y forem independentes, isto é, se as álgebras-σ σ(X) e σ(Y ) forem independentes,
então:
P X −1 (A) ∩ Y −1 (B) = P X −1 (A) × P Y −1 (B) = LX (A) × LY (B) .
Conclui-se assim, no caso de variáveis aleatórias independentes, X e Y que L(X,Y ) =
LX ⊗ LY .
Exercı́cio 283. Justifique a afirmação anterior.
O exercı́cio seguinte caracteriza a lei conjunta de um par de variáveis aleatórias em
função das leis marginais.
Exercı́cio 284. Considere as notações definidas acima.
1. Mostre que a lei marginal de X (respectivamente Y ) que representamos por LX , (respectivamente
LY ) é dada por:
∀A ∈ B(R) LX (A) = L(X,Y ) (A × R)
respectivamente,
∀B ∈ B(R) LY (B) = L(X,Y ) (R × B) .
2
2. Seja f(X,Y ) ∈ mB(R ) uma densidade do par (X, Y ). Isto é f(X,Y ) ≥ 0 e verifica
Z
∀C ∈ B(R2 ) L(X,Y ) (C) =
f(X,Y ) (x, y) d(λ ⊗ λ)(x, y)
C
em que λ é a medida de Lebesgue sobre (R, B(R)). Mostre que fX definida λ quase certamente
por
Z
fX (x) =
f(X,Y ) (x, y) dλ(y)
R
é uma densidade de lei marginal correspondente a X.
249
Capı́tulo 9
3. Mostre que X e Y são independentes sse ocorre uma das seguintes condições.
(a) L(X,Y ) ≡ LX ⊗ LY
(b) F(X,Y ) ≡ FX · FY
(c) f(X,Y ) ≡ fX · fY ,
com as hipóteses adequadas, por exemplo, para garantir a existência das densidades marginais.
No exercı́cio seguinte faz-se a construção de uma sequência (famı́lia finita) de variáveis
aleatórias independentes com leis dadas. Este resultado irá ser generalizado no teorema 44 para sucessões (famı́lia infinita numeráveis),
Exercı́cio 285. Considere, para N ∈ N∗ , medidas de probabilidade sobre (R, B(R)) denotadas por
µ1 , . . . , µ N .
1. Defina a medida produto
NN
i=1
µi sobre (RN , B(RN )).
2. Considere para i = 1, · · · , N , Πi a i-ésima projecção de RN sobre R isto é tal que
∀(x1 , . . . , xN ) ∈ RN Πi (x1 , . . . , xN ) = xi .
Mostre que Π1 , . . . , ΠN é uma sequência de variáveis aleatórias independentes sobre (Ω, A) :=
(RN , B(RN )) tais que:
∀i ∈ {1, . . . , N } LΠi = µi .
Observação 62. Os espaços produto permitem desta forma construir naturalmente sequências
de variáveis aleatórias com leis pré-estabelecidas.
9.5
Construção de uma sucessão de variáveis aleatórias independentes com leis dadas
O teorema seguinte é uma generalização do resultado obtido no exercı́cio anterior.
250
O Teorema de Fubini, Leis Conjuntas e a Construção de Sucessões de Variáveis
Aleatórias Independentes
Teorema 44. Seja (µn )n∈N∗ uma sucessão de medidas de probabilidade em
(R, B(R)). Seja
Y
∗
Ω = RN =
R := {(ωn )n∈N∗ : ∀n ∈ N∗ ωn ∈ R}
n∈N∗
e para m ∈ N∗ Πn a n-ésima projecção de Ω em R isto é:
∀(ωn )n∈N∗ ∈ Ω Πn (ω1 , ω2 , . . . , ωn , · · · ) = ωn .
Seja por definição A = σ(Πn : n ∈ N∗ ), isto é, a álgebra-σ gerada pelas projecções. Então:
N
• existe P =
n∈N∗ µn uma medida de probabilidade única sobre (Ω, A)
verificando para r ∈ N∗ e B1 , . . . , Br ∈ B(R) :
" r
#
r
Y
Y
Y
P
Bk ×
R =
µk (Bk )
k=1
k>r
k=1
• A sucessão (Πn )n∈N∗ é uma sucessão de variáveis aleatórias independentes
sobre (Ω, A, P) tais que ∀n ∈ N∗ LΠn = µn .
Demonstração. A demonstração deste resultado é tecnicamente mais exigente e pode ser
relegada para uma segunda leitura. Veja-se [2, p. 214 – 216].
9.6
Exercı́cios Complementares
0
0
Exercı́cio 286. Considere Π e Π as projecções definidas em Ω × Ω respectivamente por:
0
Ω×Ω
0
(ω, ω )
Π:
→
7−→
Ω
0
Π(ω, ω ) = ω
→
7−→
Ω
0
0
Π(ω, ω ) = ω
e
0
0
Ω×Ω
0
(ω, ω )
Π :
0
0
0
0
1. Mostre que S = {Π−1 (A) ∩ (Π )−1 (A ) : A ∈ A, A ∈ A } .
0
0
0
0
2. Mostre que A ⊗ A = σ(Π, Π ), isto é, que A ⊗ A é a mais pequena álgebra-σ sobre Ω × Ω
0
relativamente à qual Π e Π são simultaneamente aplicações mensuráveis.
0
0
3. Sejam X e Y variáveis aleatórias tais que X ∈ mA e Y ∈ mA . Seja Z definida em Ω × Ω por
0
0
0
0
∀(ω, ω ) ∈ Ω × Ω Z(ω, ω ) = X(ω) · Y (ω ) .
0
0
Mostre que Z é mensurável de (Ω × Ω , A ⊗ A ) em (R, B(R)) .
251
Capı́tulo 9
Exercı́cio 287. Sejam (X1 , τ1 ), · · · , (Xn , τn ) espaços topológicos e para cada i ∈ {1, · · · , n} seja Bi =
σ(τi ) a álgebra-σ de Borel para cada um desses espaços.
N
1. Mostre como definir n
i=1 Bi a álgebra-σ produto das álgebras σ(Bi )1≤i≤n sobre o produto cartesiano X = X1 × · · · × Xn .
N
2. Seja τ = n
i=1 τi a topologia produto sobre X, isto é, a mais pequena topologia sobre X relativamente à qual todas as projecções Πi definidas por
Πi :
X
(x1 , . . . , xn )
−→
7−→
Xi
Πi (x1 , . . . , xn ) = xi
são simultaneamente contı́nuas. Seja Bτ = σ(τ ) a álgebra-σ gerada por τ . Mostre que
Bτ
Nn
i=1
Bi ⊆
3. Mostre que se paraNcada i ∈ {1, · · · , n}(Xi , τi ) for um espaço topológico com base numerável de
n
vizinhanças então
i=1 Bi ≡ Bτ .
Q
Ω2 × Ω3 o
Exercı́cio 288. Seja (Ωi , Ai , Pi ) para i = 1, 2, 3 espaço de probabilidade, Ω = 3i=1 Ωi = Ω1 ×N
produto cartesiano usual e Πi , i = 1, 2, 3 a projecção de Ω sobre Ωi . Seja por definição A := 3i=1 Ai =
A1 ⊗ A2 ⊗ A3 = σ ((Πi )1≤i≤3 )
1. Mostre que A = σ {Π31=1 Ai : Ai ∈ Ai } .
2. Mostre que (P1 ⊗
2 ) ⊗ P3 ≡ P1 ⊗ (P2 ⊗ P3 ) e conclua que se pode definir sem ambiguidade o
QP
N3
N3
3
espaço produto
i=1 Ωi ,
i=1 Ai ,
i=1 Pi .
Exercı́cio 289. Seja no espaço de Lebesgue ([−1, 1]2 , B([−1, 1]2 ), λ ⊗ λ) a função f dada por:
(
xy
para − 1 ≤ x ≤ 1 e − 1 ≤ y ≤ 1 , (x, y) 6= (0, 0)
2
2 2
f (x, y) = (x +y )
0
para (x, y) = (0, 0) .
Mostre que os integrais iterados sobre [−1, 1]2 se existirem, são iguais mas que f não é integrável.
Exercı́cio 290. Seja no espaço de Lebesgue ([0, 1]2 , B([0, 1]2 ), λ ⊗ λ) a função f dada por:
( 2 2
x −y
para 0 < x ≤ 1 e 0 < y ≤ 1 , (x, y) 6= (0, 0)
2
2 2
f (x, y) = (x +y )
0
para (x, y) = (0, 0) .
Mostre que
1
Z
1
Z
dx
0
f (x, y)dy =
0
π
4
1
Z
Z
0
1
f (x, y)dx = −
dy
0
π
4
e explique porque é que este resultado não contradiz o teorema de Fubini.
Exercı́cio 291 (Decomposição de um bolo por camadas; layer cake decomposition). Considere o espaço
de medida ([0, +∞[, B([0, +∞[), ν) em que ν é uma medida arbitrária. Seja para cada t ∈ [0, +∞[, a
função φ(t) = ν([0, t[) que se supõe finita em cada t. Seja (X, A, µ) um qualquer espaço de medida e f
uma função mensurável positiva definida sobre X. Mostre que:
Z
Z +∞
φ(f (x))dµ(x) =
µ({x ∈ X : f (x) > t})dν(t)
X
e interprete geometricamente esta igualdade.
252
0
O Teorema de Fubini, Leis Conjuntas e a Construção de Sucessões de Variáveis
Aleatórias Independentes
Exercı́cio 292. Considere o espaço ([0, 1]2 , B([0, 1]2 ), λ ⊗ λ) e a função f definida por:

2n

para 2−n ≤ x < 2−n+1 e 2−n ≤ y < 2−n+1
2
2n+1
f (x, y) = −2
para 2−n−1 ≤ x < 2−n e 2−n ≤ y < 2−n+1


0
nos outros casos .
Mostre que:
1
Z
1
Z
1
Z
0
0
0
1
Z
f (x, y)dy = 1
dx
f (x, y)dx = 0
dy
0
e explique porque é que este resultado não contradiz o teorema de Fubini.
Exercı́cio 293. Seja ([0, +∞[, B([0, +∞[), λ) o espaço de Lebesgue e considere-se para α > 0 e para
x>0
Z x
(x − t)α−1 f (t)dλ(t) .
gα (x) =
0
Mostre que para y > 0:
y
Z
gα (x)ddλ(x) = gα+1 (y) .
α
0
Exercı́cio 294. Seja (N, P(N), µc ) o espaço discreto sobre o conjunto dos inteiros com a µc a medida
de contagem.
1. Mostre que o espaço produto de (N, P(N), µc ) por ele próprio é: (N2 , P(N2 ), µc ⊗ µc ).
2. Seja f uma função de N2 em R. Mostre que se a famı́lia (f (n, m))(n,m)∈N2 é absolutamente
somável, isto é se:
+∞
X
| f (n, m) |< +∞ ,
n,m=0
então (f (n, m))(n,m)∈N2
+∞
X
f (n, m) =
n,m=0
+∞ X
+∞
X
n=0 m=0
f (n, m) =
+∞ X
+∞
X
f (n, m) .
m=0 n=0
Observação 63. Dado que o conjunto N∗ não tem uma ordem natural não podemos usar
a definição de soma de série habitual. Assim, é conveniente definir o que entendemos
por somabilidade de uma famı́lia indexada por um conjunto qualquer. Para efeitos do
exercı́cio anterior, diremos que a famı́lia (ai )i∈I é absolutamente somável sse:
(
)
X
sup
| ai |: J ⊂ I, #J < +∞ < +∞ .
i∈J
Exercı́cio 295. Seja o espaço (N2 , P(N2 ), µc ⊗ µc ) e

−n

1 + 2
g(n, m) = −1 − 2−n


0
considere a função g de N2 em R definida por:
para n = m
para n = m + 1
nos outros casos .
Mostre que os integrais iterados são diferentes consoante a ordem de integração e explique porque é que
este resultado não contradiz o teorema de Fubini.
253
Capı́tulo 9
9.7
Resoluções
As resoluções que figuram seguidamente foram desenvolvidas no sentido de demonstrar
o que deve ser o trabalho de redacção individual que cada aluno deve realizar no seu
estudo das matérias da disciplina. Quando se resolve um exercı́cio, deve-se procurar não
só, ter uma resposta satisfatória às questões colocadas mas sempre que possı́vel aproveitar
para relembrar cuidadosamente os conhecimentos aprendidos anteriormente que melhor
elucidem o trabalho de compreensão do assunto que o exercı́cio desenvolve. Um exemplo
daquilo a que nos referimos é o uso do teorema de mudança de variável que foi estudado
quando se abordou a mudança de coordenadas nos integrais múltiplos. É claro que um
trabalho em profundidade deste tipo nem sempre é possı́vel. Mas é desejável que seja
efectuado sempre que para tal haja disponibilidade e energia suficiente.
Resolução: Exercı́cio 8 Este exercı́cio requer uma pequena revisão sobre alguns
conceitos topológicos simples.
1. Tal como no casoN
do produto de dois espaços mensuráveis há, pelo menos, duas
formas de definir ni=1 Bi .
(a) Tal como no exercı́cio 286 pode definir-se directamente
n
O
Bi = σ(Π1 , · · · , Πn ) ,
i=1
em que Πi é a i-ésima projecção, ou
(b) alternativamente pode-se definir
S = {Si × · · · × Sn : Si ∈ Bi }
e depois
Nn
i=1 Bi
= σ(S).
Naturalmente, estes dois processos devem conduzir à mesma álgebra-σ. Para proceder à verificação deste facto importa notar que:
−1
Π−1
1 (Si ) ∩ · · · ∩ Πn (Sn ) = S1 × · · · × Sn .
(9.5)
Esta igualdade mostra que S ⊆ σ(Π1 , . . . , Πn ) pelo que σ(S) ⊆ σ(Π1 , . . . , Πn ). Esta
igualdade mostra ainda que
Π−1
i (Si ) = X1 × · · · × Xi−1 × Si × Xi+1 × · · · × Xn ∈ S ⊆ σ(S)
logo Πi ∈ m(Bi , σ(S)) o que implica que σ(S) ⊇ σ(Π1 , . . . , Πn ) pela definição desta
álgebra-σ .
2. Πi sendo contı́nua de (X, τ ) em (Xi , τi ) é mensurável de (X, Bτ ) em (Xi , Bi ). A
fórmula9.5 mostra agora que
−1
∀Bik ∈ Bk Bi1 × · · · × Bin = Π−1
1 (Bi1 ) ∩ · · · ∩ Πn (Bin ) ∈ Bτ
N
logo S ⊆ Bτ e assim, σ(S) = ni=1 Bi ⊆ Bτ .
254
O Teorema de Fubini, Leis Conjuntas e a Construção de Sucessões de Variáveis
Aleatórias Independentes
3. Podemos relembrar a seguinte definição.
Definição 60. : (E, Θ) espaço topológico tem uma base numerável de
vizinhanças sse:
∃(Bn )n∈N ∈ ΘN ∀ω ∈ τ ∀x ∈ ω ∃nx ∈ N x ∈ Bnx ⊆ ω .
Exemplo 18. Sendo (rn )n∈N é uma enumeração dos racionais, a famı́lia
1
1
rn −
, rn +
m+1
m + 1 (n,m)∈N2
é uma base numerável de vizinhanças da topologia usual de R.
Exemplo 19. Analogamente se verifica que:
1
Ba r,
m+1
r∈Qn ,m∈N
é uma base numerável de vizinhanças de topologia usual de Rn .
Seja agora ω ∈ τ qualquer e x ∈ ω Admita-se que:
∃ω1 ∈ τ1 , . . . , ωn ∈ τn tais que x ∈ ω1 × · · · × ωn ⊆ ω
(9.6)
Aplicando a hipótese a cada um dos espaços topológicos (Xi , τi ) seja
∀i ∈ {1, . . . n} ∃(Bni )n∈N ∈ τiN
uma base numerável. Logo virá que para x = (x1 , . . . , xn ) ∈ ω1 × · · · × ωn isto é,
tal que xi ∈ ωi temos:
∃Bpi i,x tal que xi ∈ Bpi i,x ⊆ ωi .
Logo
ω=
[
n∈ω
[
{x} ⊆
Bp1,x × · · · × Bpn,x ⊆ ω1 × · · · × ωn ⊆ ω
p1,x ,...,pn,x ∈I1 ×···×In ⊆Nn
e, em consequência
ω=
[
Bp1,x × · · · × Bpn,x ∈ σ(S)
p1,x ,...,pn,x ∈I1 ×···×In ⊆Nn
Pode-se concluir que τ ⊆ σ(S)
N pelo que Bτ ⊆
o resultado anterior que, ni=1 Bi ≡ Bτ .
Nn
i=1 Bi
e, em conclusão final usando
255
Capı́tulo 9
A hipótese que fizemos na fórmula 9.6 leva-nos a esclarecer melhor o que é, de facto,
a topologia produto τ sobre X1 × · · · × Xn = X. Seja T = {ω1 × · · · × ωn : ωi ∈ λi } e
)
(
[
I
Θ = ω ∈ P(X) : ∃(θi )i∈I ∈ T tq. ω =
θi ,
i∈I
isto é Θ é a famı́lia das partes de X que se podem representar como uniões (arbitrárias)
de elementos de T. Então temos que:
1. Θ é uma topologia sobre X (evidente);
2. T ⊆ Θ e Θ é a mais pequena topologia com esta propriedade.
3. λ ≡ Θ.
Note-se que a primeira propriedade é de verificação imediata; a segunda resulta de toda
a topologia ser fechada para as uniões arbitrárias e a terceira resulta, mais uma vez, da
fórmula
−1
(9.7)
ω1 × · · · × ωn ≡ Π−1
1 (ω1 ) ∩ · · · ∩ Πn (ωn ) .
Com efeito:
• as projecções são contı́nuas relativamente a Θ visto que
Π−1
i (ωi ) = X1 × · · · × Xi−1 × ωi × Xi+1 × · · · × Xn ∈ T ⊆ Θ .
Logo τ ⊆ Θ visto que τ é a mais pequena topologia relativamente à qual as
projecções são contı́nuas;
• pela fórmula 9.7, T ⊆ τ e logo por definição de Θ ; Θ ⊆ τ .
Para finalizar seja ω ∈ τ e x ∈ ω como τ ≡ Θ tem-se por definição deste que
[
∃(θi )i∈I ∈ T I ω =
θi
i∈I
como x ∈ ω tem-se que existe ix ∈ I tal que x ∈ θix e, como θin ∈ T tem-se que existem
ω1 ∈ τ1 , dots, ωn ∈ τn tais que x ∈ ω1 × · · · × ωn = θix ⊆ ω.
♦
Resolução:[Exercı́cio 10] Sem necessidade de qualquer cálculo pode afirmar-se que os
dois integrais iterados, se existirem, são obviamente iguais uma vez que f (x, y) = f (y, x),
isto é, f é simétrica em relação às duas variáveis. Com efeito, tem-se que:
Z 1 Z 1
Z 1 Z 1
Z 1 Z 1
f (x, y) dx dy =
f (y, x) dy dx =
f (x, y) dy dx ,
−1
−1
−1
−1
−1
−1
em que a primeira igualdade resulta de substituir x por y e viceversa sem alterar o valor
do integral pois são de variáveis mudas e a segunda igualdade resulta da propriedade
de simetria de f . Para mostrar que f não é integrável basta mostrar que o integral do
256
O Teorema de Fubini, Leis Conjuntas e a Construção de Sucessões de Variáveis
Aleatórias Independentes
módulo de f sobre [−1, 1]2 , ou sobre um subconjunto deste produto, não é finito. Para
fazer desaparecer os módulos basta considerar [0, 1]2 . Com efeito:
x=1
Z 1
xy
y
1
y
dx = −
=
−
.
2
2
2
2
2
2(x + y ) x=0 2y 2(1 + y 2 )
0 (x + y )
Em consequência:
Z 1 Z
0
Z 1
Z 1
xy
1
y
dx
dy
=
dy
−
dy =
2 + y 2 )2
(x
2y
2(1
+
y2)
0
0
0
h y iy=1 1 y=1
−
= ln( )
ln(1 + y 2 ) y=0 = +∞ ,
2 y=0 4
pelo que se tem
Z 1 Z
−1
1
−1
1
| xy |
dx
(x2 + y 2 )2
Z
1 Z 1
dy ≥
0
0
xy
dx
(x2 + y 2 )2
dy = +∞ .
A questão natural agora é como justificar que os integrais iterados existem sendo que f
não integrável sobre [−1, 1]2 . A resposta é que é possı́vel calcular os integrais iterados
não como integrais de Lebesgue, mas sim como valores principais no sentido de Cauchy 3
determinados da mesma forma que os integrais de Riemann impróprios. Com efeito:
!
Z
Z 1 Z 1
Z
xy
lim
f (x, y) dx dy = lim
dx dy =
→0 {≤|y|≤1} δ→0 {δ≤|x|≤1} (x2 + y 2 )2
−1
−1
x=−δ x=1 !
Z
y
y
+ −
dy = 0 .
= lim
lim
−
→0 {≤|y|≤1} δ→0
2(x2 + y 2 ) x=−1
2(x2 + y 2 ) x=δ
Na aula prática, para verificação rápida que f não é integrável, foi sugerida a utilização
do teorema de mudança de variável no integral de Lebesgue que enunciamos a seguir.
Teorema 45. Sejam U e D dois abertos de Rn e ϕ : U → D um difeomorfismo
de classe C 1 . Então para qualquer função mensurável f : D → R+ ,
Z
Z
f (x)dx =
f (φ(u)) | Jϕ (u) | du
D
U
0
em que Jϕ (u) = det(ϕ (u)) é o Jacobiano de ϕ calculado em u.
Por este teorema ter-se-á passando às coordenadas polares e integrando sobre o subdomı́nio de [0, 1]2 correspondente à bola de centro em zero e raio igual a um intersectada
com [0, 1]2 , que:
Z 1 Z 2π
Z 1
Z
| xy |
dρ
dρ
dx dy ≥
| cos θ sin θ | dθ
=2
= +∞ .
2
2
2
ρ
0
0
0 ρ
[0,1]2 (x + y )
Rb
O valor principal no sentido de Cauchy de um integral a f (x)dx em que f admite uma singularidade
R x0 −
Rb
isolada em x0 ∈]a, b[ é dado por lim→0 a
f (x)dx + x0 + f (x)dx. Este valor principal pode existir
sem que os integrais impróprios correspondentes sejam convergentes.
3
257
Capı́tulo 9
♦
Observação 64. Este exercı́cio permite-nos pois concluir que uma eventual igualdade dos
integrais iterados não força a existência do integral duplo correspondente.
Resolução:[Exercı́cio 11] Tem-se que:
Z
1
0
x=1
−1
x2 − y 2
x
dx = − 2
=
.
2
2
2
2
(x + y )
x + y x=0 1 + y 2
pelo que
1 Z 1
Z
0
0
Z 1
x2 − y 2
−1
π
dx
dy
=
dy = − [arctan y]y=1
.
y=0 = −
2
(x2 + y 2 )2
1
+
y
4
0
Da mesma forma, usando a antisimetria de f :
Z
1 Z 1
0
0
x2 − y 2
dy
(x2 + y 2 )2
Z
1 Z 1
dx = −
0
0
y 2 − x2
dy
(x2 + y 2 )2
dx =
π
.
4
Podemos mostrar que f não é integrável considerando f + , a parte positiva de f , definida
por:
x2 − y 2
f + (x, y) = 2
I
(x + y 2 )2 {(x,y):0<y≤x}
com a qual se pode escrever, pelo teorema de Fubini:
Z
1 Z 1
+
f (x, y) dy
0
Z
1 Z x
dx =
0
0
Z
=
0
0
1
x2 − y 2
dy
(x2 + y 2 )2
Z
1
dx =
0
y
x2 + y 2
y=x !
dx =
y=0
dx
= +∞ .
2x
♦
Resolução:[Exercı́cio 12] A igualdade resulta de termos a seguinte cadeia de igualdades, justificada em (a), teorema de Fubini que se pode aplicar dado que estamos em
presença de funções positivas.
Z
+∞
+∞ Z
Z
µ({x ∈ X : f (x) > t}) dν(t) =
0
0
Z Z
=
+∞
X
I{x∈X:f (x)>t} dµ(x) dν(t) =(a)
!
Z
Z
f (x)
I{x∈X:f (x)>t} dν(t) dµ(x) =
X
0
dν(t) dµ(x)
X
0
Z
=
φ(f (x))dµ(x) .
X
♦
258
O Teorema de Fubini, Leis Conjuntas e a Construção de Sucessões de Variáveis
Aleatórias Independentes
Resolução:[Exercı́cio 13] A

2n

2
f (x, y) = −22n+1


0
função em estudo é dada por:
para 2−n ≤ x < 2−n+1 e 2−n ≤ y < 2−n+1
para 2−n−1 ≤ x < 2−n e 2−n ≤ y < 2−n+1
nos outros casos .
A função f é uma função em escada uma vez que é constante sobre rectângulos. Uma
representação gráfica das primeiras parcelas não nulas em que se decompõe f como soma
de funções indicatrizes de rectângulos dá:
n=1

2

2
f (x, y) = −23


0
para 1/2 ≤ x < 1 e 1/2 ≤ y < 1
para 1/4 ≤ x < 1/2 e 1/2 ≤ y < 1
nos outros casos .
É imediato verificar que o integral sobre [0, 1]2 da primeira parcela da função que
corresponde ao primeiro ramo, vale 1. O integral sobre o rectângulo definido no
segundo ramo vale −1.
n=2

4

2
f (x, y) = −25


0
para 1/4 ≤ x < 1/2 e 1/4 ≤ y < 1/2
para 1/8 ≤ x < 1/4 e 1/4 ≤ y < 1/2
nos outros casos .
Pode-se fazer exactamente a mesma observação relativamente aos integrais correspondentes a cada um dos ramos. O correspondente ao primeiro ramo vale 1 e o
correspondente ao segundo vale −1.
O leitor é fortemente aconselhado a representar graficamente as duas parcelas da função
em escada correspondentes a n = 3. Para concluir, no que toca aos integrais iterados,
podemos observar que integrando primeiro em ordem a y, por exemplo, vem (veja-se a
figura):
!
Z 1 Z 1
Z 1
+∞ Z 1/2(n+1)
X
dx
f (x, y) dy =
dx
f (x, y) dy = 1 + 0 + 0 + · · · = 1
0
0
n=0
1/2n
0
e integrando primeiro em ordem a x vem (veja-se a figura):
!
Z 1 Z 1
Z 1
+∞ Z 1/2(n+1)
X
dy
f (x, y) dx =
dy
f (x, y) dx = 0 + 0 + 0 + · · · = 0 .
0
0
n=0
1/2n
0
Este resultado não contradiz o teorema de Fubini uma vez que como se pode verificar
imediatamente (veja-se a figura):
Z 1 Z 1
Z 1 Z 1
Z
dy
| f (x, y) | dx =
dx
| f (x, y) | dy =
| f (x, y) | dx ⊗ dy = +∞
0
0
0
0
[0,1]2
259
Capı́tulo 9
♦
Resolução:[Exercı́cio 14] Trata-se de uma transformação funcional que corresponde
para as funções regulares a um integral fraccionário 4 . Com efeito, a fórmula que se
pretende demonstrar que é para y > 0:
y
Z
Z
x
gα (x)dλ(x) = gα+1 (y) com gα (x) =
α
(x − t)α−1 f (t)dλ(t) ,
0
0
0
pode ser escrita para funções f contı́nuas em ]0, +∞[ como gα+1 (y) = αgα (y).
Por aplicação do teorema de Fubini em (a), supondo que f é integrável, temos que:
Z
α
0
y
y
Z
x
gα (x)dλ(x) =
(x − t)
f (t) dλ(t) dλ(x) =(a)
0
0
Z y
Z y
Z y
α−1
=
f (t)
(x − t)
dλ(x) dλ(t) =
f (t) [(x − t)α ]x=y
x=t dλ(t) =
0
0
t
Z y
f (t)(y − t)α dλ(t) = gα+1 (y) .
=
Z
α−1
0
Note-se que para determinar os limites de integração após alteração da ordem pela qual
se calcula os integrais iterados pode ser importante a figura seguinte.
♦
Resolução:[Exercı́cio 15] Este exercı́cio desenvolve uma formulação do teorema de
Fubini num espaço discreto relevante que é aquele em que o integral corresponde à soma
de uma série.
1. Basta verificar que P(N) ⊗ P(N) = P(N2 ). Para esse efeito é suficiente mostrar
que P(N2 ) ⊆ P(N) ⊗ P(N) uma vez que a outra inclusão é imediata. Seja então
J ∈ P(N2 ). Como se tem:
J=
[
(n,m)∈J
{(n, m)} =
[
({n} × {m})
(n,m)∈J
e obviamente {n} × {m} ∈ S = {A × B : A ∈ P(N), B ∈ P(N)} tem-se necessariamente que J ∈ σ(S) = P(N) ⊗ P(N) tal como se pretendia.
2. Para demonstrarmos que a condição indicada é suficiente note-se que por aplicação
4
260
Para ter uma ideia rápida veja, por exemplo, http://mathworld.wolfram.com/FractionalIntegral.html
O Teorema de Fubini, Leis Conjuntas e a Construção de Sucessões de Variáveis
Aleatórias Independentes
do teorema da convergência monótona de Lebesgue em (a):
Z
Z
| f (n, m) | d(µc ⊗ µc )(n, m) = S S
| f (n, m) d(µc ⊗ µc ) =
N2
p∈N
(n,m)∈N2 n,m≤p {(n,m)}
Z
| f (n, m) |
=
N2
Z
= lim
p→+∞ N2
lim IS(n,m)∈N2 n,m≤p {(n,m)} d(µc ⊗ µc )(n, m) =(a)


X
I{(n,m)}  d(µc ⊗ µc )(n, m) =
| f (n, m) | 
(n,m)∈N2 n,m≤p
X
= lim
p→+∞
(n,m)∈N2
=
n,m≤p
X
= lim
p→+∞
p→+∞
(n,m)∈N2
X
Z
N2
| f (n, m) | I{(n,m)} d(µc ⊗ µc )(n, m) =
| f (n, m) |≤
sup
X
| f (n, m) |=
J⊂N2 , #J<ℵ0 (n,m)∈J
n,m≤p
| f (n, m) |< +∞ .
(n,m)∈N2
Está assim verificada uma das condições suficientes para que o teorema de Fubini
possa ser aplicado pelo que se pode concluir o pretendido.
♦
Resolução:[Exercı́cio 16] A função objecto de estudo é dada para (n, m) ∈ N × N
por

−n

1 + 2
g(n, m) = −1 − 2−n


0
para n = m
para n = m + 1
nos outros casos .
Pode observar-se desenhando rapidamente uma figura simples que os pares de inteiros
em que g é não nula se situam sobre a diagonal principal (n = m) e sobre a primeira
subdiagonal (n = m + 1), isto se n estiver nas abcissas e m nas ordenadas. Assim sendo
se calcularmos os integrais iterados - que neste caso são séries - temos que:
! +∞
+∞
∞
X
X
X
g(n, m) =
(1 + 2−n ) + (−1 − 2−n ) = 0 ,
n=1
m=1
n=1
uma vez que para cada n0 fixo, há dois pares nos quais g n— ao se anula: g(n0 , n0 ) =
1 + 2−n0 e g(n0 , n0 − 1) = −1 − 2−n0 . Por outro lado temos também que,
! +∞
+∞
∞
X
X
X
1
g(n, m) =
(1 + 2−m ) + (−1 − 2−m−1 ) = ,
2
m=1
n=1
m=1
dado que para cada m0 fixo, há também dois pares nos quais g não se anula: g(m0 , m0 ) =
1 + 2m0 e g(m0 + 1, m0 ) = −1 − 2−m0 − 1. Este resultado não contradiz o teorema
de Fubini porque nenhuma das condições suficientes para que os integrais iterados de
261
Capı́tulo 9
g fossem iguais se verifica. Com efeito, da mesma forma que calculámos os integrais
iterados de g podemos calcular os integrais iterados de | g |, obtendo-se:
! +∞
+∞
∞
X
X
X
| g(n, m) | =
(1 + 2−n ) + (1 + 2−n ) = +∞ ,
n=1
m=1
n=1
e também
+∞
X
∞
X
m=1
n=1
!
| g(n, m) |
=
+∞
X
(1 + 2−m ) + (1 + 2−m−1 ) = +∞ .
m=1
Por outro lado para qualquer J ⊂ N2 tem-se que
X
X
| g(n, m) |≤
(1 + 2−n ) + (1 + 2−n )
n:∃m∈N (n,m)∈J
(n,m)∈J
pelo que
sup
J⊂N2
X
| g(n, m) |≤
#J<ℵ0 (n,m)∈J
X
sup
J⊂N2
(2 + 2−n+1 ) = +∞ .
#J<ℵ0 n:∃m∈N (n,m)∈J
assim se confirmando que a famı́lia (g(n, m))(n,m)∈N×N não é somável.
9.8
[2]
♦
Exercı́cios
Exercı́cio 296. Sejam (X, A, µ) ainda (Y, B, ν) dois espaços de medida σ-finitos. Considere f uma
função numérica definida sobre X e µ-integrável, uma outra função numérica g definida sobre Y e
ν-integrável. Seja F a função numérica definida sobre X × Y por:
∀(x, y) ∈ X × Y F (x, y) = f (x)g(y) .
Mostre que F é µ × ν-integrável.
[3]
Exercı́cio 297. Considere o espaço topológico compacto constituı́do pelo conjunto dos reais R munido
da topologia usual τu . Seja Cc (R) o conjunto das funções contı́nuas com suporte compacto definidas sobre
R. Sejam µ e ν duas medidas finitas positivas definidas sobre a σ-álgebra de Borel de (R, τu ). Considere
finalmente ψ, a aplicação definida por:
Z Z
∀f ∈ Cc (R) ψ(f ) =
f (x + y)dµ(x)dν(y) .
R
R
1. Mostre que existe uma medida positiva µ ∗ ν sobre (R, σ(τu )) tal que se tem:
Z
∀f ∈ Cc (R) ψ(f ) =
f (z)d(µ ∗ ν)(z) .
R
2. Suponha que µ e ν são absolutamente contı́nuas relativamente a λ,a medida de Borel. Determine
então uma função h, integrável relativamente à medida de Borel, que verifica:
Z
∀f ∈ Cc (R) ψ(f ) =
f (z)h(z)dλ(z) .
R
262
O Teorema de Fubini, Leis Conjuntas e a Construção de Sucessões de Variáveis
Aleatórias Independentes
Bibliografia
[1] A. N. Kolmogorov. Foundations of the theory of probability. Chelsea Publishing Co.,
New York, 1956. Translation edited by Nathan Morrison, with an added bibliography
by A. T. Bharucha-Reid.
[2] David Williams. Probability with martingales. Cambridge Mathematical Textbooks.
Cambridge University Press, Cambridge, 1991.
263